You are on page 1of 631

Question . 1. Clinical findings are influenced by a child's developmental stage.

Which of the following features of the physical examination is most strongly


influenced by developmental stage?

Skin turgor

Respiratory rate
Explanation: Respiratory rate is highest in the neonate and
gradually becomes slower approaching adult rates in the
preadolescent-adolescent age period. Respiratory rate may be
influenced by serious pulmonary or airway infection or
inflammation in addition to non-respiratory causes, such as
fever, anxiety, acidosis, pain, heart failure, central nervous
system disease, drugs (stimulants, depressants, aspirin), and
toxins (ammonia). (See Chapter 49 in Nelson Textbook of
Pediatrics, 17th edition.)

Lung aeration

Mitral valve closure

Fontanel tension
Question . 2. The vulnerable child syndrome most influences which of the following
components of the well child assessment?

Physical examination

Observation

Developmental assessment

History
Explanation: Parents perceive vulnerability due to many real
or exaggerated risks: premature birth, difficult pregnancy,
previous death of a child, disease in other siblings, or minor
low-risk conditions in this child. The history is usually
exaggerated, embellished, and quite detailed. (See Chapter 49
in Nelson Textbook of Pediatrics, 17th edition.)

Growth
Question . 3. During examination of an 8-mo-old child, the difficulty of optimal chest
auscultation is due primarily to:

Recent meal with gastric distention

Rapid respiratory rate

Stranger anxiety
Explanation: Crying makes the chest physical examination
almost impossible, except for detection of the grossest
abnormalities,. If at all possible, the chest auscultation should
be preceded by observation and then performed in the
sleeping, calm, nursing, or feeding infant. (See Chapter 49 in
Nelson Textbook of Pediatrics, 17th edition.)
Transmitted nasal sounds

Pliable chest wall


Question . 4. Assessment of the state of well-being by observation relies mostly on
the child's:

Nutritional status

Motor ability

Visual behavior

Interaction with parents


Explanation: The interaction with the parents is most critical,
as this is the child's "normal" environment and is free of
additional factors such as stranger anxiety. As children come
with all varieties of behaviors, it is essential to ask the parents if
the observed behavior is "normal" for the child and, if not, how
it is different. (See Chapter 49 in Nelson Textbook of
Pediatrics, 17th edition.)

State variation
Question . 5. Based on an orientation to child development, when would you tell
parents the highest risk of poisoning in children is present?

6 mo

1 yr

2 yr
Explanation: Self-poisoning in toddlers occurs once they
become ambulatory and are able to walk and climb. Consider
no place safe, especially if there are older siblings. Also
consider that child-proof medicine containers will rarely be a
deterrent to a motivated toddler. Remember that plants,
berries, and liquids also present risks for ingestion in this age
group. (See Chapter 49 in Nelson Textbook of Pediatrics, 17th
edition.)

4 yr

6 yr

Question . 6. Injury control is a more appropriate term than accident prevention


because accidents are:

Not predictable

Not preventable

Random

Not due to chance


Explanation: Injuries have definable risks, are not random,
and are both predictable and preventable. The term "accident"
confuses our meaning and should no longer be used. (See
Chapter 50 in Nelson Textbook of Pediatrics, 17th edition.)

Not common
Question . 7. Motor vehicle injuries lead the list of injury deaths for all ages and are
most often associated with:

Alcohol use

Occupant injury

Driver age younger than 17 yr

Presence of more than two occupants in the car

Night driving

All of the above


Explanation: All are risk factors that can be modified by
behaviors and laws, such as a graduated driver's licensing
program. (See Chapter 50 in Nelson Textbook of Pediatrics,
17th edition.)
Question . 8. Risk factors for injury include all of the following except:

Toddler age

Female gender
Explanation: After 1-2 yr of age, males have a much higher
injury rate than females. This risk lasts until the 7th decade.
Adolescent risk-taking behavior accounts for some of the
difference during the teen years. (See Chapter 50 in Nelson
Textbook of Pediatrics, 17th edition.)

Poverty

Front-seating in an automobile

Chaotic family stress


Question . 9. The proper age at which a child may cross a busy street alone after
careful instruction is:

4-5 yr

6-7 yr

8-10 yr

10-12 yr

Developmentally, children below this age are unable to


understand the risk and judge such factors as car speed.
Chronological age does not determine readiness, which
requires instruction, trial runs with a parent, and parental
judgment of the child's ability. (See Chapter 50 in Nelson
Textbook of Pediatrics, 17th edition.)

None of the above

Question . 10. The majority of children with medical emergencies present to all of
the following for care except:

Pediatrician's office

Children's hospital emergency department


Explanation: Most children receive all or initial emergency
care at sites other than a specialized pediatric emergency room
in a children's hospital. (See Chapter 51 in Nelson Textbook of
Pediatrics, 17th edition.)

Community hospital emergency department

Clinic

Urgent care facility

Question . 11. A general pediatric office should be prepared for emergencies. This
preparedness includes all of the following except:

Training in advanced trauma care


Explanation: Although advanced life support is ideal, it is
unrealistic for a general office to be prepared for advanced
trauma care. The patient should be stabilized as best as
possible, with placement of an airway and intravascular lines,
and transported to a facility capable of caring for pediatric
trauma patients as soon as possible. (See Chapter 51 in
Nelson Textbook of Pediatrics, 17th edition.)

Education of staff in CPR

Policies for triage

Procedure for resuscitation

Transport protocols
Question . 12. A 15-mo-old child is in your office with stridor at rest and cyanosis
with the presumptive diagnosis of viral croup. You should do all of the following
except:

Transport immediately in the parent's car to the nearest


emergency department
Explanation: A cyanotic child in respiratory distress is at high
risk for a respiratory arrest. No acutely ill child should be
transported by the parents no matter how short the distance or
how long the delay for EMS to arrive. (See Chapter 51 in
Nelson Textbook of Pediatrics, 17th edition.)

Administer oxygen
Administer dexamethasone

Administer racemic epinephrine

Transport after EMS personnel arrive 20 min later

Question . 13. Enhanced 911:

Is one-touch dialing

Identifies patient location


Explanation: The location of the caller is automatically
identified. This is especially important if the caller is unable to
communicate their location (young age, coma, seizure, drug
overdose, disability). (See Chapter 51 in Nelson Textbook of
th
Pediatrics, 17 edition.)

Triages based on severity of illness

Responds within 5 min

Provides MD presence with EMS

Question . 14. Emergency Medical Treatment and Active Labor Act (EMTALA) is a
set of federal regulations that addresses the transfer of patients. Which of the
following statements regarding EMTALA is not true?

The transferring hospital must provide an appropriate medical


screening to assess if the patient has an emergency condition

If an emergency condition exists, the patient's condition must


be stabilized, or if stabilization measures exceed that hospital's
expertise, the patient must be transferred to a hospital capable
of such measures

Transfer of unstable patients is permitted under limited


circumstances

The law does not apply to pediatricians who are on call for
consultation to the emergency department
Explanation: This law is meant to benefit patients and includes
patients of all ages and their doctors. All patients must be
stabilized to the best of the ability of the emergency room staff.
All pertinent data must also be transferred with the patient to
the most appropriate hospital capable of caring for the patient's
condition. (See Chapter 51 in Nelson Textbook of Pediatrics,
th
17 edition.)
Question . 15. There are several requirements in preparing a child for transfer to a
higher level of care (e.g., from the office to the emergency department). In preparing
for transfer of a child, which of the following is not recommended?

Obtaining written consent for transfer from the patient's parent


or guardian
Copying diagnostic tests, radiographs and the child's medical
record

Calling and giving report to the appropriate transport agency

Instructing transport agency to call receiving physician to


secure acceptance for transfer
Explanation: Doctor-to-doctor communication is essential
when transferring a critically ill patient. This is not the
responsibility of an ambulance company. (See Chapter 51 in
Nelson Textbook of Pediatrics, 17th edition.)

Documenting name of transport agency and the time that the


transport occurred

Question . 16. Emergencies involving children are stressful for the child, parent, and
EMS-C providers. All of the following are useful in decreasing stress to children and
their families in emergency settings except:

Keeping the parents away during procedures or


resuscitation
Explanation: Indeed, most parents can provide additional
calming and distraction during procedures, and their presence
should be encouraged. The question of parents being present
during resuscitation is controversial, but most physicians find
that it usually does no harm and may be of value later to
grieving parents. (See Chapter 51 in Nelson Textbook of
th
Pediatrics, 17 edition.)

Training staff in calming and distraction techniques

Separating the child from other frightening sights and sounds in


the treatment area

Communicating clearly, with written instructions accompanying


verbal information whenever possible

Screening for mental health needs

Question . 17. The safest and quickest manner to transport a critically ill child from a
community hospital to the regional pediatric center is:

Have the parents drive the child from their local hospital

Request that the local paramedics transport the child

Accompany the child in the ambulance with the local


paramedics

Request that the tertiary pediatric facility assist and


transport the patient
Explanation: The care and transport of a critically ill child
requires staff with specific experience and knowledge of the
pediatric population and the illnesses necessitating
transportation. In addition, the equipment, medications, and
means to monitor children require pediatric-specific expertise.
Coordinated efforts with a pediatric transport program yield the
safest methods of transport. (See Chapter 53 in Nelson
Textbook of Pediatrics, 17th edition.)

Question . 18. The transport team from the tertiary hospital is composed of all of the
following except:

A parent who can assist in the care of the child


Explanation: Parents are not expected to provide care during
pediatric transports. Nonetheless, if room is available in the
transport vehicle, a parent may accompany the child. Usually
this is not possible, and the parent follows the transport van in
another vehicle. (See Chapter 53 in Nelson Textbook of
Pediatrics, 17th edition.)

Team members skilled in various aspects of pediatric critical


care

A dispatch service that facilitates communication with the


referring hospitals

A medical control physician who is available for telephone


consultation

Question . 19. Appropriately trained and equipped pediatric transport teams should
be able to:

Perform major surgical procedures at the referring hospitals

Provide appropriate medical care during the transport


Explanation: Appropriately trained and prepared (based on
information from the referring hospital) transport staff should be
able to care for the patient en route to the PICU. This does not
mean that a patient's condition cannot deteriorate during
transport as part of the natural history of the disease. Special
transports (ECMO) are not common and are used for only
unusual circumstances. (See Chapter 53 in Nelson Textbook of
Pediatrics, 17th edition.)

Place a patient in extremis on ECMO

Transport a patient without incident

Question . 20. The mother of a 5-yr-old near-drowning victim arrives at the pediatric
intensive care unit (PICU). She is highly upset and emotional, and forcefully
demands to see her child. The best response of the PICU staff to the mother would
be:

To sit with her and explain the procedures of the PICU,


including times permitted for visitation, the number of visitors
permitted

To direct her to a social worker who would provide a


description of the rules for visitation in the PICU

To require that she speak with the child's physician before


being permitted to visit the bedside

To direct her to the parents' waiting area, and inform her that
she will be summoned when the time is right

To take her as soon as possible to the bedside, after


having provided a brief description of what the room might
look like, what medical devices will be present, and what
level of response she might expect from her child
Explanation: It is not always possible for a parent to
immediately be brought into a child's PICU room. A health care
provider should be there for the parent to explain the patient's
condition and facilitate ongoing communication. Nonetheless,
this process should be brief, as any delay increases anxiety
and possibly mistrust. (See Chapter 54 in Nelson Textbook of
Pediatrics, 17th edition.)
Question . 21. A child has been in the PICU for 10 days and still faces at least a
week of further treatment. Various family members have consulted with a variety of
treating medical staff about prognosis. As a result, the family has heard several
contradictory versions of what the treatment plan will be. You should now:

Advise the family to speak only to you in the future


Explanation: All of these answers have been suggested
(except the letter to the administrator) and depend on the
circumstances in the PICU and the patient. Although "B" has
value, once confusion has taken over it is important for one
person to communicate with the family. Practically, this is not
always possible. (See Chapter 54 in Nelson Textbook of
Pediatrics, 17th edition.)

Schedule regular meetings where representatives of the


different services are present and consensus can be reached

Advise the family to write a letter to the hospital administrator

Advise the family not to speak to consulting physicians

Advise the family that this degree of ambiguity is unavoidable in


this setting
Question . 22. A 2-yr-old child arrives in the PICU in respiratory distress and soon
requires intubation and mechanical support. Because of a heart murmur detected 2
days later, a cardiology consultation is requested, and the fellow performing the
consultation speaks with the family, indicating the need for immediate heart surgery.
The family is distressed at this news, and wonders why you have not mentioned the
possibility of surgery. The most appropriate next step in management is to:

Ask the family to discuss the matter further with the cardiology
service

Contact the chief of cardiology and lodge a complaint about the


actions of the fellow

Convene a meeting with representatives from your service


and the cardiology service, develop a plan, and then meet
with the family to present recommendations
Explanation: When such communication catches you off
guard, regroup the team and family and discuss the events that
led to the diagnosis and surgery. Never forget the best interest
of the patient despite less-than-optimal communication. (See
Chapter 54 in Nelson Textbook of Pediatrics, 17th edition.)

Advise the family not to give much credence to the fellow's


opinions

Summon the fellow to your office and instruct him/her never to


speak to the family about treatment plans unless you are
present

Question . 23. The mother of one of your PICU patients regularly looks through the
bedside medical chart of her child. Bedside nurses report this to you and express
their discomfort with the practice. Your most appropriate response would be to:

Advise the nurses that it is the mother's right to view the chart
and nothing should be done

Report the matter to the hospital authorities

Suggest to the mother that you or your representative


would like to go through the chart with her on a regular
basis to clarify the jargon and explain the content more
fully
Explanation: Charts should not be read in isolation. Notes or
laboratory data are easily misinterpreted and require a health
care worker to help communicate their meaning and
significance. (See Chapter 54 in Nelson Textbook of Pediatrics,
17th edition.)

Instruct the mother that she may not view the chart since it
contains the writings of several different health care providers
who have not consented to her viewing it

Enlist the help of a social worker to persuade her that viewing


the chart is not appropriate

Question . 24. You inform the family of a gravely ill child in the PICU that she is very
likely to die soon. The family, consistent with their faith, wishes to apply oils to her
body and place various amulets on the bed. Your reaction should be to:

Refer the matter to the hospital attorney

Refer the matter to the chaplain

Persuade them that the application of oils and the presence of


amulets cannot possibly influence the child's health status

Inform them that so long as what they wish to do does not


pose immediate threat to their child's health, you support
their wishes
Explanation: This is a most important example of
understanding cultural issues in health, life, and dying. To this
family, not performing the ritual may prevent the child from
dying peacefully. A chaplain familiar with the family's faith is
also useful, whether employed by the hospital or present as the
family's personal spiritual advisor. (See Chapter 54 in Nelson
Textbook of Pediatrics, 17th edition.)

Inform them that hospital policies forbid applying the oil


because it is an unauthorized form of medical treatment

Question . 25. Proper use of the PRISM scoring system would include:

Decision-making in end-of-life issues for a chronically ill child

Withdrawal of support decisions for a child with multiple organ


failure

Comparison of level of disease severity between treatment


and control groups
Explanation: The Pediatric Risk of Mortality score is based on
17 physiologic variables (vital and neurologic signs, acid-base,
blood chemistries, hematologic parameters) subdivided into 26
ranges and taking into consideration age (neonate, infant, child,
adolescent). It is best in predicting mortality for populations of
patients and not for an individual PICU patient. Decision-
making at the end of life should never be based on an acute
PRISM score, especially in a chronically ill child. It has no
relevance or reliability in non-PICU patients, such as those
receiving chemotherapy. (See Chapter 56 in Nelson Textbook
of Pediatrics, 17th edition.)

Assessment of performance of a chemotherapy regimen

Question . 26. Which of the following scoring systems is useful for triage decisions?

PRISM (Pediatric RISK of Mortality)

Pediatric Trauma Score


Explanation: The pediatric trauma score is made specifically
for triage to a higher-level unit, such as a level I trauma center.
The other scores are most useful in assessing physiologic
instability resource utilization in an acute PICU setting. (See
Chapter 56 in Nelson Textbook of Pediatrics, 17th edition.)

APACHE (Acute Physiology and Chronic Health Evaluation)

TISS (Therapeutic Intervention Scoring System)

Question . 27. Regarding resuscitative efforts, the most important goal is:

Restoration of age-appropriate heart rate

Appropriate movement of the chest wall

Auscultation of equal breath sounds in both lung fields

Adequate oxygen delivery and utilization for the body


tissues
Explanation: Although all of these goals are important, they all
reflect the rescuer's ability to restore perfusion and oxygen
delivery to vital tissues. The effectiveness of resuscitation can
be assessed by visualizing good chest rise and palpating good
pulses during rescue breathing and chest compressions,
respectively. (See Chapter 57.1 in Nelson Textbook of
Pediatrics, 17th edition.)

Palpation of equal pulses in all four extremities

Question . 28. A 9-mo-old boy is brought to the emergency room in a limp and
unresponsive state. Initial examination shows a pulse rate of 35/min and occasional
irregular breaths. After initiation of CPR (including tracheal intubation), delivery of
oxygen via positive-pressure breaths, and chest compressions, multiple attempts to
insert an IV line fail. The most appropriate next step in management should be to:

Obtain an arterial blood gas sample

Place an intraosseous needle and administer fluids and


inotropic agents
Explanation: Intraosseous (IO) lines should be placed if
venous access is not obtained within 1-2 min of an arrest. The
anterior tibia is the most common site utilized. Chemistries and
other laboratory analyses can be obtained, including a blood
culture, while intravenous fluids and medications can be
delivered through an IO line. The risk of infection is very low.
Once perfusion improves, venous access is usually attainable,
and the IO line can be removed. (See Chapter 57.1 in Nelson
Textbook of Pediatrics, 17th edition.)

Obtain a "stat" head CT study to evaluate reasons for


unresponsiveness

Place a transthoracic cardiac pacemaker

Place a thoracostomy tube to evacuate a possible


pneumothorax

Question . 29. A 9-yr-old boy suddenly experiences respiratory distress while dining
in a restaurant, and despite obvious great effort to breathe, he seemingly cannot
move air. He points to his neck and appears progressively cyanotic. The most
appropriate next step should be to:

Ask the headwaiter to call 911 immediately

Give the child a glass of water and urge him to drink it quickly

Carefully review what the child may have eaten last

Deliver a series of up to five abdominal thrusts (Heimlich


maneuver)
Explanation: The child demonstrates the universal sign of
airway obstruction from a foreign body. If he is awake, finger
sweeps should not be done; the Heimlich maneuver is the
treatment of choice. If airway obstruction continues and the
patient becomes unresponsive, EMS should be activated. (See
Chapter 57.1 in Nelson Textbook of Pediatrics, 17th edition.)

Hyperextend the neck in an effort to open the airway

Question . 30. Which of the following is true regarding provision of assisted


ventilation in the early moments of an emergency?

Any technique that helps clear the upper airway and safely
produces air movement in both lung fields is acceptable

No one should attempt to deliver positive-pressure ventilation


until tracheal intubation can be accomplished

Mouth-to-mouth ventilation is preferred strongly over bag-mask


ventilation

In both infants and older children, rescue breathing should


involve the rescuer forming a seal over the patient's mouth,
with the patient's nose being occluded by the rescuer's thumb
and forefinger

In proper assisted ventilation, there should be no


movement of the abdominal wall during inspiration
Explanation: With mouth-to-mouth or with endotracheal tube
ventilation, chest rise is paramount. One should avoid air entry
to the esophagus by proper placement of the head and neck.
Techniques to open the airway differ, especially if there is head
and neck trauma. Mouth-to-mouth-and-nose ventilation is
appropriate in infants. (See Chapter 57.1 in Nelson Textbook of
Pediatrics, 17th edition.)

Question . 31. A 2-yr-old patient, previously admitted to the hospital for respiratory
distress, is found apneic and pulseless in the early morning hours. She is intubated
and ventilated adequately, and an intra-osseous needle is placed, but she remains
pulseless. The first medications you should employ are:

Lidocaine and epinephrine

Epinephrine and norepinephrine

Norepinephrine and atropine

Lidocaine and atropine

Epinephrine and atropine


Explanation: By this time in an arrest, an ECG should be
available to identify the rhythm. These are the drugs
recommended for asystole, but pulseless electrical activity
(electrical mechanical dissociation) should lead one to search
for a treatable cause of poor pulses in the presence of a
cardiac rhythm capable of effective mechanical cardiac activity
which should be able to support the cardiac output. (See
th
Chapter 57.1 in Nelson Textbook of Pediatrics, 17 edition.)

Question . 32. Causes of pulseless electrical activity include all of the following
except:
Toxins

Pulmonary embolism

Intracranial hemorrhage
Explanation: Agents that work directly on the heart (toxins,
hypoxia) or interfere with cardiac output (tamponade,
pneumothorax) may all produce pulseless electrical activity,
previously known as electrical-mechanical dissociation. If this is
present, the cause must be searched for and treated to achieve
successful resuscitation. (See Chapter 57.1 in Nelson Textbook
of Pediatrics, 17th edition.)

Pericardial tamponade

Pneumothorax

Question . 33. The drug of choice for shock refractory ventricular fibrillation is:

Magnesium

Bretylium

Aminophylline

Amiodarone
Explanation: Amiodarone is now the treatment of choice for
shock-resistant ventricular tachycardia or ventricular fibrillation.
Lidocaine is a second but effective choice. (See Chapter 57.1
in Nelson Textbook of Pediatrics, 17th edition.)

Digoxin
Question . 34. Supraventricular tachycardia is characterized by all of the following
except:

Rate >220 beats/minute in infants

Response to vagal stimuli

Rate >180 beats/minute in children

Response to adenosine

Gradual onset and gradual termination


Explanation: Supraventricular tachycardia (SVT) classically
has a sudden onset and sudden termination if it occurs
spontaneously. In adults, verapamil was once a choice as
treatment for SVT. In children with SVT and poor cardiac
output, verapamil can cause cardiac arrest. Therefore in all
ages, adenosine is the treatment of choice. (See Chapter 57.1
in Nelson Textbook of Pediatrics, 17th edition.)

Question . 35. A 9-yr-old with vomiting and diarrhea has a systolic blood pressure of
75 mm Hg. You should:

Check the fundi for papilledema

Administer 20 mL/kg of normal saline


Explanation: This is an abnormally low systolic blood
pressure. Normal blood pressure between 1 yr to 10 yr should
be 70 + 2 x age (yr), or 88 mm Hg. Normal saline or lactated
Ringer solution without glucose is the initial treatment of choice.
(See Chapter 57.1 in Nelson Textbook of Pediatrics, 17th
edition.)

Administer 20 mL/kg of lactated Ringer solution in 5% dextrose


in water

Obtain upper and lower limb blood pressure readings

Begin administration of epinephrine or atropine

Question . 36. To check the proper placement of an endotracheal tube, one should
do all of the following except:

Visualize the vocal cords

Monitor end-tidal CO2

Listen for equal breath sounds

Listen over the stomach

Obtain a lateral chest x-ray


Explanation: Most would obtain an anterior-posterior chest x-
ray. Nonetheless, in a patient with a perfusing rhythm, the most
accurate method is measuring end-tidal CO2. This is less
accurate in a patient in asystole. Looking for mist humidity in
the ETT is not 100% accurate and may be misleading
Question . 37. All of the following are anticipated reactions of tissues to the shock
state except:

Increased capillary filling time due to diminished perfusion of


tissues

Stage of "warm shock" reflecting initial vasodilation

Later stage of "cool shock" due to preservation of blood flow to


vital internal organs

Increased production of organic acids due to switch to


anaerobic metabolism

Increased production of bicarbonate due to anaerobic


metabolism
Explanation: Indeed, anaerobic metabolism produces a
severe lactic acidosis. Controversy exists about the use of
bicarbonate therapy to treat this lactic acidosis. Most agree
that, if possible, the underlying etiology (such as hypovolemia
or infection) must be treated first. Bicarbonate therapy may
increase morbidity and possibly mortality (in experimental
animals). Bicarbonate should not be used if the patient has
poor ventilation, because the CO2 generated from the
bicarbonate may exacerbate hypercarbia and produce
intracellular acidosis. (See Chapter
Question . 38. The condition of a patient in the PICU appears to be worsening, and
the supervising physician asks for the one best test to determine if shock is present.
You suggest:

A mixed venous O2 saturation measurement


Explanation: The mixed venous saturation and possibly a
serum lactate level measurement are excellent tests. The
mixed venous saturation reflects tissue oxygen extraction,
which reflects oxygen delivery and oxygen consumption. If
tissue oxygen consumption is greater than oxygen delivery, the
mixed venous saturation declines. (See Chapter 57.2 in Nelson
Textbook of Pediatrics, 17th edition.)

An arterial blood gas analysis

A venous blood gas

A hematocrit

A metabolic panel including assessment of liver and kidney


function

Question . 39. The factor most clearly predicting mortality in shock is:

Cardiac failure

Renal failure

Hepatic failure

Metabolic acidosis

Multiple organ system failure


Explanation: Multiple organ system failure, also known as
multiple organ dysfunction syndrome (MODS), is a serious
consequence of shock. In many patients, the injury many be
reversible; nonetheless, mortality increases substantially with
each dysfunctioning organ system. (See Chapter 57.2 in
Nelson Textbook of Pediatrics, 17th edition.)

Question . 40. The drug pair that meets the dual goals of stimulating the heart and
relaxing peripheral vasculature is:

Isoproterenol and epinephrine

Dopamine and dobutamine


Explanation: Dopamine acts on dopaminergic and -
adrenergic receptors of the heart, while dobutamine acts on -
receptors of the heart and the peripheral arteries. In high
doses, dopamine has -adrenergic effects. (See Chapter 57.2
in Nelson Textbook of Pediatrics, 17th edition.)

Dobutamine and amrinone

Epinephrine and norepinephrine

Amrinone and norepinephrine


Question . 41. A 5-yr-old boy presents with petechiae, fresh bruises, low-grade
fever, dizziness, and lethargy. You admit him to the hospital and start an IV infusion.
The most appropriate next step in management would be:

Administration of high-dose Solu-Medrol or Decadron

A CT scan of the head to rule out meningococcal meningitis

Collection of blood for a culture, CBC, and platelet count

Administration of 20 mL/kg of normal saline


Explanation: The dizziness and lethargy suggest poor central
nervous system perfusion and hypotension. Intravenous fluid
(crystalloid) resuscitation is urgently needed and should be
given as soon as possible. It may need to be repeated often. In
addition, intravenous antibiotics must be given rapidly, because
early therapy of meningococcal sepsis with antibiotics has
proven to reduce mortality. Steroids may be needed later if
adrenal insufficiency is demonstrated in a patient unresponsive
to fluid boluses and inotropic agents. (See Chapter 57.2 in
Nelson Textbook of Pediatrics, 17th edition.)

Administration of 1-2 mg/kg of furosemide (Lasix)

Question . 42. The normal alveolar-arterial (A-a) oxygen gradient is:

>300 mm Hg

<10 mm Hg
Explanation: The alveolar-arterial oxygen gradient in normal
children from about 1 month of age onward is usually <10 mm
Hg. Any deviation suggests a ventilation/perfusion defect,
intrapulmonary shunt, cardiac right-to-left shunt, or rarely an
alveolar diffusion defect. (See Chapter 57.3 in Nelson Textbook
of Pediatrics, 17th edition.)

50 mm Hg

>100 mm Hg
Question . 43. A patient in respiratory distress presents to your office. The most
appropriate immediate response is to:

Attempt a blood gas determination

Immediately intubate the airway and begin positive-pressure


ventilation

Call 911

Place the child in a comfortable position, reassess the


airway, and provide oxygen and other supportive
measures as necessary
Explanation: In almost all patients, oxygen will not harm a
patient in respiratory distress, and it may help. A mask may
frighten some infants, but nasal cannulas are usually well
tolerated. Oxygen should be used-albeit cautiously-in children
with chronic hypercarbia to avoid respiratory depression if the
oxygen chemoreceptors become inhibited. (See Chapter 57.3
in Nelson Textbook of Pediatrics, 17th edition.)

Question . 44. Respiratory failure accounts for PICU admissions in what percentage
of patients?

100%

80%

50%
Explanation: The percentage of children admitted with
respiratory failure varies. Nonetheless, acute airway
compromise and respiratory distress are the most common and
potentially predictable causes of "cardiac" arrest in children,
who actually have respiratory arrest. (See Chapter 57.3 in
Nelson Textbook of Pediatrics, 17th edition.)

<25%

Question . 45. Complications of mechanical ventilation include all of the following


except:

Air leak

Obstructed endotracheal tubes

Alterations of cardiac output

Reduction in nosocomial infections


Explanation: Indeed, with any indwelling device, endotracheal
tube placement increases the risk of infection. Nosocomial
acquisition of the highly resistant bacterial flora of the PICU,
poor mucociliary transport, atelectasis, suppressive broad-
spectrum antibiotics (which select for superinfection), and
mucosal barrier breakdown all increase in the risk of infection.
(See Chapter 57.4 in
Question . 46. The initial ventilator settings are determined by:

The patient's underlying disease


Explanation: The initial ventilator settings are determined by
the patient's condition. These include a patient with normal
lungs requiring ventilation for surgery or neurologic problems; a
patient with decreased compliance; or a patient with increased
airway resistance. Hypoxia and hypercarbia will require
different strategies, such as adjusting PEEP, FiO2, rate, or tidal
volume for CO2 elimination. (See Chapter 57.4 in Nelson
Textbook of Pediatrics, 17th edition.)

The patient's preferences

Standard order sets

Attempts to normalize the blood gases

Question . 47. Patients with severe forms of reactive airways disease (e.g., asthma)
who require mechanical ventilation may benefit from which of the following initial
ventilator parameters?

Rapid rates, short inspiratory times, and tidal volumes <6


mL/kg

Low rates, prolonged inspiratory/expiratory times, and low tidal


volumes (<6 mL/kg)

Low rates, prolonged inspiratory/expiratory times, and


moderate tidal volumes (8-10 mL/kg)
Explanation: Because of high airway resistance, particularly
during expiration, one must avoid air trapping, which will further
exacerbate the ongoing pulmonary pathology. (See Chapter
57.4 in Nelson Textbook of Pediatrics, 17th edition.)

Low rates, prolonged inspiratory/expiratory times, and high tidal


volumes (>15 mL/kg)

Question . 48. The most important maneuver in preserving renal function in septic
shock is:

Intravenous infusion of furosemide

Infusion of dopamine at a rate of 1-3 g/kg/min

Rapid restoration of the circulating volume


Explanation: Renal dysfunction in shock of any type may be
due to prerenal or renal etiologies. Prerenal renal dysfunction is
due to poor perfusion of the kidney secondary to hypotension.
If no intrinsic renal injury occurs, renal function will improve with
restoration of renal blood flow. If hypotension is severe and
prolonged, acute tubular or (if even more severe) cortical
necrosis may produce intrinsic renal failure. Acute tubular
necrosis is often reversible, but cortical necrosis results in
chronic renal insufficiency. (See Chapter 57.5 in Nelson
Textbook of Pediatrics, 17th edition.)

Proper antibiosis against the offending organism

Question . 49. Which of the following statements regarding continuous venovenous


hemofiltration (CVVH) is true?
Circuit patency is dependent on systemic blood pressure

Water and molecules less than 17,000 daltons in size are


removed from the bloodstream
Explanation: Removal of the molecules helps treat azotemia
but is of value in removing inflammatory cytokines in patients
with the systemic inflammatory response syndrome. (See
Chapter 57.5 in Nelson Textbook of Pediatrics, 17th edition.)

CVVH membranes are poorly biocompatible

Hemofiltration can induce hypoalbuminemia

Question . 50. Appropriate nutritional support for the child receiving mechanical
ventilation for bacterial pneumonia should include:

Amino acids, 2 g/kg/day, given intravenously

Full maintenance solution containing 35% dextrose

An age-appropriate enteral formula via nasogastric tube


Explanation: If gastrointestinal motility is normal, it is best to
provide nutrition by the enteral route, using the stomach, and
giving age-appropriate formulas. This improves nitrogen
balance but also reduces the risk of sepsis by lowering the
incidence of transmucosal migration (bacterial translocation) of
enteric bacteria. (See Chapter 57.6 in Nelson Textbook of
th
Pediatrics, 17 edition.)

An elemental formula via nasojejunal tube

Question . 51. In the severely ill child, a catabolic state ensues. Which of the
following statements is true?

Administration of growth hormone diminishes insulin resistance

Hyperglycemia is a beneficial state in the catabolic child

Branch-chain amino acids are beneficial to the previously


normal child

Intensive insulin therapy has reduced mortality in adult


ICU patients
Explanation: Insulin may reverse the catabolic state and
prevent hyperglycemia. This later metabolic problem is often a
risk factor for increased morbidity and mortality in an ICU. (See
Chapter 57
Question . 52. The differential diagnosis for the afebrile child with nausea and
vomiting should include:

Intracranial tumor
Explanation: Nausea and vomiting are common complaints in
a pediatric practice. Although most are due to non-specific
(presumed viral) infections or other identifiable infections
(gastroenteritis, otitis media, pharyngitis), the practitioner must
always be cautious of an intracranial cause. The nature of the
nausea, associated headache or vision problems, head tilt, or
cranial nerve abnormalities should suggest an intracranial
cause of vomiting. Always be cautious and, when possible,
check the fundi for papilledema. (See Chapter 57.7 in Nelson
Textbook of Pediatrics, 17th edition.)

Viral gastroenteritis

Salmonella infection

Type A influenza
Question . 53. In the child who has suffered a severe hypoxic ischemic injury, which
of the following is true?

Liver and kidney damage are irreversible

Isolated brainstem function might be preserved


Explanation: The brainstem may be preserved in the presence
of cortical cell death. This is a controversial point in the
discussion of brain death. Today we agree on the legal
definition of whole brain death, which includes the brainstem
and obviously precludes effective spontaneous ventilation.
Cortical brain death proponents want to recognize cortical
death alone as the criterion for legal death. The controversy
has not been resolved. (See Chapter 57.7 in Nelson Textbook
of Pediatrics, 17th edition.)

CT scans do not reveal abnormalities until after 1 mo following


injury

Intracranial pressure monitoring improves outcome

Question . 54. Fulminant hepatic failure will lead to encephalopathy, cerebral


edema, and brain death within several days. Which mode of support might best
prevent the progression from grade II to grade III+ hepatic encephalopathy?

Fluid restriction and furosemide infusion

Porcine hepatocyte column filtration

Continuous hemofiltration plus plasma exchange


Explanation: Various modalities have been proposed to treat
hepatic encephalopathy. In addition to controlling intracranial
pressure, preventing bleeding, and lowering ammonia levels,
many have proposed emergency liver transplant. (See Chapter
57.7 in Nelson Textbook of Pediatrics, 17th edition.)

Hemodialysis
Question . 55. A 15-yr-old boy is struck by a car while walking. On arrival in the ED,
he is alert and has no signs of upper airway obstruction. Pulse is 140/min,
respiratory rate 40 breaths/min, and blood pressure 70/50 mm Hg. Heart sounds are
distinct, but breath sounds are decreased in the left hemithorax. What is the most
appropriate next step in patient management?
Needle thoracentesis of the left hemithorax
Explanation: Although an x-ray may be helpful for diagnosis
and fluids may improve venous return, prompt evacuation of a
symptomatic hemothorax or pneumothorax is the most
appropriate step. (See Chapter 57.8 in Nelson Textbook of
Pediatrics, 17th edition.)

Upright radiograph of the chest

Pericardiocentesis

Rapid intravenous bolus of Ringer's lactate

Arterial blood gas analysis

Question . 56. A 4-yr-old girl sustains a head injury after pulling a television set onto
her. The paramedics intubate her at the scene because of inadequate respiratory
effort. On arrival in the ED, she is being ventilated at a rate of 20 breaths/min, and
her chest wall rises adequately. Pulse is 100/min, blood pressure is 100/70 mm Hg,
and peripheral perfusion is good. What is the most appropriate next step in patient
management?

Immediate head CT

Hyperventilation to attain a PCO2 of 25 mm Hg

Gentle hyperventilation and an intravenous bolus of mannitol

Evaluation of level of alertness and pupil size and


reactivity
Explanation: At this point, the patient has no signs of
increased intracranial pressure and thus doesn't need mannitol
or hyperventilation. Further clinical assessment in a stable
patient before a head CT is quite appropriate. (See Chapter
57.8 in Nelson Textbook of Pediatrics, 17th edition.)

Intravenous bolus of 3% saline

Question . 57. A 3-yr-old boy is brought to the ED after falling two stories from an
open window. He arrives appropriately immobilized and is alert. Pulse is 190/min;
respiratory rate, 28 breaths/min; and blood pressure, 70/30 mm Hg. It is clear that he
has a femur fracture. His abdomen is diffusely tender. After a 20 mL/kg bolus of
Ringer's lactate, his pulse is 180/min and blood pressure is 72/35 mm Hg. The most
appropriate next step in patient management would be:

Insertion of a central venous catheter

Emergent abdominal laparotomy

20 mL/kg bolus of Ringer's lactate


Explanation: Repeated boluses of Ringer lactate or normal
saline solution have been life-saving if administered early and
aggressively in pediatric patients with shock. With time and
signs of blood loss, red cell transfusions become indicated.
(See Chapter 57.8 in Nelson Textbook of Pediatrics, 17th
edition.)

10 mL/kg infusion of cross-matched, packed red blood cells

10 mL/kg infusion of O-negative, packed red blood cells

Question . 58. A 10-yr-old boy is brought to the ED after being kicked in the
abdomen by a horse. Pulse is 80/min; respiratory rate, 18 breaths/min; and blood
pressure, 110/70 mm Hg. His abdomen is diffusely tender. The most appropriate
diagnostic study for this patient would be:

Abdominal ultrasound study

MRI of the abdomen

Diagnostic peritoneal lavage

Abdominal CT study with intravenous contrast


Explanation: Abdominal CT is now the standard for blunt
pediatric abdominal trauma. It is most useful for solid organs
(liver, spleen, kidneys) but will also demonstrate pancreatic
injury, hemorrhage (free fluid), and intestinal injury. When
available, it has replaced peritoneal lavage. (See Chapter 57.8
in Nelson Textbook of Pediatrics, 17th edition.)

Plain abdominal radiographs

Question . 59. In the support of the child with severe ARDS, appropriate goals for
pH, oxygenation, and PCO2 include:

pH 7.40; PaO2 90 mm Hg; PCO2 40 mm Hg

pH 7.50; PaO2 125 mm Hg; PCO2 25 mm Hg

pH 7.35; PaO2 100 mm Hg; PCO2 45 mm Hg

pH 7.25; PaO2 60 mm Hg; PCO2 60 mm Hg


Explanation: This reflects the lung protection strategy with
permissive hypercarbia. There is no need for hyperoxia, and
PaO2 >90 mm Hg is a sign to reduce the FiO2 and thus
potentially decrease the risk of oxygen toxicity. (See Chapter
58 in Nelson Textbook of Pediatrics, 17th edition
Question . 60. For the child with ARDS receiving mechanical ventilation the prone
position is alternated with the supine position. This measure is used to:

Redistribute total body edema

Equalize pulmonary blood flow to dependent and independent


lung segments

Prevent atelectasis and increased consolidation in


dependent lung segments
Explanation: Body position changes help to improve
respiratory function in the previously dependent lung segments.
Although PaO2 may improve with position changes, no study
has demonstrated a survival advantage to this procedure. (See
Chapter 58 in Nelson Textbook of Pediatrics, 17th edition.)

Decrease the risk of pressure sores

Question . 61. In the management of a child with ARDS and respiratory failure, all of
the following measures are important except:

Normalize blood gases


Explanation: To avoid oxygen toxicity, excessive PEEP and
PIP, and volutrauma or barotrauma, blood gases should not be
normalized. This principle of respiratory management has been
the most significant improvement in the care of ARDS in the
last 10 years. (See Chapter 58 in Nelson Textbook of
th
Pediatrics, 17 edition.)

Use permissive hypercapnia

Accept PaO2 measurements of 60-80 mm Hg

Reduce airway pressures to prevent barotrauma and


volutrauma

Question . 62. Transplantation of part of an organ from a donor who will survive the
donation process is possible with transplantation of:

Heart or lung

Heart or liver

Cornea or heart

Kidney or liver
Explanation: Kidney transplantation is quite successful from a
related living donor. Split or partial liver transplantation from a
living related donor is also successful. Both procedures, but
particularly liver transplantation, carry risks for the donor,
including death. (See Chapter 60 in Nelson Textbook of
Pediatrics, 17th edition.)

Heart or kidney

Question . 63. Complications of transplantation particularly likely to be seen with


bone marrow transplantationinclude:

Graft-versus-host disease (GVHD) and veno-occlusive


disease (VOD)
Explanation: GVHD and VOD may occur after bone marrow
transplantation. Early in the posttransplantation period,
complications due to pancytopenia occur. Anemia,
thrombocytopenia, and neutropenia and their complications
remain the predominant risks until engraftment occurs. (See
Chapter 60 in Nelson Textbook of Pediatrics, 17th edition.)
GVHD and hyperthermia

GVHD and thrombocytosis

VOD and hypothermia

VOD and thrombocytosis

Question . 64. One of the more common pathogens responsible for pulmonary
infections in immunocompromised patients following transplantation is:

Staphylococcus epidermidis

Haemophilus influenzae

Clostridium botulinum

Streptococcus pneumoniae

Pneumocystis carinii
Explanation: Pneumocystis carinii pneumonia (PCP) is seen in
any immunosuppressed patient, particularly when T
lymphocyte function or number is reduced. Prophylaxis against
PCP has greatly reduced this potentially lethal complication.
(See Chapter 60 in Nelson Textbook of Pediatrics, 17th edition.)

Question . 65. Factors promoting successful acceptance of a transplanted kidney by


the recipient include:

Maintenance of high urine output and maintenance of lower-


than-normal blood pressure

Maintenance of high urine output and use of angiotensin-


converting enzyme (ACE) inhibitors

Maintenance of high urine output and maintenance of


higher-than-normal blood pressure
Explanation: It is essential to maintain renal perfusion and
renal blood flow. Forced diuresis is critical to achievement of
this goal. In addition, it may reduce the risk of anastomotic
vascular thrombosis. (See Chapter 60 in Nelson Textbook of
Pediatrics, 17th edition.)

Maintenance of low urine output and maintenance of lower-


than-normal blood pressure

Maintenance of low urine output and maintenance of higher-


than-normal blood pressure

Question . 66. A 2-yr-old boy is pulseless and apneic after extrication from a
swimming pool. Paramedics start CPR, intubate him, and administer 100% oxygen
while providing bag-endotracheal tube breaths. An intraosseous line is placed, and
two doses of epinephrine are administered. A pulse is found after 10 min of
resuscitative measures. The child is transferred to your ED. On arrival, he is deeply
comatose with a temperature of 34°C. He is not breathing spontaneously, his heart
rate is 120 beats/min, blood pressure is 60/25 mm Hg, and he has cold extremities.
Oxygen saturation is 85%. Finger stick glucose is 200 mg/dL. Which of the following
interventions is most likely to improve the patient's neurologic outcome?

Administration of high-dose dextrose to improve cerebral


metabolism

Fluid restriction to minimize potential cerebral edema

The addition of positive end-expiratory pressure,


administration of an isotonic fluid bolus, and an
epinephrine infusion to improve oxygenation and
circulation
Explanation: Despite many hopeful new interventions for
hypoxic-ischemic neurologic injury, none has been proven
beneficial. The best approach is to support normal oxygenation
and normal blood pressure to avoid continued cerebral hypoxia
and ischemia, respectively. Monitor blood pressure to improve
cerebral perfusion pressure and possibly to avoid secondary
ischemic injury from raised intracranial pressure. (See Chapter
th
61 in Nelson Textbook of Pediatrics, 17 edition.)

Keeping the patient hypothermic for the first 12-24 hr of his


PICU hospitalization

Hyperventilation to a PaCO2 of 25 mm Hg

Question . 67. Mr. and Mrs. Smith install a new swimming pool in their backyard.
Three months later, their 2-yr-old son is found pulseless and asystolic in the pool.
Which of the following safety measures would have been most likely to prevent this
tragedy?

A lightweight plastic swimming pool cover

A pool alarm that sounds when water movement is detected

A 5-ft-tall chain link (2.5-inch mesh) isolation fence

D A 5-ft-tall ornamental iron isolation fence (vertical bars 3


inches apart; horizontal crossbars 45 inches apart)
Explanation: Fencing is the best preventive measure. The
other interventions are dangerous (A), not proven, or provide a
false sense of security (E). Children under the age of 3-4 yr
cannot "swim" safely. (See Chapter 61 in Nelson Textbook of
Pediatrics, 17th edition.)

Swimming lessons

Question . 68. A 4-yr-old boy is admitted to the PICU after resuscitation from a
swimming pool submersion. He was pulseless and apneic at the scene. On
presentation to the PICU, he is intubated and mechanically ventilated. His vital signs
are normal and he is comatose, with a Glasgow Coma Score of 4. His initial arterial
blood gas is pH 7.12, PaCO2 35 mm Hg, and PaO 2 145 mm Hg. In counseling the
family regarding the child's prognosis, which of the following results is most likely to
represent a favorable outcome?
A normal head CT scan

Normal intracranial pressure after placement of a Camino


monitor

A serum glucose of 180 mg/dL on admission to the PICU

Resolution of metabolic acidosis on arterial blood gas


measurement

Spontaneous purposeful movement 12 hr after admission


Explanation: Although his initial clinical appearance sounds
ominous, the presence of clinical improvement during 6-12 or
12-24 hr is the best predictor of intact outcome. Unfortunately,
no laboratory or radiologic test has had any valuable predictive
power. Serial examination is the best method to follow the
course and predict the outcome. Indeed, abnormalities (such
as those seen on CT scans) are usually self-evident from
severity of the abnormalities on clinical examination. (See Cha
Question . 69. A 16-yr-old boy with 50% body surface area burns from a house fire
had been on controlled positive-pressure ventilation. His condition continues to
deteriorate, necessitating high FiO2 and PEEP of at least 12.5 cm H2O. Of the
following, the most therapeutic approach is:

Continued positive-pressure ventilation

ECMO

Nitric oxide therapy


Explanation: Inhaled nitric oxide (NO) has shown promise in
managing respiratory (hypoxic) failure after burns. Therapy is
usually started at 5 ppm and titrated to 30 ppm. Hyperbaric
oxygenation is of value in severe carbon monoxide poisoning,
while ECMO is of value if the patient doesn't respond to inhaled
NO. High-frequency ventilation should also be used before
ECMO. (See Chapter 62 in Nelson Textbook of Pediatrics, 17th
edition.)

Hyperbaric oxygen

Question . 70. A 5-yr-old girl who accidentally spilled hot water on her face and
trunk and is brought to the emergency room. Which of the following is the best
method for estimating body surface area burn?

Rule of 9s

Rule of palm

The Lund and Browder chart


Explanation: Body surface area (BSA) of various anatomic
sites changes with growth and development. The head has a
greater body surface area in the youngest children. (See
Chapter 62 in Nelson Textbook of Pediatrics, 17th edition.)

A growth chart
Question . 71. A 10-yr-old boy spilled gasoline on his legs. His pants became
ignited and he suffered 20% body surface area burns. Of the following, the most
important treatment is:

7-day course of penicillin

5-day course of penicillin

10-day course of penicillin

None of the above


Explanation: Initial treatment is to remove the clothing and
place warm saline dressings over the wound. Penicillin is not
an immediate therapy, and there is controversy about whether
it should be used at all. (See Chapter 62 in Nelson Textbook of
th
Pediatrics, 17 edition.)

Question . 72. A 4-yr-old girl sustained a 40% second- and third-degree total body
surface area burn from scalding hot water. Of the following, the most therapeutic
approach is:

Aggressive use of topical antibacterial agents with frequent


dressing changes

Use of intravenous appropriate antibiotics

Excision of the burn wounds and grafting


Explanation: To prevent infection and to facilitate healing,
excision of the wound and removal of all devitalized and dead
tissue must be performed rapidly. Grafting follows. (See
Chapter 62 in Nelson Textbook of Pediatrics, 17th edition.)

Use of topical analgesics

Question . 73. A 10-yr-old boy sustained 30% body surface area burns and had
been requiring dressing changes for physical therapy. Which of the following
regimens will provide the best pain management? A. B. C.

Morphine bolus

Morphine continuous infusion

Morphine and Versed bolus

Oral morphine and Ativan


Explanation: Preemptive narcotics before the procedure and
an anxiolytic are the best ways to avoid future behavioral
problems and to provide appropriate pain relief. Boluses may
be added to preemptive therapy
Question . 74. All of the following statements about brain death are true except:

Brain death alone can be used as a justification for withdrawing


all life support
Declaration of brain death in children younger than 2 mo of age
requires two assessments, separated by at least 48 hr

Declaration of brain death in a child 5 yr of age can be made on


clinical criteria alone

Brain death can be declared even if certain brainstem


reflexes (e.g., papillary response) are still present
Explanation: Currently, the legal and medically acceptable
definition of brain death is whole brain death, which includes
the brainstem and cortex. Some suggest that neocortical brain
death should be used, as noted in patients in a persistent
vegetative state (PVS). This remains objectionable because of
the rare patients who wake up from PVS. (See Chapter 64 in
Nelson Textbook of Pediatrics, 17th edition.)

Acceptance of brain death is sufficient to declare full legal


death and should facilitate more organ donation opportunities
Question . 75. Ancillary tests to confirm brain death include all of the following
except:

Electroencephalogram

Head ultrasound study


Explanation: Head ultrasonography demonstrates structural
problems, such as presence of blood and edema, but does not
help define function, such as electrical activity or blood flow.
With whole brain death, four-vessel angiography remains the
gold standard. With further experience, Doppler sonography
may be of value. (See Chapter 64 in Nelson Textbook of
Pediatrics, 17th edition.)

Radionuclide angiogram

Apnea test

Cerebral angiogram
Question . 76. Correct performance of an apnea test requires:

Pre-oxygenation with 100% O2 for several minutes

B Significant elevation of the CO2 level in the blood without


resultant respiratory efforts

Maintenance of acceptable O2 arterial saturation throughout the


test (usually 2-4 min)

DAll of the above


Explanation: The apnea test helps define absent brain stem
reflexes related to respirations. It may take some time, usually
>10 min, for the PCO2 to rise to the acceptable level of 60 mm
Hg. (See Chapter 64 in Nelson Textbook of Pediatrics, 17th
edition.)

None of the above


Question . 77. Fentanyl anesthesia in neonates does all of the following except:

Avoid postoperative hypotension after PDA ligation

Provide analgesia

Cause hyperglycemia
Explanation: Fentanyl is an effective and safe analgesic
anesthetic in newborns and has improved the outcome of
complex surgeries in the neonatal period. Use before, during,
and after the procedure has reduced morbidity and probably
mortality. (See Chapter 65 in Nelson Textbook of Pediatrics,
17th edition.)

Avoid hypoglycemia

Prevent acidosis

Question . 78. Deep sedation is associated with all of the following except:

Loss of airway protective reflexes

Loss of airway patency

Loss of cardiovascular stability

Apnea

Seizures
Explanation: Most agents producing deep sedation will
actually suppress physical and electrical seizure activity. Deep
sedation is a risk factor for a cardiopulmonary arrest and
should only be performed in a setting equipped for response to
apnea and bradycardia. (See Chapter 65 in
Question . 79. Akinesia is:

Analgesia

Sedation

Absence of movement
Explanation: Indeed, akinesia induced by neuromuscular
blocking agents (muscle relaxants) has no effect on reducing
pain or producing amnesia. It simply means no movement.
Sedation and analgesia must also be used when a patient
receives neuromuscular blocking agents. (See Chapter 65 in
Nelson Textbook of Pediatrics, 17th edition.)

Amnesia

Nystagmus
Question . 80. Risks associated with the use of succinylcholine include all of the
following except:
Seizures
Explanation: With much better and safer neuromuscular
blocking muscle relaxants, succinylcholine is rarely used in the
operating room. (See Chapter 65 in Nelson Textbook of
Pediatrics, 17th edition.)

Hyperkalemia

Malignant hyperpyrexia

Myoglobinuria

Elevated intracranial pressure

Question . 81. Malignant hyperthermia is associated with all of the following except:

Hypercarbia

Masseter spasm

Metabolic acidosis

Muscular dystrophy

Morphine
Explanation: Narcotic agents are not associated with the
development of this potentially serious and lethal familial
condition. The spectrum may range from masseter spasms or a
mild increase in intraoperative temperature and CO2 elimination
to severe muscle rigidity, hyperpyrexia, metabolic acidosis, and
cardiovascular collapse. (See Chapter 65 in Nelson Textbook
of Pediatrics, 17th edition.)

A positive family history for the disorder

Question . 82. A 10-yr-old girl with metastatic primitive neuroectodermal tumor


(PNET) of the bone is a hospice patient. She is on oral morphine and is experiencing
increased frequency of uncontrollable pain. Her mother is hesitant to change to
intravenous morphine because of fear that her daughter will become addicted to the
medication. The most appropriate response to her stated fear would be:

Remind her that her daughter is terminally ill, so it doesn't


matter if she becomes addicted

Tell her that the oral morphine will be maintained for now, but
make no promises about the future

Educate her about addiction, tolerance and dependence;


describe the potential risks and benefits of intravenous
morphine; and help her make a decision based on the
priorities of her child and the family
Explanation: Many people (lay and medical) believe in myths
and don't understand opiate drugs. There is a great difference
between addiction and tolerance to the effects of an opiate.
(See Chapter 66 in Nelson Textbook of Pediatrics, 17th edition.)

Ask her to trust your judgment that this is the best route to take

Question . 83. Other approaches to pain management that might be considered for
the patient described in Question 82 include:

Adjuvant analgesics

Complementary interventions, such as massage or


hypnotherapy

Spiritual care

All of the above


Explanation: Adding other non-opiate analgesic medications
and using other non-drug related therapies are especially
useful in children with poorly controlled chronic pain.
Combination NSAID and opiates may improve control and
reduce opiate dose, while behavioral approaches help with
anticipation, coping, and articulation of pain. (See Chapter 66 in
Nelson Textbook of Pediatrics, 17th edition.)

Question . 84. When the patient described in Question 82 learns that the pain
medication might be changed, she tells the hospice nurse that she is no longer
hurting. What is the likely reason she might have misled the nurse?

Because she knows her mother is against the idea.

Because she is afraid of needles

Because she doesn't want to bother the nurse

All of the above


Explanation: The statement by a child who should be in pain
that he or she no longer hurts often indicates a fear of
something else, such as changing from an oral medication to a
parenteral one (fear of needles) or loss of approval by a parent.
Always be cautious about what children with chronic pain say,
because what they say is often mediated by many important
related factors
Question . 85. A bright 14-yr-old boy has had persistent right leg pain for 6 mo after
a minor soccer injury has healed. He describes the pain as sharp, shooting, and
burning. Results of all imaging studies are normal. He is able to walk on the leg and
is attending school. The type of pain the patient is experiencing is most likely to be:

Neuropathic
Explanation: Neuropathic pain may be difficult to diagnose.
This patient has classic symptoms of what used to be called
reflex sympathetic dystrophy but is now called complex
regional pain syndrome, type I. (See Chapter 66 in Nelson
Textbook of Pediatrics, 17th edition.)

Somatic
Visceral

Peripheral

Question . 86. Which type of medication should first be considered to treat the pain
in the child described in Question 85?

Opioids

Tricyclic antidepressants and/or anticonvulsants


Explanation: Neuropathic pain is quite resistant to opiates.
Low-dose tricyclic antidepressants, with or without various
anticonvulsant medications, are helpful. (See Chapter 66 in
Nelson Textbook of Pediatrics, 17th edition.)

NSAIDs

Corticosteroids
Question . 87. Other types of treatment that may be considered for the child
described in Question 85 include:

Physical therapy
Explanation: Physical therapy is very useful in rehabilitation. It
also teaches the patient to use the extremity without an
increase in disability, a fear many patients have with chronic
neuropathic extremity pain. (See Chapter 66 in Nelson
Textbook of Pediatrics, 17th edition.)

Crutches to eliminate weight bearing

Home schooling to reduce stress

All of the above

Question . 88. A 3-yr-old girl is scheduled to have a bone marrow aspiration for the
first time. Which of the following combination of interventions is most likely to be
helpful to her?

Education abut the procedure and hypnotherapy

Oral sucrose and a pacifier

Distraction and a local anesthetic


Explanation: Local anesthesia plus distraction is a helpful
combination for bone marrow aspiration. Some would also
premedicate with a benzodiazepine. Unless the parents will
create a stressful situation, they should be present to help their
child and even create a helpful distraction. (See Chapter 66 in
Nelson Textbook of Pediatrics, 17th edition.)

Separation from her parents and a benzodiazepine

Question . 89. Indications for admission to the hospital after a burn injury may
include all of the following except:

Suspected child abuse

Electric burns through an extremity

Perineal burns

Poor follow-up

No tetanus immunization
Explanation: Lack of immunization against tetanus may be
managed with tetanus toxoid and (if a wound is large or dirty)
with tetanus immune globulin. The other choices indicate that
the patient is at high risk and requires hospitalization. (See
Chapter 62 in Nelson Textbook of Pediatrics, 17th edition.)

Inhalation injury
Question . 90. A 3-mo-old, formerly a 29-wk premature infant, has been scheduled
for repair of bilateral inguinal hernias. The infant had received mechanical ventilation
for the first 6 days of life and had apnea of prematurity that resolved 5 wk previously.
The infant is feeding well and gaining weight and has no requirement for
supplemental oxygen. The hematocrit is 28%. The HMO clerk approves the surgery
on an outpatient basis. All of the following statements are true except:

Risk of postoperative apnea is increased by anemia

Accepted standard of care includes overnight inpatient apnea


monitoring for this infant after general anesthesia because of a
significant risk of postoperative apnea

Postponing the surgery incurs a small but real risk of


incarceration, with complications that may include bowel
obstruction and infarction of testes or ovaries

After spinal anesthetic, monitoring for apnea is not


required, and the infant can be sent home from the
postanesthetic care unit on the day of surgery
Explanation: Postoperative severe apnea can occur in former
premature infants up to roughly 50-55 wk postconception, even
after apnea of prematurity has resolved or has never been
present, despite premature birth. The risk of postoperative
apnea decreases with age. Anemia is an independent risk
factor for apnea, but preoperative transfusion is not
recommended for this hematocrit finding. Former premature
infants up to perhaps 50-60 wk postconceptual age should be
monitored for at least 12-18 hr after anesthesia. If apnea does
not occur in the first 12 hr, it is unlikely to occur thereafter.
Retrospective analysis of anesthetic risk supports delaying
elective surgery in the first 1-2 mo of life. These risks must be
balanced against findings of other studies that suggest that
delay in repairing inguinal hernia may result in incarceration
that cannot be reduced and in additional complications
associated with more emergent surgery. Although apnea is
reported to be much less common after spinal anesthesia for
inguinal surgery in formerly premature infants, current practice
is still to monitor in hospital for apnea for at least 12 hr after a
spinal anesthesia. (See Chapter 65 in Nelson Textbook of
Pediatrics, 17th edition.)

Question . 91. A 12-yr-old, 45-kg, previously healthy child is now on the pediatric
ward 2 hr after repair of a forearm fracture under general anesthesia. You are called
because the child has a fever of 40.6°C, a respiratory rate of 60 breaths/ min, a
heart rate of 140 beats/min, and skin color described as "a bit off." Over the phone,
the nurse reviews the anesthetic record and reports that general anesthesia was
administered after a "rapid-sequence induction" because the child had eaten a large
meal just before the injury. The nurse notes from the record that anesthetic induction
was performed with thiopental sodium and succinylcholine. Anesthesia was
maintained with halothane in a mixture of nitrous oxide and oxygen, and the child
received morphine, 3 mg IV, before awakening. The anesthesia record notes stable
vital signs during surgery, which lasted 30 minutes. All of the following are true
except:

Prior to your examining the patient, based on this history, your


two primary considerations in the differential diagnosis are
aspiration pneumonitis and malignant hyperthermia

Because a rapid-sequence induction was successful and the


intraoperative vital signs were stable, aspiration pneumonitis
was prevented, and its likelihood among the differential
diagnostic posibilities is very low

Atelectasis is a more common cause of postoperative


fever than either malignant hyperthermia or aspiration
pneumonitis; this child's severity of fever, tachypnea, and
"off" color are typical for the diagnosis of atelectasis
Explanation: Atelectasis is the most common cause of fever
immediately after surgery, but the severity of the fever, the
degree of tachypnea, and the "color isn't too good" description
(whether reflecting cyanosis or impaired circulation) are
atypical for ordinary postoperative atelectasis. Malignant
hyperthermia is an inherited muscle disorder that produces
acute hypermetabolism, increased CO2 production,
rhabdomyolysis, and fever. Clinically significant aspiration
pneumonitis is comparatively uncommon in children after
anesthesia and surgery but should be suspected if tachypnea,
hypoxemia, and fever are present postoperatively. Although a
rapid-sequence induction may reduce the likelihood of
aspiration, it does not completely prevent aspiration of gastric
contents. Aspiration may occur, as often on emergence as on
induction. Auscultation of the chest would most probably show
rales, rhonchi, or wheezes, and chest radiograph would
(eventually) show infiltrates. (See Chapter 65 in Nelson
Textbook of Pediatrics, 17th edition.)
Untreated pain increases respiratory rates, but rarely to this degree
Question . 92. In response to social overtures (being held, hugged, kissed, talked
to), a febrile infant does not smile, has a dull, expressionless face, and is not alert to
stimuli. The most appropriate approach to management is to:

Administer ceftriaxone IM after a blood culture and have the


parent and child return to the office in the morning

If the child is older than 6 mo, obtain a blood culture and have
the parents return to the office if the patient remains febrile
Administer acetaminophen and reassess after the infant is no
longer febrile

Administer ceftriaxone after obtaining a blood, urine, and


CSF specimens for culture and admit the child to the
hospital
Explanation: Clinical observation of young patients is critical in
helping you to evaluate and distinguish the degree of risk of
infection and physiologic impairment. In addition to observing
color, tone, grunting, or a bulging fontanel, the response to
social stimuli is valuable. This 3-mo-old had pneumococcal
meningitis. (See Chapter 49 in Nelson Textbook of Pediatrics,
17th edition.)

Administer a normal saline bolus of 20 mL/kg and reevaluate in


1 hr

Question . 93. Paradoxical irritability may be present with all of the following except:

Osteomyelitis

Appendicitis

Extremity cellulitis

Meningitis

Pneumonia
Explanation: Paradoxic irritability is present when a child
becomes anxious and cries during attempts to cuddle and hold
the patient. Movement of a painful extremity, abdomen, or neck
may elicit this response
Question . 94. A previously healthy 7-mo-old white baby boy presents one summer
day with a temperature of 41.1°C, a pulse of 190/min, a respiratory rate of 70
breaths/min, and a blood pressure of 65/20 mm Hg. He has a 1-day history of
diarrhea (five stools in 24 hr) and is now unresponsive to verbal commands or
painful stimuli. The most appropriate initial therapy is:

Application of cooling blankets

Administration of aspirin (100 mg/kg)

Administration of ceftriaxone (150 mg/kg)

Administration of dantrolene (10 mg/kg)

Administration of normal saline (20-40 mL/kg)


Explanation: Despite an uncertain etiology, the physiologic
condition is that of shock. The circulation needs to be re-
established to perfuse vital organs.

Question . 95. After receiving normal saline pushes, the patient in Question 97
remains unconscious. A lumbar puncture reveals 3 WBCs/cu mm, 10 RBCs/cu mm,
a protein level of 30 mg/dL, and a glucose level of 75 mg/dL. After the lumbar
puncture, he is noted to be bleeding at venipuncture sites. The most likely diagnosis
is:

Herpes simplex encephalitis

Meningococcemia

Salicylate poisoning

Hemorrhagic shock encephalopathy syndrome


Explanation: Hemorrhagic shock encephalopathy syndrome
may look like heat stroke, but it is a distinct disorder
characterized by encephalopathy, shock, fever, disseminated
intravascular coagulopathy, and other organ failure (heart,
liver). It has a high mortality rate and morbidity. (See Chapter
57 in Nelson Textbook of Pediatrics, 17th edition.)

Malignant hyperthermia

Question . 96. A burn wound characterized by the absence of painful sensation,


bleeding, or capillary refilling is best classified as:

First degree

Moderate to severe

Second degree

Midlevel

Full thickness
Explanation: This defines a full-thickness burn (also known as
a third-degree burn). (See Chapter

Question . 97. A 12-yr-old boy with spina bifida experiences respiratory distress
during induction of anesthesia for an orthopedic procedure. He has been otherwise
well prior to this hospital admission. Past medical history reveals surgery for closure
of the spina bifida at age 3 days, placement of a ventricular peritoneal shunt at 1 mo
of life, and release of contractures at 6 yr of life. He is on ampicillin prophylaxis for
recurrent urinary tract infections since birth and has to be catheterized for urination.
The most likely diagnosis is:

Ampicillin hypersensitivity

Urosepsis

Reactive airway disease

Status epilepticus

Latex anaphylaxis
Explanation: Latex allergy is common in children with multiple
surgical procedures and those who have required
catheterization for urinary retention. The presentation includes
urticaria, wheezing, and hypotension. Bananas may cross-
react with latex. (See Chapter 65 in Nelson Textbook
Question . 1. Autosomal dominance inheritance is characterized by all of the
following except:

It affects individuals in every generation

It has a high spontaneous mutation rate

It affects males more commonly than females


Explanation: All others are very typical of autosomal dominant
inheritance. The sex ratio should be equal. Often the parents
are not affected because the disease is a spontaneous
mutation in the child. Neurofibromatosis is an example of an
autosomal dominant disease. (See Chapter 69 in Nelson
Textbook of Pediatrics, 17th edition.)

Phenotypically normal parents do not transmit the disease to


the child

There is a 50% chance of transmission

Question . 2. Autosomal recessive disorders are characterized by all of the


following except:

Equal sex ratio

Consanguinity

Greater incidence than that of autosomal dominant


disorders
Explanation: Autosomal dominant disorders are more
common because only one gene is needed for the disease to
be manifested. In autosomal recessive disorders, two genes
are needed. (See Chapter 69 in Nelson Textbook of Pediatrics,
17th edition.)

Asymptomatic carrier state in parents

Recurrence risk in sibling of 25%

Question . 3. The risk of an autosomal recessive disease in the offspring of a


consanguineous mating between first cousins is:

2-4%

6-8%
Explanation: This risk is twice the risk of an autosomal
recessive trait being manifest in non-consanguineous mating.
(See Chapter 69 in Nelson Textbook of Pediatrics, 17th edition.)

10-12%

50%

Zero
Question . 4. A disease affecting all of the daughters but none of the sons of an
affected father, and 50% of the sons or daughters of an affected mother, is most
likely:

X-linked recessive

X-linked dominant
Explanation: This is the classic pattern of an X-linked
dominant condition. These are rare conditions, which include
hypophosphatemic rickets and incontinentia pigmenti (IP). IP is
lethal to the male fetus and thus no males are born alive with
this disorder. (See Chapter 69 in Nelson Textbook of
Pediatrics, 17th ed.

Mitochondrial deletion syndrome

Autosomal dominant

Autosomal recessive

Question . 5. A disease passed on only through the mother and affecting both sons
and daughters, with variable manifestations in the affected siblings, is most likely:

Mitochondrial inheritance
Explanation: Because mitochondria in the fetus are usually
derived only from the ovum, mitochondrial inheritance typically
passes disorders through the mother but to both her sons and
daughters. (See Chapter 69 in Nelson Textbook of Pediatrics,
17th edition.)

Multifactorial inheritance

X-linked recessive inheritance

X-linked dominant inheritance

Environmentally induced

Question . 6. What does a patient with 45 XX (t13q2.1-14q1.3) have?

Turner syndrome

Female with Robertsonian translocation of chromosomes 13


and 14

Female with reciprocal translocation of chromosomes 13


and 14
Explanation: There are only 45 chromosomes and two sex
chromosomes. Therefore, there is one autosomal chromosome
missing. The nomenclature states it is a translocation with
chromosomal fusion but no net loss of chromosome DNA.
Carriers of translocations usually have a normal phenotype.
(See Chapter 70 in Nelson Textbook of Pediatrics, 17th edition.)
Male with reciprocal translocation of chromosomes 13 and 14

Female with pericentric inversion involving 13q and 14q

Question . 7. Uniparental disomy:

Covers/uncovers imprinting

Can lead to autosomal recessive disorders

Is associated with advanced maternal age

Is associated with mosaicism for trisomy

All of the above


Explanation: This is a fascinating area of genetics whereby
two chromosomes come to the embryo from the same parent.
If three chromosomes are present in the embryo (a trisomy),
one may drop out, producing the uniparental disomy. (See
Chapter 70 in Nelson Textbook of
Question . 8. A baby is born with classic features of Down syndrome. Indications for
chromosome studies include:

Maternal age older than 35 yr

Family history of Down syndrome

Prospective adoption of the child

Maternal age younger than 35 yr

All of the above?such studies are indicated in all children


so affected
Explanation: Chromosome studies should always be
performed in a child with features suggestive of a trisomy. In
the case of trisomy 21, if the neonate has a translocation,
chromosome studies should also be done on both parents to
determine if either is a translocation carrier. If one is a carrier,
the risk of an affected sibling is about 30%. (See Chapter
Question . 9. Prior to genetic counseling, all of the following should be available or
performed except:

Physical examination

Family history

Screening laboratory studies


Explanation: Laboratory tests are usually chosen on the basis
of the history and physical examination. Counseling will help
reveal the need for laboratory studies. (See Chapter 72 in
Nelson Textbook of Pediatrics, 17th edition.)

Pregnancy and delivery history


Review of medical records

Question . 10. A mentally retarded 15-yr-old boy is found to have macro-orchidism


and large, prominent ears. The most likely diagnosis is:

Cerebral giantism

Acromegaly

Hypothyroidism

Trisomy 21

Fragile X syndrome
Explanation: Fragile X syndrome is a common chromosomal
cause of mental retardation in boys. Affected boys have allelic
expansion of trinucleotide repeats to more than 200 (normal is
6-54 repeats). (See Chapter 70 in Nelson Textbook of
Pediatrics, 17th edition.)

Question . 11. Patients with Turner syndrome should undergo careful analysis of
their chromosomes for Y chromosome material because they may:

Become masculinized

Grow tall

Become pregnant

Experience gonadoblastoma
Explanation: Y chromosome material is present in 5-10% of
girls with Turner syndrome. Gonadoblastoma may develop in
the ovary, thus necessitating bilateral oophorectomy as a
preventive measure. (See Chapter 70 in Nelson Textbook of
Pediatrics, 17th edition.)

None of the above

Question . 12. Kearns-Sayre syndrome and Leber hereditary optic neuropathy are
noted in both males and females but are inherited only through the mother. These
conditions are examples of:

Uniparental disomy

Mitochondrial inheritance
Explanation: Mitochondrial inheritance of the diseases listed in
the question involves mutation of the mitochondrial genome,
which originated solely from the ovum. (See Chapter 77 in
Nelson Textbook of Pediatrics, 17th edition.)

Anticipation

X-linked recessive inheritance


X-linked dominant inheritance

Question . 13. Tay-Sachs disease is best described as:

Only affecting Ashkenazi Jews

Having a single genetic defect

Producing disease in only males

Having genetic heterogenicity


Explanation: This autosomal recessive disorder, due to a
defect in the protein hexosaminidase A, has a degree of
genetic heterogenicity in that the defect in Ashkenazi Jews is a
frameshift mutation, whereas that in French Canadians is due
to a missing gene segment. (See Chapter 69 in Nelson
Textbook of Pediatrics, 17th edition.)

Having an adult-onset variety

Question . 14. FISH in genetic testing is best described as:

A way to prepare RNA

An enzyme assay to detect mutations

Scraping of the buccal cells

A test to identify the chromosomal location of an affected


gene
Explanation: Fluorescence in situ hybridization to a
chromosome region with a deletion or mutated gene is a
powerful new tool in genetic diagnosis. (See Chapter 68 in
Nelson Textbook of Pediatrics, 17th edition.)

Fibroblast inhibition selective histology

Question . 15. The polymerase chain reaction is best described as:

A method to produce many antigenic epitopes

A method to amplify small quantities of DNA or RNA


Explanation: This powerful tool is helpful in genetic research
and diagnosis. It also has been valuable in the rapid diagnosis
of infectious diseases such as herpes simplex virus and
tuberculosis. (See Chapter 68 in Nelson Textbook of
Pediatrics, 17th edition.)

A Southern blot

A Northern blot
A Western blot

Question . 16. Trinucleotide repeats are implicated in the etiology of all of the
following except:

Fragile X syndrome

Neurofibromatosis
Explanation: Multiple repeats of trinucleotides in the coding
region and the untranslated or translated region of these genes
produce significant disease in the all of the other named
choices. (See Chapter 68 in Nelson Textbook of Pediatrics,
17th edition.)

Friedreich ataxia

Spinocerebellar ataxia type I

Myotonic dystrophy

Question . 17. A 1-yr-old presents with a disease that is classically an autosomal


recessive trait (such as cystic fibrosis). The father is tested and, with 99%
confidence, he is demonstrated to be negative for the carrier state. The most likely
explanation is:

Mutation to an autosomal dominant trait

Uniparental isodisomy transmission from the mother


Explanation: Inheritance of two copies of the same affected
chromosome from the mother is the best explanation. Although
the mother is an obligate carrier, two copies of the affected
chromosome are transmitted to the offspring; the normal
chromosome was not transmitted. (See Chapter 70 in Nelson
Textbook of Pediatrics, 17th edition.)

Uniparental isodisomy transmission from the father

Imprinting of the missing recessive gene

Mitochondrial recombination

Question . 18. A newborn infant is noted to have dysmorphic features. The


pregnancy was complicated by breech presentation, decreased fetal movements,
and polyhydramnios. The child demonstrates hypotonia, a flat face, flattened
occiput, epicanthal folds, and abdominal distention. The most likely cause of this
child's dysmorphology is:

Trisomy 13

Trisomy 18

Edwards syndrome

Trisomy 8
Trisomy 21
Explanation: Trisomy 21, or Down syndrome, is also
associated with other cardiac, gastrointestinal, and skeletal
problems. (See Chapter 70 in Nelson Textbook of Pediatrics,
17th

Question . 19. To evaluate the abdominal distention in the patient described in


Question 18, an x-ray study of the kidneys, ureters, and bladder (KUB) is performed
and reveals a "double-bubble" sign. The best explanation for the neonate's
abdominal distention is:

Hirschsprung disease

Meconium ileus

Meconium plug

Duodenal atresia
Explanation: Duodenal atresia is common in neonates with
trisomy 21 and may produce polyhydramnios. After birth,
intestinal obstruction requires that the child be NPO and
undergo intestinal decompression prior to surgery. (See
Chapter 70 in Nelson Textbook of Pediatrics, 17th edition.)

Pyloric atresia
ALLERGY
Question . 1. Which of the following are characteristic of allergens?

Proteins of molecular weight <10 kd

Proteins of molecular weight 10-70 kd


Explanation: Most allergens are proteins that have
molecular weights of 10-70 kd. Molecules smaller than 10
kd would not bridge adjacent IgE antibody molecules on the
surface of mast cells or basophils. Most molecules larger
than 70 kd would not pass through mucosal surfaces
needed to reach antigen-presenting cells for stimulation of
the immune system. Allergens frequently function in their
natural state as proteolytic enzymes, which may contribute
to increased mucosal permeability and sensitization. (See
Chapter 130 in Nelson Textbook of Pediatrics, 17th ed.)

Proteins of molecular weight >70 kd

Lipopolysaccharides

Carbohydrates

Question . 2. Which of the following factors is characteristic of an atopic


response?

Th1 release of cytokines promoting phagocytosis

Th1 release of cytokines promoting synthesis of opsonizing


antibodies

Th1 and Th2 release of cytokines promoting synthesis of


complement-fixing antibodies

Th2 release of cytokines promoting phagocytosis

Th2 release of cytokines promoting synthesis of IgE


antibodies
Explanation: Nonatopic subjects respond with the
proliferation of T helper type 1 (Th1) cells, which secrete
Th1 type cytokines (e.g., IFN- ) involved in the elicitation of
allergen-specific IgG antibodies. Th1 cells are generally
involved in the eradication of intracellular organisms such
as mycobacteria, because of the ability of Th1 cytokines to
activate phagocytes and promote the production of
opsonizing and complement-fixing antibodies. However,
genetically predisposed atopic individuals respond with a
brisk expansion of T helper type 2 (Th2) cells that secrete
cytokines favoring IgE synthesis. (See Chapter 130 in
Nelson Textbook of Pediatrics, 17th ed.)
Question . 3. Which of the following types of cells are distributed throughout
connective tissues, often adjacent to blood vessels and below epithelial
surfaces that are exposed to the external environment, and release a diverse
array of mediators of allergic inflammation?

Eosinophils

Basophils

Mast cells
Explanation: Mast cells contain or produce a diverse array
of mediators of allergic inflammation. (See Chapter 130 in
Nelson Textbook of Pediatrics, 17th ed.)

Th2 cells

Dendritic cells

Question . 4. Which of the following antigen-presenting cells are actively


phagocytic and reside in peripheral sites such as the skin, intestinal lamina
propria, and lungs?

Eosinophils

Basophils

Mast cells

Th2 cells

Dendritic cells
Explanation: Antigen-presenting cells (APCs) are a
heterogeneous group of cells that present antigens in the
context of the major histocompatibility complex (MHC).
Dendritic cells are actively phagocytic cells that reside in
peripheral sites such as the skin, intestinal lamina propria,
and lungs. (See Chapter 130 in Nelson Textbook of
Pediatrics, 17th ed.)
Question . 5. Which of the following statements best describes the
relationship between allergic disorders and a possible genetic basis?

Allergic disorders are a response to only environmental


factors

Allergic disorders are a response to only environmental


factors and infectious agents

Asthma and allergic rhinitis are the only allergic disorders


with a familial predisposition

Any familial predisposition is related to polymorphisms


of a single gene located on chromosome 10
Explanation: Both environmental and genetic factors are
important in allergic diseases. The clinical expression of
these diseases is a complex interaction of many genetic
loci and polymorphisms in each of these genes. (See
Chapter 130 in Nelson Textbook of Pediatrics, 17th ed.)

Any familial predisposition is related to many genetic loci


and also many polymorphisms

Question . 6. Which of the following factors may contribute to the worldwide


rise in prevalence of allergic diseases, particularly in Westernized metropolitan
areas?

Increasing genetic polymorphisms of CD14

Increased numbers of children in group daycare

Excessive use of antibiotics in first 2 yr of life


Explanation: Widespread antibiotic use, particularly in
young children, alters the microbial flora in the
gastrointestinal tract and may produce an environment that
is less effective in driving a Th1 response. (See Chapter
130 in Nelson Textbook of Pediatrics, 17th ed.)

Reduced exposure to pollutants in Westernized


metropolitan areas since 1980

Reduced exposure to indoor allergens


Question . 7. All of the following may be signs of moderate to severe airway
obstruction resulting from allergic response except:

Dennie lines (Dennie-Morgan folds)


Explanation: Dennie lines (Dennie-Morgan folds) are
prominent symmetric skinfolds that extend in an arc from
the inner canthus beneath and parallel to the lower lid
margin. Like allergic shiners and the allergic salute, they
are signs of persistent rhinorrhea associated with allergic
rhinitis. A "silent chest' in a patient with asthma (answer E)
is a severe sign suggesting inspiratory and expiratory
obstruction. Cyanosis is always present in such severe
cases. (See Chapter 131 in Nelson Textbook of Pediatrics,
17th ed.)

Supraclavicular and intercostal retractions

Cyanosis

Pulsus paradoxus

Respiratory distress with minimal wheezing and a few


crackles

Question . 8. A 7-yr-old boy with asthma has roughness over the extensor
surfaces of the upper arms and thighs, which is caused by keratin plugs
lodged in the openings of hair follicles. This physical finding is termed:

Keratosis pilaris
Explanation: Xerosis, or dry skin, is the most common skin
abnormality of allergic children. Keratosis pilaris, often
found on the extensor surfaces of the upper arms and
thighs, is characterized by roughness of the skin caused by
discrete follicular papules. These are the result of
hyperkeratosis with keratin plugs lodged in the openings of
hair follicles, and re-form after removal. (See Chapter 131
in Nelson Textbook of Pediatrics, 17th ed.)

Fibroepitheliosis

Hidradenitis

Xerosis

Acrochordon
Question . 9. The radioallergosorbent test (RAST) determines:

Bronchial reactivity to subcutaneous serotonin

Bronchial reactivity after inhalation bronchial provocation


test

The proportion of total allergic immunoglobulin

Antigen-specific serum IgE concentrations


Explanation: The RAST (radioallergosorbent test)
determines the serum IgE concentrations against specific
antigens. The RAST correlates well with medical history
and allergy skin testing but is somewhat less sensitive than
skin testing. (See Chapter 131 in Nelson Textbook of
Pediatrics, 17th ed.)

The overall allergic risk profile based on absolute


eosinophil count, total IgE, and skin test results

Question . 10. All of the following statements regarding skin testing for allergic
reactivity are true except:

Antihistamines given prior to testing may inhibit the reaction

Intradermal tests are more sensitive than puncture tests

Positive skin test results by intradermal testing


correlate better than results by puncture tests with
clinical symptoms
Explanation: Positive skin test results obtained by the
puncture technique correlate better than the more sensitive,
less specific intradermal tests with measurements of
specific IgE antibody and with the appearance of clinical
symptoms on exposure to the allergen. (See Chapter 131
in Nelson Textbook of Pediatrics, 17th ed.)

The reaction peaks within approximately 20 min and


usually resolves over 20-30 min

Larger reactions have greater clinical relevance


Question . 11. Which of the following is an advantage of skin testing over
RAST to determine specific IgE?

Skin testing is not affected by administration of


antihistamines

Skin testing has greater sensitivity than RAST


Explanation: Because skin tests are more sensitive than
RAST, they are more reliable than RAST in confirming risk
of life-threatening anaphylactic conditions. All of the other
responses are incorrect. (See Chapter 131 in Nelson
Textbook of Pediatrics, 17th ed.)

Skin testing is semiquantitative

Skin testing is associated with less risk of allergic reaction

Skin testing is not confounded by dermographism

Question . 12. Which of the following physical findings would be least likely on
examination of a child with moderate to severe asthma?

Tachypnea

Wheezing

Clubbing
Explanation: Digital clubbing (hypertrophic pulmonary
osteoarthropathy) is rarely observed in children with
uncomplicated asthma and should prompt evaluation to
exclude other potential diagnoses. (See Chapter 131 in
Nelson Textbook of Pediatrics, 17th ed.)

Decreased air exchange over the right middle lobe

An increased anterior-posterior diameter of the chest


Question . 13. Recommendations to the parents of a child with dust mite
allergy to help reduce dust mite exposure should include all of the following
except:

Use a humidifier regularly


Explanation: Household humidity should be kept at less
than 50% to inhibit survival of mites. Use of vaporizers
should be avoided. Dehumidifiers may be necessary in
damp basements. The air conditioning should be set at the
lowest level during the warmer months. Clothes and
bedding should be washed in hot water (>130°F) to kill dust
mites. Carpeted flooring is not recommended. Carpet and
upholstered furniture, if retained, should be vacuumed
weekly using a vacuum with a HEPA filter. (See Chapter
132 in Nelson Textbook of Pediatrics, 17th ed.)

Place the mattress and pillow in allergen-proof


encasements

Wash bed linens in hot water weekly

Remove the old carpet from the bedroom

Question . 14. All of the following statements regarding decreasing exposure


to cat allergens are true except:

Removing the cat from the home is the most effective


means of reducing exposure to cat allergen

Keeping the cat out of the child's bedroom and other rooms
where the sensitized child spends large amounts of time
reduces cat allergen exposure

Washing the cat regularly reduces cat allergen exposure

Using HEPA-filtered air cleaners does not reduce cat


allergen exposure
Explanation: Advice to remove a pet cat from the home or
keep it outdoors is often ignored. In contrast to dust mite
allergens, cat allergen is light and remains suspended in
the air for long periods of time. Regular vacuuming with a
HEPA-filtered and double-thickness-bag vacuum cleaner is
encouraged. (See Chapter 132 in Nelson Textbook of
Pediatrics, 17th ed.)

Removing carpet decreases cat allergen exposure


Question . 15. A 12-yr-old girl with moderate to severe asthma is sensitive to
cat dander. Her family elects to remove the pet cat from the house, but to
retain the present carpeting and upholstered furniture. What is the length of
time required before the levels of cat allergen drop to levels found in homes
without a cat?

Immediately

2 days

2 wk

2 mo

6 mo
Explanation: Cat owners who remove the cat from the
home without also removing carpeting and upholstered
furniture, and thoroughly wiping down all walls and hard
surfaces, should be informed not to expect immediate
results. It may take 6 months to 1 year for the levels of cat
allergen to drop to a level found in homes without a cat.
(See Chapter 132 in Nelson Textbook of Pediatrics, 17th
ed.)

Question . 16. Which of the following statements regarding antihistamines is


true?

Classification of antihistamines from type I to type VI is


based on increasing antihistamine activity

Second-generation antihistamines are distinguished by


greater effectiveness than first-generation antihistamines

Antihistamines should not be administered in combination


with decongestants

Antihistamines are more effective in treating than


preventing the action of histamine

The choice of antihistamines should be based on


associated adverse effects and cost
Explanation: There is little reason to choose one
antihistamine over another except for avoidance of adverse
effects, such as sedation, impairment of function, and cost.
The chemical classification of antihistamines (type I to type
VI) does not have functional significance. Second-
generation antihistamines have fewer sedative adverse
effects. (See Chapter 132 in Nelson Textbook of Pediatrics,
17th ed.)
Question . 17. Which of the following is an advantage of second-generation
antihistamines over first-generation antihistamines?

Second-generation antihistamines are often less expensive

Second-generation antihistamines are more frequently


available in oral preparations

Second-generation antihistamines have less of a


sedative effect and produce less cognitive impairment
Explanation: One of the primary advantages of second-
generation antihistamines is that they are nonsedating or
much less so than first-generation antihistamines. (See
Chapter 132 in Nelson Textbook of Pediatrics, 17th ed.)

Many more second-generation antihistamines are available


as over-the-counter medications

Second-generation antihistamines are generally more


effective than first-generation antihistamines

Question . 18. Which of the following statements regarding the use of


cromolyn in the management of asthma is true?

Cromolyn prevents antibody-mediated mast cell


degranulation and mediator release

Cromolyn prevents non-antibody-mediated mast cell


degranulation

Cromolyn has no bronchodilator properties

The incidence of adverse effects is low

All of the above


Explanation: Cromolyn prevents bronchoconstriction
caused by immunologic as well as nonimmunologic stimuli
(e.g., frigid air, exercise). It has no bronchodilator
properties and is useful only if given prophylactically. (See
Chapter 132 in Nelson Textbook of Pediatrics, 17th ed.)
Question . 19. The type of adrenergic activity of drugs most desirable in
treatment of asthma is:

2
Explanation: Agents with greater 2-selective activity
provide effective bronchodilation with less cardiac
stimulation (e.g., increase in heart rate) than may occur
with agents with both 1 and 2 activities. (See Chapter 132
in Nelson Textbook of Pediatrics, 17th ed.)

Question . 20. A 4-yr-old boy experiences perennial clear rhinorrhea, nasal


congestion, conjunctival injection, allergic shiners, nasal and ocular pruritus,
and occasional fits of sneezing. An environmental history is significant for two
cats in the home and flooding of the basement when it rains. He keeps twenty
stuffed animals on his bed and sleeps with a feather pillow on an old mattress.
He lives in a warm climate. Seasonal worsening of his symptoms has not been
observed. He has perennial allergic rhinitis. Which of the following groups of
allergens would be the most likely to contribute to his symptoms?

Dust mites, tree pollens, and weed pollens

Dust mites, animal danders, and molds


Explanation: Perennial allergic rhinitis is most often
associated with indoor allergens: house dust mites, animal
danders, and molds. (See Chapter 133 in Nelson Textbook
of Pediatrics, 17th ed.)

Tree, weed, and grass pollens

Tree pollen, grass pollen, and milk protein


Question . 21. A 7-yr-old girl presents with allergic nasal symptoms that are
prominent from the middle of August through the first frost. Which of the
following allergens is the most likely cause of her symptoms?

Milk protein

Tree pollen

Grass pollen

Weed pollen
Explanation: In temperate climates, airborne pollen
responsible for SAR appears in distinct phases: trees
pollinate in the spring, grasses in the early summer, and
weeds in the late summer. (See Chapter 133 in Nelson
Textbook of Pediatrics, 17th ed.)

Question . 22. A teenage boy presents in April with symptoms consistent with
seasonal allergic rhinitis. On examination of his nose, which of the following
findings suggest the need for further evaluation to exclude another diagnosis?

Nasal polyps
Explanation: Nasal polyps and nasal septal deviation are
structural disorders that can mimic allergic rhinitis. (See
Chapter 133 in Nelson Textbook of Pediatrics, 17th ed.)

Pale-to-purple nasal mucosa

Thin, clear nasal secretions

A transverse nasal crease

Continuous open-mouth breathing

Question . 23. A 12-yr-old presents with sneezing, clear rhinorrhea, and nasal
itching. Physical examination reveals boggy, pale nasal edema with a clear
discharge. The most likely diagnosis is:

Foreign body

Vasomotor rhinitis

Neutrophilic rhinitis

Nasal mastocytosis

Allergic rhinitis
Explanation: Allergic rhinitis is often seasonal and
associated with allergic conjunctivitis. Eosinophils
predominate in the nasal secretions.Chapter 133

Question . 24. Two weeks later, the patient described in Question 23


complains of headache, poor nasal airflow requiring mouth breathing, fever,
and a change in the nature of the nasal discharge to mucopurulent discharge.
The most likely diagnosis is:

Sinusitis
Explanation: Sinusitis is a possible complication of allergic
rhinitis. A change in the nature of the nasal discharge,
facial pain, and fever may all herald the onset of sinusitis.
(See Chapter 133 in Nelson Textbook of Pediatrics, 17th
ed.)

Foreign body

Rhinitis medicamentosa

Choanal stenosis

Ciliary dyskinesia

Question . 25. A 12-yr-old child presents with watery rhinorrhea, paroxysmal


sneezing, and nasal obstruction. The serum IgE level is normal, and skin test
results are negative. The physical examination is remarkable only for swollen
turbinates and clear nasal secretions. A trial of antihistamine-decongestant
therapy for 3 wk has not relieved symptoms. Which of the following is the
recommended management?

Institute strict measures to avoid outdoor allergen


exposure.

Begin seasonal use of oral sympathomimetic drugs.

Begin seasonal use of topical intranasal


corticosteroids.
Explanation: Topical intranasal corticosteroids (e.g.,
fluticasone, budesonide) should be used in children with
allergic rhinitis that is resistant to antihistamine-
decongestant therapy. A consultation with an allergist is
recommended for patients with allergic rhinitis that does not
respond to intranasal corticosteroids. (See Chapter 133 in
Nelson Textbook of Pediatrics, 17th ed.)

Give a 10-day course of amoxicillin

Give a 10- to 14-day course of cefpodoxime


Question . 26. Which of the following is most useful in establishing the
diagnosis of seasonal allergic rhinitis?

History of good clinical response to an intranasal


corticosteroid preparation

History of exacerbation of symptoms in the spring


Explanation: Seasonal allergic rhinitis follows a well-
defined course of cyclical exacerbation, whereas perennial
allergic rhinitis causes year-round symptoms.Chapter 133

Elevated serum IgE level

Positive result on skin testing for the house dust mite


allergen

Nasal eosinophils

Question . 27. Common triggers of asthma in children include all of the


following except:
Secondary tobacco smoke

Ozone

Cold air

Exercise

Gelatin
Explanation: Asthma symptoms may be provoked by
numerous events or exposures.Chapter 134

Question . 28. The parents of a 3-yr-old girl with a history of several previous
coughing and wheezing exacerbations are wondering if their toddler is likely to
develop persistent asthma. Which of the following is a strong risk factor for
persistent asthma in toddlers with recurrent wheezing?

Eczema
Explanation: Only a minority of young children who
experience recurrent wheezing will go on to have persistent
asthma in later childhood. Several risk factors have been
identified. Chapter 134

Colic

Living on a farm

Female gender
Otitis media with effusion

Question . 29. A 4-yr-old boy with asthma has had mild wheezing only four
times since you began treating him 6 mo ago with theophylline (Slo-bid
Gyrocaps) twice each day. He previously experienced coughing and wheezing
at least three times each week. (A peak serum theophylline concentration 5
mo ago was 16 g/mL). For the past 4 days, he has again experienced mild
coughing and wheezing responsive to inhaled albuterol. Two days ago, an
emergency department physician began treatment with erythromycin-
sulfisoxazole (Pediazole) for otitis media. This morning the youngster began
vomiting. The likely cause of the vomiting is:

Provocation by coughing (post-tussive emesis)

Sequelae of otitis media

Theophylline toxicity
Explanation: The erythromycin (a macrolide antibiotic)
component of Pediazole inhibits hepatic theophylline
metabolism, thus potentially producing theophylline toxicity.
(See Chapter 134 in Nelson Textbook of Pediatrics, 17th
ed.)

Albuterol toxicity

Pediazole intolerance

Question . 30. A 10-yr-old child has intermittent symptoms of mild asthma.


The most appropriate treatment option is:

Environmental control and patient education only?no


medication is indicated

Oral theophylline

Cromolyn

Inhaled 2-agonist as needed for symptoms


Explanation: For mild intermittent symptoms of asthma,
recommended treatment is with a short-acting inhaled 2-
agonist as needed for symptoms. The intensity of treatment
depends on the severity of exacerbations. The need for
short-acting inhaled 2-agonist use more than two times a
week may indicate the need to initiate long-term-control
therapy. (See Chapter 134 in Nelson Textbook of
Pediatrics, 17th ed.)

Daily inhaled corticosteroid


Question . 31. The child described in Question 30 experiences worsening of
symptoms, which are now persistent and of moderate severity. The most
appropriate treatment option is:

Oral theophylline

Inhaled 2-agonist as needed for symptoms

Daily inhaled corticosteroid and oral theophylline

Daily inhaled corticosteroid and a long-acting inhaled


2-agonist
Explanation: For moderate persistent symptoms of
asthma, recommended treatment is with a daily-inhaled
corticosteroid and a long-acting inhaled 2-agonist.
Alternatives to the inhaled 2-agonist are sustained-release
theophylline and a leukotriene receptor antagonist. In
addition, for moderate persistent symptoms of asthma, a
short-acting 2-agonist is also used as needed for quick
relief of symptoms.Chapter 134

Daily inhaled corticosteroid, a long-acting inhaled 2-


agonist, and oral theophylline

Question . 32. A 12-yr-old asthmatic boy has developed an asthma


exacerbation in the past few days. Asthma symptoms have continued to
progress despite frequent albuterol use at home. He comes to the emergency
department with chest tightness, dyspnea, and wheezing, and in moderate
respiratory distress. In this setting, management should include all of the
following except:

Close monitoring

Supplemental oxygen

Inhaled albuterol

Theophylline
Explanation: Initial emergency department management of
an asthma exacerbation includes close monitoring of
clinical status, treatment with supplemental oxygen, inhaled
-agonist every 20 min for 1 hr, and if necessary, systemic
glucocorticoids (2 mg/kg/day) given either orally or
intravenously. Inhaled ipratropium may be added to the -
agonist treatment if no significant response is seen with the
first inhaled -agonist treatment. If a child responds poorly
to intensive therapy with nebulized albuterol, ipratropium,
and parenteral glucocorticoids, then adding intravenous
theophylline could be considered.Chapter 134
Systemic glucocorticoids

Question . 33. A 7-yr-old girl has had intermittent asthma symptoms over the
past 5 yr. Her asthma symptoms have been treated with inhaled albuterol as
needed. She mostly has exercise-induced asthma symptoms, which happens
on most school days except when she uses her albuterol inhaler before going
to recess and physical education classes. In the past year, she has had two
asthma exacerbations with viral upper respiratory tract infections, and she has
used a total of 5 albuterol metered-dose inhalers. The most appropriate
management for this asthmatic girl is:

Continue albuterol as needed and before physical exercise


activities

Begin daily controller medication with an inhaled


glucocorticoid, initially used more frequently to gain
control, then a reduced amount in a few months to
maintain control
Explanation: Low-dose inhaled glucocorticoids,
leukotriene pathway modifiers, and cromolyn/nedocromil
are the recommended controllers for mild persistent
asthmatics; sustained-release theophylline is an alternative.
Chapter 134

Begin daily inhaled glucocorticoid in a low dose, increasing


the dose monthly until good control is obtained

Administer daily oral glucocorticoid treatment for one week,


with concurrent daily inhaled glucocorticoid

Begin use of a long-acting inhaled -agonist each morning

Question . 34. Components of the U.S. National Asthma Education &


Prevention Program (NAEPP) guidelines include all of the following except:

Regular assessment and monitoring

Control of factors contributing to asthma severity

Asthma pharmacotherapy, especially the use of anti-


inflammatory controller medications

Genetic profiling
Explanation: The NAEPP guidelines were recently
adapted for childhood asthma in a joint-effort publication of
the American Academy of Allergy, Asthma & Immunology
with the U.S. National Institutes of Health's National Heart,
Lung and Blood Institute and the American Academy of
Pediatrics entitled Pediatric Asthma: Promoting Best
Practice.Chapter 134
Patient education

Question . 35. Features characteristically associated with atopic dermatitis


include all of the following except:

Allergic rhinitis or asthma

Elevated serum IgE level

Peripheral blood eosinophilia

Lymphopenia
Explanation: Most patients with atopic dermatitis have
peripheral blood eosinophilia and elevated serum IgE level.
Nearly 80% of patients with atopic dermatitis develop
allergic rhinitis and/or asthma.

Question . 36. Major features of atopic dermatitis in children include all of the
following except:

Pruritus

Facial and extensor eczema

Angioedema
Explanation: Angioedema is similar to urticaria but has
deeper tissue involvement. Urticaria and angioedema are
not characteristic features of atopic dermatitis

Chronic or relapsing course

Personal or family history of atopic disease

Question . 37. A 2-yr-old is diagnosed with atopic dermatitis. Which of the


following environmental modifications is recommended?

A bland diet, especially minimizing meats

Installation of wool carpeting instead of synthetic carpeting

Use of a liquid rather than powder laundry detergent,


and adding a second rinse cycle
Explanation: Using a liquid rather than a powder laundry
detergent and adding a second rinse cycle will facilitate
removal of the detergent. Soaps should have minimal
defatting activity and a neutral pHChapter 135

Use of soaps that are especially effective in removing fatty


substances
Bathing less often than daily

Question . 38. The most appropriate prognosis to convey to the parents of the
2-yr-old with atopic dermatitis described in Question 37 is:

The child will be asymptomatic with environmental


modifications

Symptoms will gradually worsen during childhood and


persist stably through adulthood

Symptoms will exhibit a remittent but progressively


worsening course through adulthood

Symptoms will gradually decrease over the next


several years with an approximately 50% chance of
spontaneous improvement
Explanation: Atopic dermatitis generally tends to be more
severe and persistent in young children. With control of
trigger factors and appropriate local treatment, reasonable
but not complete resolution of symptoms is usually
possible. Periods of remission appear more frequently as
the child grows older. Spontaneous resolution of atopic
dermatitis has been reported to occur after age 5 yr in 40-
60% of patients affected during infancy, particularly if their
disease is mild. Recent studies have reported that atopic
dermatitis disappears in approximately 20% of children
followed from infancy until adolescence, but it had become
less severe in 65%. (See Chapter 135 in Nelson Textbook
of Pediatrics, 17th ed.)

Symptoms will resolve completely at puberty

Question . 39. Which of the following is the major feature of atopic dermatitis?

Onset shortly before or during puberty

Pruritus
Explanation: All patients with atopic dermatitis have
pruritus. However, not all patients with atopic dermatitis
have other allergic symptoms, elevated IgE levels, or S.
aureus skin infections. (See Chapter 135 in Nelson
Textbook of Pediatrics, 17th ed.)

C. Staphylococcus aureus cutaneous infections

Elevated serum IgE

Immediate skin test reactivity to allergens


Question . 40. A 5-yr-old boy with severe atopic dermatitis develops illness
with dozens of vesicles primarily covering areas of skin previously affected by
atopic dermatitis. The distribution crosses many dermatomes. Findings include
fever and lymphadenopathy. The most likely diagnosis is:

Chickenpox

Zoster

Kaposi varicelliform eruption


Explanation: Kaposi varicelliform eruption, or eczema
herpeticum, results from herpes simplex virus infection of
skin with altered immunity, usually from atopic dermatitis.
Kaposi varicelliform eruption is clinically distinguished from
zoster by its random distribution, which may involve many
dermatomes. Additionally, lesions of eczema herpeticum
are often isolated and are not grouped, as are the vesicles
of zoster. Similar eruptions have been described in
association with vaccinia virus (smallpox vaccination) and
coxsackievirus infections. (See Chapter 135 in Nelson
Textbook of Pediatrics, 17th ed.)

Eczema vaccinatum

Coxsackievirus infection

Question . 41. A 14-yr-old presents with acute-onset urticaria that has


gradually worsened over the past 10 days. Detailed history reveals no clues to
the possible etiology. Findings on physical examination are normal except for
urticaria. Which of the following diagnostic options is recommended?

Systematic elimination diets to determine a possible


ingestant cause

Allergy skin testing


Explanation: No laboratory test confirms or excludes the
diagnosis of urticaria. Allergy skin testing can be helpful in
sorting out causes of acute urticaria, especially when
supported by historical evidence. Drugs and foods are the
most common causes of acute urticaria. A skin biopsy is
indicated only if urticarial vasculitis is suspected. (See
Chapter 136 in Nelson Textbook of Pediatrics, 17th ed.)

Serum IgE and RAST

Skin biopsy

None of the above


Question . 42. Which of the following treatment options is recommended for
the patient described in Question 42?

A bland diet

Wearing cotton garments

Oral antihistamine
Explanation: Antihistamines are usually effective for
treatment of urticaria. Diphenhydramine and hydroxyzine
are effective but also cause sedation. A nonsedating
antihistamine (e.g., Loratadine) is often the preferred
therapy for urticaria for school-aged children to minimize
the effect on learning and school performance. (See
Chapter 136 in Nelson Textbook of Pediatrics, 17th ed.)

Oral prednisone

Topical corticosteroid

Question . 43. Which of the following laboratory tests is most likely to give
abnormal results in a patient with chronic urticaria?

Serum IgE level determination

Skin prick testing for egg sensitivity

C4 level assay

Assay for antibodies to thyroglobulin


Explanation: There is an increased association of chronic
urticaria with Hashimoto thyroiditis. Such patients generally
have antibodies to thyroglobulin, or a microsomal-derived
antigen (peroxidate) even if they are euthyroid. The
incidence of abnormal thyroid function (either increased or
decreased T4 and/or increased or decreased TSH) is
approximately 20%. Patients with chronic urticaria usually
have normal IgE levels. (See Chapter 136 in Nelson
Textbook of Pediatrics, 17th ed.)

Heterophile antibody testing


Question . 44. A 12-yr-old girl with repeated episodes of streptococcal
pharyngitis experiences another episode of sore throat. The rapid strep test
result is positive, and oral amoxicillin is started, with the first dose given in the
office. One hour later, she experiences a "funny feeling" and a tingling
sensation around her mouth. Next she becomes apprehensive, has difficulty
swallowing, and develops a hoarse voice. On arrival at the emergency
department, she has giant urticaria and the following vital signs: pulse 130,
respiratory rate 32/min, blood pressure 70/30 mm Hg, and temperature
37.2°C. The most appropriate therapy is administration of:

Epinephrine
Explanation: Intramuscular epinephrine is the treatment of
choice. If the blood pressure does not respond, lactated
Ringer's solution should be administered. Benadryl,
cimetidine, and prednisone are second-line therapeutic
agents to be administered after epinephrine and fluids.
(See Chapter 137 in Nelson Textbook of Pediatrics, 17th
ed.)

Prednisone

Diphenhydramine

Albuterol

Lactated Ringer's solution

Question . 45. The most likely diagnosis for the patient described in Question
45 is:

Streptococcal toxic shock

Scarlet fever

Stevens-Johnson syndrome

Reye syndrome

Anaphylaxis
Explanation: Anaphylaxis to penicillin usually occurs within
30-90 min of administration of this drug. Anaphylactic shock
is often missed as a diagnosis unless a complete history is
obtained and there is a high index of suspicion. (See
Chapter 137 in Nelson Textbook of Pediatrics, 17th ed.)
Question . 46. The mother of an 8-yr-old boy with acute streptococcal
tonsillitis calls to report that now, within 15 min after the first dose of oral
penicillin V that you prescribed, he is complaining of itching and has developed
hives. Which of the following should you recommend?

A dose of oral Benadryl, with instructions to call again if he


has not improved within 30 min

Immediate return to your office or the nearest


emergency department
Explanation: The urticarial reaction described in the
question may develop into anaphylaxis; the latter requires
emergency treatment. In addition, the penicillin V should be
stopped and a substitute nonpenicillin antibiotic chosen.
(See Chapter 137 in Nelson Textbook of Pediatrics, 17th
ed.)

Careful monitoring at home, with instructions to return to


your office or the nearest emergency department if he
becomes short of breath or loses consciousness

Schedule a visit for a laboratory test to determine serum


trypticase level

Substitution of erythromycin for penicillin

Question . 47. All of the following statements regarding anaphylaxis are true
except:

Virtually any foreign substance can elicit an anaphylactic


reaction

Most anaphylactic reactions are due to drugs, latex, foods,


and Hymenoptera venom

Oral drugs carry a higher risk of anaphylaxis than that


associated with injected drugs
Explanation: Reactions to medications can be reduced
and minimized by using oral medications in preference to
injected forms. (See Chapter 137 in Nelson Textbook of
Pediatrics, 17th ed.)

Anaphylactic reactions to foods usually begin within


minutes to 2 hr of exposure

Exercise alone can elicit an anaphylactoid reaction


Question . 48. Administration of which of the following drugs is the treatment
of choice for anaphylaxis?

Diphenhydramine orally

Diphenhydramine by intravenous infusion

Aqueous epinephrine (1:1,000) by subcutaneous injection

Aqueous epinephrine (1:1,000) by intramuscular


injection
Explanation: The principal treatment of choice for
anaphylaxis is aqueous epinephrine, 1:1,000, 0.01 mL/kg
(maximum 0.3 mL for a child or 0.5 mL for an adult) by
intramuscular injection, which can achieve more rapid
effective concentrations than obtainable with subcutaneous
injection. Intravenous epinephrine may be added as a
continuous drip for persistent shock. Intramuscular or
intravenous H1 and H2 antagonist antihistamines, oxygen,
intravenous fluids, inhaled -agonists, and corticosteroids
may also be required.

Aqueous epinephrine (1:1,000) by intravenous infusion

Question . 49. A 16-yr-old with history of anaphylaxis to Hymenoptera suffers


a sting on an extremity. The first-aid kit that is available includes aqueous
epinephrine 1:1,000 and other necessary medical supplies. All of the following
measures for management of this sting are appropriate except:

Infiltration of one half of the epinephrine dose


subcutaneously around the site of the sting

Repeat doses of aqueous epinephrine at 15-min intervals if


necessary

Placement of a tourniquet above the site of the sting

Incision of and suction of venom from the site of the


sting
Explanation: With anaphylaxis due to injection of allergen
extract or to a Hymenoptera sting on an extremity, one half
of the dose of epinephrine may be diluted in 2 mL of normal
saline and infiltrated subcutaneously at the site of the sting
to slow absorption. Doses can be repeated at 15-min
intervals if necessary. A tourniquet above the site can also
slow systemic distribution. The tourniquet can be loosened
after improvement or briefly at intervals of 3 min. Immediate
transport to an appropriate medical facility should be
arranged
Transport to an emergency department

Question . 50. The most common single cause of anaphylaxis outside of the
hospital is:

Insect sting allergy

Drug allergy

Food allergy
Explanation: Food allergy is the most common cause of
anaphylaxis occurring outside of the hospital, accounting
for about one half of the anaphylactic reactions reported in
pediatric surveys. (See Chapter 137 in Nelson Textbook of
Pediatrics, 17th ed.)

Latex allergy

Food-associated exercise-induced anaphylaxis

Question . 51. A 12-yr-old child with a history of allergy to yellow jackets is


stung and immediately begins experiencing tightness in the chest and
wheezing. The drug of first choice for management of this child is:

Inhaled albuterol

Subcutaneous epinephrine

Intramuscular diphenhydramine

Intramuscular epinephrine
Explanation: The principal treatment of choice of
anaphylaxis is aqueous epinephrine, 1:1,000, 0.01 mL/kg
(maximum 0.3 mL for a child or 0.5 mL for an adult) by
intramuscular injection, which can achieve more rapid
effective concentrations than obtainable with subcutaneous
injection. (See Chapter 137 in Nelson Textbook of
Pediatrics, 17th ed.)

Oral corticosteroids
Question . 52. Which of the following would be the optimal long-term
management of the child described in Question 51?

Daily oral non-sedating antihistamine

Daily low-dose oral corticosteroid

Daily inhaled corticosteroid

Inhaled corticosteroid immediately upon insect sting

Immunotherapy
Explanation: Children experiencing systemic anaphylactic
reactions to an insect sting should be evaluated and treated
with immunotherapy, which is >90% protective. (See
Chapter 137 in Nelson Textbook of Pediatrics, 17th ed.)

Question . 53. A 2-yr-old child who has completed 8 days of a 10-day course
of cefaclor presents with low-grade fever, malaise, irritability,
lymphadenopathy, and a generalized erythematous rash that is mildly pruritic.
The most likely diagnosis is:

Partially treated meningitis

Infectious mononucleosis

Kawasaki disease

Type I hypersensitivity reaction

Type III hypersensitivity reaction


Explanation: Serum sickness is a classic example of a
type III hypersensitivity reaction, or immune complex
disease. The symptoms develop as antibodies appear
against the antigen at a time when the antigen is still
present. Immune complexes may stimulate complement
and deposit in joints, the skin, and the renal glomeruli. (See
Chapter 138 in Nelson Textbook of Pediatrics, 17th ed.)
Question . 54. A 14-yr-old child received equine-derived antivenom for a
snake bite 5 yr ago and now requires it again. Results of skin testing to the
product are negative. Which of the following statements is true?

Premedication with corticosteroids is warranted to prevent


serum sickness

Negative skin tests indicate that it is highly unlikely that he


will develop serum sickness

He should not receive this product more than once

Serum sickness may begin within a few days of


administration of the antivenom
Explanation: Because he received the preparation
previously, he may experience an accelerated form of
serum sickness starting before the usual time course of 7-
12 days following injection. Premedication with
corticosteroids does not prevent serum sickness. Skin
testing helps to identify the potential for immediate-type
hypersensitivity (IgE antibody-mediated) to the serum
components but does not predict serum sickness (a type III,
immune complex-mediated hypersensitivity reaction). If
there is no alternative treatment, then there is no
contraindication to receive the product more than once.
(See Chapter 138 in Nelson Textbook of Pediatrics, 17th
ed.)

Question . 55. Risk factors for adverse drug reactions include:

Topical administration (compared with parenteral


administration)

Low dose (compared with high dose)

Frequent, intermittent dosing frequency (compared


with prolonged, continuous dosing)
Explanation: Risk factors for adverse drug reactions
include previous exposure, previous reaction, age (20-49
yr), route of administration (parenteral), dose (high), and
dosing schedule (intermittent), as well as genetic
predisposition (e.g., in slow acetylators). Frequent,
intermittent administration is more likely to elicit
sensitization than prolonged, continual administration. (See
Chapter 139 in Nelson Textbook of Pediatrics, 17th ed.)

No previous exposure (compared with previous


administration)

All of the above


Question . 56. Which of the following statements concerning adverse drug
reactions is true?

Adverse drug reactions are primarily IgE mediated

Drug-induced thrombocytopenia results from circulating


immune complexes

Both parental and topical exposures to a drug increase


the risk for an adverse reaction
Explanation: Parenteral administration poses greater risk
than topical administration, but both contribute to risk for an
adverse reaction. Adverse drug reactions are immune
complex reactions (Gell and Coombs type III)

Approximately 80% of patients with a history of penicillin


allergy will have evidence of penicillin-specific IgE
antibodies on testing

Epidermal detachment of >30% suggests Stevens-Johnson


syndrome

Question . 57. A 7-yr-old boy presents with fever and otalgia. On examination,
he has a bulging right tympanic membrane. As you hand his mother a
prescription for amoxicillin, she informs you that when the child was 4 yr old,
he broke out in an itchy rash during treatment with amoxicillin. The most
appropriate approach to management of this patient would be:

Reassure the mother that since more than 2 yr have


passed, it is highly unlikely that the child is still allergic and
he can now take the amoxicillin safely

Explain to the mother that most adverse drug reactions to


amoxicillin are not IgE mediated and that amoxicillin can be
safely given.

Prescribe a cephalosporin and explain to the mother that


there is no cross-reaction between penicillins and
cephalosporins

Prescribe a macrolide antibiotic and explain to the


mother that there is no cross-reaction between
penicillins and macrolides
Explanation: Risk factors for adverse drug reactions
include previous exposure and previous reaction. A
macrolide is recommended for otitis media in penicillin-
allergic patients

Give the child a prescription for amoxicillin, and instruct the


mother to pre-treat him with diphenhydramine (which is
available without a prescription)

Question . 58. The parents of a 6-yr-old girl relate a history of urticarial


reaction and vomiting following administration of amoxicillin in the past. Skin
testing to major and minor determinants of penicillin is positive. Which of the
following statements regarding administration of a cephalosporin constitutes
appropriate advice for the parents?

The child can receive a cephalosporin with no greater risk


of anaphylaxis than in the general population

There is a 2% risk of anaphylaxis to a cephalosporin


Explanation: Although the risk of allergic reactions to
cephalosporins in patients with positive skin tests to
penicillin appears to be low (less than 2%), anaphylactic
reactions after administration of a cephalosporin have
occurred in patients with a positive history of penicillin
anaphylaxis. If a patient has a history of penicillin allergy
and requires a cephalosporin, skin testing to major and
minor determinants of penicillin should preferably be done
to determine if the patient has penicillin-specific IgE
antibodies. If results of skin tests are negative, the patient
can receive a cephalosporin with no greater risk than in the
general population. If skin tests are positive to penicillin,
recommendations may include administration of an
alternative antibiotic, cautious graded challenge with
appropriate monitoring, in view of the 2% risk of an
anaphylactic reaction, and desensitization to the required
cephalosporin. (See Chapter 139 in Nelson Textbook of
Pediatrics, 17th ed.)

There is a 9% risk of anaphylaxis to a first-generation


cephalosporin but an almost 0% risk of anaphylaxis to a
fourth generation cephalosporin

There is a 9% risk of anaphylaxis to a cephalosporin

There is a 50% risk of anaphylaxis to a cephalosporin


Question . 59. A 14-yr-old girl, who has a long-standing seizure disorder for
which she takes phenytoin, develops fever and a urinary tract infection and is
prescribed trimethoprim-sulfamethoxazole. After 9 days of antibiotic treatment
she has recurrence of fever and develops confluent purpuric macules on her
face and trunk with erosive mucosal lesions of her mouth and conjunctivae. A
skin biopsy reveals 8% epidermal detachment. Which of the following best
describes this disorder?

Toxic shock syndrome

Anticonvulsant hypersensitivity syndrome

Allergy to sulfamethoxazole

Stevens-Johnson syndrome
Explanation: Stevens-Johnson syndrome is a blistering
mucocutaneous disorder induced by drugs, classically
sulfonamides. Epidermal detachment of less than 10%
suggests Stevens-Johnson syndrome. (See Chapter 139 in
Nelson Textbook of Pediatrics, 17th ed.)

Toxic epidermal necrolysis

Question . 60. All of the following may be manifestations of insect allergy


except:

Rhinitis and conjunctivitis

Asthma

Wheal and flare

Anaphylaxis

Uveitis
Explanation: Clinical findings in allergy caused by insects
are similar to those occurring with usual inhalant allergens
(e.g., rhinitis, conjunctivitis, asthma). Biting insects may
cause local reactions that do not involve IgE. Venom from
stinging insects causes IgE-mediated sensitivity that may
lead to urticaria and anaphylaxis. (See Chapter 140 in
Nelson Textbook of Pediatrics, 17th ed.)
Question . 61. All of the following statements concerning allergic reactions to
stinging insects are true except:

The majority are due to Hymenoptera

There is substantial cross-reactivity among vespid venoms

Systemic reactions can occur after the first sting

Most reactions are IgE mediated

Negative results on skin testing and RAST reliably


exclude the likelihood of anaphylaxis
Explanation: There are patients with convincing histories
of sting anaphylaxis with negative skin test results and
RAST results. (See Chapter 140 in Nelson Textbook of
Pediatrics, 17th ed.)

Question . 62. Immunotherapy provides symptomatic improvement in all of the


following except:

Ragweed allergy

Local reaction to bee sting


Explanation: Local reactions to Hymenoptera venom in
children are not managed by immunotherapy. (See Chapter
140 in Nelson Textbook of Pediatrics, 17th ed.)

Tree pollen allergy

House dust mite allergy

Anaphylaxis to a wasp sting


Question . 63. An 8-yr-old boy experienced immediate urticaria surrounding a
large local reaction to a honeybee sting 2 mo ago. He had no other symptoms.
Skin testing with honeybee venom has been strongly positive at a weak
concentration. Appropriate recommendations include all of the following
except:

Hymenoptera venom immunotherapy


Explanation: Immunotherapy is indicated only for systemic
reactions. Individuals with local reactions are not at
increased risk for severe systemic reactions on a
subsequent sting and are not candidates for Hymenoptera
venom immunotherapy. (See Chapter 140 in Nelson
Textbook of Pediatrics, 17th ed.)

An epinephrine auto-injector (EpiPen) for administration


after a subsequent sting

Wearing shoes when outdoors

A Medic-Alert bracelet

Wearing long pants

Question . 64. A 10-yr-old girl was stung on her left cheek by a yellow jacket.
She is experiencing pain. By 4 hr following the sting the left side of her face is
so swollen that her left eye is virtually closed. There are no other complaints.
The best course of action would be:

Apply cold compresses, and consider antihistamines


and pain medication
Explanation: The child has experienced a large local
reaction to the sting. Supportive care directed at the
reaction is appropriate. Individuals who have experienced
only large local reactions, or children younger than 17 yr
who have experienced systemic reactions confined to the
skin (generalized urticaria), are not at significantly
increased risk for a severe systemic reaction upon
subsequent stings, so testing for allergy and providing
emergency medications are not warranted. (See Chapter
140 in Nelson Textbook of Pediatrics, 17th ed.)

Perform or refer her for skin testing to Hymenoptera venom

Prescribe self-injectable epinephrine and provide


instructions to school/camp

All of the above


Question . 65. A 7-yr-old boy was stung by an unidentified insect and within
minutes developed generalized urticaria, a repetitive cough, difficulty
breathing, and extreme dizziness. He was treated in the emergency
department with antihistamines, epinephrine, and corticosteroids. Which of the
following statements is accurate?

If skin tests to Hymenoptera venom are performed 1 wk


later and results are negative, he is not a candidate for
venom immunotherapy

Testing and venom immunotherapy cannot be undertaken


until the insect is identified

Venom immunotherapy could reduce the risk for a


severe anaphylaxis on a subsequent sting from more
than 50% to less than 3%
Explanation: Venom immunotherapy is highly effective in
reducing the risk of anaphylaxis. While venom
immunotherapy carries some risks for local and systemic
adverse effects, the benefits outweigh the risks for those at
high risk for anaphylaxis from a subsequent sting. Those at
high risk include any individual with positive results on skin
tests/RAST who experienced a systemic reaction to a sting
with symptoms beyond generalized skin rashes (e.g.,
respiratory, cardiovascular reactions) or those 17 yr of age
and older with systemic reactions confined to the skin
(generalized urticaria). Test results may be negative during
a refractory period in the weeks following the reaction, so
they should be repeated, along with RAST, after 4-6 wk if
they are negative initially. It is not necessary to know
exactly which insect caused the sting before proceeding
with testing and treatment. Although venom immunotherapy
may not be indicated for patients without identifiable IgE to
the venom, in cases of anaphylaxis proximate to a sting,
patients should be equipped with self-administered
epinephrine because the risk for a subsequent anaphylactic
reaction is increased. (See Chapter 140 in Nelson
Textbook of Pediatrics, 17th ed.)

If results of venom skin tests are negative, he does not


need to have self-administered epinephrine readily
available
Question . 66. A 15-yr-old with a history of seasonal hay fever now also has
itchy eyes, profuse tearing, and reddened and edematous conjunctivae. A
treatment option effective for the ocular symptoms would be:

Topical antihistamines

Topical decongestants

Topical mast cell stabilizers

Topical nonsteroidal anti-inflammatory drugs

All of the above?each is an effective secondary


treatment regimen for ocular allergies
Explanation: Allergic conjunctivitis in the patient with hay
fever generally responds well to treatment regimens
including topical application of antihistamines, topical
decongestants, topical mast cell stabilizers, and topical
nonsteroidal anti-inflammatory drugs. Children often
complain of stinging or burning with use of topical
ophthalmic preparations and usually prefer oral
antihistamines for allergic conjunctivitis. (See Chapter 141
in Nelson Textbook of Pediatrics, 17th ed.)

Question . 67. The patient described in Question 66 continues to have


symptoms. The most appropriate next step in management would be:

Combination therapy such as with an antihistamine and a


vasoconstrictive agent

Immunotherapy

Topical corticosteroids

Oral corticosteroids

All of the above?each is an effective tertiary treatment


regimen for ocular allergies
Explanation: Tertiary treatment of ocular allergy includes
topical, or rarely oral, corticosteroids. Local administration
of topical corticosteroids may be associated with increased
intraocular pressure, viral infections, and cataract
formation. Allergen immunotherapy can be very effective in
seasonal and perennial allergic conjunctivitis, especially
when associated with rhinitis. It can decrease the need for
oral or topical medications to control allergy symptoms.
(See Chapter 141 in Nelson Textbook of Pediatrics, 17th
ed.)
Question . 68. All of the following statements concerning allergic reactions to
foods are true except:

Skin tests are of little diagnostic value for cell-mediated


gastrointestinal hypersensitivity

Cow's milk sensitivity is the most common cause of protein-


induced enteropathy

Gastrointestinal anaphylaxis is mediated by IgA


Explanation: Gastrointestinal anaphylaxis generally
presents as acute abdominal pain and vomiting that
accompanies other IgE-mediated allergic symptoms. (See
Chapter 142 in Nelson Textbook of Pediatrics, 17th ed.)

The majority of children with positive results on prick skin


tests to a food will not react when the food is ingested

Elimination diets are the only means to establish the


diagnosis of food allergies

Question . 69. Which of the following is an uncommon clinical manifestation of


food allergies?

Acute urticaria

Angioedema

Wheezing

Diarrhea

Chronic fatigue
Explanation: Chronic fatigue is not recognized to be
caused by food allergies. Acute urticaria and angioedema
(but not chronic urticaria and angioedema), acute
rhinoconjunctivitis, bronchospasm (wheezing), vomiting,
and protracted diarrhea are all manifestations of food
allergies. (See Box 142-1 and Chapter 142 in Nelson
Textbook of Pediatrics, 17th ed.)
Question . 70. All of the following foods are characteristically associated with
allergy except:

Peanuts

Tree nuts

Legumes
Explanation: Peanuts, tree nuts, eggs, and seafood all are
characteristically associated with food allergies. (See
Chapter 142 in Nelson Textbook of Pediatrics, 17th ed.)

Eggs

Seafood

Question . 71. Because of a strong family history on both sides, the parents of
a newborn baby ask for guidance about preventing their child from developing
an allergy to peanuts. Which of the following approaches is recommended?

Begin and extend breast-feeding until age 2 yr, with


exclusion of peanuts from the mother's diet while breast-
feeding

Begin and extend breast-feeding until age 2 yr, with the


mother ingesting gradually increasing amounts of creamy
peanut butter from 18-24 mo of age

Begin and continue breast-feeding as routinely


recommended, with the mother regularly ingesting small
amounts of peanuts but not introducing peanuts in the
child's diet until age 1 yr

Begin and continue breast-feeding as routinely


recommended, excluding peanuts from the mother's
diet while breast-feeding and from the child's diet until
age 3 yr
Explanation: There is no consensus on whether food
allergies can be prevented. However, several authorities
recommend delaying introduction of major food allergens to
infants from atopic families. Recommendations include
promotion of breast-feeding with maternal exclusion of
peanut and nut products from the mother's diet and delay in
introducing major allergenic foods: cow's milk until 1 yr of
age; egg until 18-24 mo of age, and peanuts, tree nuts, and
seafood until 3 yr of age. (See Chapter 142 in Nelson
Textbook of Pediatrics, 17th ed.)

Use only creamy peanut butter and not chunky peanut


butter or whole peanuts in the child's diet (after 1 yr of age)
Question . 72. A 6-mo-old infant develops protracted projectile vomiting, and
lethargy about 2 hr after ingesting a milk formula. The most likely diagnosis is:

Generalized anaphylaxis

Milk-induced enterocolitis syndrome


Explanation: Food protein-induced enterocolitis syndrome
typically manifests in the first several months of life with
irritability, protracted vomiting and diarrhea, not infrequently
resulting in dehydration. Vomiting generally occurs 1-3 hr
following feeding, and continued exposure may result in
bloody diarrhea, anemia, abdominal distention, and failure
to thrive. Symptoms are most commonly provoked by cow's
milk- or soy protein-based formulas but occasionally result
from food proteins passed in maternal breast milk. (See
Chapter 142 in Nelson Textbook of Pediatrics, 17th ed.)

Gastrointestinal anaphylaxis

Allergic eosinophilic esophagitis

Allergic eosinophilic gastroenteritis

Question . 73. Which of the following is the most definitive test for diagnosing
a food protein-induced enterocolitis?

Positive clinical history

Positive food challenge


Explanation: Unfortunately there are no laboratory studies
that help identify foods responsible for cell-mediated
reactions. Consequently, elimination diets followed by food
challenges are the only way to establish the diagnosis.
(See Chapter 142 in Nelson Textbook of Pediatrics, 17th
ed.)

Positive result on skin prick test

Positive RAST result

Quantitative IgE level


IMMUNOLOGY
Question . 1. A 40-day-old, previously healthy, full-term female infant
manifests fever, lethargy, and poor feeding for 12 hr. Physical examination
reveals a lethargic child with vital signs of respiratory rate 70/min, heart rate
185/min, mean blood pressure 25 mm Hg, and temperature 39.5°C. Peripheral
perfusion is poor. The chest examination reveals retractions, the abdomen is
soft and reveals persistence of the umbilical cord, and the extremities are cool.
Laboratory studies reveal a white blood cell count of 67,800/mm3 and a
platelet count of 105,000/mm3. Family history reveals that a male sibling died
suddenly at the age of 2 mo, 10 yr prior to the birth of this child. The most likely
diagnosis is:

Chronic granulomatous disease

Congenital leukemia

Kostmann syndrome

Leukocyte adhesion deficiency


Explanation: This infant has the clinical picture of sepsis
with additional findings of delayed separation of the
umbilical cord, extreme leukocytosis, and a family history of
early childhood death. This pattern is suggestive of a
leukocyte adhesion deficiency, which predisposes to
sepsis.

Neutrophil myeloperoxidase deficiency

Question . 2. Evaluation of immune function should be initiated for otherwise


healthy children with which of the following infections?

A life-threatening bacterial infection (e.g., sepsis,


meningitis)

A systemic fungal infection (e.g., coccidioidomycosis)

Eight or more upper respiratory tract infections within 12


mo

Infection with unusual organisms (e.g., Nocardia)


Explanation: Immune evaluations should be initiated for
children with unusual, chronic, or recurrent infections, such
as (1) two or more systemic or serious bacterial infections;
(2) three or more serious respiratory or documented
bacterial soft tissue infections within 12 mo; (3) infections at
unusual sites; (4) infections with unusual organisms; and
(5) infections with common childhood pathogens but of
unusual severity.

All of the above


Question . 3. A 3-yr-old girl has a history of recurrent, serious skin and soft
tissue infections caused by Staphylococcus aureus and group A
streptococcus. The initial screening laboratory evaluation for possible
immunodeficiency includes all of the following except:

Complete blood count and manual differential

Absolute lymphocyte count

Platelet count

CH50
Explanation: The CH50 assay is a screening test for
complement defects and is not recommended for the initial
evaluation of immunodeficiency unless there is history of
infection with encapsulated organisms. The CBC, platelet
count, and immunoglobulin levels assay are cost-effective
screening tests. (See Chapter 112 in Nelson Textbook of
Pediatrics, 17th ed.)

Immunoglobulin levels

Question . 4. Which of the following is an effective screening test for T-cell


function?

Absolute lymphocyte count

Flow cytometry for CD4 (helper) and CD8 (cytotoxic) T cells

Respiratory burst assay

Candida skin test


Explanation: The Candida skin test is a cost-effective test
of T-cell function. The absolute lymphocyte count and flow
cytometry measure T-cell numbers and subsets but not
function. (See Chapter 112 in Nelson Textbook of
Pediatrics, 17th ed.)

Mumps antibody titer after mumps vaccination


Question . 5. Infections characteristic of T-cell immunodeficiency include all of
the following except:

Pneumocystis carinii pneumonia

Cryptosporidium diarrhea

Severe disseminated chickenpox (varicella)

Serratia marcescens abscess


Explanation: Abscess formation is not characteristic of T-
cell immune deficiency, even though Serratia marcescens
is an unusual cause of infections in children. S.
marcescens abscess suggests chronic granulomatous
disease (CGD). (See Chapter 112 in Nelson Textbook of
Pediatrics, 17th ed.)

Strongyloides stercoralis hyperinfection syndrome

Question . 6. Useful test for evaluation of possible B-cell (antibody) deficiency


include all of the following except:

Isohemagglutinins

Antibodies to tetanus

Flow cytometry for CD3 cells


Explanation: CD3 flow cytometry detects all T cells, which
normally constitute about 70% of peripheral circulating
lymphocytes. The other studies all provide useful
information on B-cell function and antibody production.
(See Chapter 112 in Nelson Textbook of Pediatrics, 17th
ed.)

Serum IgA level

Total IgG level


Question . 7. A 2-yr-old boy is diagnosed with an immune deficiency. There is
no other family history suggestive of immunodeficiency. Which of the following
constitutes appropriate advice for the parents regarding possible future
pregnancies?

Immune deficiencies primarily occur as spontaneous


mutations and therefore no genetic counseling is
necessary.

Inherited immunodeficiencies reflect the complex


interactions of multiple genes with variable expressivity,
and therefore genetic counseling is not accurate.

The absence of other immunodeficiencies in this family


indicates that this immunodeficiency most likely does not
have a genetic component.

This immune deficiency is almost certainly X-linked.

None of the above.


Explanation: Many of the recognized immunodeficiencies
are inherited, and the specific genes for many of these
have been identified. Specific genetic tests are available for
many of these immunodeficiencies. The family history will
often be positive in X-linked immunodeficiency syndromes,
but not all immune deficiencies are X-linked. (See Chapter
112 in Nelson Textbook of Pediatrics, 17th ed.)

Question . 8. Complications of intravenous immunoglobulin (IVIG) therapy


include all of the following except:

Anaphylaxis

Fluid overload

Transmission of HIV

Aseptic meningitis
Explanation: AIDS has not been reported as a
complication of the use of intravenous immunoglobulin
(IVIG) prepared from human donors. Donors are screened
for human immunodeficiency virus, hepatitis B virus, and
hepatitis C virus; furthermore, preparative methods
inactivate these viruses. The remaining choices are rare
but reported complications of IVIG therapy. Nonetheless,
IVIG has been a remarkable aid in the treatment of patients
with congenital antibody deficiency states. (See Chapter
114 in Nelson Textbook of Pediatrics, 17th ed.)

Systemic reactions
Question . 9. A 1-yr-old child is found to have a normal WBC count but no
circulating B cells, small tonsils, and no palpable lymph nodes. Serum
concentrations of IgG, IgA, IgM, and IgE are below the 95% lower limits for
age. The most likely diagnosis is:

Ataxia-telangiectasia

X-linked lymphoproliferative syndrome

DiGeorge syndrome

Common variable immunodeficiency

X-linked agammaglobulinemia
Explanation: X-linked agammaglobulinemia (XLA), or
Bruton agammaglobulinemia, is suggested by low
concentrations of all Ig classes. Affected patients are also
unable to respond to immunizations, unlike infants with
transient agammaglobulinemia of infancy. (See Chapter
114 in Nelson Textbook of Pediatrics, 17th ed.)

Question . 10. The most common defined immunodeficiency disorder is:

Common variable immunodeficiency

Selective IgA deficiency


Explanation: Selective IgA deficiency is the most common
well-defined immunodeficiency disorder, with a frequency
of 0.33%. Most affected persons are healthy, but this
deficiency is occasionally associated with illness. (See
Chapter 114 in Nelson Textbook of Pediatrics, 17th ed.)

X-linked agammaglobulinemia

X-linked lymphoproliferative syndrome

Ataxia-telangiectasia
Question . 11. The X-linked lymphoproliferative (XLP) syndrome is classically
associated with overwhelming infection by which of the following agents?

Epstein-Barr virus
Explanation: The genetic defect for X-linked
lymphoproliferative (XLP) syndrome also predisposes to
severe primary EBV infection, which is associated with 50%
mortality in affected patients. (See Chapter 114 in Nelson
Textbook of Pediatrics, 17th ed.)

Enteroviruses

Catalase-positive bacteria

Neisseria meningitidis

Pneumocystis carinii

Question . 12. Features of the complete DiGeorge syndrome include


susceptibility to infection and:

Neonatal hypocalcemia

Anomalies of the great vessels

Graft versus host disease after blood transfusion with


nonirradiated blood

Micrognathia

Onset of infections after age 12 mo


Explanation: Because T-lymphocyte deficiency is present
at birth, most patients with complete (severe) DiGeorge
syndrome manifest serious infections before 1 yr of age.
Many patients have partial or incomplete DiGeorge
syndrome; these patients may have sufficient lymphocyte
activity to avoid serious infections. (See Chapter 115 in
Nelson Textbook of Pediatrics, 17th ed.)
Question . 13. All of the following statements regarding DiGeorge syndrome
are true except:

It occurs approximately equally in both males and females.

Variable hypoplasia of the thymus and parathyroids is more


common than complete aplasia.

Children with partial DiGeorge syndrome may have little


trouble with infections.

Absolute lymphocyte counts are usually markedly low.


Explanation: In persons with DiGeorge syndrome, the
absolute lymphocyte count is usually only moderately low
for age. Lymphocyte responses to mitogen stimulation are
absent, reduced, or normal, depending on the degree of
thymic deficiency. (See Chapter 115 in Nelson Textbook of
Pediatrics, 17th ed.)

Concentrations of serum immunoglobulins are usually


normal.

Question . 14. All of the following are prominent features of Wiskott-Aldrich


syndrome except:

Atopic dermatitis

Thrombocytopenia

Recurrent infections with encapsulated bacteria

Autosomal dominant inheritance


Explanation: The combination of atopic dermatitis,
thrombocytopenic purpura, and susceptibility to infection in
males is the classic presentation of Wiskott-Aldrich
syndrome, which is an X-linked recessive syndrome. (See
Chapter 116 in Nelson Textbook of Pediatrics, 17th ed.)

More frequent occurrence in males


Question 15. manifestations of hyper IgE syndrome frequently include:

Recurrent pneumonia

Pneumatoceles

Recurrent fractures

Hyperextensible joints

All of the above


Explanation: The hyper IgE syndrome is inherited as an
autosomal dominant trait with variable expressivity that is
associated with recurrent pneumonias and pneumatoceles.
Abnormalities in dentition, bones, and connective tissue are
common. (See Chapter 116 in Nelson Textbook of
Pediatrics, 17th ed.)

Question . 16. The recommended treatment for severe combined


immunodeficiency is:

Gene therapy

Monthly IVIG

Monthly IVIG and IFN-

Monthly IVIG and IL-8 monoclonal antibody

Stem cell transplantation


Explanation: Stem cell transplantation remains the most
important and effective therapy for severe combined
immunodeficiency. These patients do not require
pretransplantation conditioning or prophylaxis of graft
versus host disease. (See Chapter 116 in Nelson Textbook
of Pediatrics, 17th ed.)
Question . 17. All of the following statements regarding monocytes and
neutrophils are true except:

Monocytes, unlike neutrophils, have an unlimited capacity


to divide.

Monocytes remain longer in the circulation than do


neutrophils.

Monocytes in tissues (macrophages) can persist for


months.

Only neutrophils kill organisms by ingestion.


Explanation: Neutrophils and monocytes share many
primary functions, including the ability to ingest organisms.
Neutrophils persist for 6 hr in the circulation. Monocytes
persist for 26-104 hr in the circulation and can persist in
tissues as macrophages for months. (See Chapters 117
and 118 in Nelson Textbook of Pediatrics, 17th ed.)

There are many varieties of mononuclear phagocytes, but


only one type of neutrophil

Question . 18. Monocytes leave the bloodstream and differentiate in the


tissues into macrophages with variable morphology and function. All of the
following cell types are monocyte-derived except:

Dendritic cells

Microglial cells

Kupffer cells

Osteoblasts
Explanation: Tissue macrophages derived from
monocytes include osteoclasts, which are large
multinucleated cells associated with the absorption and
removal of bone. In contrast, osteoblasts arise from
fibroblasts and, when mature, are associated with the
production of bone. (See Chapters 118 in Nelson Textbook
of Pediatrics, 17th ed.)

Multinucleated giant cells


Question . 19. A 6-yr-old girl has had intermittent fever, decreased appetite,
and weight loss since starting first grade 4 mo ago. Physical examination
reveals generalized lymphadenopathy, and a chest film shows prominent para-
aortic nodes. Past medical history is negative for infection except for
Salmonella paratyphi septicemia and liver abscess at 3 yr of age, successfully
treated with antibiotics. Cervical lymph node biopsy shows marked histiocytic
infiltration but no granulomas or giant cells; the acid-fast stain is positive. The
most likely diagnosis is:

Congenital hypogammaglobulinemia

Leukocyte mycobactericidal defect


Explanation: Individuals with inherited deficiency in
macrophage receptors for IFN- or lymphocyte receptors
for IL-12, or in IL-12 itself, suffer a severe, profound, and
selective susceptibility to infection by nontuberculous
mycobacteria such as Mycobacterium avium or bacille
Calmette-Guérin (BCG). About half of these patients have
had disseminated Salmonella infection. These
abnormalities are now grouped under the term leukocyte
mycobactericidal defects. (See Chapters 118 in Nelson
Textbook of Pediatrics, 17th ed.)

Severe combined immunodeficiency disease

Normal child

Langerhans cell histiocytosis

Question . 20. All of the following are typically associated with an eosinophilic
response and eosinophilia except:

Allergic rhinitis

Hypersensitivity drug reactions

Trichinosis (Trichinella spiralis)

Pinworms (Enterobius vermicularis)


Explanation: Eosinophilia is associated with tissue-
invasive helminthic parasites but not parasitic infections
that are solely intraluminal, such as giardiasis (Giardia
lamblia) or pinworm infestation (Enterobius vermicularis).
(See Chapter 119 in Nelson Textbook of Pediatrics, 17th
ed.)

Wiskott-Aldrich syndrome
Question . 21. Delayed separation of the umbilical cord after birth suggests
which of the following types of immune dysfunction?

B-cell defect

T-cell defect

Combined B- and T-cell defect

Phagocyte function defect


Explanation: Children with phagocyte dysfunction may
have a history of delayed separation of the umbilical cord,
frequently also associated with infection of the cord stump.
(See Chapter 120 in Nelson Textbook of Pediatrics, 17th
ed.)

Complement component deficiency

Question . 22. The phagocytic defect of chronic granulomatous disease is:

Defect of transendothelial migration

Inability to ingest microorganisms

Inability to kill some microorganisms


Explanation: Chronic granulomatous disease is
characterized a defect in the generation of microbial
oxygen metabolites. Neutrophils and monocytes have the
ability to ingest but not the ability to kill catalase-positive
organisms. (See Chapter 120 in Nelson Textbook of
Pediatrics, 17th ed.)

Myeloperoxidase deficiency

Excessive formation of H2O2


Question . 23. All of the following are associated with chronic granulomatous
disease (CGD) except:

X-linked and autosomal recessive inheritance

Pyloric outlet obstruction

Aspergillus pneumonia

Perianal abscess

Hypogammaglobulinemia
Explanation: Because of chronic or recurrent infections,
most patients with CGD demonstrate
hypergammaglobulinemia. (See Chapter 120 in Nelson
Textbook of Pediatrics, 17th ed.)

Question . 24. A 5-yr-old boy presents with his third episode of painful cervical
lymphadenitis. Each was treated with incision and drainage, and cultures grew
Staphylococcus aureus. At the age of 2 yr, he required surgical aspiration of a
liver abscess. The most important laboratory test is:

PCR assay for ADA deficiency

Fluorescence assay using dihydrorhodamine 123


Explanation: This boy has a history suggestive of chronic
granulomatous disease. The nitroblue tetrazolium (NBT) is
the classic test for chronic granulomatous disease. The
NBT test measures the neutrophils' ability to generate
superoxide anion and thereby kill ingested bacteria. It is
being rapidly replaced by a more accurate flow cytometry
fluorescence assay using dihydrorhodamine 123 (DHR).
(See Chapter 120 in Nelson Textbook of Pediatrics, 17th
ed.)

MAC-1 assay

Neutrophil count

Bone marrow aspiration


Question . 25. The most likely diagnosis for the patient described in Question
24 is:

Bruton agammaglobulinemia

AIDS

Chronic granulomatous disease


Explanation: Chronic granulomatous disease (CGD) in the
patient described in the question is most commonly
inherited as an X-linked disorder (in approximately two
thirds of cases) and is associated with an absence of
cytochrome b. NBT testing reveals failure to generate
intracellular superoxide anion. (See Chapter 120 in Nelson
Textbook of Pediatrics, 17th ed.)

Kostmann disease

Cyclic neutropenia

Question . 26. Long-term effective therapy for the patient described in


Question 24 is best accomplished with:

Intravenous immunoglobulin

IFN-
Explanation: IFN- increases superoxide anion generation
in vitro and reduces the incidence of new infections. Long-
term use of trimethoprim-sulfamethoxazole may also be
effective in reducing infections. (See Chapter 120 in Nelson
Textbook of Pediatrics, 17th ed.)

IL-2

Bone marrow transplantation

Granulocyte transfusion
Question . 27. Neutropenia is noted in children in all of the following
conditions except:

Kostmann disease

Viral infection

Maternal preeclampsia

Hunter syndrome
Explanation: Transient, benign neutropenia associated
with various non-life-threatening viral infections is the most
common cause of neutropenia in previously healthy
children. Neonatal neutropenia due to alloimmune,
autoimmune, or preeclamptic processes is often
asymptomatic and transient. (See Chapter 121 in Nelson
Textbook of Pediatrics, 17th ed.)

Shwachman-Diamond syndrome

Question . 28. A 5-mo-old girl presents with diarrhea and malabsorption, and
on initial laboratory testing has a WBC count of 900/mm3. Which of the
following is the most likely diagnosis?

Cystic fibrosis

Shwachman-Diamond syndrome
Explanation: Schwachman-Diamond syndrome is an
autosomal recessive disorder characterized by digestive
abnormalities and leukopenia. (See Chapter 121 in Nelson
Textbook of Pediatrics, 17th ed.)

Cyclic neutropenia

Chronic granulomatous disease

Severe combined immunodeficiency


Question . 29. All of the following may be associated with neutropenia except:

Leukocyte adhesion deficiency


Explanation: Persons with leukocyte adhesion deficiency
have impaired transendothelial migration. The circulating
neutrophil count with infection is typically above
30,000/mm3 and can surpass 100,000 per mm3, with a
paucity of neutrophils in the infected tissues. (See Chapter
121 in Nelson Textbook of Pediatrics, 17th ed.)

Shwachman-Diamond syndrome

Cartilage-hair hypoplasia

Chédiak-Higashi syndrome

Glycogen storage disease type Ib

Question . 30. Features associated with cyclic neutropenia include:

An oscillatory period of 21 ± 3 days

Oral ulcerations and stomatitis

Serious infections including pneumonia

Septicemia, notably with Clostridium perfringens

All of the above


Explanation: Severe infections including pneumonia and
life-threatening sepsis, notably with Clostridium perfringens,
may occur. (See Chapter 121 in Nelson Textbook of
Pediatrics, 17th ed.)
Question . 31. A 6-mo-old child presents with recurrent cellulitis and
bacteremia due to Staphylococcus aureus. The white blood cell count is
2500/mm3 with 5% neutrophils, 10% eosinophils, 35% monocytes, and 50%
lymphocytes. The platelet count is 650,000/mm3. A brother and a female
cousin died at the ages of 18 mo and 2 yr, respectively. The most likely
diagnosis is:

AIDS

Severe combined immunodeficiency

Kostmann disease
Explanation: Kostmann disease, an autosomal recessive
severe infantile form of agranulocytosis, manifests with
persistently low absolute neutrophil counts (<200/mm3) and
severe, recurrent, and at times lethal (by age 3 yr) infection.
(See Chapter 121 in Nelson Textbook of Pediatrics, 17th
ed.)

Cyclic neutropenia

Chronic granulomatous disease

Question . 32. Long-term treatment of the disease described in Question 31 is


best accomplished with:

Prophylactic antibiotics

Intravenous immunoglobulin monthly

IFN-

Recombinant human G-CSF


Explanation: Recombinant human granulocyte colony-
stimulating factor (rhG-CSF) is the treatment of choice and
has dramatically improved the neutrophil count while
reducing the incidence and severity of infection. (See
Chapter 121 in Nelson Textbook of Pediatrics, 17th ed.)

Stem cell transplantation


Question . 33. A 12-yr-old boy has a total white blood cell count of
13,000/mm3, with 60% neutrophils, 14% bands, 25% lymphocytes, and 1%
monocytes. Which of the following describes this result?

Neutrophilia

Leukocytosis

Shift to the left


Explanation: Band forms normally constitute 1-5% of the
circulating neutrophils. An increase in band forms is termed
a "shift to the left." The total white count is normal, and
therefore there is not a leukocytosis. The percentage of
neutrophils is within normal limits, and therefore there is not
a neutrophilia. The percentage of lymphocytes is within
normal limits, and the absolute lymphocyte count is
2,800/mm3, and therefore there is not a lymphopenia. (See
Chapter 122 in Nelson Textbook of Pediatrics, 17th ed.)

Leukocytosis and shift to the left

Neutrophilia, shift to the left, and lymphopenia

Question . 34. Repeated meningococcal infections suggest which of the


following types of immune disorder?

B-cell defect

T-cell defect

Combined B- and T-cell defect

Phagocyte function defect

Complement component deficiency


Explanation: Congenital deficiencies of the terminal
components of complement (especially C5, C6, C7, or C8)
have been associated with repeated meningococcal and
extragenital gonococcal infections. (See Chapter 124.2 in
Nelson Textbook of Pediatrics, 17th ed.)
Question . 35. A 10-yr-old boy undergoes stem cell transplantation with stem
cells harvested from his 14-yr-old sister. Which of the following terms
describes this type of transplantation?

Autologous

Syngeneic

Familiogeneic

Allogeneic
Explanation: Cells for autologous transplantation (after
removal of cancer cells) are obtained from the patient. Cells
for syngeneic transplantation are obtained from an identical
twin. Cells for allogeneic transplantation are obtained from
a nonidentical person. (See Chapter 125 in Nelson
Textbook of Pediatrics, 17th ed.)

Xenogeneic

Question . 36. A 4-mo-old girl has severe persistent neutropenia (absolute


neutrophil count 200 cells/mm3), thrombocytopenia (platelet count
<20,000/mm3), and reticulocytopenia (reticulocyte count <1% when anemia is
present). She has an HLA-matched family donor. The treatment of choice is:

IFN-

IFN- and IL-8

IFN- (, platelet-derived growth factor, and monthly IVIG

Recombinant human G-CSF, platelet-derived growth factor,


and monthly (as needed) red blood cell transfusions

Stem cell transplantation


Explanation: Stem cell transplantation is the treatment of
choice for patients with severe aplastic anemia who have
an HLA-matched family donor. (See Chapter 125 in Nelson
Textbook of Pediatrics, 17th ed.)
Question . 37. The most important factor contributing to graft failure and graft
rejection is:

HLA disparity
Explanation: HLA disparity is the most important variable
influencing graft failure and graft rejection. (See Chapter
126 in Nelson Textbook of Pediatrics, 17th ed.)

Pretransplantation alloimmunization by transfusions

The conditioning regimen

Transplanted stem cell dose

Post-transplantation viral infections

Question . 38. A 9-yr-old boy undergoes stem cell transplantation for acute
myelogenous leukemia in remission. Approximately 4 mo after transplantation,
he has persistent maculopapular rash over 25% of his body, generalized
erythroderma, bilirubin 8.4 mg/dL, and diarrhea with >1,000 mL of feces per
day. He complains of xerostomia. The most likely diagnosis is:

Post-transplantation Sj gren syndrome

Acute graft versus host disease

Chronic graft versus host disease


Explanation: Chronic graft versus host disease resembles
a multisystem autoimmune process with maculopapular
rash, hyperbilirubinemia, and diarrhea, usually after 100
days after transplantation. (See Chapter 127 in Nelson
Textbook of Pediatrics, 17th ed.)

Acute cytomegalovirus infection

Reactivated cytomegalovirus infection


Question . 39. Which of the following statements regarding cyclosporine is
true?

Adverse effects, other than immunosuppression, are


uncommon.

Cyclosporine should generally be used with tacrolimus for


synergy.

Cyclosporine is as effective as methotrexate for post-stem


cell transplantation immunosuppression.

The only significant drug interaction requiring dosage


adjustment is with ketoconazole.

The dosage should be adjusted regularly based on


drug levels.
Explanation: The renal toxicity associated with
cyclosporine can be reduced by adjusting dosing based on
blood cyclosporine levels. (See Chapter 128 in Nelson
Textbook of Pediatrics, 17th ed.)

Question . 40. Well-recognized late effects of total body irradiation as part of


the preparative regimen for stem cell transplantation include all of the following
except:

Leukoencephalopathy

Secondary malignancies

Hyperthyroidism
Explanation: The use of total body irradiation (TBI) with or
without additional conventional irradiation involving the
thyroid gland, may result in hypothyroidism, not
hyperthyroidism. (See Chapter 129 in Nelson Textbook of
Pediatrics, 17th ed.)

Growth depression

Cataracts
Question . 1. A 3-yr-old girl has fever. Urine is collected by catheterization for
culture. What is the minimum concentration of bacterial growth in the culture that
would generally be considered diagnostic of a urinary tract infection in this young
girl?

1 bacterium/mL

10 bacteria/mL

103 bacteria/mL
Explanation: Urine colony counts of 103 organisms/mL or greater
are generally considered significant in urine collected by
catheterization, and of 105 organisms/mL or greater in urine collected
by clean-catch (midstream) void. Urine obtained by suprapubic
puncture should normally be sterile; any bacterial growth is
considered significant. (See Chapter 160 in Nelson Textbook of
Pediatrics, 17th ed.)

105 bacteria/mL

None of the above: urine must be obtained by suprapubic aspiration


to be considered diagnostic in children of this age

Question 2. 2. Which of the following defines the concept of bacterial tolerance?

MIC in vivo >4 times the MIC in vitro

MBC in vivo greater than the MIC in vivo

MIC in vivo >2 times the MIC in vitro

MIC >4 times the MBC

MBC >4 times the MIC


Explanation: A mean bactericidal concentration (MBC) > 4 times the
minimum inhibitory concentration (MIC) defines bacterial tolerance.

Question 3. 3. A 14-yr-old boy in Connecticut has an illness characterized by fever


and an erythematous rash. An initial Borrelia burgdorferi IgG antibody titer was 1:32.
Repeat antibody titer 5 wk later is 1:64. The most likely interpretation of these
serologic tests is:

Acute Lyme disease

Atypical Lyme disease

Relapsing disease
Syphilis

None of the above


Explanation: These two antibody titers are not different and reflect
past Lyme disease, not acute Lyme disease. Serologic tests are
performed on serial twofold dilutions of serum. The actual titer is
probably between 1:32 and 1:64 in both sera. To confirm acute
infection using IgG testing, it is necessary to demonstrate either
seroconversion (i.e., from seronegative to seropositive) or a
significant rise in IgG titer; a fourfold increase in a convalescent titer
obtained 2-3 wk after the acute titer indicates a significant change in
titer and is considered diagnostic in most situations.

Question . 4. Amoxicillin was begun 4 days ago for treatment of otitis media
in a 4-yr-old boy. He now presents with signs of meningitis and is found to
have a pleocytosis with126 leukocytes/mm3. The diagnostic stain of the
cerebrospinal fluid that would be most useful is:

Gram stain
Explanation: The Gram stain remains the most useful
diagnostic stain for bacterial meningitis. It is a rapid,
inexpensive method for demonstrating the presence of
bacteria and fungi. (See Chapter 160 in Nelson Textbook of
Pediatrics, 17th ed.)

Methenamine silver stain

Kinyoun stain

Trichrome stain

Wright stain

Question 5. 5. A 12-yr-old girl has a positive throat culture for group A streptococci.
Which of the following antimicrobial susceptibility tests should be performed?

Bauer-Kirby (agar disk diffusion)


E-test
Minimum inhibitory concentration (MIC)
Minimum inhibitory concentration (MIC) and minimum bactericidal
concentration (MBC)
None of the above
Explanation: Group A streptococci have a predictable antimicrobial
susceptibility pattern and are universally susceptible to penicillin.
Bacitracin susceptibility is generally used to identify the organisms as
group A streptococci. Susceptibility testing specifically to penicillin is
not necessary.

Question 6. 6. A 17-yr-old boy is suspected of having an acute infection. Serologic


testing of acute and convalescent sera, taken 4 wk apart, is performed at a reference
laboratory for antibodies against the suspected organism. All of the following
results support recent infection except:

Positive IgM antibody titer and negative IgG antibody titer

Positive IgM antibody titer and positive IgG antibody titer

A fall in IgG titer from 1:64 to 1:8

A rise in IgG titer from 1:64 to 1:128


Explanation: A positive IgM antibody titer, seroconversion, or a 2-
tube (fourfold) difference in antibody titers supports recent infection.
Depending on the timing of samples relative to the onset of infection,
the IgG antibody titers may demonstrate seroconversion (from
seronegative to seropositive), a fourfold increase, or a fourfold
decrease if taken later. The titers of 1:64 and 1:128 are only 1 tube
(twofold) different; this may reflect the interpretation of a titer that is
between the two dilutions tested. Because this difference is only 1
tube (twofold), this difference does not necessarily indicate a change.
(See Chapter 160 in Nelson Textbook of Pediatrics, 17th ed.)

Absence of IgG antibody titer in the acute serum and IgG antibody
titer of 1:16 in the convalescent serum

Question 7. 7. Most evidence suggests that fever is an adaptive response, and that
antipyretics provide symptomatic relief but do not change the course of diseases
except in selected circumstances. Patients with which of the following underlying
conditions should be treated with antipyretics for fever as an essential part of
treatment?

Osteomyelitis

Diabetes mellitus

Inflammatory bowel disease

Chronic lung disease


Explanation: Antipyretic therapy is directly beneficial in high-risk
patients who have chronic cardiopulmonary diseases, metabolic
disorders, or neurologic diseases. (See Chapter 161 in Nelson
Textbook of Pediatrics, 17th ed.)
Meningitis

Question 8. 8. Which of the following is a periodic fever syndrome?

Rat bite fever

Colorado tick fever

Cyclic neutropenia
Explanation: Periodic fever is used to narrowly describe fever
syndromes with a regular periodicity, such as cyclic neutropenia, and
the syndrome of periodic fever, aphthous stomatitis, pharyngitis, and
adenopathy (PFAPA). (See Chapter 161 in Nelson Textbook of
Pediatrics, 17th ed.)

Hectic fever

Marburg fever

Ques.9.A 4-yr-old girl has a temperature of 41.8°C. Which of the following is the most
likely cause?

Cerebellar disorders

Hypothalamic disorders
Explanation: Temperatures in excess of 41°C are most often
associated with a noninfectious cause. Causes of very high
temperatures (>41°C) include central fever (resulting from central
nervous system dysfunction involving the hypothalamus), malignant
hyperthermia, malignant neuroleptic syndrome, drug fever, or
heatstroke. (See Chapter 161 in Nelson Textbook of Pediatrics, 17th
ed.)

Excessive sweating

Ventricular shunt malfunction

Most bacterial infections

10. A 16-yr-old boy who has recently traveled to sub-Saharan Africa is diagnosed
with Plasmodium vivax malaria. The fever pattern is one of febrile periods every
other day. The term that best characterizes this fever pattern is:

Hectic fever

Remittent fever
Biphasic fever

Tertian fever
Explanation: Tertian fever occurs on the 1st and 3rd days (e.g.,
malaria caused by Plasmodium vivax), and quartan fever occurs on
the 1st and 4th days (e.g., malaria caused by Plasmodium malariae).
(See Chapter 161 in Nelson Textbook of Pediatrics, 17th ed.)

Periodic fever

12. An 8-yr-old boy with sickle cell disease presents with fever and symptoms of an
acute illness. You consider treating him as an outpatient with intramuscular
ceftriaxone. Which of the following findings should lead to hospitalization?

Temperature > 40°C

WBC count < 5,000 cells/µL

WBC count > 30,000>cells/µL

Pulmonary infiltrates

Any of the above


Explanation: Febrile children with sickle cell disease are at an
increased risk for overwhelming sepsis. Any of the manifestations
listed requires inpatient management for presumed bacterial sepsis.
The combination of splenic hypofunction and a deficiency of the
properdin system of complement activation places patients with
sickle cell disease at even greater risk for bacteremia from
encapsulated organisms.

13. Pyelonephritis is a frequent cause of fever in young infants. All of the following
patient groups have an increased risk of urinary tract infections except:

Infant girls

Uncircumcised infant boys

Infants with urinary tract anomalies

Infants with Mediterranean ancestors


Explanation: Ancestry rarely predisposes to urinary tract infections,
whereas the other choices are all significant risk factors. (See Chapter
162 in Nelson Textbook of Pediatrics, 17th ed.)

Infants with vesicoureteral reflux


14. All of the following organisms are recognized as an important cause of occult
bacteremia of infants and children except:

Haemophilus influenzae type b

Moraxella catarrhalis
Explanation: Occult bacteremia (bacteremia without an obvious
focus of infection) due to Streptococcus pneumoniae, H. influenzae
type b, N. meningitidis, or nontyphoidal Salmonella occurs in
approximately 4% of relatively well-appearing children between 3 and
36 mo of age with fever (rectal temperature >38.0°C). S. pneumoniae
accounts for 90% of cases of occult bacteremia. (See Chapter 162 in
Nelson Textbook of Pediatrics, 17th ed.)

Neisseria meningitidis

Salmonella

Streptococcus pneumoniae

15. A 22-day-old infant is noted by his mother to have a rectal temperature of 38.3°C.
She reports that he has been acting normal and appears generally well. He was born
at 37 wk of gestation, went home with his mother at 24 hr of life, and has done well
since. There is no known underlying illness. The physical examination is normal.
The infant's WBC count is 19,500/mm3 and the absolute band count is 850/mm3.
There are 4 WBCs/mm3 in an unspun urine sample, and results of a Gram stain of
the urine are negative. This infant fails to meet criteria for low risk of serious
bacterial infection because:

He was born prematurely.

The WBC count is elevated.


Explanation: Infants younger than 3 mo with fever who appear
generally well; who have been previously healthy; who have no
evidence of skin, soft tissue, bone, joint, or ear infection; and who
have a total white blood cell (WBC) count of 5,000-15,000/µL, an
absolute band count of <1,500/µL, and normal urinalysis results are
unlikely to have a serious bacterial infection. The negative predictive
value with 95% confidence of these criteria for any serious bacterial
infection is >98%, and for bacteremia, >99%. Neonates are at higher
risk of sepsis and meningitis caused by group B streptococci and
other organisms. (See Chapter 162 in Nelson Textbook of Pediatrics,
17th ed.)

The absolute band count is elevated.

There is a urinary tract infection.


He is less than 1 mo of age.

16. A 10-mo-old black male infant from an urban area presents to the emergency
department with a temperature of 39.3°C and an unremarkable physical examination.
Which of the following is a risk factor for occult bacteremia in this child?

Absence of signs of pneumonia on physical examination

Race

Socioeconomic status

Sex

Fever
Explanation: Socioeconomic status, race, gender, and age do not
appear to affect the risk for occult bacteremia among children 3-36
mo of age.

17. The triage history of a 9-mo-old child presenting with a temperature of 39.2°C
shows the immunization history as "up-to-date." Further review of the medical
record shows that the patient has had three doses of Hib conjugate vaccine. Of the
following immunizations, which is the most important to document before
determining any management decisions?

Hepatitis B

DTaP

IPV

Pneumococcal conjugate vaccine


Explanation: "Up-to-date" is not a satisfactory immunization history
for a 6-mo-old infant presenting with a temperature of 39°C.
Conjugate pneumococcal vaccination is recommended for all children
beginning at 2 mo of age and decreases the risk for occult
bacteremia caused by S. pneumoniae. (See Chapter 162 in Nelson
Textbook of Pediatrics, 17th ed.)

Meningococcal conjugate vaccine

18. The laboratory calls at 8:45 A.M. to inform you that a blood culture obtained in the
emergency department yesterday from a 10-mo-old febrile girl who is followed in your
practice is growing Streptococcus pneumoniae. You contact her parents, and they report
that she is "doing well." The most appropriate next step in management would be to:

Instruct the family that you will telephone a prescription for amoxicillin
to the pharmacy, and that they should pick up the prescription and
start treating their child this morning.

Inform the parents that the blood culture was positive and that they
should have the child return to your office if new symptoms develop.

Instruct the parents that they should have the child return to your
office if new symptoms develop; in addition, since the medical record
shows that she has been immunized against S. pneumoniae and has
received the three doses recommended by this age, you should
report the vaccine failure to the state health department.

Instruct the parents to bring the child to your office as soon as


possible to be re-examined.
Explanation: If S. pneumoniae is found in the initial blood culture,
the child should return for re-evaluation as soon as possible once the
culture results are known. (See Chapter 162 in Nelson Textbook of
Pediatrics, 17th ed.)

Instruct the parents to call 911 and have their child taken to the
emergency department immediately.

20. A 1-yr-old child presents with a history of recurrent fevers for several weeks. On
physical examination you find that the pupils fail to constrict. This finding is associated with:

Toxoplasmosis

Ectodermal dysplasia

Juvenile rheumatoid arthritis

Hypothalamic dysfunction
Explanation: Fever of unknown origin is sometimes due to
hypothalamic dysfunction. A clue to this disorder is failure of pupillary
constriction due to absence of the sphincter constrictor muscle of the
eye. This muscle develops embryologically when the hypothalamus
is also undergoing differentiation. (See Chapter 162 in Nelson
Textbook of Pediatrics, 17th ed.)

Thyrotoxicosis

22. Thirty minutes after admission, an 18-yr-old girl with meningococcemia has just finished
receiving the last of her 60 mL/kg infusion of normal saline. The nurse tells you that the
patient's heart rate is still 120/min, that her blood pressure is 70/30 mm Hg, and that pink,
frothy material is being suctioned from the patient's endotracheal tube. The patient's
extremities are warm, with bounding pulses. The next step in the resuscitation is to:

Administer 20 mL/kg of lactated Ringer's solution


Administer 10 mL/kg of 25% albumin

Administer NaHCO3

Administer nitroprusside

Administer norepinephrine
Explanation: This patient is exhibiting signs of fluid overload-
pulmonary edema-and therefore should be given pharmacologic
therapy to restore her blood pressure. Because the patient's pulses
are good and she is well perfused, a pressor is indicated. (See
Chapter 163 in Nelson Textbook of Pediatrics, 17th ed.)

24. In the case described in Question 23, the most appropriate antibacterial regimen to use
until culture results are known would be:

Ceftriaxone

Ceftriaxone and clindamycin

Piperacillin and gentamicin

Ampicillin and gentamicin

Ceftriaxone and vancomycin


Explanation: The cloudy cerebrospinal fluid indicates meningitis. In
this age group S. pneumoniae should be considered as a cause, and
therefore vancomycin in addition to ceftriaxone is indicated until
antimicrobial susceptibilities are known. The vaccination history for S.
pneumoniae should be elicited, which provides some indication of the
probability of S. pneumoniae infection. However, even completed
immunization does not provide absolute protection to the vaccine
serotypes, and the vaccine does not provide protection against all
serotypes.
26. The laboratory results for the patient described in Question 25 are significant for BUN
20 mg/dL, creatinine 1.5 mg/dL, HCO3 16 mmol/L, hematocrit 30%, WBC count 4,500/mm3
with differential count pending, platelets 100,000/mm3, prothrombin time (PT) 20 sec, AST
550 U/L, and ALT 1,200 U/L. As you are studying these results the baby has a focal
seizure, which resolves spontaneously. However, the nurse reports that the patient is
intermittently apneic. The most appropriate next step in the management would be to:

Obtain a stat head CT study and ophthalmology consultation.

Intubate the patient and add acyclovir to the antibacterial


regimen.
Explanation: Focal seizures in a febrile neonate with evidence of
systemic shock and hepatic dysfunction suggest the possibility of
systemic herpes simplex virus infection. Depressed mental status,
airway compromise due to seizures, and the need for preservation of
cardiorespiratory status in shock dictate intubation in this patient.
(See Chapter 163 in Nelson Textbook of Pediatrics, 17th ed.)

Consult gastroenterology for possible Reye syndrome and obtain a


serum ammonia level.

Consult nephrology and measure serum phosphate and ionized


calcium.

Perform a lumbar puncture prior to administering antibacterial agents.

27. A 6-mo-old girl with streptococcal bacteremia admitted earlier to the PICU is now
unresponsive, hypotensive, and tachycardic. She is intubated, sedated, and receiving the
last part of her third 20 mL/kg normal saline bolus. Her blood pressure is 120/70 mm Hg,
and her heart rate is 120/min. Despite some improvement in peripheral pulses, her
peripheral extremities remain cool, with delayed capillary refill. The recommended next
step in management of this girl is:

Administration of 20 mL/kg of 5% albumin

Administration of 20 mL/kg hetastarch

Administration of 20 mL/kg normal saline and institution of


milrinone
Explanation: This patient exhibits what used to be called "cold
shock." Compensatory mechanisms include peripheral
vasoconstriction, which conserves blood flow to central organs in
addition to maintaining blood pressure. There may also be a
component of myocardial dysfunction due to cytokines released
during the SIRS response. These patients are best served by an
agent that has inotropic as well as afterload-reducing properties.
(See Chapter 163 in Nelson Textbook of Pediatrics, 17th ed.)

Administration of 20 mL/kg normal saline and institution of dopamine

Administration of 20 mL/kg normal saline and institution of labetalol

28. Which of the following statements concerning infections in immunocompromised


persons is true?

Fever is an insensitive sign of infection.

Normal skin flora can cause life-threatening infection.


Explanation: In immunocompromised persons, fever is a sensitive
and specific sign of infection. Almost any organism can cause severe
and even life-threatening infection. Absolute neutrophil counts of
<500/mm3 are predictive of infection, with the risk directly
proportional to the duration and depth of neutropenia. Multiple
infections, either concomitant or sequential, are common. (See
Chapter 164 in Nelson Textbook of Pediatrics, 17th ed.)

Absolute neutrophil counts of <5000 cells/mm3 are predictive of


infection.

The risk of serious infection is highest in the first 48 hr of


neutropenia.

Multiple infections are uncommon.

29. A 4-yr-old boy who was previously well is found to have a Staphylococcus aureus
hepatic abscess. His maternal uncle died during childhood from "an infection" that was also
thought to be caused by S. aureus. Of the following, which is the most likely diagnosis?

Congenital neutropenia

Cyclic neutropenia

Severe combined immunodeficiency syndrome

Chronic granulomatous disease


Explanation: S. aureus is the most common organism causing
infections, including liver abscess, that are associated with chronic
granulomatous disease. These infections usually respond well to
intravenous antimicrobial therapy. (See Chapter 164 in Nelson
Textbook of Pediatrics, 17th ed.)

None of the above

30. The boy described in Question 29 should receive specific antibiotic therapy directed
against S. aureus and also:

Granulocyte colony-stimulating factor

Interferon gamma and trimethoprim-sulfamethoxazole


Explanation: The management of chronic granulomatous disease
includes lifelong treatment with interferon- and also trimethoprim-
sulfamethoxazole, as well as additional antimicrobial therapy as
needed for acute infections. (See Chapter 164 in Nelson Textbook of
Pediatrics, 17th ed.)

Intramuscular immunoglobulin
Cyclosporine

Amphotericin B

31. A 13-yr-old girl with hyperthyroidism has chronic infection of her skin and nails with
Candida albicans. She has never required hospitalization or had life-threatening infection.
She had chickenpox at age 11 mo from which she had an uneventful recovery, and has
received her other routine childhood vaccinations without problems. Which of the following
tests often yields an abnormal result in a child with this condition?

Antibody level against varicella zoster virus

Antibody level against H. influenza type b

Neutrophil count

Delayed type hypersensitivity testing against Candida


Explanation: This girl has mucocutaneous candidiasis. This is often
associated with an absent delayed-type hypersensitivity response to
skin testing with Candida, despite chronic infection. (See Chapter 164
in Nelson Textbook of Pediatrics, 17th ed.)

Total IgG level


32. A 2-yr-old boy presents in late summer with fever, lethargy, and a stiff neck. CBC
shows a hemoglobin of 13.2 g/dL, platelet count of 323,000/mm3, and white blood cell
count of 14,200/mm3 with a differential of 32% granulocytes, 3% bands, 56% lymphocytes,
and 9% mononuclear cells. A lumbar puncture CSF specimen contains 230
leukocytes/mm3 with 80% lymphocytes and 20% granulocytes. The CSF glucose is 60
mg/dL (blood glucose is 100 mg/dL), and CSF protein is 55 mg/dL. Gram stain shows
moderate white blood cells and no organisms. After 4 days the boy's clinical status remains
unchanged. Results of bacterial cultures of the blood, urine, and CSF are negative, and a
polymerase chain reaction assay of the CSF for herpes simplex virus is negative. The
patient's past medical history reveals that he has had many ear infections and required
bilateral myringotomy tubes at age 13 mo. He was hospitalized once for pneumonia but
responded well to antibiotics.Which of the following immunologic laboratory abnormalities
would be most consistent with this child's history?

Very low to undetectable IgG levels


Explanation: This boy is most likely to have X-linked
agammaglobulinemia, with low to undetectable levels of IgG. His
current illness is probably an enterovirus meningoencephalitis. (See
Chapter 164 in Nelson Textbook of Pediatrics, 17th ed.)

Abnormal result on NBT test

Abnormal neutrophil killing of S. aureus

Very low to undetectable levels of CD4 cells


Elevated levels of IgE

33. A child with fever is found to have a white blood cell count of 1,400/mm3 with a
differential count of 10% neutrophils, 2% band forms, 85% lymphocytes, and 3%
mononuclear cells. Which of the following scenarios indicate the greatest risk for this child
to have a new serious bacterial complication?

The child is a previously healthy 3-yr-old who attends out-of-home


daycare. He had a 5-day history of rhinorrhea, congestion, and
maculopapular rash. A number of classmates have likewise been ill.

The child has leukemia and received chemotherapy 20 days ago. His
WBC count 2 days ago was 800/mm3, with 1 % neutrophils, 0% band
forms, 65% lymphocytes, and 44% mononuclear cells.

The child has leukemia and received chemotherapy 7 days ago.


His WBC count 2 days ago was 3,400/mm3, with 19%
neutrophils, 8% band forms, 65% lymphocytes, 4% mononuclear
cells, and 4% atypical lymphocytes.
Explanation: The child with neutropenia whose counts are on the
way down is at higher risk for bacterial complications of
chemotherapy-associated neutropenia than a child whose counts are
recovering or one who is neutropenic because of bone marrow
suppression from viruses or medications or cyclic neutropenia. (See
Chapter 164 in Nelson Textbook of Pediatrics, 17th ed.)

The child was recently diagnosed with a seizure disorder and was
started on carbamazepine (Tegretol) 2 weeks ago.

The child has monthly recurrence of aphthous ulcers.

34. A 4-yr-old girl underwent liver transplantation 4 mo ago after liver failure from biliary
atresia. She has done relatively well, with only one episode of acute cellular rejection at 1
mo after transplantation. She is currently on cyclosporine and prednisone given orally. She
takes no other medications. She presents to your office with fever of 3 days' duration,
tachypnea, and cough. The clinical examination reveals a moderately ill child with
increased respiratory effort and perioral cyanosis. The lungs are clear on auscultation.
Chest radiograph shows a diffuse interstitial infiltrate White blood cell count is 5,400/mm3
with a differential count showing 56% neutrophils, 3% band forms, 37% lymphocytes, and
4% eosinophils. Bronchoalveolar lavage is performed, and a diagnosis is established.
Which of the following is the least likely pathogen in this patient?

Cytomegalovirus

Pneumocystis carinii

S. pneumoniae (pneumococcus)
Explanation: Although pneumococcal pneumonia can occur in both
immunocompetent and immunocompromised persons, unusual
pathogens are more likely in a post-transplantation patient receiving
immunosuppressive therapy. (See Chapter 164 in Nelson Textbook of
Pediatrics, 17th ed.)

Adenovirus

Parainfluenza virus

35. A 6-yr-old boy with acute lymphocytic leukemia has had a central venous catheter
(Hickman catheter) in place for 1 yr to facilitate administration of chemotherapy. He was
brought into the clinic with a temperature of 39°C. His mother reports that he was well until
an hour after she routinely flushed his line with heparin, when he developed the fever and
had rigors. Aside from increased heart rate, his vital signs and findings on physical
examination are normal. He is not neutropenic. The most appropriate approach to
management of this child is:

Immediately removing the Hickman catheter and beginning therapy


with vancomycin and ceftazidime

Obtaining blood cultures via the catheter lumen and a peripheral


vein followed by initiation of antibiotic therapy with vancomycin
and ceftazidime
Explanation: The risk of catheter-associated infection is significant.
Antibiotics should be administered after blood cultures are obtained.
Antibiotic treatment is successful for most systemic bacterial
infections without removal of the device. Antibiotic therapy should be
directed to the isolated pathogen and given for a total of 10-14 days.
Until identification and susceptibility testing are available, empirical
therapy with a third-generation cephalosporin or aminoglycoside plus
vancomycin is indicated. (See Chapter 165 in Nelson Textbook of
Pediatrics, 17th ed.)

Obtaining blood cultures via the catheter lumen and a peripheral


vein, with follow-up in the clinic the next day and subsequent
treatment based on culture results

Administering urokinase through the Hickman catheter

Admitting the patient to the hospital for observation, with no antibiotic


therapy as long as the cardiovascular status is stable

36. A 9-yr-old boy with cancer had an indwelling Broviac catheter placed 8 mo ago for
cancer chemotherapy. He presents with fever and erythema over the catheter tunnel,
without other localizing symptoms. Blood culture specimens are taken. Which of the
following antibiotics is recommended?
Nafcillin

Cephalexin

Ceftriaxone

Clindamycin

Vancomycin
Explanation: Vancomycin is the choice for empirical treatment of
indwelling line and prosthesis infections caused by coagulase-
negative staphylococci. Most coagulase-negative staphylococci are
resistant to nafcillin. (See Chapter 165 in Nelson Textbook of
Pediatrics, 17th ed.)

37. An 18-mo-old infant has a CSF shunt in place because of congenital hydrocephalus.
He underwent a surgical revision of the shunt 4 mo ago because of malfunction. He
presents to the emergency department with increased irritability and decreased appetite of
3 days' duration. The rectal temperature is 38.7°C. There are no focal signs of infection.
The most appropriate management of this child would be:
If the WBC count is normal, discharge the child with instructions to
take acetaminophen every 4 hr for the fever.

Obtain a blood culture specimen and a CSF culture specimen via a


lumbar puncture.

Obtain a blood culture specimen and a CSF culture specimen


via a tap of the CNS shunt.
Explanation: Culture of CSF obtained from the shunt is the most
useful test to diagnose a CSF shunt infection. (See Chapter 165 in
Nelson Textbook of Pediatrics, 17th ed.)

Perform a radionuclide dye study to determine if the shunt is


functioning properly.

Begin oral antibiotic therapy with amoxicillin/clavulanate.

38. A 9-yr-old boy presents with fever >39°C for 4 days, myalgias, watery diarrhea,
conjunctival infection, diffuse erythroderma, strawberry tongue, blood pressure of 105/45
mm Hg, and moderately elevated hepatic transaminases. The most likely diagnosis is:

Staphylococcal scalded skin syndrome

Kawasaki disease

Toxic shock syndrome


Explanation: Toxic shock syndrome and Kawasaki disease share
many features, but Kawasaki disease is not accompanied by
hypotension and shock. Kawasaki disease also typically occurs in
children younger than 5 yr. Toxic shock syndrome can complicate
focal infections caused by TSST-1-producing strains of
Staphylococcus aureus. (See Chapter 166.2 in Nelson Textbook of
Pediatrics, 17th ed.)

Stevens-Johnson syndrome

Toxic epidermal necrolysis

39. Strains of Staphylococcus aureus can produce which of the following toxins?

Exfoliatin A and B

Hemolysins

Enterotoxins A through E

Toxic shock syndrome toxin-1 (TSST-1)

All of the above


Explanation: Many strains of S. aureus produce toxins. Exfoliatins A
and B are associated with localized (e.g., bullous impetigo) or
generalized (e.g., scalded skin syndrome) eruptions. Hemolysins act
on cell membranes. Staphylococcal enterotoxins A, B, C1, C2, D, and
E are associated with food poisoning. TSST-1 is the toxin of toxic
shock syndrome.

40. A 16-yr-old girl experiences abrupt onset of high fever, vomiting, and diarrhea, with a
diffuse sunburn-like rash, hyperemia of the pharyngeal and conjunctival membranes,
oliguria, and postural hypotension. The most likely diagnosis is:

Hemolytic-uremic syndrome

Kawasaki disease

Rocky Mountain spotted fever

Toxic shock syndrome


Explanation: The diagnosis of toxic shock syndrome is based on the
clinical manifestations. In severe cases, the hypotension may
progress to shock. Kawasaki disease is uncommon after age 5 yr, is
not as severe or as progressive, and is not associated with
hypotension or diarrhea. (See Chapter 166.2 in Nelson Textbook of
Pediatrics, 17th ed.)
Stevens-Johnson syndrome

41. Recommended therapy for the patient described in Question 40 is:

Renal dialysis

IVIG

Doxycycline

An antistaphylococcal antibiotic
Explanation: Treatment of toxic shock includes an
antistaphylococcal antibiotic and aggressive intravenous fluid therapy
to prevent or treat hypotension. Corticosteroids and intravenous
immunoglobulin are reserved for severe cases. (See Chapter 166.2 in
Nelson Textbook of Pediatrics, 17th ed.)

Corticosteroids

42. Which of the following organisms characteristically produce(s) an exopolysaccharide


protective biofilm (slime layer)?

Proteus mirabilis

Coagulase-negative staphylococci
Explanation: Slime produced by coagulase-negative staphylococci
surrounds the organism, resists phagocytosis, and enhances
adhesion to foreign surfaces, including indwelling catheters and
prostheses. (See Chapter 166.3 in Nelson Textbook of Pediatrics, 17th
ed.)

Group A streptococci

Haemophilus influenzae

Moraxella catarrhalis
43. Coagulase-negative staphylococci are commonly associated with all of the following
except:

Toxic shock syndrome


Explanation: Coagulase-negative staphylococci are associated with
nosocomial neonatal infections, urinary tract infections associated
with urinary catheters or following urinary tract surgery, infections of
central venous catheters and cerebrospinal fluid shunts. Toxic shock
syndrome is caused by TSST-producing Staphylococcus aureus.
(See Chapter 166.3 in Nelson Textbook of Pediatrics, 17th ed.)

Neonatal bacteremia
Urinary tract infection

Intravascular catheter-associated infection

44. A 12-yr-old child is being treated with nafcillin for Staphylococcus aureus bacteremia.
However, cultures of specimens taken 4 days after presentation remain positive. Which of
the following is the least likely cause of the positive cultures?
Antimicrobial resistance to nafcillin

Endocarditis

Septic thrombophlebitis

An undrained abscess

Penicillin allergy
Explanation: Persistent Staphylococcus aureus bacteremia
suggests the possibility of antimicrobial resistance, an endovascular
focus such as endocarditis, or focal infection that requires drainage.
45. A throat culture from a 16-yr-old boy with osteosarcoma and a Broviac catheter grows
coagulase-negative staphylococci. The recommended course of management is:

Intravenous nafcillin

Vancomycin mouthwashes

Intravenous vancomycin

Removal of the Broviac catheter and administration of intravenous


vancomycin

None of the above: no treatment is indicated


Explanation: Coagulase-negative staphylococci are part of the
normal flora of the skin, throat, mouth, vagina, and urethra. In the
absence of specific symptoms, no treatment is indicated.
46. In regions where penicillin-resistant pneumococci are prevalent, patients who are
strongly suspected of having bacterial meningitis should receive which of the following
antimicrobial agents added to ceftriaxone or cefotaxime as empirical antimicrobial therapy?

Clindamycin

Azithromycin

Vancomycin
Explanation: Vancomycin is the current treatment of choice for
penicillin/cephalosporin-resistant pneumococcal infection. (See
Chapter 167 in Nelson Textbook of Pediatrics, 17th ed.)

Streptomycin

Any of the above

47. A 7-mo-old infant presents to your office with fever, lethargy, and a stiff neck. A lumbar
puncture reveals cloudy cerebrospinal fluid. Blood and CSF specimens are sent for culture.
Which of the following is the most appropriate next step in the management of this infant?

MRI scan

Intravenous penicillin

Intravenous ceftriaxone

Intravenous vancomycin and ceftriaxone


Explanation: Intravenous vancomycin and ceftriaxone are
recommended. Vancomycin is started empirically because of the
possibility of penicillin/cephalosporin-resistant pneumococci. (See
Chapter 167 in Nelson Textbook of Pediatrics, 17th ed.)

Withholding of antibiotics until culture results are available

48. A 26-mo-old child with sickle cell disease appears in your office for the first time. He
has a history of a prior hospitalization for pneumococcal bacteremia. Currently the child is
on penicillin prophylaxis, but he has not received either the conjugate or polysaccharide
pneumococcal vaccine. Which of the following is the optimal means for protecting this child
from developing another episode of invasive pneumococcal disease?

Give one dose of the pneumococcal conjugate vaccine and continue


penicillin prophylaxis.

Give one dose of the pneumococcal polysaccharide vaccine and


continue penicillin prophylaxis.

Give one dose of the pneumococcal conjugate vaccine followed


one month later by one dose of the pneumococcal
polysaccharide vaccine, and continue penicillin prophylaxis.
Explanation: Immunization with the conjugate polysaccharide
vaccine is recommended for all infants. High-risk children >2 yr of
age, such as those with asplenia, sickle cell disease, some types of
immune deficiency (e.g., antibody deficiencies), HIV infection, or
chronic lung, heart, or kidney disease (including nephrotic
syndrome), may benefit also from the 23-valent pneumococcal
polysaccharide vaccine. (See Chapter 167 in Nelson Textbook of
Pediatrics, 17th ed.)
Give one dose of the pneumococcal conjugate vaccine and switch
from penicillin prophylaxis to Augmentin prophylaxis.

Give one dose of pneumococcal conjugate vaccine and switch to


intramuscular benzathine penicillin prophylaxis.
49. A previously healthy 2-yr-old child in your practice who has been fully immunized with
the pneumococcal conjugate vaccine has now developed pneumococcal meningitis. The
most likely reason for development of pneumococcal meningitis after receipt of the vaccine
in this child is:

An underlying primary immunodeficiency

A pneumococcal serotype that the vaccine did not protect


against
Explanation: The currently available heptavalent vaccine contains
conjugated capsular polysaccharides of serotypes 4, 6B, 9V, 14, 19F,
23F, and 18C. These serotypes cause >80% of pneumococcal
infections among children. (See Chapter 167 in Nelson Textbook of
Pediatrics, 17th ed.)

Failure of the vaccine to elicit a protective immune response against


a pneumococcal serotype contained in the vaccine

An anatomic defect with communication between the middle ear and


the subarachnoid space

Cardiac defect with right-to-left shunting

50. An 8-mo-old infant who has received all recommended vaccines at the appropriate age
presents to your office with a temperature of 39.5°C, increased respiratory rate, and
crackles in the left lung. A chest radiograph shows a left lower lobe infiltrate with a large
pleural effusion. The peripheral white blood cell count is 22,000/mm3, with 60% neutrophils
and 15% bands. The child is admitted to the hospital but is not thought to require intensive
care or intubation.The most appropriate next step in management would be to:

Start intravenous penicillin and follow closely.

Start intravenous cefuroxime and follow closely.

Start intravenous vancomycin and ceftriaxone and follow closely.

Have a surgeon perform a pleural tap followed by placement of a


chest tube, and start intravenous penicillin.

Have a surgeon perform a pleural tap followed by placement of a


chest tube, and start intravenous ceftriaxone.
Explanation: The likely organisms causing this infection are either
pneumococci or group A streptococci. For invasive pneumococcal
infections outside the central nervous system (e.g. lobar pneumonia
with or without bacteremia), high-dose cefotaxime or ceftriaxone is
usually effective even for those infections caused by cephalosporin-
intermediate or-resistant strains (MIC 1.0-2.0+ mg/L).

51. A 4-yr-old boy presents with well-demarcated, perianal erythema associated with anal
pruritus and blood-streaked stools. The most likely etiology is:

Lupus erythematosus

Crohn disease

Clostridium difficile infection

Staphylococcus aureus infection

Group A streptococcal infection


Explanation: Perianal streptococcal disease is a distinct clinical
entity caused by group A streptococci.
52. Scarlet fever is caused by:

Staphylococcus aureus strains that produce exfoliatins

Staphylococcus aureus strains that produce enterotoxins

Group A streptococci strains that produce pyrogenic exotoxins


Explanation: Scarlet fever is the result of infection with a strain of
group A streptococci that produces one of three pyrogenic
(erythrogenic) exotoxins (A, B, or C). (See Chapter 168 in Nelson
Textbook of Pediatrics, 17th ed.)

Human herpesvirus type 6

Parvovirus B19

Question . 53. All of the following are features of scarlet fever except:

Pastia lines

Desquamation

White strawberry tongue

Red strawberry tongue


Preauricular lymphadenopathy
Explanation: The classic rash of scarlet fever has a texture of
gooseflesh or coarse sandpaper. Pastia lines are areas of
hyperpigmentation that do not blanch with pressure that may appear
in creases, particularly in the antecubital fossae. White strawberry
tongue is characteristic of the early illness; as the white coat
desquamates, the red strawberry tongue persists. Skin desquamation
begins toward the end of the first week of illness. Preauricular
lymphadenopathy is not typical
Question . 54. A newborn develops sepsis and shock. The pathogen that most commonly
causes systemic and focal infections in the newborn is:

Staphylococcus aureus

Group A streptococci

Group B streptococci
Explanation: Group B streptococcal organisms are the major cause
of severe systemic and focal infections in the newborn. Coagulase-
negative staphylococcal infections are the most common nosocomial
infections in the neonatal intensive care unit. (See Chapter 169 in
Nelson Textbook of Pediatrics, 17th ed.)

Escherichia coli

Herpes simplex virus

Question . 55. All of the following statements concerning perinatal group B streptococcal
neonatal infections are true except:

Approximately 20-40% of pregnant women are colonized.

Colonization rates are increased in women older than 40 yr, in


whites, and in higher socioeconomic groups.
Explanation: Group B streptococcal colonization rates are increased
in women younger than 20 years of age, in African Americans, and
among lower socioeconomic groups. (See Chapter 169 in Nelson
Textbook of Pediatrics, 17th ed.)

Pregnant women who are colonized are usually asymptomatic.

Approximately 40-70% of infants born to colonized women become


colonized.

Approximately 0.5-2% of colonized infants become infected.

Question . 56. The recommended regimen for selective intrapartum prophylaxis for group
B streptococcal infection is:

Penicillin G 5,000,000 units IV to the mother at the onset of labor

Penicillin G 5,000,000 units IV to the mother at the onset of


labor, followed by 2,500,000 units IV every 4 hr until delivery
Explanation: For group B streptococcal prophylaxis, the AAP and
CDC recommend administration of penicillin G, 5,000,000 units IV, to
the mother at the onset of labor, followed by 2,500,000 units IV every
4 hr until delivery. Ampicillin is an alternative. For penicillin-intolerant
women, cefazolin may be used. For penicillin-allergic women at high
risk for anaphylaxis, clindamycin or erythromycin should be used.
(See Chapter 169 in Nelson Textbook of Pediatrics, 17th ed.)

Penicillin G 5,000,000 units IV to the mother at the onset of labor,


followed by continuous infusion of 500,000 units per hour until
delivery

Crystalline penicillin G 50,000 units IM to the newborn

Procaine penicillin G 150,000 units IM to the newborn

Question . 57. Regarding early-onset versus late-onset neonatal infection, all of the
following are true except:

Group B streptococci are the most common cause of early-onset


infection.

Early-onset disease usually presents within the first 7 days of life,


whereas late-onset disease presents after 7 days.

Maternal chemoprophylaxis has led to striking decreases in


both early-onset and late-onset disease.
Explanation: In the 1990s, widespread implementation of maternal
chemoprophylaxis led to a striking 65% decrease in the incidence of
early-onset neonatal group B streptococcal disease in the United
States, from 1.7 per 1,000 live births to 0.6 per 1,000 live births, while
the incidence of late-onset disease remained essentially stable at
approximately 0.4 per 1,000. (See Chapter 169 and Figure 169-1 in
Nelson Textbook of Pediatrics, 17th ed.)

Early-onset disease is often associated with maternal obstetric


complications, whereas late-onset disease is not.

Early-onset disease usually presents as sepsis and pneumonia,


whereas late-onset disease usually presents as bacteremia and
meningitis.
Question . 58. The risk of early-onset group B streptococcal infection in an infant of a 34-
yr-old mother is increased with all of the following except:

Heavy maternal vaginal or rectal colonization with group B


streptococci

Prolonged rupture of membranes (greater than 18 hr)

Maternal fever with temperature greater than 38°C

Maternal bacteriuria with group B streptococci during her last


pregnancy
Explanation: Risk factors for early-onset disease include heavy
maternal vaginal or rectal colonization with group B streptococci,
prolonged rupture of membranes, intrapartum fever, prematurity,
maternal bacteriuria during pregnancy, and previous delivery of an
infant who developed group B streptococcal disease. (See Chapter
169 in Nelson Textbook of Pediatrics, 17th ed.)

Previous infant with invasive group B streptococcal infection

Question . 59. Which of the following pregnant women should receive prophylaxis against
group B streptococci?

A woman who tested positive for group B streptococci at 37 wk of


gestation and is having a planned cesarean section (without rupture
of membranes)

A woman who tested negative for group B streptococci at 35 wk of


gestation and goes into spontaneous labor at 36 wk

A woman whose group B streptococcal status is unknown and


who had a previous infant at home with invasive group B
streptococcal infection
Explanation: Any woman with a positive prenatal screening culture,
group B streptococcal (GBS) bacteriuria during pregnancy, or a
previous infant with invasive GBS disease should receive intrapartum
antibiotics. (See Chapter 169 in Nelson Textbook of Pediatrics, 17th
ed.)

A woman who tested negative for group B streptococci at 37 wk and


has had prolonged rupture of membranes (>20 hr) when she delivers
at 39 wk of gestation

Question . 60. All of the following infants routinely require a sepsis evaluation (at least a
complete blood count with differential, and blood culture) except:

An infant born at 36 wk whose mother had not been tested for


group B streptococci
Explanation: Guidelines from the Centers for Disease Control and
Prevention (CDC) provide an algorithm for the empirical management
of a newborn whose mother received intrapartum antimicrobial
agents for prevention of early-onset group B streptococcal disease or
suspected chorioamnionitis. (See Chapter 169 and Figure 169-2 in
Nelson Textbook of Pediatrics, 17th ed.)

An infant born at 38 wk whose mother had a culture that was positive


for group B streptococci and who was delivered 2 hr after the mother
received her first dose of antibiotics

An infant born at 40 wk whose mother had a culture that was


negative for group B streptococci but was treated for suspected
chorioamnionitis

An infant born at 38 wk whose mother had a culture that was positive


for group B streptococci and had spontaneous rupture of membranes
but then delivered by cesarean section for failure to progress

An infant born at 39 wk whose mother had a culture that was


negative for group B streptococci

Question . 61. A 19-yr-old freshman in college presents with signs of pharyngitis. His
throat culture is positive for group C streptococci. The recommended management is:

Oral penicillin as for group A streptococcal infection


Explanation: Penicillin is the drug of choice for treating infections
due to either group C or group G streptococci. (See Chapter 170 in
Nelson Textbook of Pediatrics, 17th ed.)

Oral clindamycin

Oral erythromycin

Intramuscular ceftriaxone

None of the above: no treatment is necessary with a positive culture


for group C streptococci

Question . 62. A 12-yr-old girl is hospitalized in intensive care. She has a Foley catheter,
and a urine culture is positive for Enterococcus. In addition to removing the catheter, which
antibiotic is the recommended agent for treatment of Enterococcus infections?

Penicillin

Ampicillin
Explanation: In general, in the immunocompetent person, minor
localized infections due to Enterococcus can be treated with
ampicillin alone. (See Chapter 171 in Nelson Textbook of Pediatrics,
17th ed.)

Clindamycin

Erythromycin

Vancomycin

Question . 63. As a visiting physician in a developing country, you are asked to see a 2-yr-
old child who presents with a history of fever, dysphagia, and lethargy. Physical
examination shows a gray-brown leather-like adherent membrane over the posterior
oropharynx and hypopharynx. The most likely diagnosis is:

Epiglottitis

Bacterial tracheitis

Group A streptococcal infection

Vincent angina

Diphtheria
Explanation: The leather-like adherent membrane, extension
beyond the faucial area, relative lack of fever, and dysphagia help
differentiate diphtheria from exudative pharyngitis due to group A
streptococci and Epstein-Barr virus. (See Chapter 172 in Nelson
Textbook of Pediatrics, 17th

Question . 64. Which of the following routes of transmission has been associated with
several large outbreaks of human listeriosis?

Aerosol transmission

Person-to-person spread

Zoonotic transmission

Drinking contaminated water

Food-borne transmission
Explanation: Listeria monocytogenes may contaminate dairy
products (milk, cheese) and may survive, replicate, and spread in a
family's refrigerator. (See Chapter 173 in Nelson Textbook of
Question . 65. Which of the following is the recommended treatment for neonatal
listeriosis?
Ceftriaxone

Ampicillin with or without an aminoglycoside


Explanation: Listeria isolates are usually sensitive to penicillin,
ampicillin, erythromycin, and tetracycline but are not susceptible to
the cephalosporins, including the third-generation cephalosporins.
The addition of an aminoglycoside (e.g., gentamicin) lowers the
minimum bactericidal concentration. (See Chapter 173 in Nelson
Textbook of Pediatrics, 17th ed.)

Cefotaxime with or without an aminoglycoside

Erythromycin

Vancomycin

Question . 66. All of the following represent risk factors for infection with Listeria
monocytogenes except:

Age younger than 1 mo

Cancer chemotherapy

Previous treatment with broad-spectrum antibiotics


Explanation: Likely routes of spread of Listeria include food-borne
infection, zoonotic spread, and person-to-person transmission,
especially to immunocompromised persons and via vertical
transmission to newborns.

Ingestion of unpasteurized cheese

Occupational exposure as a veterinarian

Question . 67. All of the following are effective in the prevention of infection due to Listeria
monocytogenes except:

Use of the approved Listeria vaccine for high-risk individuals


Explanation: There is no Listeria vaccine available.

Avoidance of unpasteurized dairy food

Careful handwashing to prevent nosocomial spread in the hospital

Prophylaxis with trimethoprim/sulfamethoxazole for


immunocompromised patients
Thorough cooking of meats

Question . 68. A 16-yr-old girl presents with signs and symptoms of appendicitis. Her past
medical history is significant only for sexual activity and placement of an IUD 1 yr
previously. She undergoes an appendectomy, in which her appendix is found to be normal.
One month postoperatively, she has local pain and has an irregular, hard mass in her
ileocecal area. The most likely diagnosis is:
Yersinia pseudoappendicitis

Lymphoma

Inflammatory bowel disease

Pelvic actinomycosis
Explanation: Pelvic actinomycosis is an unusual complication of IUD
use.

Amebiasis

Question . 69. Nocardia infection is an acute, subacute, or chronic suppurative infection


that primarily causes pulmonary disease in immunocompromised patients. The hallmark of
Nocardia infection is:

A tendency for remissions and exacerbations


Explanation: Nocardia infection is characterized by remissions and
exacerbations. (See Chapter 175 in Nelson Textbook of Pediatrics,
17th ed.)

Involvement of bone

A self-limited disease with scar formation

Association in the pelvis with IUD placement

An association with sickle cell disease

Question . 70. All of the following serogroups are included in the quadrivalent
meningococcus vaccine except:

B
Explanation: Meningococcal group B polysaccharide is poorly
immunogenic. There is no vaccine available for this serogroup. (See
Chapter 176 in Nelson Textbook of Pediatrics, 17th ed.)
C
W135
Y

Question . 71. Which of the following is associated with a poorer prognosis for persons
presenting with meningococcal disease?

Presence of petechiae for <12 hr


Explanation: Poor prognostic signs for meningococcal disease
include hypotension and shock, purpura fulminans, seizures,
leukopenia, thrombocytopenia, and high circulating levels of
endotoxin and tumor necrosis factor. The presence of petechiae for
less than 12 hr before admission, hyperpyrexia, and the absence of
meningitis reflect rapid clinical progression and poorer prognosis.
(See Chapter 176 in Nelson Textbook of Pediatrics, 17th ed.)

Meningitis

Thrombocytosis

Leukocytosis
Low circulating levels of tumor necrosis factor

Question . 72. Which of the following contacts should receive rifampin chemoprophylaxis
after diagnosis of invasive Neisseria meningitidis in a child?

Unimmunized or partially immunized children younger than 4 yr of


age in the same household

All unimmunized or partially immunized children in the same


household

All children and adults in the same household if there is an


unimmunized or partially immunized child younger than 48 mo

Unimmunized or partially immunized children in the same household


and in the same daycare facility

All children and adults in the same household or daycare facility


regardless of immunization history
Explanation: Chemoprophylaxis for contacts of a person with proven
or suspected N. meningitidis infection is indicated for all household,
daycare, and nursery care contacts. The index patient should also
receive rifampin prophylaxis if penicillin was used for treatment. (See
Chapter 176 in Nelson Textbook of Pediatrics, 17th ed.)

Question . 73. A 7-mo-old girl presents with a temperature of 103.8°F, blood pressure of
70/30 mm Hg, diffuse petechiae first noted 4 hr before presentation, platelet count of
88,000/mm3, and white blood cell count of 4300/mm3, with 23% neutrophils and 42%
bands. The infant has received all recommended vaccinations. Which of the following is the
most likely bacterial etiology of this presentation?

Staphylococcus aureus

Streptococcus pneumoniae

Neisseria meningitidis
Explanation: The presence of fever, petechiae, and hypotension
suggest meningococcemia. (See Chapter 176 in Nelson Textbook of
Pediatrics, 17th ed.)

Haemophilus influenzae type b

Escherichia coli O157:H7

Question . 74. A 15-mo-old child who attends a child care center 5 days per week is
diagnosed with meningococcal infection. Which of the following agents should be used for
prophylaxis for the child care classmates of this infant?

Chloramphenicol

Cefixime

Rifampin
Explanation: Rifampin or ceftriaxone is recommended for
prophylaxis of close contacts of patients with meningococcal disease.
(See Chapter 176 in Nelson Textbook of Pediatrics, 17th ed.)

Ciprofloxacin

Trimethoprim-sulfamethoxazole

Question . 75. Parents of a college-bound high school senior ask whether their child
should receive a meningococcal vaccine. Which of the following statements should
comprise part of the information that is provided to them?

The quadrivalent polysaccharide vaccine will prevent over 90% of the


cases of meningococcal infection among college students.

Vaccine recipients may develop scattered petechiae 5 to 7 days after


vaccination.

The highest risk of meningococcal disease among college students


occurs near the end of each semester.
The risk for college students living on campus is severalfold
higher than for their non-collegiate peers.
Explanation: College students, especially freshmen living in
dormitories, are at about 2- to 8-fold higher risk than non-collegiate
peers for meningococcemia. (See Chapter 176 in Nelson Textbook of
Pediatrics, 17th ed.)

Freshmen living off-campus have the highest risk of meningococcal


infection.

Question . 76. A 6-yr-old child with meningococcal meningitis develops a swollen left knee
on the fifth day of antibiotic treatment. Which of the following is the most likely etiology of
this finding?

Hemorrhage into the joint occurring as a result of disseminated


intravascular coagulation

Progression of septic arthritis that was unrecognized at the onset of


illness

Nonspecific edema from progressive sepsis-related capillary leak

Immune complex deposition resulting from production of anti-


meningococcal antibodies
Explanation: Nonsuppurative complications of meningococcal
disease appear to be immune complex-mediated and become
apparent 4-9 days after the onset of illness. Arthritis and cutaneous
vasculitis (erythema nodosum) are most common. The arthritis
usually is mono- or oligoarticular, involves large joints, and is
accompanied by sterile effusions that respond to nonsteroidal anti-
inflammatory agents. Long-term sequelae are uncommon. (See
Chapter 176 in Nelson Textbook of Pediatrics, 17th ed.)

Secondary bacterial infection from the immunosuppressive effects of


meningococcal infection

Question . 77. A 13-yr-old boy develops fever with petechiae and a few purpura. Mild
hypotension responds to intravenous fluids without the need for pressor support. A blood
culture yields Neisseria meningitidis. The patient improves greatly after 7 days of antibiotic
therapy and is ready for discharge. Which of the following is most likely to reveal an
abnormality that may have predisposed him to develop meningococcal disease?

Quantitative immunoglobulins (IgM, IgG, and IgA)

Serial complete blood counts to assess for cyclic neutropenia

Genotype analysis of the TNF-alpha promoter region


Screening assay for deficiencies of complement factors (CH50)
Explanation: Persons with primary complement deficiency have an
increased risk of developing meningococcal disease, underscoring
the important role of complement in host defense against
meningococci. Of individuals with properdin, factor D, or terminal-
component deficiencies, 50-60% will develop serious bacterial
infections, caused almost solely by N. meningitidis. (See Chapter 176
in Nelson Textbook of Pediatrics, 17th ed.)

Total lymphocyte subsets (B cells, CD4+ T cells, CD8+ T cells)

Question . 78. Which of the following statements about the epidemiology of


meningococcal infections is true?

Serogroup A strains are common in developed countries.

Serogroup B strains continue to cause about half of the cases


among children in developed countries.
Explanation: The vast majority of meningococcal disease worldwide
is caused by serogroups A, B, C, W135, and Y. Historically,
serogroups B and C each accounted for nearly half of cases among
young children in developed countries. Cases caused by serogroup Y
strains increased during the 1990s, predominantly among
adolescents and adults, such that serogroups B, C, and Y now each
account for about one third of cases in these age groups. (See
Chapter 176 in Nelson Textbook of Pediatrics, 17th ed.)

The frequency of serogroup Y disease has decreased since 1990.

Elementary school-aged children have higher rates of disease than


those in 15- to 19-yr-olds.

Purpura fulminans is the most common clinical presentation during


childhood.

Question . 79. In addition to genital tract infections, well-recognized forms of Neisseria


gonorrhoeae infection include all of the following except:

Asymptomatic infection

Lymphadenitis
Explanation: Gonococcal infections range from asymptomatic
carriage to localized urogenital infections, culture-negative
tenosynovitis, suppurative arthritis, and disseminated gonococcal
infection with bacteremia. Most genital tract infections in children are
symptomatic, but as many as 80% of sexually active mature females
with urogenital gonorrhea infections are asymptomatic. Conjunctivitis
occurs in neonates born to mothers with genital tract gonorrhea. (See
Chapter 177 in Nelson Textbook of Pediatrics, 17th ed.)

Conjunctivitis

Suppurative arthritis

Disseminated disease with bacteremia

Question . 80. Which of the following drugs is recommended for initial therapy for
nondisseminated gonococcal disease in children and adults?

Penicillin

Tetracycline

Ceftriaxone
Explanation: Even with disseminated disease (bacteremia, arthritis,
meningitis), ceftriaxone is the drug of choice because of the high rate
of penicillin resistance. (See Chapter 177 in Nelson Textbook of
Pediatrics, 17th ed.)

Cefazolin

Erythromycin

Question . 81. The Fitz-Hugh-Curtis syndrome is characterized by:

Right upper quadrant pain caused by gonococcal perihepatitis


Explanation: In addition to gonococcal disease, Fitz-Hugh-Curtis
syndrome may also be due to Chlamydia. (See Chapter 177 in Nelson
Textbook of Pediatrics, 17th ed.)

Polyarticular arthritis and rash of disseminated gonococcal infection

Lower quadrant pain caused by gonococcal endometritis

Gonococcal meningitis

Monoarticular arthritis and urethral exudate caused by N.


gonorrhoeae

Question . 82. Which of the following contacts should receive rifampin chemoprophylaxis
after diagnosis of invasive Haemophilus influenzae type b infection in a child?
Unimmunized or partially immunized children younger than 4 yr in the
same household

All children and adults in the same household if there is an


unimmunized or partially immunized child younger than 48 mo
Explanation: Chemoprophylaxis for contacts of a person with proven
H. influenzae type b infection is indicated if the close contact group
includes one or more children younger than 48 mo of age who are
not fully immunized; under these circumstances, rifampin prophylaxis
is indicated for all members of the close contact group, including the
index patient. (See Chapter 178 in Nelson Textbook of Pediatrics, 17th
ed.)

Unimmunized or partially immunized children younger than 48 mo in


the same household and in the same daycare facility

All children and adults in the same household or daycare facility


regardless of immunization history

All immunocompormised children and adults in the same household


or daycare facility regardless of immunization history.

Question . 83. After diagnosis of pertussis in a toddler, erythromycin should be given to


the patient and to which family members?

Only those with a cough

Only those younger than 7 yr

Only those who are incompletely immunized

Only those with compromised immunity

All family members regardless of age, symptoms, or


immunization status
Explanation: Pertussis is a "family" disease with various degrees of
symptoms and colonization. (See Chapter 180 in Nelson Textbook of
Pediatrics, 17th ed.)

Question . 84. Which of the following contacts should receive erythromycin after diagnosis
of pertussis in a child?

Unimmunized children younger than 4 years in the same household

All unimmunized children in the same household


Symptomatic children and adults in the same household

Unimmunized children in the same household and in the same


daycare, and symptomatic adults in the same household

All children and adults in the same household or daycare


regardless of immunization history or symptoms
Explanation: Antimicrobial prophylaxis for pertussis is always given
to all household contacts and other close contacts, such as those in
daycare, regardless of age, history of immunization, or symptoms.
(See Chapter 180 in Nelson Textbook of Pediatrics, 17th ed.)

Question . 85. The only absolute contraindication to subsequent administrations of


acellular pertussis vaccine is:

Collapse or shocklike state within 2 days of pertussis vaccination

Persistent, inconsolable crying lasting longer than 3 hr within 2 days


of pertussis vaccination

An anaphylactic reaction to a previous dose of whole-cell


pertussis vaccine
Explanation: The more serious adverse events associated with DTP
vaccine, such as high fever, persistent crying, and hyperresponsive
episodes, occur significantly less frequently with DTaP vaccine and
are not considered contraindications to further doses. (See Chapter
180 in Nelson Textbook of Pediatrics, 17th ed.)

Convulsions with or without fever within 3 days of pertussis


vaccination

History of sudden infant death syndrome (SIDS) in the family

Question . 86. A 4-wk-old infant has the history of gagging and gasping episodes of acute
onset, with mild cough and rhinorrhea. The infant is afebrile and appears well, and findings
on lung auscultation are normal. Complete blood count reveals hemoglobin 12.0 gm/dL,
WBC count 42,000/mm3 (10% neutrophils, 3% monocytes, 87% lymphocytes), and platelet
count 650,000/mm3. The most likely etiologic agent of this illness is:

Chlamydia trachomatis

Rhinovirus

Bordetella pertussis
Explanation: Leukocytosis (15,000-100,000 cells/mm3) due to
absolute lymphocytosis is characteristic in the catarrhal stage of
pertussis. (See Chapter 180 in Nelson Textbook of Pediatrics, 17th
ed.)
Group B streptococci

Respiratory syncytial virus

Question . 87. You are advising the mother of a 2-mo-old infant about recommended
immunizations. She has read about pertussis vaccine and questions you on the acellular
pertussis vaccine you are recommending. You should advise her that compared with
whole-cell pertussis vaccine, the acellular vaccine:

Is more effective in preventing pertussis

Has a longer protective effect

Is less likely to cause seizures


Explanation: Mild local and systemic adverse events as well as
more serious events (including high fever, persistent crying for longer
than 3 hr, hypotonic hyporesponsive episodes, and seizures) occur
significantly less frequently among infants who receive DTaP than
among those who receive DTP vaccine. (See Chapter 180 in Nelson
Textbook of Pediatrics, 17th ed.)

Is less likely to cause encephalopathy

Is less likely to cause anaphylaxis

Question . 88. A 6-wk-old infant is proved to have pertussis. His mother has had a bad
cough for 2 wk, and his father is well. There are also three siblings (ages 20 mo, 6 yr, and
8 yr) living in the household; all are appropriately immunized for age and are
asymptomatic. In addition to the infant, for whom erythromycin is prescribed, which of the
following household contacts should also be treated?

Mother

Mother, father, and the 8-yr-old sibling

Mother and the 20-mo-old sibling

All three of the siblings

All of the household contacts


Explanation: Erythromycin (40-50 mg/kg/24 hr divided qid PO;
maximum 2 g/24 hr) for 14 days should be administered promptly to
all household contacts and other close contacts of the index patient
with pertussis, such as those in daycare, regardless of age, history of
immunization, or symptoms. Clarithromycin and azithromycin are
potential but not proven alternative agents for those who cannot
tolerate erythromycin. (See Chapter 180 in Nelson Textbook of
Pediatrics, 17th ed.)

Question . 89. Which of the following is recognized to be associated with infantile


hypertrophic pyloric stenosis?

Maternal azithromycin administered during pregnancy

Any maternal macrolide administered during pregnancy

Erythromycin administered during the first 2 wk of life


Explanation: A 7- to 10-fold relative risk for infantile hypertrophic
pyloric stenosis (IHPS) has been reported in infants younger than 6
wk of age treated with orally administered erythromycin. The highest
risk appears to be in the first 2 wk of life in term infants, and with
courses of 14 days or longer. The risk of IHPS after treatment with
azithromycin or clarithromycin is unknown. (See Chapter 180 in
Nelson Textbook of Pediatrics, 17th ed.)

Any macrolide administered during the first 2 wk of life

Any macrolide administered during pregnancy or during the first 2 wk


of life

Question . 90. A 1-mo-old infant develops bloody diarrhea associated with fever. The most
likely agent causing this illness is:

Nontyphoidal Salmonella
Explanation: Salmonella is by far the most common cause of febrile
gastroenteritis in early infancy. Campylobacter is second in frequency
to Salmonella as a bacterial cause of enteritis in infancy. Yersinia is
rare. Shigella is rare in infancy but common in children 1-3 yr of age.
Rotavirus rarely causes bloody diarrhea. (See Chapter 181.1 in Nelson
Textbook of Pediatrics, 17th ed.)

Shigella

Campylobacter

Yersinia

Rotavirus
Question . 91. Isolation of Salmonella serotype marina from an infant suggests that:

The child is in contact with an iguana.


Explanation: Salmonella ser. marina is usually acquired by contact
with an iguana. (See Chapter 181.1 in Nelson Textbook of Pediatrics,
17th ed.)

The child lives near a marina.

The child lives in Marina, California.

The child has been exposed to raw seafood.

Nothing: the isolation of this serotype has no special epidemiologic


significance.

Question . 92. A child who has recently returned from a visit to a developing country
experiences a 2-wk illness characterized by gradually increasing fever with temperature
that eventually reaches 104°F, associated with headache, malaise, cough, and abdominal
pain. The most likely diagnosis is:

Cholera

Diphtheria

Shigellosis

Typhoid fever
Explanation: Typhoid fever may or may not be associated with
diarrhea or constipation and is a prolonged, serious illness. (See
Chapter 181.2 in Nelson Textbook of Pediatrics, 17th ed.)

Tetanus

Question . 93. A 10-mo-old child presents to the emergency department with a 4-day
history of fever, with temperatures to 105°, and watery diarrhea, and has just experienced
a generalized seizure. The most likely cause of this syndrome is:

Salmonella gastroenteritis

Aeromonas gastroenteritis

Shigella gastroenteritis
Explanation: Shigella usually causes diarrhea and fever and
sometimes, particularly in young infants, seizures. (See Chapter 182 in
Nelson Textbook of Pediatrics, 17th ed.)
Rotavirus gastroenteritis

Drug ingestion
Question . 94. A 3-yr-old boy acutely develops severe abdominal pain, vomiting, fever
with temperatures to 105°F, and diarrhea that initially is watery but becomes grossly
bloody. Stool culture specimens are obtained, and oral rehydration is initiated. His 8-
yr-old sister had a similar presentation 9 days ago, including a stool culture that was
positive for Shigella flexneri. The next step in management should be to administer:

Loperamide

Ampicillin intravenously

Ceftriaxone parenterally
Explanation: Shigella infection is the most likely treatable
etiology for this acute dysenteric syndrome. Loperamide may
prolong illness. Ampicillin and TMP-SMZ are poor choices for
empirical management because of the frequency of resistant
Shigella. (See Chapter 182 in Nelson Textbook of Pediatrics, 17th
ed.)

Trimethoprim-sulfamethoxazole

None of the above: no antibiotics should be administered until


culture results are known

Question . 95. The symptoms and fever in the child described in Question 94 resolve
over the ensuing 3 days. The boy's stool culture is reported as negative for enteric
pathogens. The recommended management is to:

Continue loperamide

Discontinue all antibiotics

Continue the antibiotic for 5 days


Explanation: A child who has typical dysentery and who responds
to initial empirical antibiotic treatment should be continued on that
drug for a full 5-day course even if the stool culture is negative.
Stool cultures have low sensitivity for culturing Shigella. (See
Chapter 182 in Nelson Textbook of Pediatrics, 17th ed.)

Continue the antibiotic for 14 days

Continue the antibiotic for 14 days and also obtain specimens for
culture from all other family members for Shigella
Question . 96. Hemolytic-uremic syndrome is typically a complication of infection with E.
coli O157:H7. The same process, microangiopathic hemolytic anemia with renal failure,
can also follow infection with:

Salmonella typhi or Campylobacter jejuni

Shigella flexneri

Shigella sonnei

Shigella dysenteriae serotype 1


Explanation: Only Shigella dysenteriae serotype 1 and certain Shiga
toxin-producing E. coli (STEC) commonly produce significant
amounts of Shiga toxin and cause hemolytic-uremic syndrome. (See
Chapters 182 and 183 in Nelson Textbook of Pediatrics, 17th ed.)

Any species of Shigella

Question . 97. A 20-mo-old child develops hemolytic anemia, anuria, azotemia, and
thrombocytopenia after a bout of febrile bloody diarrhea. The most likely etiologic agent of
this illness is:

Campylobacter jejuni

Salmonella typhi

Enterohemorrhagic Escherichia coli


Explanation: Shiga toxin-producing E. coli O157:H7 is an
enterohemorrhagic pathogen and is responsible for most episodes of
hemolytic-uremic syndrome. Shigella dysenteriae is occasionally
responsible for the hemolytic-uremic syndrome. (See Chapters 182 in
Nelson Textbook of Pediatrics, 17th ed.)

Aeromonas

Non-typhi Salmonella

Question . 98. A 2-yr-old girl has an acute afebrile diarrheal syndrome characterized by
abdominal pain, vomiting, and grossly bloody stools. A stool culture on MacConkey sorbitol
media shows growth of bacteria that suggests E. coli O157:H7. A fecal toxin
electroimmunoassay suggests that a Shiga toxin is present. The most appropriate next
step in the management of this girl is:

Administration of loperamide
Oral antibiotic therapy, with the choice based on susceptibility of the
Shiga toxin-producing E. coli

A parenteral antibiotic, with the choice based on susceptibility of the


Shiga toxin-producing E. coli

Hospitalization in intensive care and treatment with intravenous


immune globulin (IVIG)

Careful follow-up evaluation to monitor for development of


thrombocytopenia, anemia, and/or renal failure

Question . 100. Which of the following indicates a poor prognosis for cholera in children?

Illness complicated by hypokalemia

Acidosis

Acute tubular necrosis

Tachycardia and tachypnea

Hypoglycemia and seizures


Explanation: Coma and death are more common with cholera that is
complicated by seizures associated with hypoglycemia. In one study,
14.3% of children with cholera complicated by hypoglycemia died,
compared with 0.7% of children without hypoglycemia. After
dehydration, hypoglycemia is the most common life-threatening
consequence of cholera in children. (See Chapter 184 in Nelson
Textbook of Pediatrics, 17th ed.)

Question . 101. A 1-yr-old child experiences ascending paralysis with peripheral


neuropathy. The cranial nerves are intact. The cerebrospinal fluid is normal except for an
elevated protein level. Which of the following is the likely infectious agent precipitating this
syndrome?

Corynebacterium diphtheriae

Clostridium botulinum

S. dysenteriae serotype 1

Campylobacter jejuni
Explanation: C. jejuni is a commonly associated pathogen in
Guillain-Barré syndrome (autoimmune polyneuropathy). (See Chapter
185 in Nelson Textbook of Pediatrics, 17th ed.)
Clostridium tetani

Question . 102. Which of the following blood products has been shown to be associated
with the greatest risk for transfusion-associated disease due to Y. enterocolitica?

Albumin

Fresh frozen plasma

Platelet concentrates

Intravenous immune globulin

Packed RBCs after 2-wk storage


Explanation: Y. enterocolitica infection most often develops from
ingestion of contaminated food (animal products, milk, water), from
contact with animals (dogs), or from human-to-human contact.
Transfusion-related disease remains a risk but is uncommon. (See
Chapter 186.1 in Nelson Textbook of Pediatrics, 17th ed.)

Question . 103. Of the following diagnostic tests, which is least useful in establishing a
diagnosis of mesenteric lymphadenitis due to Yersinia pseudotuberculosis?

Stool culture
Explanation: Many patients affected by Y. pseudotuberculosis do
not have diarrhea, and thus a stool culture is not even considered as
part of the diagnostic evaluation. If the extent of infection is limited to
the mesenteric lymph nodes, the stool culture results may be
negative. (See Chapter 186.2 in Nelson Textbook of Pediatrics, 17th
ed.)

Mesenteric lymph node histology

Abdominal ultrasonography

Mesenteric lymph node culture

Endoscopy

Question . 104. An adolescent in rural New Mexico living in a household with domestic
cats that roam freely presents with fever and chills of sudden onset and extraordinarily
painful lymphadenopathy. He appears toxic. The most likely etiology is:

Bartonella henselae infection


Francisella tularensis infection

Yersinia pestis infection


Explanation: The unusually severe lymph node tenderness,
especially in the presence of systemic toxicity, distinguishes bubonic
plague caused by Y. pestis from the typical lymphadenopathy
associated with cat-scratch disease or tularemia. (See Chapter 186.3
in Nelson Textbook of Pediatrics, 17

Rickettsia rickettsii infection

Hantavirus pulmonary syndrome

Question . 105. A 3-yr-old girl experiences mild enteritis. A stool culture identifies the
presumed etiology. Her symptoms are resolved by the time the culture result is known.
Which of the following organisms is least likely to be the cause of her illness?

Salmonella

Campylobacter jejuni

Aeromonas hydrophila

Plesiomonas shigelloides

Pseudomonas aeruginosa (nonmucoid strains)


Explanation: Aeromonas and Plesiomonas are common causes of
self-limited diarrheal illness. See Chapter 187 in Nelson Textbook of
Pediatrics, 17th ed.)

Question . 106. A 13-yr-old girl with cystic fibrosis experiences an acute exacerbation
requiring hospitalization for antibiotic therapy. Which of the following organisms is the most
likely cause of her exacerbation?

Aeromonas

Plesiomonas

Pseudomonas aeruginosa (nonmucoid strains)

Stenotrophomonas maltophilia

Burkholderia cepacia
Explanation: Mucoid strains of P. aeruginosa are common in
persons with cystic fibrosis, causing insidious but progressive
respiratory deterioration, but are rarely encountered in other persons.
B. cepacia is an opportunist that rarely infects immunocompetent
persons but is common in persons with cystic fibrosis and causes an
acute respiratory syndrome with fever, leukocytosis, and progressive
respiratory failure. (See Chapters 188.1 and 188.2 in Nelson
Textbook of Pediatrics, 17th ed.)

Question . 107. A 12-yr-old boy with cystic fibrosis experiences an acute exacerbation and
is admitted for intravenous antibiotic therapy. Sputum culture reveals Pseudomonas
aeruginosa. The antibiotic recommended for treatment is:

Ampicillin-sulbactam

Ceftazidime
Explanation: Appropriate single agents for treatment of
Pseudomonas aeruginosa infection include ceftazidime,
cefoperazone, ticarcillin-clavulanate, and piperacillin-tazobactam.
Gentamicin or another aminoglycoside may be used concomitantly
for synergistic effect. (See Chapter 188.1 in Nelson Textbook of
Pediatrics, 17th ed.)

Cefotetan

Ceftriaxone

Ciprofloxacin

Question . 108. In which of the following patients, all age 10 yr, would isolation of
Pseudomonas aeruginosa from the sputum be likely?

A child with tetralogy of Fallot undergoing cardiac repair

A child with cystic fibrosis who has moderate to severe lung


disease
Explanation: Most human infections due to Pseudomonas and
Burkholderia are opportunistic and occur among low birthweight
infants and in older infants and children with impaired host defenses,
such as those with traumatic wounds, cystic fibrosis, malignancies,
extensive burns, malnutrition (especially in impoverished
populations), and primary immunodeficiencies as well as those
receiving immunosuppressive therapy. (See Chapter 188.1 in Nelson
Textbook of Pediatrics, 17th ed.)

A previously well child presenting with a temperature of 40°C, a left


lower lobe infiltrate, and a white blood cell count of 20,000/mm3
A child with asthma previously under good control who presents with
wheezing unresponsive to bronchodilators

An adolescent who has recently been using a hot tub

Question . 109. An 18-yr-old adolescent hospitalized in intensive care develops


symptomatic nosocomial bacteremia due to Pseudomonas aeruginosa. All of the following
are appropriate agents for treatment of this infection except:

Ceftazidime

Cefotaxime
Explanation: Cefotaxime does not provide coverage for
Pseudomonas aeruginosa. (See Chapter 188.1 in Nelson Textbook of
Pediatrics, 17th ed.)

Piperacillin/tazobactam

Ciprofloxacin

Ticarcillin/clavulanate

Question . 110. All of the following are means of transmission of Francisella tularensis
except:

Tick bite

Contact with infected animals

Consumption of contaminated foods

Inhalation

Person-to-person spread
Explanation: F. tularensis, the cause of tularemia, can be
transmitted by many different modes, but person-to-person
transmission has not occurred. (See Chapter 189 in Nelson Textbook
of Pediatrics, 17th ed.)

Question . 111. All of the following have been shown to be routes of transmission for
tularemia except:

The bite of an infected tick

The bite of a mosquito


Aerosol transmission

Drinking contaminated water

Person-to-person transmission
Explanation: Tularemia, an important zoonotic infection, is most
often acquired from a tick bite or directly from rabbits. Flies, fleas,
mosquitos, lice, and many animals (squirrels, beavers, birds) are
additional vectors. (See Chapter 189 in Nelson Textbook of Pediatrics,
17th ed.)

Question . 112. The recommended treatment for tularemia is administration of:

Gentamicin or streptomycin
Explanation: Gentamicin or streptomycin is the agent of choice for
treatment of tularemia. Chloramphenicol and third-generation
cephalosporins are associated with a high clinical failure rate. (See
Chapter 189 in Nelson Textbook of Pediatrics, 17th ed.)

Ceftriaxone or cefotaxime

Trimethoprim-sulfamethoxazole

Chloramphenicol

Vancomycin

Question . 113. A 12-yr-old boy develops progressive symptoms of fever, lassitude,


arthralgias, headache, and abdominal pain. Physical examination shows
hepatosplenomegaly. Further questioning discovers that he and his family live in a rural
area and include unpasteurized dairy products in their diet. The most likely etiologic agent
of this illness is:

Actinomyces

Bartonella henselae

Brucella
Explanation: A history of exposure to animals or ingestion of
unpasteurized dairy produces is helpful in the diagnosis of
brucellosis. (See Chapter 190 in Nelson Textbook of Pediatrics, 17th
ed.)

Francisella tularensis
Yersinia enterocolitica

Question . 114. A 16-yr-old boy is suspected of having legionellosis. Which of the following
is the most useful test for prompt diagnosis of Legionella pneumonia?

Culture of respiratory secretions

Detection of antigen in respiratory secretions

Detection of antigen in urine


Explanation: Because of the difficulty in culture techniques and the
inconsistency of seroconversion, the urinary antigen test for
Legionella is a useful method in the prompt diagnosis of Legionella
infection. (See Chapter 191 in Nelson Textbook of Pediatrics, 17th ed.)

Serologic testing for IgM antibodies

Serologic testing for total (IgM and IgG) antibodies

Question . 115. For the patient described in Question 114, the diagnostic test confirms
legionellosis. The recommended treatment for Legionella infection is drug therapy with:

Gentamicin or streptomycin

Ceftriaxone or cefotaxime

Trimethoprim-sulfamethoxazole

Erythromycin
Explanation: Erythromycin, with or without rifampin, has been
established empirically as effective therapy for legionellosis. The
newer macrolides (azithromycin and clarithromycin) and the
quinolones also have excellent activity in vitro. (See Chapter 191 in
Nelson Textbook of Pediatrics, 17th ed.)

Vancomycin

Question . 116. Cat-scratch disease classically presents as lymphadenopathy, sometimes


accompanied by fever. The most common clinical manifestation of atypical cat-scratch
disease is:

Seizures and coma

Systemic disease

Erythema nodosum
Oculoglandular syndrome of Parinaud
Explanation: The oculoglandular syndrome (conjunctivitis, lymph
node) is the most frequent atypical manifestation of cat-scratch
disease. (See Chapter 192.2 in Nelson Textbook of Pediatrics, 17th
ed.)

Neuroretinitis

Question . 117. All of the following statements regarding cat-scratch disease are true
except:

The causative organism is Bartonella henselae.

The diagnosis can be confirmed only by culture of the organism.


Explanation: The diagnosis of cat-scratch disease, caused by B.
henselae, is usually confirmed by serologic testing but can also be
confirmed by culture. (See Chapter 192.2 in Nelson Textbook of
Pediatrics, 17th ed.)

Treatment affords minimal, if any, benefit.

Recovery usually occurs slowly over many months.

The prognosis is generally excellent.

Question . 118. Which of the following is the characteristic incubation period for
development of cutaneous papules from Bartonella henselae after a cat bite or scratch?

<24 hr

2-4 days (range: 1-7 days)

7-12 days (range: 3-30 days)


Explanation: The lag phase after a cat scratch may be 3-30 days,
with an average incubation of 7-12 days. (See Chapter 192.2 in Nelson
Textbook of Pediatrics, 17th ed.)

6-8 mo (range: 4-12 mo)

Question . 119. All of the following statements concerning the symptomatology of cat-
scratch disease are true except:

Lymph nodes are tender in 80% of patients

Lymph nodes vary in size from 1 to 12 cm


Fever (>38.1°C) occurs in 90% of patients
Explanation: Fever occurs in only 30% of cases of cat-scratch
disease. (See Chapter 192.2 in Nelson Textbook of Pediatrics, 17th
ed.)

Malaise and fatigue occur in 30% of patients

Splenomegaly occurs in 9% of patients

Question . 120. A 3-yr-old presents to your office with a unilateral conjunctivitis that has
been present for 5 days. You palpate an enlarged preauricular node on the same side. She
has no history of recent travel, but her family did receive a kitten as a pet approximately 2
mo ago. The most likely etiologic agent responsible for her condition is:

Borrelia burgdorferi

Francisella tularensis

Bartonella henselae
Explanation: Bartonella henselae is the cause of Parinaud
oculoglandular syndrome (unilateral conjunctivitis accompanied by
preauricular lymphadenopathy). Direct inoculation of the eye,
perhaps by contaminated hands after contact with a cat, is the
presumed mode of spread. (See Chapter 192.2 in Nelson Textbook of
Pediatrics, 17th ed.)

Staphylococcus aureus

Toxoplasma gondii

Question . 121. A 5-yr-boy presents to your office with an enlarged axillary node that has
been present for 3 wk. He had visited the emergency department of a local hospital for the
same problem 2 wk ago and was prescribed a course of cephalexin. His family reports that
he took most of the prescribed doses, but with no change in the node. He initially had a
low-grade fever and fatigue, which has resolved. He has no known exposure to infection
although the family has 3 birds, 2 dogs, and 2 cats. Physical examination shows a large, 3-
cm left axillary node that is tender. There is no other lymphadenopathy or abnormalities
except for healing scratches on both arms. The most reasonable therapeutic approach
would be:

Reassurance and observation


Explanation: This is a typical presentation of cat-scratch disease
caused by Bartonella henselae. There is no treatment that is clearly
beneficial. Unless there are complications, conservative care and
observation constitute appropriate management. (See Chapter 192.2
in Nelson Textbook of Pediatrics, 17th ed.)
Repeating the course of cephalexin

Hospital admission for intravenous oxacillin therpay

Referral for immediate node removal

Azithromycin therapy for 14 days

Question . 122. A family returns to your travel clinic after a three-week vacation to the
Peruvian Andes. Their 12-yr-old boy has become ill with fever, malaise, and a yellowish,
pale color. On physical examination, he is pale, tachycardic with mild icterus, and has
generalized lymphadenopathy. The most likely etiologic agent is:

Yersinia pestis

Rickettsia rickettsii

Bartonella bacilliformis
Explanation: Bartonellosis, caused by Bartonella bacilliformis, is
found only in the valleys of the Andes Mountains in Peru, Ecuador,
Colombia, Chile, and Bolivia. (See Chapter 192.1 in Nelson Textbook
of Pediatrics, 17th ed.)

Staphylococcus aureus

Salmonella

Question . 123. The preferred treatment for infant botulism is:

Penicillin G

Clindamycin

Hydration and cathartics

Botulism immune globulin (BIG)


Explanation: Botulism immune globulin can be obtained from the
California Department of Health Services. Use of BIG significantly
reduces both mean hospital stay and hospital costs. In many
countries, only supportive care can be provided, because BIG is not
universally available. (See Chapter 193 in Nelson Textbook of
Pediatrics, 17th ed.)

Supportive treatment only, including intensive care if necessary


Question . 124. Over a 6-hr period, 4 children who attend the same elementary school
present to the emergency department with vomiting, double vision, slurred speech, fatigued
appearance, and weakness of the neck and arms, but no fever. Laboratory results include
normal results on complete blood count and spinal fluid analysis. Their parents report that
other classmates are having similar symptoms, as are a parent and a teacher who
chaperoned the class on its field trip yesterday to the State Capitol building. You suspect
that these illnesses represent an outbreak, from either natural occurrence or bioterrorism.
Which of the following steps should be taken immediately?

Administer parenteral broad-spectrum antibiotics.

Obtain chest and abdominal MRI scans on the most severely


affected patients.

Perform an edrophonium (Tensilon) test on the most severely


affected patients.

Administer gastric lavage and catharsis.

Telephone the local or state health department to report a


possible outbreak of botulism.
Explanation: Outbreaks of food-borne botulism have been reported
in North America associated with many different types of foods.
Suspected botulism represents a medical and public health
emergency that is immediately reportable by telephone in most U.S.
health care jurisdictions. (See Chapter 193 in Nelson Textbook of
Pediatrics, 17th ed.)

Question . 125. Which of the following findings on the history or physical examination
excludes a diagnosis of infant botulism?

No ingestion of honey or corn syrup

Normal latch, suck, and swallow


Explanation: In infants, bulbar palsies are evident as poor feeding,
weak suck, feeble cry, drooling, and even obstructive apnea. (See
Chapter 193 in Nelson Textbook of Pediatrics, 17th ed.)

Soft feces

Normal deep tendon reflexes

Metabolic acidosis

Question . 126. A 2-mo-old infant is suspected of having infant botulism. Which is the best
means to confirm the diagnosis?

Lumbar puncture
CT scan

Muscle biopsy

Electromyography

Fecal specimen
Explanation: The diagnosis of botulism is unequivocally established
by demonstrating the presence of botulinum toxin in serum or of C.
botulinum toxin or organisms in wound material or feces. (See
Chapter 193 in Nelson Textbook of Pediatrics, 17th ed.)

Question . 127. A 2-mo-old child is seen in your clinic for the first time. The child was born
at home, and this is the first well-child visit. Risk factors for infant botulism that should be
communicated to the parents include:

Gardening

Home construction

Frozen vegetables

Honey
Explanation: The one identified, avoidable source of botulinum
spores for infants is honey. Honey is an unsafe food for any child
younger than 1 yr of age. (See Chapter 193 in Nelson Textbook of
Pediatrics, 17th ed.)

All of the above

Question . 128. An 18-yr-old adolescent presents to the emergency department with a nail
puncture wound of his foot. The injury occurred 1 hr ago in a school play yard. His wound
is cleansed and debrided. His last tetanus immunization was at age 12 yr, as confirmed in
his medical records. Which of the following measures is recommended?

Tetanus vaccination alone

TD

Td

DTaP

None of the above: no immunization is necessary


Explanation: A tetanus toxoid booster (preferably Td) is given to all
persons with any wound if their tetanus immunization status is
unknown or incomplete. A booster is given to injured persons who
have completed their primary immunization series if (1) the wound is
clean and minor but more than 10 yr have passed since the last
booster, or (2) the wound is more serious and more than 5 yr have
passed since the last booster. With delayed wound care, active
immunization should be started at once. (See Chapter 194 and Table
194-1 in Nelson Textbook of Pediatrics, 17th ed.)

Question . 129. A 12-yr-old boy presents to the emergency department with a nail
puncture wound of his foot. The injury occurred 1 hr ago in a school play yard. His wound
is cleansed and debrided. His last tetanus immunization was at age 4 yr, as confirmed in
his medical records. Which of the following is recommended?

Tetanus vaccination alone

Tetanus vaccination plus tetanus immune globulin

Td
Explanation: A tetanus toxoid booster (preferably Td) is given to all
persons with any wound if their tetanus immunization status is
unknown or incomplete. A booster is given to injured persons who
have completed their primary immunization series if (1) the wound is
clean and minor but more than 10 yr have passed since the last
booster, or (2) the wound is more serious and more than 5 yr have
passed since the last booster. With delayed wound care, active
immunization should be started at once. (See Chapter 194 and Table
194-1 in Nelson Textbook of Pediatrics, 17th ed.)

DTaP

None of the above: no immunization is necessary

Question . 130. An 12-yr-old boy presents to the emergency department with a nail
puncture wound of his foot. The injury occurred 1 hr ago in a school play yard. His wound
is cleansed and debrided. His parents came to the United States 4 yr ago from Mexico. He
has not had medical evaluation in the United States since, and his vaccination records from
childhood are not available, although his parents report that "all his childhood
immunizations were completed." Which of the following is recommended?

Tetanus vaccination alone

Tetanus vaccination plus tetanus immune globulin

Td
Explanation: A tetanus toxoid booster (preferably Td) is given to all
persons with any wound if their tetanus immunization status is
unknown or incomplete. A booster is given to injured persons who
have completed their primary immunization series if (1) the wound is
clean and minor but more than 10 yr have passed since the last
booster, or (2) the wound is more serious and more than 5 yr have
passed since the last booster. With delayed wound care, active
immunization should be started at once. (See Chapter 194 and Table
194-1 in Nelson Textbook of Pediatrics, 17th ed.)

DTaP

None of the above?no immunization is necessary

Question . 131. A 15-yr-old white girl develops bloody diarrhea on her sixth day of
hospitalization for therapy of presumed bacterial pneumonia. She has been receiving
intravenous ampicillin. Her fever and tachypnea have resolved, and her oxygenation is now
normal. In fact, she was scheduled for discharge. Which of the following actions would be
most appropriate?

Switch the antibiotic to a third-generation cephalosporin

Discontinue ampicillin
Explanation: The first and essential step in treatment of Clostridium
difficile-associated diarrhea is the discontinuation of the current
antibiotics, if at all possible. In most instances this course combined
with appropriate fluid and electrolyte replacement is sufficient. (See
Chapter 195 in Nelson Textbook of Pediatrics, 17th ed.)

Obtain ventilation-perfusion scan

Obtain CT scan of the abdomen

Add erythromycin or doxycycline

Question . 132. All of the following statements regarding Clostridium difficile-associated


diarrhea are true except:

Antibiotic-associated diarrhea is often related to production of a toxin


by C. difficile.

Newborn and young infants are commonly colonized by C. difficile.

The antibiotic that most commonly causes C. difficile colitis is


gentamicin .
Explanation: Virtually all known antibiotics have been implicated;
penicillins, broad-spectrum cephalosporins, and clindamycin are the
most frequent offenders. (See Chapter 195 in Nelson Textbook of
Pediatrics, 17th ed.)

Most children with antibiotic-associated diarrhea will improve without


specific antibiotic treatment.
Treatment of C. difficile colitis should include stopping antibiotics
whenever feasible.

Question . 133. All of the following are risk factors for Clostridium difficile-associated
diarrhea except:

Antibiotics that alter normal gut flora but do not affect growth of C.
difficile

Chemotherapy

Bowel surgery

Oral ingestion of Lactobacillus (e.g., yogurt)


Explanation: Use of antibiotics that impair growth of normal gut flora
but not C. difficile is the most common risk factor for C. difficile-
associated diarrhea. (See Chapter 195 in Nelson Textbook of
Pediatrics, 17th ed.)

Bowel stasis

Question . 134. A previously healthy 6-mo-old child who just completed antibiotic
treatment for acute otitis media and now is healthy and asymptomatic undergoes a stool
culture to check for antibiotic-associated diarrhea. The stool culture grows C. difficile. The
preferred antibiotic treatment is:

Oral clindamycin

Oral vancomycin

Intravenous vancomycin

Oral metronidazole

None of the above: antibiotic treatment is not indicated


Explanation: difficile is frequently isolated from the stool of healthy
infants. The interpretation of a positive stool culture or toxin requires
clinical correlation. Treatment would not be indicated for an
asymptomatic infant. (See Chapter 195 in Nelson Textbook of
Pediatrics, 17th ed.)

Question . 135. Which of the following best describes the pathogenesis of Clostridium
difficile-associated diarrhea?

Invasion of the small bowel wall by C. difficile


Invasion of the large bowel wall by C. difficile

Production of toxins
Explanation: Clostridium difficile strains may produce two toxins, A
and B. Both toxins are internalized and act within cells to modify
proteins, resulting in cell death. (See Chapter 195 in Nelson Textbook
of Pediatrics, 17th ed.)

Enteroaggregative strains of C. difficile

None of the above: the pathogenesis is unknown

Question . 136. All of the following may be clues to anaerobic infection except:

Pus that shows no growth on routine laboratory testing

Highly variable morphology of organisms seen on Gram stain

Sweet-smelling odor
Explanation: Anaerobic infections should be suspected with foul-
smelling pus, which is present in half of anaerobic infections. In
addition, the culture may be negative if the specimen is not handled
properly under anaerobic conditions. (See Chapter 196 in Nelson
Textbook of Pediatrics, 17th ed.)

Gas formation in tissues

Tissue necrosis

Question . 137. All of the following antimicrobials can provide good anti-anaerobic
coverage except:

Clindamycin

Cefoxitin

Meropenem

Gentamicin
Explanation: Antibiotics that are active against anaerobic organisms
include metronidazole, penicillins combined with -lactamase
inhibitors (ampicillin-sulbactam, ticarcillin-clavulanate, and
piperacillin-tazobactam), carbapenems (imipenem and meropenem),
clindamycin, cefoxitin, and chloramphenicol. Penicillin and
vancomycin are active against the gram-positive anaerobes. (See
Chapter 196 in Nelson Textbook of Pediatrics, 17th ed.)
Metronidazole

Question . 138. A 10-yr-old boy who presents with right lower quadrant pain is found to
have a ruptured appendix. Which of the following organisms are most likely to cause
postoperative infection in such cases?

Peptostreptococcus and Escherichia coli

Bacteroides fragilis and Escherichia col


Explanation: Bacteroides fragilis and other Bacteroides species are
the most common anaerobes causing infection, which is usually
mixed. (See Chapter 196 in Nelson Textbook of Pediatrics, 17th ed.)i

Staphylococcus aureus and Prevotella melaninogenica

Klebsiella pneumoniae and Enterococcus faecalis

Escherichia coli and Klebsiella pneumoniae

Question . 139. In the patient described in Question 138, which of the following specimens
is most likely to yield the responsible pathogen?

Blood

Stool

Throat swab

Peritoneal fluid
Explanation: Rupture of the gut leads to spillage of gut flora into the
peritoneal cavity, and thus the peritoneal fluid is the best source for a
culture specimen. Anaerobic bacteremia is uncommon but may
occur. (See Chapter 196 in Nelson Textbook of Pediatrics, 17th ed.)

Urine

Question . 140. Which of the following infections is most likely to involve anaerobes?

Neonatal meningitis

Tubo-ovarian abscess
Explanation: Pelvic inflammatory disease and tubo-ovarian
abscesses are frequently due to mixed aerobes and anaerobes. (See
Chapter 196 in Nelson Textbook of Pediatrics, 17th ed)

Pharyngitis
Otitis media

Paronychia

Question . 141. A 9-yr-old child is found to have a positive PPD test result on routine
screening. There are no symptoms. Laboratory tests, including chest film, are normal.
Which of the following is the best interpretation of this finding?

Mycobacterium tuberculosis infection


Explanation: Tuberculosis infection typically occurs after inhalation
of M. tuberculosis. A reactive skin test and the absence of clinical
and radiographic manifestations of disease are the hallmark of this
stage. (See Chapter 197 in Nelson Textbook of Pediatrics, 17th ed.)

Mycobacterium tuberculosis disease

Mycobacterium tuberculosis or atypical Mycobacterium infection

Mycobacterium tuberculosis or atypical Mycobacterium disease

Recent exposure to Mycobacterium tuberculosis but neither infection


nor disease

Question . 142. In the child described in Question 141, the risk of developing clinical and
radiographic evidence of tuberculosis without institution of therapy is:

<1%

5-10%
Explanation: Without appropriate treatment, 5-10% of asymptomatic
persons with M. tuberculosis infection will experience disease in the
future. (See Chapter 197 in Nelson Textbook of Pediatrics, 17th ed.)

50%

90-95%

Almost 100%

Question . 143. The most common means of transmission of Mycobacterium tuberculosis


is:

Inhalation of organisms originating from soil or other environmental


sources

Ingestion of organisms originating from soil or other environmental


sources
Person-to-person spread by direct contact with infected discharge or
contaminated fomite

Person-to-person spread by infected airborne droplets


Explanation: In almost all cases, transmission of M. tuberculosis is
from person-to-person spread by airborne mucus droplet nuclei,
usually from an actively infected person with cavitary tuberculosis
and coughing. (See Chapter 197 in Nelson Textbook of Pediatrics,
17th ed.)

Question . 144. A true statement concerning the 5-unit PPD skin test for tuberculosis is:

It is administered by subcutaneous injection.

The reaction is measured 24 to 48 hr after administration.

Persons with tuberculosis meningitis often do not react to the


PPD skin test.
Explanation: The 5-TU PPD skin test is administered by intradermal
injection and is read by measuring the amount of induration (not
erythema) 48-72 hr after administration. Corticosteroids may
decrease the reaction, but this is variable. Up to 50% of persons with
tuberculous meningitis or disseminated disease do not react initially
to PPD; most become reactive after several months of therapy. (See
Chapter 197 in Nelson Textbook of Pediatrics, 17th ed.)

Corticosteroid therapy may increase the amount of reaction.

Reaction is measured by the amount of erythema and induration.

Question . 145. A 3-yr-old child in whose mother tuberculosis was just diagnosed has a
positive PPD skin test result. A chest film shows a localized, nonspecific infiltrate in the
peripheral segments of the right lower lobe. The most appropriate course of action to
confirm the diagnosis of tuberculosis disease in this child is:

Culture of sputum

Culture of pulmonary secretions obtained by bronchoscopy

Culture of gastric contents obtained by gastric aspiration


Explanation: Pulmonary tuberculosis is best confirmed by culture
and isolation of M. tuberculosis. The best culture specimen in young
children is three consecutive early-morning gastric aspirates obtained
before the child has arisen and before peristalsis has emptied the
stomach contents. The yield is approximately 50%, but the yield from
bronchoscopy is even lower. Negative cultures never exclude the
diagnosis of tuberculosis. (See Chapter 197 in Nelson Textbook of
Pediatrics, 17th ed.)

Percutaneous lung and pleural biopsies for culture and


histopathology

Segmental lobectomy for culture and histopathology

Question . 146. A true statement concerning drug resistance of Mycobacterium


tuberculosis is:

Naturally resistant organisms occur at a frequency of about 102 to 103


.

Drug resistance is transferable between organisms.

Drug resistance to each drug is independent of resistance to


other drugs.
Explanation: M. tuberculosis drug resistance is chromosomally
mediated and is not transferable. The estimated frequency of
naturally drug-resistant organisms varies from 105 to 108. Drug
resistance to any one drug is independent of resistance to other
drugs. (See Chapter 197 in Nelson Textbook of Pediatrics, 17th ed.)

Isoniazid, rifampin, and pyrazinamide are bacteriostatic.

None of the above.

Question . 147. All of the following statements concerning isoniazid are true except:

It penetrates all body tissues and fluids.

It can be given orally or intramuscularly.

Rapid acetylation is associated with lower rates of peripheral


neuritis and hepatotoxicity.
Explanation: Rapid acetylation of isoniazid is more frequent in
African-Americans and Asians than among whites. There is no
correlation between acetylation rate and either efficacy of treatment
or adverse events in children. (See Chapter 197 in Nelson Textbook of
Pediatrics, 17th ed.)

Peripheral neuritis is rare in children.

Hepatotoxicity is rare in children.

Question . 148. The recommended treatment for active pulmonary tuberculosis in children
is:
Isoniazid for 9 mo

Isoniazid and rifampin for 6 mo

Isoniazid, rifampin, and pyrazinamide for 6-9 mo

Isoniazid and rifampin for 6 mo, with pyrazinamide during the


first 2 mo
Explanation: The American Academy of Pediatrics and the Centers
for Disease Control and Prevention recommend that children with
pulmonary tuberculosis be treated with 6 mo of isoniazid and rifampin
supplemented with pyrazinamide for the first 2 mo. (See Chapter 197
in Nelson Textbook of Pediatrics, 17th ed.)

Isoniazid and rifampin for 6 mo, with ethambutol during the first 2 mo

Question . 149. A 2-wk-old neonate experiences high fever, severe respiratory distress,
and hepatomegaly. The chest film shows a fine, nodular infiltrate throughout both lungs,
and congenital tuberculosis is suspected. All of the following are expected additional
findings in this newborn except:

Positive PPD skin test result


Explanation: The PPD test usually does not yield positive results
initially in children with congenital tuberculosis, but results become
positive in 1-3 mo. The most important clue to diagnosis is a family
history of tuberculosis. (See Chapter 197 in Nelson Textbook of
Pediatrics, 17th ed.)

Positive PPD skin test results in family members

Acid-fast organisms on gastric aspirate

Meningitis

Hepatitis

Question . 150. The mother of a newborn is found to have an abnormal-appearing


admission chest film and acid-fast bacilli on sputum smear. The mother has no other
symptoms and is ready to be discharged from the hospital and is willing to comply with her
recommended treatment. The recommended management strategy for the newborn is to:

Treat the mother and isolate her from the newborn until she has been
treated for 2 wk.

Treat the mother and isolate her from the newborn until she has three
consecutive negative sputum smears and cultures.
Treat the mother; no isolation is necessary.

Treat the mother and treat the infant with isoniazid and rifampin for 6
mo, with pyrazinamide during the first 2 mo; no isolation is
necessary.

Treat the mother and treat the infant with isoniazid until the
mother is sputum culture-negative for 3 mo; no isolation is
necessary.
Explanation: Isoniazid therapy for newborns has been so effective
that separation of the mother and infant is no longer mandatory
unless the mother is ill enough to require hospitalization or is
expected not to adhere to her treatment regimen. (See Chapter 197 in
Nelson Textbook of Pediatrics, 17th ed.)

Question . 151. The form of leprosy characterized by a single large lesion that slowly
enlarges, heavy cellular infiltration in the dermis, destruction of the cutaneous nerve fibers,
a vigorous and specific cell-mediated immune response, and granulomas composed of
epithelioid cells and lymphocytes, but few or absent bacilli, is:

Lepromatous leprosy
Explanation: Tuberculoid and lepromatous leprosy constitute the
ends of the spectrum of leprosy. Tuberculoid leprosy responds well to
treatment, but complete resolution may take 8-12 mo. Response of
lepromatous leprosy to therapy may take 2-5 yr. (See Chapter 198 in
Nelson Textbook of Pediatrics, 17th ed.)

Borderline lepromatous leprosy

Borderline leprosy

Borderline tuberculoid leprosy

Tuberculoid leprosy

Question . 152. The disease associated most frequently with atypical mycobacteria in
children is:

Cellulitis

Lymphadenitis
Explanation: Lymphadenitis of the superior or anterior cervical or
submandibular areas is the most frequent manifestation of atypical
mycobacterial infections in children. (See Chapter 199 in Nelson
Textbook of Pediatrics, 17th ed.)

Skeletal infections
Pneumonia

Urinary tract infections

Question . 153. The recommended treatment for cervical lymphadenitis caused by atypical
mycobacteria is:

Isoniazid and rifampin for 6 mo

Isoniazid and rifampin for 6 mo, with pyrazinamide during the first 2
mo

Clarithromycin and rifampin for 6 mo

Complete surgical excision followed by clarithromycin and rifabutin


for 6 mo

Complete surgical excision alone


Explanation: Complete surgical excision is the preferred treatment
of lymphadenitis caused by atypical mycobacteria. Antimycobacterial
therapy is necessary only if there is concern for M. tuberculosis
infection or if chronic drainage develops. (See Chapter 199 in Nelson
Textbook of Pediatrics, 17th ed.)

Question . 154. Which of the following therapeutic options is the optimal method to
manage symptomatic nontuberculous mycobacterial lymphadenitis?

Complete surgical excision


Explanation: The treatment of choice for symptomatic
nontuberculous mycobacterial lymphadenitis is complete excision.
Fine-needle aspiration may help with the diagnosis, but excisional
biopsy is the cure. (See Chapter 199 in Nelson Textbook of Pediatrics,
17th ed.)

Administration of isoniazid and rifampin for 6 mo

Await suppuration; then incise and drain

Perform partial biopsy; begin antituberculosis therapy

Oral clarithromycin therapy continued until the swelling resolves

Question . 155. A 16-yr-old adolescent developed a nontender, solitary nodule that slowly
enlarged over several weeks. It began at the site of an abrasion that he incurred while
cleaning his fish aquarium. The most likely infecting organism in this case is:

Sporothrix schenckii
Mycobacterium avium complex

Mycobacterium marinum
Explanation: "Swimming pool granuloma" or "fish tank granuloma" is
caused by M. marinum and develops after inoculation of the
organism at the site of a minor abrasion. (See Chapter 199 in Nelson
Textbook of Pediatrics, 17th ed.)

Mycobacterium leprae

Chlamydia trachomatis (LGV biovar)

Question . 156. Which of the following serologic patterns is most consistent with
successful treatment of primary syphilis in an adolescent 2 yr previously?

Negative VDRL, negative MHA-TP

Negative VDRL, positive MHA-TP


Explanation: Nontreponemal tests (VDRL, RPR) can be quantified
and usually become nonreactive with 1 yr of adequate therapy for
primary syphilis and within 2 yr of adequate therapy for secondary
syphilis. Treponemal tests (MHA-TP, FTA-ABS) are not quantified
and usually remain positive for life, even with adequate therapy. (See
Chapter 200 in Nelson Textbook of Pediatrics, 17th ed.)

VDRL 1:16, RPR 1:8

VDRL 1:8, negative MHA-TP

VDRL 1:8, positive MHA-TP

Question . 157. In a newborn whose mother was treated for syphilis during pregnancy, all
of the following are risk factors for congenital syphilis except:

Treatment of the mother with erythromycin

Treatment of the mother with doxycycline

Change in maternal VDRL titer from 1:32 at treatment to 1:16 at


delivery

Treatment of the mother more than 30 days before delivery


Explanation: Risk factors for congenital syphilis are maternal
treatment that was inadequate, unknown, or undocumented;
treatment given at less than 30 days before delivery; treatment with a
nonpenicillin regimen; and serial maternal VDRL titers that do not
decrease sufficiently (at least fourfold) to demonstrate a cure. (See
Chapter 200 in Nelson Textbook of Pediatrics, 17th ed.)

VDRL titer 1:32; RPR negative.

Question . 158. A full-term male newborn whose mother had reactive Venereal Disease
Research Laboratory (VDRL) and microhemagglutination assay-Treponema pallidum
(MHA-TP) results at the time of delivery was evaluated. He was anemic and had
thrombocytopenia and mild hepatomegaly. He also had a desquamative skin rash
consistent with congenital syphilis. His CSF was clear, with 5 white blood cells (WBCs), 0
RBCs, protein of 80 mg/dL, and glucose of 49 mg/dL; the CSF VDRL result was
nonreactive. Based on this examination, which of the following is true?

The newborn has symptomatic congenital syphilis but not


neurosyphilis and can therefore be treated with benzathine penicillin.

The newborn may be treated with a combination of ampicillin and


gentamicin for 7 to 10 days.

Neurosyphilis cannot be excluded in this newborn; therefore, he


should be treated for neurosyphilis.
Explanation: This infant requires a complete course of therapy for
neurosyphilis. (See Chapter 200 in Nelson Textbook of Pediatrics,
17th ed.)

All cases of neurosyphilis would have CSF pleocytosis and a reactive


CSF VDRL result.

An MHA-TP test of the CSF is necessary to diagnose neurosyphilis.

Question . 159. A sexually active adolescent who has never been treated for syphilis is
found to have a negative result on a VDRL test and a positive result on an MHA-TP. The
most likely explanation is:

Primary syphilis

Secondary syphilis

Latent syphilis

Lyme disease
Explanation: There is limited cross-reactivity of the treponemal test
(MHA-TP, FTA-ABS) material with other spirochetes. Only T.
pallidum and Borrelia burgdorferi, the causative organism of Lyme
disease, are endemic in the United States. Th nontreponemal tests
(VDRL, RPR) are uniformly nonreactive in Lyme disease. (See
Chapter 200 in Nelson Textbook of Pediatrics, 17th ed.)
False-negative results on VDRL testing

Question . 160. The radiologist calls to report that a plain x-ray that you ordered to
evaluate a hip-click shows periostitis. This finding suggests congenital infection by which of
the following agents?

Cytomegalovirus and rubella virus

Toxoplasma gondii

Syphilis
Explanation: Periostitis, occurring in the long bones, is most typical
of congenital syphilis. Osteochondritis is a common finding in
congenital cytomegalovirus, rubella, and syphilis infections. (See
Chapter 200 in Nelson Textbook of Pediatrics, 17th ed.)

Herpes simplex virus

Parvovirus B19

Question . 161. A sexually active adolescent who lives in a rural area with contact with
farm animals presented 1 wk ago with an acute febrile illness associated with headache,
emesis, and myalgias, which all resolved briefly. He now returns with hematuria,
proteinuria, hepatomegaly, and icterus. Which diagnosis is most likely?

Relapsing fever

Leptospirosis
Explanation: A biphasic course is characteristic of icteric
leptospirosis (Weil syndrome). Hepatorenal dysfunction follows
anicteric leptospirosis in less than 10% of cases. (See Chapter 202 in
Nelson Textbook of Pediatrics, 17th ed.)

Infection mononucleosis

Hantavirus pulmonary syndrome

Acute HIV infection

Question . 162. A 6-yr-old child is brought to your office because a tiny tick was found and
removed from his forearm. The parents are unsure how long the tick had been attached,
although they thought that it probably had not been there for more than 1 day. They live in
an area in which Lyme disease is common. The next step in the proper treatment of this
patient should be to:

Send the tick to be tested for evidence of B. burgdorferi.


Reassure the parents that the risk of infection is small and have
them observe the area around the bite for the development of a
rash.
Explanation: The risk of Lyme disease is very small, and observing
the patient for the development of a rash is reasonable. Most patients
with Lyme disease experience erythema migrans even if a tick bite is
not remembered. (See Chapter 204 in Nelson Textbook of Pediatrics,
17th ed.)

Begin prophylactic treatment with amoxicillin.

Order a serologic test for antibodies against B. burgdorferi in the


child.

Wait 1 mo and then order a serologic test for antibodies against B.


burgdorferi in the child.

Question . 163. The preferred agent for treatment of Lyme disease in a child 13 yr of age
is:

Doxycycline

Amoxicillin
Explanation: Doxycycline (for 14-21 days) is the treatment of choice
for Lyme borreliosis in children older than 8 yr of age, but amoxicillin
(for 14-21 days) is the treatment of choice in children younger than 8
yr. Where effective alternatives are available, children younger than 8
yr should not be treated with doxycycline because it may cause
permanent discoloration of the teeth. Erythromycin is an alternative
for persons who cannot take either doxycycline or amoxicillin. (See
Chapter 204 in Nelson Textbook of Pediatrics, 17th ed.)

Ceftriaxone

Erythromycin

Trimethoprim-sulfamethoxazole

Question . 164. A 4-yr-old boy is brought to your office because of a circular reddish rash
on the right side of his chest. The child has been afebrile and has had no other systemic
symptoms. The rash is not pruritic. The child's parents state that they have recently
returned from a vacation in Massachusetts on Cape Cod and that a small tick had been
removed from the same area where the rash is now. The only abnormality on the
examination is the circular, flat, erythematous rash, which is about 6 cm in diameter and is
not tender. The most appropriate next step in treating this patient is to:

Order a test for serum antibodies against Borrelia burgdorferi to


confirm that the child has Lyme disease

Begin treatment with doxycycline

Begin treatment with amoxicillin


Explanation: Amoxicillin is the treatment of choice for this early
lesion of erythema migrans. (See Chapter 204 in Nelson Textbook of
Pediatrics, 17th ed.)

Begin treatment with ceftriaxone

Perform a lumbar puncture to be certain that the child's central


nervous system is not involved

Question . 165. The mother of a 5-yr-old boy notices a new "mole" on her child's neck.
Upon closer inspection, she realizes that it is actually a small tick. She removes it with
tweezers and brings both the tick and the child to your office. They live in northern
Westchester County, New York, in a house surrounded by woods. She is not sure how
long the tick might have been on the child. The most appropriate next step in management
would be to:

Send the tick to be tested by the polymerase chain reaction assay to


ascertain whether it is infected with B. burgdorferi.

Reassure the mother and order a serologic test for antibodies against
B. burgdorferi on the child in 1 mo.

Begin prophylactic treatment with a single dose of doxycycline.

Begin prophylactic treatment with a 10-day course of amoxicillin.

Reassure the mother and instruct her to call if a rash


subsequently develops at the site of the bite.
Explanation: The risk of Lyme disease is very small, and observing
the patient for the development of a rash is reasonable. Most patients
with Lyme disease experience erythema migrans even if a tick bite is
not remembered. (See Chapter 204 in Nelson Textbook of Pediatrics,
17th ed.)

Question . 166. A 13-yr-old boy comes to your office with a chief complaint of fatigue and
intermittent arthralgia and headache for 8 mo. His mother is concerned about Lyme
disease (they live in Connecticut and she recently heard a lecture about Lyme disease at
the public library). She denies any stress at home or at school. There has been no weight
loss. She has not seen any ticks on her son, although he does spend time outdoors. The
child is quiet and answers your questions in monosyllables. Findings on physical
examination are entirely normal, with no evidence of synovitis. The most appropriate next
step in management would be to:
Reassure the mother and order a serologic test for antibodies against
B. burgdorferi.

Reassure the mother and explain that the child is completely normal.

Reassure the mother and order a CT scan of the brain.

Reassure the mother and explain why it is very unlikely that her
child has Lyme disease in the absence of any objective findings.
Explanation: There is no evidence of Lyme disease. It would be
advisable to discuss other possible non-organic causes for the
symptoms. (See Chapter 204 in Nelson Textbook of Pediatrics, 17th
ed.)

Reassure the mother and order tests for infectious mononucleosis


and other possible chronic infections.

Question . 167. A 10-yr-old boy comes to your office with a chief complaint of facial nerve
palsy. He lives in an area of New Jersey in which Lyme disease is endemic but has not had
any recognized tick bites. He has never had fever blisters or cold sores. He has been
afebrile and otherwise asymptomatic. On physical examination, there is a large (8-cm)
erythematous annular rash on his upper back and a unilateral peripheral palsy of the facial
nerve. The neck is supple, and the child appears otherwise well. The most appropriate next
step in management would be to:

Begin treatment for Lyme disease with doxycycline


administered orally.
Explanation: Paralysis of the facial (7th) cranial nerve is relatively
common in children with Lyme disease and may be the initial or the
only manifestation of infection. The paralysis usually lasts 2 to 8 wk
and resolves completely in most cases. There is no evidence that the
clinical course of the facial palsy with Lyme disease is affected by
antimicrobial treatment. (See Chapter 204 in Nelson Textbook of
Pediatrics, 17th ed.)

Begin treatment for Lyme disease with ceftriaxone administered


intravenously.

Defer treatment pending the results of a serologic test for antibodies


against B. burgdorferi.

Obtain a CSF specimen by lumbar puncture.

Obtain a bone marrow aspirate.

question . 168. You have just diagnosed erythema migrans in a 14-yr-old boy. His parents
are concerned about his prognosis because of Lyme disease in a cousin that seems to be
untreatable. Which of the following is the most accurate characterization of the prognosis
for Lyme disease?

Lyme disease is difficult to treat and may require multiple sequential


courses of different antibiotics.

Lyme disease is difficult to treat and may require therapy with the
same antibiotic for several months.

Erythema migrans often proceeds to late Lyme disease, even if


treated appropriately.

Lyme disease among some families may be more resistant to


treatment.

The prognosis for children treated for Lyme disease is excellent.


Explanation: There is a widespread misconception that Lyme
disease is difficult to treat successfully and that chronic symptoms
and clinical recurrences are common. In fact, the most common
reason for apparent treatment failure is misdiagnosis in patients who
do not have Lyme disease. The impression that Lyme disease
requires prolonged treatment, including intravenous antimicrobial
therapy, and that treatment is often unsuccessful can be attributed to
the treatment of patients whose symptoms were not due to Lyme
disease. (See Chapter 204 in Nelson Textbook of Pediatrics, 17th ed.)

Question . 169. The recommended agent for treatment of pneumonia caused by


Mycoplasma pneumoniae is:

Amoxicillin or ampicillin

Ceftriaxone, cefotaxime, or cefotetan

Erythromycin, clarithromycin, or azithromycin


Explanation: The newer macrolide antibiotics clarithromycin and
azithromycin are preferred because they are tolerated better than
erythromycin. Doxycycline is an alternative for patients older than 8 yr. (See
Chapter 205 in Nelson Textbook of Pediatrics, 17th ed.)

Gentamicin or kanamycin

Trimethoprim-sulfamethoxazole

Question . 170. An 18-yr-old sexually active boy complains of penile discomfort and
dysuria that has developed slowly over the past 12 days. Physical examination reveals a
scanty, mucoid white urethral discharge. A Gram stain is most likely to reveal:

Polymorphonuclear leukocytes and gram-negative diplococci


Polymorphonuclear leukocytes and gram-positive diplococci

Polymorphonuclear leukocytes and gram-negative bacilli

Polymorphonuclear leukocytes and gram-negative bacilli

Polymorphonuclear leukocytes only


Explanation: Penile discomfort and dysuria accompanied by a
scanty, mucoid-white urethral discharge are a classic presentation of
nongonococcal urethritis in males. This is caused by Ureaplasma
urealyticum and Mycoplasma hominis. The diagnosis of
nongonococcal urethritis is confirmed by Gram stain of urethral
discharge showing at least three polymorphonuclear leukocytes per
oil-immersion field and the absence of gram-negative diplococci.
(See Chapter 206 in Nelson Textbook of Pediatrics, 17th ed.)

Question . 171. For the patient described in Question 170, which of the following is a
recommended treatment?

Azithromycin
Explanation: Nongonococcal urethritis in adolescents and adults is
treated with azithromycin (1 g PO as a single dose) or doxycycline
(100 mg bid PO for 7 days). Sexual partners should also be treated
to avoid recurrent disease. (See Chapter 206 in Nelson Textbook of
Pediatrics, 17th ed.)

Benzathine penicillin G weekly for 3 doses

Cefotetan

Ceftriaxone

Gentamicin

Question . 172. A 2-mo-old infant presents with conjunctivitis, tachypnea, and a mild
cough. There is no fever. Physical examination reveals the presence of crackles bilaterally.
The most likely etiologic agent is:

Influenza virus

Parainfluenza virus

Respiratory syncytial virus

Adenovirus
Chlamydia trachomatis
Explanation: C. trachomatis infection is characterized by insidious
onset of persistent cough and tachypnea, with the notable absence of
fever. Rales are common, but wheezes are uncommon, which helps
distinguish C. trachomatis from RSV pneumonia. (See Chapter 208.3
in Nelson Textbook of Pediatrics, 17th ed.)

Question . 173. Manifestations of Chlamydia trachomatis pneumonia include all of the


following except:

Fever
Explanation: C. trachomatis pneumonia is not associated with fever.
(See Chapter 208.3 in Nelson Textbook of Pediatrics, 17th ed.)

Conjunctivitis

Cough

Tachypnea

Eosinophilia

Question . 174. The recommended treatment for Chlamydia trachomatis conjunctivitis or


pneumonia is:

Erythromycin eye drops for 7 days

Erythromycin eye drops and oral erythromycin for 7 days

Oral erythromycin for 2 wk


Explanation: The recommended treatment for C. trachomatis
conjunctivitis or pneumonia is erythromycin 50 mg/kg/24 hr in 2 or 4
divided doses orally for 14 days. Topical therapy of conjunctivitis
does not reduce the high risk of subsequent pneumonia. (See Chapter
208.3 in Nelson Textbook of Pediatrics, 17th ed.)

Trimethoprim-sulfamethoxazole for 7 days

Ceftriaxone given as a single intramuscular dose

Question . 175. A 15-yr-old boy presents to your office with complaint of fever, malaise,
headache, cough, and shortness of breath. A chest radiograph reveals left upper and lower
lobe infiltrates. His WBC count is elevated. He states that he recently received a pet
cockatiel that became ill and died. The most likely diagnosis is:

Mycoplasma pneumonia
Pneumococcal pneumonia

Psittacosis
Explanation: Psittacosis, a bird-borne chlamydial disease, produces
severe pneumonia with systemic manifestations similar to those of
Mycoplasma pneumoniae or Legionnaires' disease. (See Chapter 209
in Nelson Textbook of Pediatrics, 17th ed.)

Q fever

Legionnaires' disease

Question . 176. The most appropriate next step in diagnosis for the case described in
Question 175 is:

Blood culture

Throat culture

Serologic testing
Explanation: Culture is of poor value, but serologic studies are the
diagnostic tests of choice for psittacosis. (See Chapter 209 in Nelson
Textbook of Pediatrics, 17th ed.)

Bronchoalveolar lavage and cultures

MRI of the chest

Question . 177. All of the following laboratory findings are suggestive of Rocky Mountain
spotted fever except:

Normal to slightly low leukocyte count

Shift to the left

Reticulocytopenia
Explanation: Early laboratory clues to Rocky Mountain spotted fever
include normal to slightly low leukocyte count, a shift to the left
(increased bands), thrombocytopenia, and low serum sodium. (See
Chapter 210.1 in Nelson Textbook of Pediatrics, 17th ed.)

Thrombocytopenia

Low serum sodium

Question . 178. An 8-yr-old child presents with headache, fever, anorexia, and myalgias
with onset 3 days ago, now accompanied by a petechial rash that is prominent on the
extremities, including the palms and soles (Figure). The most likely diagnosis is:

Infective endocarditis

Meningococcemia

Rocky Mountain spotted fever


Explanation: The rash early in the illness of Rocky Mountain spotted
fever is prominent over the extremities. Peripheral edema of the
wrists and ankles is also evident. (See Chapter 210.1 in Nelson
Textbook of Pediatrics, 17th ed.)

Henoch-Sch nlein purpura

Systemic lupus erythematosus

Question . 179. The recommended agent for treatment of Rocky Mountain spotted fever in
a child younger than 8 yr is:

Ceftriaxone

Chloramphenicol

Doxycycline
Explanation: Chloramphenicol and tetracyclines have proven
efficacy against Rocky Mountain spotted fever, but chloramphenicol
may be associated with higher mortality. Dental staining is unlikely
with a single course of a tetracycline. Doxycycline is recommended
because the risk of dental staining is less than with other
tetracyclines. (See Chapter 210.1 in Nelson Textbook of Pediatrics,
17th ed.)

Clindamycin

Quinidine

Question . 180. All of the following may be characteristic of Rocky Mountain spotted fever
except:

History of tick exposure

Presentation between April and September

Headache and myalgias

Rash that begins centrally and spreads to the periphery


Explanation: The diagnosis of Rocky Mountain spotted fever should
be suspected in persons with an acute febrile illness with headache
and myalgias, especially during the spring through the fall with a
history of tick exposure. The maculopapular rash appears on the
extremities, including the ankles, wrists, and lower legs. The rash
then spreads rapidly to involve the entire body, including the palms
and soles, as the rash progresses to petechiae and purpura. (See
Chapter 210.1 in Nelson Textbook of Pediatrics, 17th ed.)

Rash involving the palms and soles

Question . 181. The recommended treatment for human ehrlichiosis is:

Ceftriaxone

Chloramphenicol

Doxycycline
Explanation: Unlike with Rickettsia rickettsii, the cause of Rocky
Mountain spotted fever, chloramphenicol is not effective against
Ehrlichia spp. Dental staining is unlikely with a single course of a
tetracycline. Doxycycline is recommended because the risk of dental
staining is less than with other tetracyclines. (See Chapter 213 in
Nelson Textbook of Pediatrics, 17th ed.)

Clindamycin

Quinidine

Question . 182. A 5-yr-old boy has had fever, headache, abdominal pain, and muscle
aches for the preceding 3 to 4 days. His temperature is 103.4°F, heart rate is 130/min, and
respiratory rate is 40/min. He appears acutely ill and dehydrated. He has no rash. His
family had recently been camping in rural Wisconsin, but there is no history of tick bite.
Laboratory findings include WBC count of 2,300/mm3, 24% segmented neutrophils, 65%
bands, and 8% lymphocytes, and platelet count of 57,000/mm3. The AST level is 465 IU/L.
A peripheral blood smear reveals small blue clusters of bacteria-like bodies in an
aggregate within the cytoplasm of 1% of circulating mononuclear leukocytes. The most
likely diagnosis is:

Meningococcemia

Staphylococcal septicemia
Rocky Mountain spotted fever

Ehrlichiosis
Explanation: Human ehrlichiosis is a rickettsial multisystem illness,
which may be accompanied by a rash. It is endemic and probably is
more common than reported. (See Chapter 213 in Nelson Textbook of
Pediatrics, 17th ed.)

Hemolytic-uremic syndrome

Question . 183. A 17-yr-old adolescent who works on his family's sheep ranch presents in
August with an influenza-like illness with interstitial pneumonitis. There is no history of a
tick bite. Laboratory tests reveal leukopenia and elevated serum transaminase levels.
Which of the following pathogens is the most likely cause of this illness?

Bartonella henselae

Leptospira

Coxiella burnetii
Explanation: Acute Q fever is transmitted by inhalation of infectious
aerosols; arthropod vectors are rarely implicated in human cases.
(See Chapter 214 in Nelson Textbook of Pediatrics, 17th ed.)

Ancylostoma

Echinococcus

Question . 184. What is the most appropriate next step in diagnosis for the case described
in Question 183?

Blood culture

Throat culture

Serologic testing
Explanation: The diagnosis of Q fever is most easily confirmed
serologically by testing acute and convalescent sera (2-4 wk apart),
which should show a fourfold increase in indirect fluorescent antibody
titers to phase I and phase II antigens. (See Chapter 214 in Nelson
Textbook of Pediatrics, 17th ed.)

Bronchoalveolar lavage and cultures


MRI of the chest

Question . 185. Which of the following is the most significant risk factor for fungal sepsis in
a premature infant?
Meconium aspiration

Intravenous lipid infusion

Previous administration of vancomycin

Postconceptional age

Heavy colonization with Candida


Explanation: The inability of the newborn infant to localize Candida
colonization facilitates overgrowth of Candida on mucocutaneous
surfaces, which is a major predisposing factor for neonatal Candida
infections. (See Chapter 215 in Nelson Textbook of Pediatrics, 17th
ed.)

Question . 186. The recommended treatment of a central line infection due to Candida is:

Amphotericin B therapy

Flucytosine therapy

Removal of the central line and amphotericin B therapy


Explanation: Treatment of Candida central line infections includes
removal of the central line as well as parenteral amphotericin B for 2-
3 wk. (See Chapter 215 in Nelson Textbook of Pediatrics, 17th ed.)

Removal of the central line and flucytosine therapy

Daily iodine antisepsis of the catheter site

Question . 187. A premature newborn on broad-spectrum antibiotics with a central venous


catheter appears to be in shock, and you suspect either bacteria or Candida. The
recommended culture studies for this patient include:

Blood cultures using only routine blood culture media


Explanation: Candida grows readily on routine blood culture media,
and therefore special media are unnecessary. Cultures should also
include the possibility of antibiotic-resistant bacteria. (See Chapter 215
in Nelson Textbook of Pediatrics, 17th ed.)

Routine blood cultures and also fungal cultures using Sabouraud


culture media
Routine blood cultures and also fungal cultures using brain-heart
infusion media

Routine blood cultures and also fungal cultures using Bordet-Gengou


culture media

Only fungal cultures, because the newborn is on antibiotics

Question . 188. The microbiology laboratory calls to report that an organism has been
detected in a blood culture, and the rapid germ tube test is positive. Which of the following
organisms has been identified?

Aspergillus

Candida albicans
Explanation: Candida albicans is the only species that forms a germ
tube when suspended in rabbit or human serum and incubated for 1-
2 hr. (See Chapter 215 in Nelson Textbook of Pediatrics, 17th ed.)

Candida krusei

Cryptococcus

Malassezia furfur

Question . 189. An otherwise healthy 5-yr-old girl presents with focal back pain of insidious
onset. Her father is a pigeon fancier, and the child helps him care for the pigeons. Her past
medical history is also remarkable for an episode of "bronchitis" 1 yr ago. A plain film of the
back reveals destruction of the T8 vertebral body. The most likely diagnosis is:

Aspergillosis

Blastomycosis

Cryptococcosis
Explanation: Pigeon breeders and laboratory personnel who work
with Cryptococcus are at greatest risk for cryptococcosis. (See
Chapter 216 in Nelson Textbook of Pediatrics, 17th ed.)

Histiocytosis

Histoplasmosis
Question . 190. The use of intravenous lipids predisposes most significantly to catheter-
related infections and fungemia due to which of the following organisms?

Aspergillus

Candida albicans

Cryptococcus

Histoplasma

Malassezia furfur
Explanation: Catheter-related infections and fungemia with
Malassezia furfur occur almost exclusively in patients receiving
intravenous lipids. The use of lipid emulsions containing medium-
chain triglycerides inhibits the growth of Malassezia. (See Chapter 217
in Nelson Textbook of Pediatrics, 17th ed.)

Question . 191. A newborn in the NICU is intubated and on total parenteral nutrition,
including lipid emulsions. A blood culture yields Malassezia furfur. The most important
element of treatment for this infection is:

Intravenous itraconazole

Intravenous amphotericin B

Oral fluconazole

Discontinuing intravenous lipid infusion


Explanation: Catheter-related infections and fungemia with
Malassezia furfur occur almost exclusively in patients receiving
intravenous lipids. The use of lipid emulsions containing medium-
chain triglycerides inhibits the growth of Malassezia. (See Chapter 217
in Nelson Textbook of Pediatrics, 17th ed.)

Topical econazole applied to the catheter site

Question . 192. A 17-yr-old boy with cystic fibrosis develops recurrent bronchospasm and
transient pulmonary infiltrates. Laboratory testing reveals mycelia in the sputum, and a
peripheral eosinophilia. The most likely causative organism is:

Burkholderia cepacia

Stenotrophomonas maltophilia

Aspergillus
Explanation: Allergic bronchopulmonary aspergillosis complicates
chronic pulmonary disease in approximately 10% of persons with
cystic fibrosis. (See Chapter 218.1 in Nelson Textbook of Pediatrics,
17th ed.)

Histoplasma

Malassezia furfur

Question . 193. All of the following are requirements for the diagnosis of allergic
bronchopulmonary aspergillosis except:

Asthma

Immediate cutaneous reactivity to Aspergillus fumigatus antigens by


RAST

Elevated total serum IgE level

Peripheral eosinophilia

Pulmonary hyperinflation and hilar lymphadenopathy


Explanation: Hyperinflation due to air trapping is not always noted
with allergic bronchopulmonary aspergillosis, nor is hilar
lymphadenopathy. The inflammatory response does not necessarily
produce either of these lesions. (See Chapter 218.1 in Nelson
Textbook of Pediatrics, 17th ed.)

Question . 194. A 16-yr-old boy is brought to your office for a second opinion. He was well
until 2 wk ago when he developed a fever and cough. He was seen at a local urgent care
center, where a chest film demonstrated focal calcifications throughout both lungs with
some hilar calcifications noted bilaterally. A 5 U PPD placed at that time was negative;
Candida controls were positive. His history is remarkable for an episode of "bronchitis" 2 yr
ago following a spelunking adventure in Kentucky. The most likely diagnosis is:

Aspergillosis

Blastomycosis

Cryptococcosis

Histiocytosis

Histoplasmosis
Explanation: Histoplasma capsulatum thrives in soil rich in nitrates
such as areas heavily contaminated with bird droppings, and with bat
guano in caves and along bridges frequented by bats. (See Chapter
219 in Nelson Textbook of Pediatrics, 17th ed.)

Question . 195. Which of the following organisms is the most likely cause of fungal
pneumonia in immunocompetent persons living in the Ohio and Mississippi River valleys?

Aspergillus

Blastomyces dermatitidis

Coccidioides immitis

Cryptococcus neoformans

Histoplasma capsulatum
Explanation: H. capsulatum is found in the soil throughout the
Midwestern United States. Blastomyces is found in the same areas
but is less common except in the northern Midwestern states (e.g.,
Wisconsin). (See Chapter 219 in Nelson Textbook of Pediatrics, 17th
ed.)

Question . 196. An adolescent with a history of spelunking presents with weight loss,
fatigue, dyspnea, and fever of onset 2 wk previously. Chest film shows evidence of
pneumonia. The most likely causative organism is:

Aspergillus

Blastomyces dermatitidis

Coccidioides immitis

Cryptococcus neoformans

Histoplasma capsulatum
Explanation: H. capsulatum thrives in soil rich in nitrates such as
areas heavily contaminated bat guano, in caves and along bridges
frequented by bats. (See Chapter 219 in Nelson Textbook of
Pediatrics, 17th ed.)

Question . 197. The most likely cause of fungal pneumonia in immunocompetent persons
living in arid areas of California, Arizona, and southwestern Texas is:

Aspergillus

Blastomyces dermatitidis

Coccidioides immitis
Explanation: C. immitis is found in arid areas of California's San
Joaquin Valley (Valley fever), central and southern Arizona, and
southwestern Texas. (See Chapter 221 in Nelson Textbook of
Pediatrics, 17th ed.)

Cryptococcus neoformans

Histoplasma capsulatum

Question . 198. A 4-yr-old girl develops nodular lymphangitis after being bitten by a cat.
The most likely etiologic agent is:

Aeromonas hydrophila

Mycobacterium marinum

Nocardia brasiliensis

Pasteurella multocida

Sporothrix schenckii
Explanation: Sporothrix thrives in decaying vegetation but also can
be transmitted by bites and scratches of animals, most frequently
cats and armadillos. (See Chapter 223 in Nelson Textbook of
Pediatrics, 17th ed.)

Question . 199. Which of the following statements concerning measles virus is true?

There is no animal reservoir

There is no vector

There is no transmissible latent virus

There is only one serotype

All of the above


Explanation: Several features of measles support the possibility of
global eradication, including the absence of an animal reservoir,
vector, transmissible latent virus, and the presence of only one
serotype. (See Chapter 225 in Nelson Textbook of Pediatrics, 17th
ed.)

Question . 200. All of the following vaccines may be given to children with severe
combined immunodeficiency syndrome except:
Diphtheria, tetanus, pertussis (DTaP) vaccine

Measles virus vaccine


Explanation: The live measles virus is contraindicated in patients
with T-cell immunodeficiency because disseminated disease may
occur. Nonetheless, it is recommended that HIV-infected patients be
given this vaccine because measles itself is a serious illness once
AIDS develops. (See Chapters 225 and 282 in Nelson Textbook of
Pediatrics, 17th ed.)

Salk poliovirus vaccine

Hepatitis B virus vaccine

Pneumococcal vaccine

Question . 201. Which of the following statements concerning measles and vitamin A is
true?

There is not a confirmed relationship of vitamin A to the prognosis of


measles

Measles causes vitamin A deficiency

Treatment with vitamin A reduces measles severity for children


in developing countries
Explanation: There is an apparent correlation between retinol
concentration and measles severity. Treatment with vitamin A
reduces morbidity and mortality in children with severe measles in
developing countries and is recommended for selected children with
severe measles in the United States. (See Chapter 225 in Nelson
Textbook of Pediatrics, 17th ed.)

Subacute sclerosing panencephalitis (SSPE) is more likely in children


with underlying vitamin A deficiency

Treatment with vitamin A reduces the incidence of SSPE

Question . 202. All of the following are characteristic manifestations of congenital rubella
syndrome except:

Snuffles
Explanation: Congenital rubella affects virtually all organ systems.
Snuffles is a sign of congenital syphilis. (See Chapter 226 in Nelson
Textbook of Pediatrics, 17th ed.)

Intrauterine growth retardation


Cataracts

Structural cardiac defects

Sensorineural hearing loss

Question . 203. The most common complication of mumps in childhood is:

Arthritis

Meningoencephalitis
Explanation: Meningoencephalitis is the most common complication
of mumps in childhood, but appears to occur in more than two-thirds
of cases, with clinical symptoms in more than 10% of cases. (See
Chapter 227 in Nelson Textbook of Pediatrics, 17th ed.)

Myocarditis

Orchitis

Pancreatitis

Question . 204. A 14-yr-old boy and his twin sister each developed mumps. Their parents
had repeatedly declined for them to receive MMR vaccination. They ask about the
prognosis for this disease in their children, especially the possibility of infertility. Which of
the following statements most accurately describes the relationship between mumps and
infertility?

Infertility occurs as a sequela of mumps only among prepubertal


children

Infertility is more common among females than among males

Infertility rate among males with mumps orchitis is approximately 30-


40%

Infertility among males is rare even with bilateral orchitis


Explanation: Infertility among males with mumps is rare, even with
bilateral orchitis. There is no evidence of impairment of fertility among
women with mumps oophoritis. Mumps orchitis and oophoritis are
rare among prepubescent boys and girls, and are more common
among postpubertal men (14-35%) and women (7%). (See Chapter
227 in Nelson Textbook of Pediatrics, 17th ed.)
Infertility rate among females with mumps oophoritis is approximately
30-40%

Question . 205. The percentage of poliovirus infections that are inapparent is:

5-10%

20-25%

50%

75-80%

90-95%
Explanation: Approximately 90-95% of poliovirus infections are
inapparent, causing no paralytic disease and no sequelae. (See
Chapter 228 in Nelson Textbook of Pediatrics, 17th ed.)

Question . 206. Most infections with poliovirus result in illness that is best characterized
as:

Subclinical illness
Explanation: Approximately 90-95% of poliovirus infections are
inapparent, causing no paralytic disease and no sequelae. (See
Chapter 228 in Nelson Textbook of Pediatrics, 17th ed.)

Nonspecific symptoms (e.g., mild fever, malaise, headache)

Aseptic meningitis

Aseptic meningitis with encephalitis (meningoencephalitis)

Paralytic poliomyelitis

Question . 207. Which of the following features distinguishes paralytic polio from Guillain-
Barré syndrome?

Pleocytosis is uncommon in paralytic polio

Paralysis is usually asymmetric in paralytic polio


Explanation: Paralytic polio is characterized by aseptic meningitis
accompanied by asymmetric flaccid paralysis without sensory loss. In
Guillain-Barré syndrome, the paralysis is characteristically symmetric,
and sensory changes (paresthesias) are common. Pleocytosis is
common in polio, whereas the cerebrospinal fluid in Guillain-Barré
syndrome usually shows only elevated protein and occasionally a few
cells. (See Chapter 228 in Nelson Textbook of Pediatrics, 17th ed.)

The paralysis of polio is usually spastic

Sensory changes are common in paralytic polio

Paralytic polio only occurs in unimmunized persons

Question . 208. All of the following statements concerning the risk of vaccine-associated
paralytic poliomyelitis (VAPP) are true except:

The risk is associated only with OPV and not IPV

The risk is higher after the second and third doses than after the
first dose
Explanation: Vaccine-associated paralytic poliomyelitis (VAPP),
which follows reversion of the OPV strain to a neurovirulent strain,
has accounted for all cases of polio in the United States since 1979.
The risk is higher in vaccinees than in contacts, after the first dose,
and in immunocompromised persons. (See Chapters 228 and 282 in
Nelson Textbook of Pediatrics, 17th ed.)

The risk is higher for immunocompromised persons

The risk is present for household contacts as well as vaccinees

The risk for VAPP in the United States has exceeded the risk for wild-
type polio since 1979

Question . 209. A 5-yr-old boy who lives in an urban area is hospitalized because of low-
grade fever, flaccid paralysis of both legs, sensory changes, and absent ankle deep tendon
reflexes. The child received only two immunizations with OPV, at the ages of 2 and 6 mo.
The most probable diagnosis is:

Acute paralytic poliomyelitis due to wild poliovirus

Paralysis due to nonpolio enteroviruses

Vaccine-associated poliomyelitis

Guillain-Barré syndrome
Explanation: Guillain-Barré syndrome, or autoimmune peripheral
neuropathy, is symmetric and involves sensory but more so motor
nerves. Polio is an anterior horn cell disease and is purely motor.
(See Chapter 229 in Nelson Textbook of Pediatrics, 17th ed.)

Tick-bite paralysis
Question . 210. All of the following may be manifestations of enterovirus infection except:

Aseptic meningitis

Herpangina

Hand, foot, and mouth disease

Aphthous stomatitis
Explanation: Infections due to enteroviruses have a wide spectrum
of clinical manifestations. Herpangina is usually caused by echovirus
type 9. Hand-foot-and-mouth disease is usually caused by
coxsackievirus A16. Diarrhea is probably less common than is
taught. (See Chapter 229 in Nelson Textbook of Pediatrics, 17th ed.)

Vomiting and diarrhea

Question . 211. Advantages of inactivated poliovirus vaccine over live poliomyelitis


vaccine include:

Induction of a high level of intestinal mucosal immunity

Oral administration

No requirement for a booster dose at age 4-6 yr

No association with vaccine-associated paralytic poliomyelitis


(VAPP)
Explanation: Vaccine-associated paralytic poliomyelitis (VAPP) is
not associated with the inactivated poliovirus vaccine. (See Chapters
228 and 282 in Nelson Textbook of Pediatrics, 17th ed.)

Longer duration of immunity

Question . 212. Which of the following factors is associated with increased risk of
enterovirus infection?

Wintertime in temperate climates

Upper socioeconomic class

Recent receipt of poliovirus vaccine

Young age
Explanation: Factors associated with increased incidence and/or
severity of enterovirus infection include young age, male sex, poor
hygiene, overcrowding, and low socioeconomic status; >25% of
symptomatic enterovirus infections occur in children younger than 1
yr of age. Breast-feeding reduces the risk of infection in infants. (See
Chapter 229 in Nelson Textbook of Pediatrics, 17th ed.)

Breast-feeding

Question . 213. Which of the following is an accurate description of an enterovirus?

A large, negative-sense RNA virus

A member of a genus containing 11 different serotypes

An unenveloped virus able to replicate in the intestinal tract


Explanation: Enteroviruses are small, unenveloped viruses that are
able to replicate in the intestinal tract. (See Chapter 229 in Nelson
Textbook of Pediatrics, 17th ed.)

A small virus restricted to intestinal tract mucosal infection

A virus whose infection is primarily limited by cellular immunity

Question . 214. All of the following are infections typically caused by enteroviruses except:

Neonatal sepsis

Osteomyelitis
Explanation: Although more than 60 different serotypes of
enteroviruses have been identified, 11 account for the majority of
disease. No enterovirus disease is uniquely associated with any
specific serotype; however, certain manifestations are preferentially
associated with specific serotypes. Osteomyelitis is not associated
with enteroviruses. (See Chapter 229 in Nelson Textbook of
Pediatrics, 17th ed.)

Myocarditis

Hand, foot, and mouth[HBJ1] disease

Meningitis

Question . 215. Which of the following is a feature of epidemics of hand-foot-and-mouth


disease associated with enterovirus 71?

High rates of encephalitis and cardiopulmonary complications


Hand, foot, and mouth[HBJ2] disease caused by enterovirus 71 is
frequently more severe than that due to coxsackievirus A16, with
high rates of associated neurologic disease including aseptic
meningitis, encephalitis, and paralysis. Recent outbreaks in Malaysia,
Japan, and Taiwan have been notable for brainstem
encephalomyelitis, neurogenic pulmonary edema, pulmonary
hemorrhage, shock, and rapid progression to death, especially in
young children. (See Chapter 229 in Nelson Textbook of Pediatrics,
17th ed.)

High rates of concomitant hemorrhagic conjunctivitis

Highest mortality in adolescents and young adults

Occurrence of disease primarily in immunocompromised hosts

High rates of severe gastrointestinal complications

Question . 216. Which of the following statements regarding neonatal enterovirus


infections is true?

They are much less common than infections due to herpes simplex
virus and cytomegalovirus

They are invariably mild, benign illnesses

They are best treated with ribavirin

They may cause life-threatening hepatitis and coagulopathy


Explanation: Most symptomatic neonates with neonatal enterovirus
infection have benign courses, but a minority has severe disease that
may be dominated by any combination of sepsis,
meningoencephalitis, myocarditis, hepatitis, coagulopathy, and
pneumonitis. (See Chapter 229 in Nelson Textbook of Pediatrics, 17th
ed.)

They generally occur only in extremely low birthweight infants

Question . 217. Which of the following statements regarding the diagnosis of enterovirus
infections is true?

Useful diagnostic techniques are not available

Polymerase chain reaction assay is very sensitive


Explanation: PCR assay detects the majority of enteroviruses (but
frequently not echoviruses 22 and 23) and has been applied to a
variety of specimens, including cerebrospinal fluid, serum, urine, and
conjunctival, nasopharyngeal, throat, and rectal specimens. (See
Chapter 229 in Nelson Textbook of Pediatrics, 17th ed.)

Useful diagnostic specimens are limited to throat and rectal swabs

All enteroviruses grow well in cell culture

Antigen detection is a useful way to detect all enteroviruses

Question . 218. All of the following may be manifestations of parvovirus B19 infection
except:

Facial rash ("slapped-cheek" appearance)

Lacy, reticulated rash over the trunk and proximal extremities

Arthritis

Transient reversal of the CD4:CD8[HBJ3] ratio


Explanation: The rash occurs in three stages, which are not always
clinically distinguishable. After the initial "slapped-cheek"
appearance, the rash spreads to become a lacy, reticulated rash over
the trunk and proximal extremities. Arthritis and arthralgia are much
more common in adults, especially in females, than in children. The
transient arrest of erythropoiesis is usually clinically silent in
previously healthy persons. (See Chapter 230 in Nelson Textbook of
Pediatrics, 17th ed.)

Reticulocytopenia

Question . 219. During springtime, an infectious disease spread through a small


community in the United States. The principal signs and symptoms were fever, mild rash,
and arthralgia. One pregnant woman in late gestation contracted the illness but recovered
without sequelae. However, 1 mo later she gave birth to a stillborn infant. The pathology
report listed the diagnosis "hydrops fetalis." Which of the following congenital infections is
the most likely etiology?

Congenital rubella virus infection

Congenital cytomegalovirus infection

Congenital parvovirus B19 infection


Explanation: Parvovirus B19, the agent of fifth disease (erythema
infectiosum), produces congenital infection of the fetal erythrocyte
precursor cells, producing transient fetal anemia. If the anemia is
severe, it produces nonimmune hydrops with the possibility of
intrauterine fetal demise. Intrauterine (umbilical venous) blood
transfusion is curative but poses risk. (See Chapter 230 in Nelson
Textbook of Pediatrics, 17th ed.)

Congenital herpes simplex virus infection

Congenital HIV infection

Question . 220. All of the following statements regarding herpes simplex virus (HSV)
infections in neonates are true except:

Most cases are caused by HSV type 2

Women with primary HSV genital tract infection are more likely to
transmit infection to their offspring than women with recurrent HSV
infection

Most mothers of newborns with perinatal HSV infection have a


history of genital HSV infection
Explanation: Only 15-20% of mothers of newborns with perinatal
HSV have a history of obvious HSV infection, and only about 25%
have any relevant symptoms at birth. (See Chapter 231 in Nelson
Textbook of Pediatrics, 17th ed.)

Most mothers of newborns with perinatal HSV infection are


asymptomatic at delivery

Most cases are transmitted at delivery and are not true congenital
infections

Question . 221. Recommended management for a mother with active genital HSV
infection during labor is:

Culture of blood from the newborn, with treatment based on culture


results

Culture of blood from the newborn, with empirical acyclovir therapy

Intravenous acyclovir treatment for the mother

Cesarean section within 4 hr of rupture of membranes


Explanation: Both the American Academy of Pediatrics and the
American College of Obstetrics and Gynecology recommend
cesarean section if primary, first-episode, or recurrent HSV lesions
are present on the mother at the onset of labor. Only 15-20% of
mothers of newborns with perinatal HSV have a history of HSV
infection. (See Chapter 231 in Nelson Textbook of Pediatrics, 17th ed.)

Intravenous acyclovir treatment for the mother and cesarean section


within 4 hr of rupture of membranes
Question . 222. A 3-yr-old boy presents with a 7-day history of fever, cervical
lymphadenopathy, foul breath, and painful oral lesions on his tongue, gums, and lips. For
the past 3 days he has had a red, painful swollen area about the nail of his right thumb with
an area of fluid by the nail bed, unresponsive to warm soaks and a first-generation
cephalosporin. The most likely etiologic agent is:

Staphylococcus aureus

Mucocutaneous candidiasis

Coxsackievirus

Adenovirus

Herpes simplex virus


Explanation: The child has herpetic gingivostomatitis. He has
autoinoculated his thumb by sucking, and herpetic whitlow has
developed. (See Chapter 231 in Nelson Textbook of Pediatrics, 17th
ed.)

Question . 223. A 13-mo-old previously healthy child presents on New Year's Eve with a
2-day history of fever, lethargy, and irritability. Earlier this afternoon he began to have
twitching movements of his left arm and on the left side of his face. His immunizations are
up-to-date. Physical examination reveals fever with a temperature of 39°C and left-sided
weakness, with no rashes. Examination of the cerebrospinal fluid (CSF) reveals 70
WBCs/mm3 with 85% lymphocytes, 400 RBCs/mm3, protein of 140 mg/dL, glucose 80
mg/dL, and negative results on Gram stain. MRI scan reveals right temporal abnormalities.
The diagnostic study most likely to identify a treatable illness in a timely fashion is:

Viral CSF culture

Herpes simplex virus polymerase chain reaction (PCR) assay


Explanation: The focal seizure, focal physical findings, the CSF
profile, and temporal lobe lesion on MRI are highly suggestive of
herpes simplex virus encephalitis. The fastest, most specific means
of diagnosis is by PCR assay of CSF. (See Chapter 231 in Nelson
Textbook of Pediatrics, 17th ed.)

Acute and convalescent antibody titers

Comparison of maternal and infant antibody titers

CSF bacterial culture

Question . 224. For the patient described in Question 223, the most appropriate empirical
therapy to begin while awaiting definitive diagnosis is:
Ceftriaxone

Nafcillin, cefotaxime, and metronidazole

Acyclovir
Explanation: The focal seizure, focal physical findings, the CSF
profile, and temporal lobe lesion on MRI are highly suggestive of
herpes simplex virus encephalitis. Acyclovir must be included in the
initial treatment. (See Chapter 231 in Nelson Textbook of Pediatrics,
17th ed.)

Amphotericin B

Isoniazid, rifampin, pyrazinamide, and streptomycin

Question . 225. Most infants with localized neonatal herpes encephalitis become
symptomatic at the age of:

0-2 days

2-8 days

8-12 days
Explanation: Localized skin, eye, and mouth infection and also
disseminated infection occur at a mean of 5-6 days post partum,
whereas localized CNS infection occurs later at a mean of 8-12 days
post partum. (See Chapter 231 in Nelson Textbook of Pediatrics, 17th
ed.)

13-21 days

Older than 21 days

Question . 226. At 6 mo after stem cell transplantation, a 4-yr-old boy has had several
episodes of recurrent oral herpes simplex virus infection, each responsive to episodic
acyclovir treatment. For the past 2 wk he has been experiencing progressive oral and lip
lesions that have spread to his face and neck, in spite of first oral and then high-dose
intravenous acyclovir. The viral culture is positive for herpes simplex virus. The most
appropriate treatment to begin during the wait for results of further studies on the virus is:

Valacyclovir

Famciclovir

Ganciclovir
Foscarnet
Explanation: The repeated antiviral treatment of herpes simplex
virus disease in an immunocompromised person promotes the
development of resistant HSV. Foscarnet should be used during the
wait for results of viral susceptibility testing. (See Chapter 231 in
Nelson Textbook of Pediatrics, 17th ed.)

Gamma interferon

Question . 227. A 9-day-old neonate, born by vaginal delivery to a 21-yr-old healthy


woman, is presented with fever, lethargy, and poor feeding. There are no diagnostic
findings on physical examination, and results of sepsis evaluation, including a CBC and
cerebrospinal fluid studies, are unremarkable. Ampicillin and cefotaxime are begun. Two
days later all bacterial cultures are negative but the child's clinical condition worsens, with
falling blood pressure, decreased level of consciousness, thrombocytopenia, and elevated
liver enzymes. The indicated change in treatment is:

Addition of amphotericin

Addition of vancomycin

Addition of ribavirin

Addition of acyclovir
Explanation: The history, inability to confirm bacterial infection, and
worsening clinical condition are suggestive of neonatal herpes
simplex virus infection. Acyclovir therapy should be initiated. Repeat
cerebrospinal fluid analysis or MRI might also be recommended.
(See Chapter 231 in Nelson Textbook of Pediatrics, 17th ed.)

Substitution with meropenem and amikacin

Question . 228. An otherwise healthy 16-yr-old high school wrestling star presents with a
6-mo history of a recurrent vesicular eruption on one side of his face. He has no other
history of unusual infections and is HIV negative. His immunizations are appropriate for his
age, and he has never had chickenpox. The most likely diagnosis is:

Recurrent group A streptococcal infection

Shingles

Recurrent herpes simplex virus


Explanation: This is a characteristic history of herpes gladiatorum.
(See Chapter 231 in Nelson Textbook of Pediatrics, 17th ed.)

Recurrent facial mat burns

Traumatic candidiasis
Question . 229. A 10-mo-old girl is presented in January with a 2-day history of fever with
temperatures to 103.5°F and refusal to eat. Physical examination reveals a temperature of
40°C, general irritability, and numerous ulcers on the anterior portion of her buccal
mucosa, gums, and tongue. Appearance of the throat is unremarkable. There are bilateral
tender enlarged cervical lymph nodes. The recommended therapeutic agent is:

Acyclovir
Explanation: The fever, irritability, and mucosal ulcerations suggest
acquired herpes simplex virus infection. Acyclovir therapy is the
treatment of choice. (See Chapter 231 in Nelson Textbook of
Pediatrics, 17th ed.)

Azithromycin

Nystatin

Penicillin

Pleconaril

Question . 230. Which of the following statements concerning antiviral treatment of


varicella-zoster virus infections is true?

It carries a high risk of drug toxicity

It significantly modifies the course of chickenpox in


immunocompetent persons

It significantly modifies the course of zoster


Explanation: Acyclovir treatment of otherwise healthy persons with
varicella is acceptable but is not recommended by the American
Academy of Pediatrics. Antiviral treatment of zoster is associated
with a less severe disease and greatly decreased risk for
postherpetic neuralgia. (See Chapter 232 in Nelson Textbook of
Pediatrics, 17th ed.)

It is associated with a greater risk of recurrences of zoster

Antiviral resistance by varicella-zoster virus is common

Question . 231. A 12-yr-old girl develops varicella. Her parents ask about the risk for
herpes zoster. Which of the following statements regarding herpes zoster is/are true?

Increasing age and altered cell-mediated immunity are risk factors for
herpes zoster

The lifetime risk of herpes zoster is about 10-20 %


Varicella-zoster virus can usually be cultured from zoster lesions

The incidence and the duration of postherpetic neuralgia are directly


correlated with age

All of the above


Explanation: Varicella-zoster virus establishes latent infection in
sensory ganglia cells in all individuals who experience primary
infection. Subsequent reactivation of latent virus causes herpes
zoster. (See Chapter 232 in Nelson Textbook of Pediatrics, 17th ed.)

Question . 232. A 2-yr-old healthy girl presents to your office for the first time. She was
seen by another physician 2 wk ago and was told at that time that she was behind in her
immunizations. Review of available records confirms that she has still not received varicella
vaccine. All of the following are contraindications to varicella vaccine except:

Past history of chickenpox, according to the mother

Her younger sibling currently has chickenpox

She currently has an upper respiratory tract infection with


temperature of 100°F

Administration of MMR vaccine 2 wk ago


Explanation: MMR and varicella vaccine should be administered
either simultaneously or at least 4 wk apart. Past or even current
chickenpox is not a contraindication to vaccination (although
physician-diagnosed chickenpox or serologic confirmation of
immunity can be accepted in lieu of vaccination). HIV-infected
children with CD4 percentage greater than 25% may also receive
varicella vaccine. (See Chapter 232 in Nelson Textbook of Pediatrics,
17th ed.)

HIV infection with a CD4 count of 50/mm3

Question . 233. Which of the following sets of serologic titers is most consistent with acute
primary Epstein-Barr virus infection?

IgM-VCA negative; IgG-VCA negative; EA 1:40; EBNA negative

IgM-VCA negative; IgG-VCA 1:160; EA 1:40; EBNA negative

IgM-VCA 1:16; IgG-VCA 1:160; EA 1:40; EBNA negative


Explanation: The IgM-VCA is the best single test to identify acute
EBV infection. Anti-EBNA antibodies appear 3-4 mo after infection
and can be used to distinguish recent from past infection. (See
Chapter 233 in Nelson Textbook of Pediatrics, 17th ed.)
IgM-VCA 1:16; IgG-VCA 1:160; EA 1:40; EBNA 1:8

IgM-VCA 1:16; IgG-VCA 1:160; EA negative; EBNA 1:8

Question . 234. All of the following tumors are associated with Epstein-Barr virus except:

Burkitt lymphoma

Kaposi sarcoma
Explanation: EBV is associated with several malignancies, including
nasopharyngeal carcinoma, Burkitt lymphoma, Hodgkin disease, and
lymphoproliferative diseases and leiomyosarcomas in
immunocompromised persons. Kaposi sarcoma is associated with
HHV-8. (See Chapter 233 in Nelson Textbook of Pediatrics, 17th ed.)

Leiomyosarcoma

Lymphoproliferative disease in immunocompromised persons

Nasopharyngeal carcinoma

Question . 235. All of the following organisms are recognized as potential causes of an
infectious mononucleosis-like syndrome except:

Cytomegalovirus

Epstein-Barr virus

Human immunodeficiency virus (HIV)

Parvovirus B19
Explanation: Infectious mononucleosis-like illnesses may also be
caused by primary infection with cytomegalovirus, T. gondii,
adenovirus, hepatitis virus, HIV, and possibly rubella virus. (See
Chapter 233 in Nelson Textbook of Pediatrics, 17th ed.)

Toxoplasma gondii

Question . 236. All of the following statements concerning congenital cytomegalovirus


infection are true except:

It is the most common congenital infection

Approximately 5% of infected newborns have severe disease

Approximately 60% of infected newborns have mild disease


Explanation: Only 5% of infected newborns with congenital
cytomegalovirus infection have severe disease, and another 5% have
mild disease. Most infected newborns are asymptomatic. Treatment
has not been shown to be beneficial and is considered experimental.
(See Chapter 234 in Nelson Textbook of Pediatrics, 17th ed.)

The diagnosis is best confirmed by neonatal urine culture

Treatment with ganciclovir is recommended

Question . 237. On examination of a full-term newborn, the physician notes mild


hepatomegaly. Other physical findings are normal, including head circumference and
appearance of the retinas. A urine culture grows cytomegalovirus (CMV). Results of head
ultrasonography are normal. Subsequent testing discloses no metabolic disorders. The
deficit most likely to occur in the next year is:

Visual loss

Hearing loss
Explanation: Sensorineural hearing loss is a risk after asymptomatic
congenital CMV infection. The incidence of neonatal CMV-positive
urine is much greater than the incidence of symptomatic neonatal
CMV inclusion disease (e.g., microencephaly, retinitis, being small for
gestational age, petechiae). (See Chapter 234 in Nelson Textbook of
Pediatrics, 17th ed.)

Cirrhosis

Patent ductus arteriosus

Immunoglobulin deficiency

Question . 238. Which of the following newborns is at greatest risk for symptomatic
cytomegalovirus (CMV) infection?

A 2,985-g boy born at 41 wk of gestation to a 27-yr-old G2P2


mother who had primary CMV infection during her first
pregnancy
Explanation: The incidence of congenital CMV infection ranges from
0.2 to 2.4% of all live births, with the higher rates among populations
with a lower economic standard of living. The risk for fetal infection is
greatest with maternal primary CMV infection (30%) and much lower
with recurrent infection (<1%). (See Chapter 234 in Nelson Textbook
of Pediatrics, 17th ed.)

A 1-wk-old 2,412-g breast-fed girl born at 34 wk of gestation to a 21-


yr-old mother whose breast milk has detectable levels of CMV
A 2,634-g exclusively bottle-fed girl born at 38 wk of gestation to a
16-yr-old G1P1 mother who smoked heavily throughout the pregnancy

A 3,420-g boy born at 35 wk of gestation to a 38-yr-old G5P4 mother


who works in a daycare setting and has school-aged children

A 2,835-g boy born to a 26-yr-old mother who had no prenatal care


and is a recent immigrant from Uganda

Question . 239. An 18-yr-old male patient is followed in the infectious diseases clinic for
human immunodeficiency virus infection. His most recent CD4 lymphocyte count is
97/mm3. Over the last week he has developed increasing dyspnea and coughing, visual
changes, diarrhea, and fever. You suspect that cytomegalovirus infection is responsible for
these symptoms. Which of the following laboratory tests would be most helpful in
establishing the diagnosis?

CMV IgG antibody titer

CMV IgM antibody titer

Glycoprotein H epitopes

Viral culture of the blood buffy coat for CMV


Explanation: In immunocompromised patients, excretion of CMV in
urine, increases in IgG titers, and even the presence of IgM
antibodies are common, making the distinction between primary and
recurrent infections more difficult. Demonstrating viremia by buffy
coat culture or detection of CMV DNA implies active disease and
worse prognosis (See Chapter 234 in Nelson Textbook of Pediatrics,
17th ed.)

Centrifugation-enhanced rapid culture system for early antigen (shell


vial) from a urine specimen

Question . 240. A 1,050-g male infant is born at 31 wk of gestation to a migrant farm


worker who had poor prenatal care. The mother has a history of gonorrhea. He requires
ventilatory support and has a mean arterial blood pressure of 27 mm Hg despite 3
intravenous boluses of normal saline. He has a petechial rash and is microcephalic. He
thrusts his tongue forward and is jittery. Laboratory studies reveal an alanine
aminotransferase level of 346 U/L, serum glucose of 61 mg/dL, and white blood cell count
of 20,000/mm3, with neutrophils 60%, bands 15%, and lymphocytes 20%. The platelet
count is 60,000/mm3. A computed tomography study of the head shows periventricular
calcifications. The most likely diagnosis is:

Group B streptococcal sepsis


Congenital toxoplasmosis

Congenital Neisseria gonorrhoeae sepsis

Perinatal listeriosis

Congenital cytomegalovirus infection


Explanation: Petechial rash and thrombocytopenia in a small-for-
gestational-age newborn suggest congenital infection. The CT scan
shows periventricular calcifications, which are characteristic of
congenital CMV infection. (See Chapter 234 in Nelson Textbook of
Pediatrics, 17th ed.)

Question . 241. The patient described in Question 240 is discharged from the neonatal
intensive care unit at 9 mo of age. His discharge diagnoses include chronic lung disease,
cerebral palsy, a seizure disorder, resolved necrotizing enterocolitis, and retinopathy of
prematurity. Because of the infection that was present at birth, the primary care physician
should regularly order which of the following screening tests?

Alanine and aspartate aminotransferase assays

Auditory evoked response


Explanation: CMV infection is a leading cause of sensorineural
hearing loss, which occurs in approximately 7% of infected infants.
(See Chapter 234 in Nelson Textbook of Pediatrics, 17th ed.)

Lymphocyte subsets

Neural imaging

Platelet counts

Question . 242. Which of the following statements regarding treatment of cytomegalovirus


disease in children is true?

Acyclovir and CMV IVIG constitute the regimen of choice for life-
threatening CMV infections
Explanation: Ganciclovir combined with immune globulin, either
standard intravenous immunoglobulin (IVIG) or hyperimmune CMV
IVIG, has been used to treat life-threatening CMV infections in
immunocompromised hosts (e.g., bone marrow, heart, and kidney
transplant recipients and patients with AIDS). (See Chapter 234 in
Nelson Textbook of Pediatrics, 17th ed.)

Ganciclovir administration often results in severe neutropenia and


liver dysfunction
CMV retinitis and gastrointestinal disease are cured by ganciclovir

Acyclovir is recommended to shorten the duration of heterophile-


negative mononucleosis

Cidofovir is recommended prophylactically in renal transplantation


patients to prevent recurrent disease

Question . 243. The peak incidence of HHV-6 infection is seen in which of the following
age groups?

Newborns, in whom it is a congenital infection

Children 0-5 yr
Explanation: Approximately 60-90% of children 12 mo of age and
80-100% of children 3-5 yr of age have antibodies to HHV-6. Most
newborns are seropositive as a result of transplacental transfer of
maternal antibodies. (See Chapter 235 in Nelson Textbook of
Pediatrics, 17th ed.)

Children 5-10 yr

Children 10-15 yr

Adolescents 16 yr and older, in whom it is acquired as a sexually


transmitted infection

Question . 244. A 7-mo-old child presents in late October with 3 days of fever with
temperatures to 103.5°F, a mildly injected pharynx, mild cervical lymphadenopathy, and
diarrhea. The child has been behaving normally and eating well and has no other
symptoms. On the fourth day of the illness the fever resolves, and a generalized measles-
like rash appears 12 hr later. The child appears normal on physical examination. The most
likely diagnosis is:

Measles

Rubella

Drug reaction to antipyretics

HHV-6 infection
Explanation: HHV-6 is the agent of roseola (erythema subitum), the
childhood exanthem present in the infant described in the question.
(See Chapter 235 in Nelson Textbook of Pediatrics, 17th ed.)

Enteroviral infection
Question . 245. For the patient described in Question 244, the most appropriate next step
in management would be to:

Perform a lumbar puncture

Begin antiviral treatment with acyclovir

Begin antiviral treatment with ganciclovir

Report the case to the local public health department

Reassure the parents


Explanation: The generally benign nature of roseola precludes
consideration of antiviral therapy. Although HHV-6 is inhibited by
ganciclovir, cidofovir, and foscarnet (but not acyclovir), the clinical
efficacy and benefit of antiviral therapy for roseola have not been
established. (See Chapter 235 in Nelson Textbook of Pediatrics, 17th
ed.)

Question . 246. The recommended antiviral treatment of HHV-6 infection complicated by


seizures is:

Administration of acyclovir

Administration of ganciclovir

Administration of famciclovir

Administration of foscarnet

Symptomatic treatment only


Explanation: The generally benign nature of roseola precludes
consideration of antiviral therapy. Although HHV-6 is inhibited by
ganciclovir, cidofovir, and foscarnet (but not acyclovir), the clinical
efficacy and benefit of antiviral therapy for roseola has not been
established. (See Chapter 235 in Nelson Textbook of Pediatrics, 17th
ed.)

Question . 247. A 2-yr-old child of an African mother is noted to have poor weight gain,
with development of Pneumocystis carinii pneumonia (PCP), oral candidiasis, and
cutaneous Kaposi sarcoma lesions. Further evaluation reveals that the child is HIV
seropositive, with an HIV viral load of 100,000 copies/mL and a CD4 cell count of 150
cells/ L. Treatment for PCP and candidiasis is initiated. The most appropriate next step in
management of the Kaposi sarcoma lesions would be:

Surgical excision

Combination chemotherapy
Local radiation

Intralesional injection with cidofovir

Initiation of antiretroviral therapy


Explanation: Several antiviral compounds inhibit HHV-8 in vitro,
including ganciclovir, foscarnet, and cidofovir. However, the benefit of
specific antiviral therapy for HHV-8-associated disease has not yet
been established. Introduction of highly active antiretroviral therapy
(HAART) has dramatically improved survival of AIDS patients with
KS. (See Chapter 236 in Nelson Textbook of Pediatrics, 17th ed.)

Question . 248. All of the following may be complications of influenza except:

Pneumonia

Otitis media

Reye syndrome

Hemolytic-uremic syndrome
Explanation: Otitis media and pneumonia, either viral or bacterial,
are common complications of influenza in children. Reye syndrome
may follow influenza, usually if salicylates are given for antipyresis,
which is why salicylates are not recommended for persons with
influenza or flulike illnesses. (See Chapter 237 in Nelson Textbook of
Pediatrics, 17th ed.)

Acute myositis

Question . 249. A 4-yr-old child is brought to your clinic for well child care. Indications for
influenza vaccine include all of the following except:

Immunosuppression caused by medications

Residence in a chronic care facility

Family member who is at risk of complications of influenza

Family history of SIDS


Explanation: Annual influenza vaccine is recommended for
individuals at high risk for influenza and its complications, including
individuals with chronic pulmonary (e.g., asthma), cardiac, renal, or
metabolic disorders (e.g., diabetes mellitus) or with
hemoglobinopathies; for persons undergoing immunosuppression or
receiving long-term aspirin therapy; for family members at risk for
influenza; and forwomen who will be in the second or third trimester
of pregnancy during the influenza season. (See Chapters 237 and
282 in Nelson Textbook of Pediatrics, 17th ed.)

Mother in the second or third trimester of pregnancy

Question . 250. Which of the following is true concerning rimantadine and amantadine for
treatment of influenza?

They are both approved for treatment of influenza A illness in


children

They are equally effective against influenza A and B viruses

They are both completely excreted unchanged in the urine

They can reduce the severity and duration of influenza A illness


when administered within 48 hr of onset of illness
Explanation: When administered within the first 48 hours of illness,
amantidine or rimantadine hastens recovery from influenza by 1 to 2
days. Amantadine and rimantadine are effective only against
influenza A strains. Amantadine is indicated for treatment and
prophylaxis of influenza in both children and adults, whereas
rimantadine is indicated in children only for prophylaxis. Amantadine
is associated with much more frequent central nervous systems
adverse effects than rimantadine. (See Chapter 237 in Nelson
Textbook of Pediatrics, 17th ed.)

They have identical adverse effect profiles

Question . 251. Patient groups that should be targeted for annual influenza vaccination
include:

Persons aged 65 yr and older

Persons with cardiovascular disease

Otherwise healthy children with asthma

Women who will be in the second or third trimester of pregnancy


during influenza season

All the above


Explanation: All of these groups should receive annual influenza
vaccination. (See Chapters 237 and 282 in Nelson Textbook of
Pediatrics, 17th ed.)

quastion . 252. A 5-mo-old boy and an 18-mo-old girl, both healthy, are presented to your
office in November for physical examination prior to entry to daycare. Which of the
following is true concerning the recommendation for influenza vacation of these children?

Influenza vaccine is not recommended for either

Influenza vaccine is recommended for the 5-mo-old but not the 18-
mo-old

Influenza vaccine is not recommended for the 5-mo-old but is


recommended for the 18-mo-old
Explanation: Recently, the recommendations for influenza vaccine
have been extended to encourage vaccination, to the extent feasible,
of all healthy children who will be 6-23 mo of age during influenza
season (October-March) because their risk of hospitalization is
similar to that in the elderly. The vaccine is not given to children
younger than 6 mo of age. (See Chapter 237 in Nelson Textbook of
Pediatrics, 17th ed.)

Influenza vaccine is recommended for both

Indications depend on whether either infant is in a vaccine target


group

Question . 253. A 4-yr-old child has an acute illness with coryza, barky cough,
hoarseness, and anorexia. There is no fever, and lower respiratory tract findings are
normal. The most likely etiologic agent is:

Influenza virus

Parainfluenza virus
Explanation: Parainfluenza viruses account for approximately half of
cases of croup and, unlike influenza virus and RSV infections, are not
usually associated with fever or lower respiratory tract symptoms.
(See Chapter 238 in Nelson Textbook of Pediatrics, 17th ed.)

Respiratory syncytial virus

Calicivirus

Adenovirus
Question . 254. Following your request for a plain x-rays of the neck of a 2-yr-old child, the
radiologist calls you to report finding a steeple sign. The most likely etiologic agent is:

Influenza virus

Parainfluenza virus
Explanation: The steeple sign is visible on an anteroposterior
radiograph of the neck. It is the progressive narrowing of the
subglottic region that has the appearance of a tall, tapering church
steeple. It is a sign of the subglottic edema that is typical of croup,
which is most frequently caused by parainfluenza viruses. (See
Chapter 238 in Nelson Textbook of Pediatrics, 17th ed.)

Respiratory syncytial virus

Adenovirus

Chlamydia trachomatis

Question . 255. A 7-mo-old infant has rhinorrhea, cough, audible wheezes, and chest
retractions. There is a low-grade fever and irritability. The most likely etiologic agent is:
Influenza virus

Parainfluenza virus

Respiratory syncytial virus


Explanation: Respiratory syncytial virus is the most common cause
of bronchiolitis, which has the highest incidence from 2 to 7 mo of
age and usually occurs in the winter months.

Adenovirus

Chlamydia trachomatis

Question . 256. Prophylaxis with palivizumab or RSV-IVIG against respiratory syncytial


virus (RSV) is indicated for all of the following patients except:

Children younger than 2 yr with congenital cyanotic heart


disease
Explanation: RSV-IVIG is contraindicated and palivizumab is not
recommended for infants with cyanotic heart disease. In studies of
RSV-IVIG given for prophylaxis, mortality is higher in treated patients
with heart disease. (See Chapter 239 in Nelson Textbook of
Pediatrics, 17th ed.)

Children younger than 2 yr with bronchopulmonary dysplasia who


require oxygen

Infants up to 12 mo of age who were born at 28 wk of gestation or


less

Infants up to 6 mo of age who were born at 32 wk of gestation or less

Question . 257. Which of the following is/are frequently caused by adenoviruses?


Pharyngitis
Pertussis-like syndrome

Conjunctivitis

Acute diarrhea

All of the above


Explanation: Adenoviruses cause a wide array of clinical illnesses,
including pharyngitis, pneumonia, a pertussis-like syndrome,
pharyngoconjunctival fever, conjunctivitis, myocarditis, diarrhea, and
hemorrhagic cystitis. Leukocytosis and a high ESR may also be
present. (See Chapter 240 in Nelson Textbook of Pediatrics, 17th ed.)

Question . 258. Hemorrhagic cystitis, conjunctivitis, pneumonia, and diarrhea all have
been linked to:
Respiratory syncytial virus

Adenovirus
Explanation: Adenovirus 11 or 21 is probably the cause. (See
Chapter 240 in Nelson Textbook of Pediatrics, 17th ed.)

Rhinovirus

Herpes simplex virus

Parainfluenza virus

Question . 259. The cell receptor for most rhinoviruses is:

ICAM-1
Explanation: ICAM-1 (intercellular adhesion molecule 1) is present
on the epithelium covering the lymphoepithelium of the adenoids and
other epithelial cells of the nose and is the cell receptor for most
rhinoviruses. (See Chapter 241 in Nelson Textbook of Pediatrics, 17th
ed.)

CD23

Tumor necrosis factor (TNF)

IL-10

IFN-

Question . 260. All of the following viruses may be associated with gastroenteritis except:

Rotavirus
Polyomavirus
Explanation: Rotaviruses, astrovirus, adenoviruses, and
caliciviruses (e.g., Norwalk virus) are the medically important
pathogens of human viral gastroenteritis. (See Chapter 242 in Nelson
Textbook of Pediatrics, 17th ed.)

Astrovirus

Adenovirus

Calicivirus

Question . 261. Papillomaviruses may be associated with:

Common skin warts

Laryngeal papillomatosis

Condylomata acuminata

Cervical cancer

All of the above


Explanation: Papillomaviruses cause a variety of proliferative
cutaneous and mucosal lesions. Condylomata acuminata are genital
warts. Laryngeal papillomatosis follows acquisition of
papillomaviruses during passage through an infected birth canal.
(See Chapter 243 in Nelson Textbook of Pediatrics, 17th ed.)

Question . 262. Which of the following statements regarding treatment for papillomavirus
infection is true?

Warts, especially hand warts, often resolve spontaneously


Explanation: Untreated warts may spontaneously regress, remain
unchanged, or increase in size and number. There are many
effective treatments, some of which are patient-applied and some of
which are physician-applied, and some are painful and some are
painless. Treatment of cervical warts may not decrease the risk of
cervical cancer. (See Chapter 243 in Nelson Textbook of Pediatrics,
17th ed.)

Effective treatments are painful

Effective treatments are physician-applied

Most cervical warts should be treated by a generalist


Treatment of cervical warts greatly decreases the risk of cervical
cancer

Question . 263. Which of the following statements concerning eastern equine encephalitis
is true?

It is the most common of the arboviral encephalitides

The area of highest incidence is the northeastern United States

Asymptomatic infections are uncommon

Death or residual neurologic deficits are common


Explanation: Eastern equine encephalitis has a low incidence of a
few cases each year along the Atlantic and Gulf States. The case-to-
infection ratio is 1:8 in children and 1:29 in adults. Unlike the other
arboviral encephalitides, it carries a poor prognosis. The case fatality
rate is 33-75%, with many survivors having residual neurologic
deficits. No specific treatment is available. (See Chapter 244 in Nelson
Textbook of Pediatrics, 17th ed.)

Zanamivir is an effective agent but is not usually required

Question . 264. This morning an 8-yr-old boy experienced the abrupt onset of fever,
irritability, and headache, followed by lethargy, confusion, seizures, and coma. He had
accompanied his parents on a picnic and boating outing in the saltwater marshes along the
New Jersey coastline 1 wk earlier, on September 3. The peripheral white blood cell count
demonstrates a marked leukocytosis. Which of the following is the most likely diagnosis?

Meningococcal meningitis

Tuberculous meningitis

Dengue hemorrhagic fever

Lyme disease

Eastern equine encephalitis


Explanation: The clinical picture is typical of viral encephalitis, which
is also suggested by the time of year. The history of outdoor activity
in freshwater swamps in New Jersey suggests Eastern equine
encephalitis. (See Chapter 244 in Nelson Textbook of Pediatrics, 17th
ed.)

Question . 265. A 4 yr-old boy attends the clinic with a minor upper respiratory tract
infection. During the interview the mother tells you that the family shortly will leave the
United States for several years' residence in northern Thailand. His vaccinations are up to
date for his age. What is your recommendation with regard to travel immunization?

Vaccination against Japanese encephalitis


Explanation: Travelers to countries endemic for Japanese
encephalitis who will be staying for 1 mo or longer in rural areas
should be vaccinated against Japanese encephalitis virus. (See
Chapter 245.2 in Nelson Textbook of Pediatrics, 17th ed.)

Vaccination against yellow fever

Vaccination against Western equine encephalitis

Immune globulin to prevent typhoid fever

No additional immunizations are indicated

Question . 266. An 8-yr-old girl presents with high fever of 5 days' duration, cool
extremities, a rapid and weak pulse, and a petechial rash. Pulse pressure is less than 20
mm Hg, and a small effusion is detected in the right pleural space. The child lived in
Thailand for 2 yr prior to immigrating to the United States and has just returned from
Thailand after visiting to see her grandparents. The most likely diagnosis is:

Dengue hemorrhagic fever


Explanation: The clinical picture of high fever, shock, and a
petechial rash is characteristic of dengue hemorrhagic fever. The
history of previously living in Thailand also suggests that the recent
trip resulted in second infection with dengue virus, which is
associated with more severe infections. (See Chapter 246 in Nelson
Textbook of Pediatrics, 17th ed.)

Hepatitis A

Relapsing fever

Typhoid fever

Yellow fever

Question . 267. All of the following are typical symptoms of dengue fever except:

Headache

A transient macular rash for the first 1-2 days of fever

A maculopapular rash after defervescence

Mild interstitial pneumonia


Explanation: Dengue is characterized by high fever, headache,
myalgias and arthralgias, a transient blanching rash for the first 1-2
days of fever, and a maculopapular rash, sometimes with
desquamation, that appears after defervescence. (See Chapter 246 in
Nelson Textbook of Pediatrics, 17th ed.)

Arthralgias

Question . 268. All of the following may be features of dengue hemorrhagic fever except:

Hematocrit decreased by >=20%


Explanation: Dengue hemorrhagic fever is associated with
increased capillary permeability, which leads to hemoconcentration
(an increased hematocrit) and hypoalbuminemia. Dengue shock
syndrome is dengue hemorrhagic fever plus hypotension or a narrow
pulse pressure. (See Chapter 246 in Nelson Textbook of Pediatrics,
17th ed.)

Pleural effusion (by chest radiograph)

Hypoalbuminemia

Thrombocytopenia (<=100,000/mm3)

Fever

Question . 269. A 16-yr-old female high school student is seen for pretravel counseling.
Departing next month with a group of classmates, she will spend 7 days on a bird-watching
excursion in the Amazonian portion of Peru. Your recommendation with regard to travel
immunization is:

Vaccination against malaria

Vaccination against Lyme disease

Vaccination against yellow fever


Explanation: All persons traveling to areas of South America and
Africa in which yellow fever is endemic are possible candidates for
vaccination, but length of stay, exact locations to be visited, and
environmental or occupational exposure may determine the specific
risk and individual need for vaccination. (See Chapter 247 in Nelson
Textbook of Pediatrics, 17th ed.)

Immune globulin to prevent typhoid fever

Five days of chloramphenicol on arrival


Question . 270. A 15-yr-old boy has been in Arizona on a camping trip. He presents with
fever and myalgia accompanied by cough, vomiting, diarrhea, and mid-abdominal pain. On
physical examination he is dyspneic and tachypneic. His blood pressure is 90/55 mm Hg.
The most likely diagnosis is:

Yellow fever

Colorado tick fever

Borreliosis

Hantavirus pulmonary syndrome


Explanation: The clinical manifestations of fever, myalgia, cough,
and acute respiratory distress are characteristic of hantavirus
pulmonary syndrome. (See Chapter 250 in Nelson Textbook of
Pediatrics, 17th ed.)

Dengue hemorrhagic fever

Question . 271. Which of the following is a risk factor for hantavirus pulmonary syndrome
caused by Sin Nombre virus?

Tick bites

Mosquito bites

Exposure to infected rodents


Explanation: Sin nombre virus is shed from saliva, urine, and feces
of the deer mouse, Peromyscus maniculatus. (See Chapter 250 in
Nelson Textbook of Pediatrics, 17th ed.)

Swimming in stagnant or brackish water

Contaminated food

Question . 272. A child suffers a provoked bite from a stray dog that was captured by
animal control and appears healthy. The most appropriate action would be to:

Confine and observe the dog for 10 days for signs suggestive of
rabies
Explanation: Healthy dogs, cats, and ferrets should be held for a 10-
day observation. Postexposure prophylaxis for rabies should be
given to the bitten person at the first sign of rabies in the animal,
which should be euthanized immediately and tested. (See Chapter 251
in Nelson Textbook of Pediatrics, 17th ed.)
Submit the dog's head for examination for rabies

Begin rabies vaccination

Administer human rabies immune globulin (HRIG) and begin rabies


vaccination

None of the above?no postexposure prophylaxis is necessary


because it was a provoked attack

Question . 273. Which of the following describes the recommended use of HRIG for
postexposure rabies prophylaxis in the United States?

Administer one half of the HRIG dose at the exposure site and the
remainder at another site intramuscularly, always with rabies vaccine

Administer as much of the HRIG dose as possible at the


exposure site and the remainder at another site intramuscularly,
always with rabies vaccine
Explanation: As much as possible of the dose of HRIG (20 IU/kg)
should be infiltrated into the wound and the area around the wound,
with the remainder administered intramuscularly at another site.
HRIG should always be given with vaccine for postexposure
prophylaxis of unimmunized persons. (See Chapter 251 in Nelson
Textbook of Pediatrics, 17th ed.)

Administer one half of the HRIG dose at the exposure site and the
remainder at another site intramuscularly, with rabies vaccine for
high-risk bites

Administer as much of the HRIG dose as possible at the exposure


site and the remainder at another site intramuscularly, with rabies
vaccine for high-risk bites

Rabies immune globulin is no longer recommended in the United


States

Question . 274. Animals that should be regarded in the United States as rabid include:

Rabbits and hares

Feral mice and rats

Squirrels and chipmunks

Bats
Explanation: Bats, skunks, raccoons, foxes, and most other
carnivores should be regarded as rabid unless the animal is proved
rabies-negative by laboratory testing. Bites of squirrels, hamsters,
guinea pigs, gerbils, chipmunks, rats, mice, other small rodents,
rabbits, and hares almost never require prophylaxis; local public
health officials should be consulted. (See Chapter 251 in Nelson
Textbook of Pediatrics, 17th ed.)

Woodchucks and beavers

Question . 275. An 18-yr-old patient with AIDS develops motor weakness, visual field
defects, and speech and cognitive impairment with dementia, confusion, and personality
change. The most likely etiologic agent is:

Pneumocystis carinii

Cryptococcus

Mycobacterium avium complex

Cytomegalovirus

JC virus
Explanation: Progressive multifocal leukoencephalopathy (PML) is
caused by JC virus. More than half of cases occur in HIV-infected
individuals. (See Chapter 252 in Nelson Textbook of Pediatrics, 17th
ed.)

Question . 276. All of the following are features of prion proteins except:

They are glycoproteins

They are susceptible to chemical and physical treatments


Explanation: Prion proteins cause the transmissible spongiform
encephalopathies. Their primary structure is encoded by the host.
Prion proteins from several species are similar but not identical in
structure. They are very resistant to chemical and physical
treatments. (See Chapter 253 in Nelson Textbook of Pediatrics, 17th
ed.)

They are transmissible

Their primary structure is encoded by the host

Prions from different species are very similar in structure

Question . 277. All of the following statements regarding the spongiform encephalopathies
are true except:
Creutzfeldt-Jakob disease (CJD) is the most common human
spongiform encephalopathy

Kuru now occurs primarily in young adults


Explanation: Kuru once affected many children, adolescents, and
adults of Papua New Guinea but is now recognized only in older
adults. This indicates that ritual cannibalism was probably the only
mechanism of spread. (See Chapter 253 in Nelson Textbook of
Pediatrics, 17th ed.)

Some forms show autosomal dominant inheritance

There is no effective treatment

Most patients die within 1 yr

Question . 278. All of the following statements regarding HIV gp120 are true except:

It shows little heterogeneity among HIV strains


Explanation: HIV gp120 has significant heterogeneity among HIV
strains, which is one reason for the difficulty in developing an
effective HIV vaccine. (See Chapter 254 in Nelson Textbook of
Pediatrics, 17th ed.)

It is a transmembrane glycoprotein

It is highly immunogenic

It is a major component of the viral envelope

It binds with the CD4 cell receptor

Question . 280. An HIV-seropositive woman comes to her physician for advice because
she is pregnant. She has never been on any antiviral medication because she has
remained healthy without symptoms of AIDS. However, she wonders whether antiviral
treatment might diminish the likelihood of fetal HIV infection. Which of the following
statements regarding such treatment is correct?

Treatment will produce maternal thrombocytopenia

Treatment will result in viral resistance in all infected infants

Treatment will decrease the likelihood of fetal infection but adversely


affect the woman, so it should be avoided

Treatment will decrease the likelihood of fetal infection by


greater than 50%
Explanation: Treatment of HIV-positive mothers can dramatically
reduce the incidence of HIV infection in infants. Treatment reduces
the overall risk of infection from approximately 25-30% to 8%.
Treatment begins any time after 14 wk of gestation and continues
during labor and delivery and for another 6 wk (in the infants). (See
Chapter 254 in Nelson Textbook of Pediatrics, 17th ed.)

Treatment will adversely affect the fetus and should be avoided

Question . 281. All of the following statements about HIV infection are true except:

HIV infection is the most common cause of immunodeficiency in


adults and children

HIV-infected children commonly have hypergammaglobulinemia

Stem cell transplantation is not considered a useful therapeutic


option for AIDS

A positive HIV antibody test is diagnostic of HIV infection in


children and adults
Explanation: In infants and young children (<18 mo of age) it is not
possible to distinguish readily between passively acquired and
actively produced HIV antibody. In the absence of clinical signs or
symptoms, repeat testing at age 18 mo or older is necessary to
confirm infection. (See Chapter 254 in Nelson Textbook of Pediatrics,
17th ed.)

The risk of vertical transmission of HIV can be reduced by perinatal


zidovudine administration

Question . 282. Mechanisms responsible for vertical transmission of HIV infection include:

Intrauterine fetal infection

Intrapartum (peripartum) transmission

Breast-feeding

All of the above


Explanation: Most cases of vertical-transmitted HIV infection occur
from intrapartum transmission, but HIV can also be vertically
transmitted in utero or by breast-feeding. (See Chapter 254 in Nelson
Textbook of Pediatrics, 17th ed.)
Question . 283. All of the following may be diagnostic of HIV infection in an 8-mo-old child
except:

Positive result on p24 antigen assay

Positive HIV culture

Positive HIV Western immunoblot assay


Explanation: Serologic diagnosis of HIV infection by ELISA and
Western immunoblot analysis is reliable only after 18 mo of age.
Before this age, residual maternal antibodies acquired
transplacentally may be responsible for the positive serologic test
results. (See Chapter 254 in Nelson Textbook of Pediatrics, 17th ed.)

Positive result on HIV DNA assay

Positive result on HIV RNA assay

Question . 284. All of the following statements concerning HIV infection are true except:

HIV suppression is best achieved by regularly rotating


antiretroviral regimens
Explanation: Sustainable suppression of HIV is best achieved by
combination antiretroviral therapy to which the patient has not been
exposed previously and that is not cross-resistant to drugs given to
the patient previously. (See Chapter 254 in Nelson Textbook of
Pediatrics, 17th ed.)

Therapy can reduce HIV burden to undetectable levels

Viral burden predicts disease progression

CD4 cell counts reflect the risk of opportunistic infections

Adherence to therapy is crucial

Question . 285. All of the following statements concerning treatment of HIV infection are
true except:

Multiple-drug regimens are superior to single-drug therapy

Protease inhibitors prevent uncoating of infectious virions


Explanation: Protease inhibitors act by preventing packaging of
infectious virions before they leave the infected cell. (See Chapter 254
in Nelson Textbook of Pediatrics, 17th ed.)
Nucleoside and non-nucleoside reverse transcriptase inhibitors act at
different sites of reverse transcription

Drug-drug interactions are common with protease inhibitors

Antiretroviral treatment is most successful in treatment-naive patients

Question . 286. Which of the following infants born to HIV-infected mothers should receive
prophylaxis for Pneumocystis carinii?

Infants with a positive result on HIV culture, DNA or RNA PCR assay,
or p24 antigen assay

Infants with an AIDS-defining diagnosis

Infants who have had a first episode of P. carinii pneumonia

Infants with a CD4 cell count of less than750/mm3 or a CD4


percentage of less than 15%

All infants 6 wk to 1 yr of age born to HIV-infected mothers


Explanation: All infants between 6 wk and 1 yr of age should receive
prophylaxis for P. carinii regardless of CD4 cell count and
percentage. After 12 mo of age, prophylaxis is prescribed according
to the CD4 cell count and percentage. (See Chapter 268 in Nelson
Textbook of Pediatrics, 17th ed.)

Question . 287. Components of the recommended prophylactic regimen to prevent vertical


HIV transmission include:

Zidovudine given orally to the mother after the first trimester

Zidovudine given intravenously to the mother during delivery

Zidovudine given orally to the newborn for the first 6 wk of life

All of the above


Explanation: The regimen of prenatal oral zidovudine during the
second and third trimesters, intravenous zidovudine during labor, and
postnatal zidovudine given to the infant reduces vertical HIV
transmission to as low as 3-4%. Regimens of only one or two of
these components provide some reduction. Combination prophylaxis
with drugs more potent than zidovudine and that may provide greater
effectiveness is being studied. (See Chapter 254 in Nelson Textbook
of Pediatrics, 17th ed.)
Question . 288. Management for a newly diagnosed, asymptomatic 2-yr-old HIV-infected
child with a CD4 cell count of 45/mm3 and HIV RNA of 110,000 copies/mL should include
all of the following except:

Trimethoprim-sulfamethoxazole prophylaxis for Pneumocystis carinii


infection

Clarithromycin prophylaxis for Mycobacterium avium complex


infection

Monthly IVIG prophylaxis for bacterial infections


Explanation: Monthly IVIG to prevent bacterial infections is
recommended only for HIV-infected children who have had two or
more serious bacterial infections within 1 yr, have a documented
inability to make antigen-specific immunoglobulin, or have
hypogammaglobulinemia. (See Chapter 254 in Nelson Textbook of
Pediatrics, 17th ed.)

Tuberculin skin testing

Antiretroviral therapy

Question . 289. The rate of transmission of maternal HIV infection to offspring without any
antiretroviral treatment is:

1%

4%

12-30%
Explanation: Most large studies in the United States and Europe
have documented vertical transmission rates in untreated women of
12-30%. Transmission rates in Africa and Haiti are higher (25-52%).
(See Chapter 254 in Nelson Textbook of Pediatrics, 17th ed.)

50-60%

70-90%

Question . 290. The role of elective cesarean section in preventing perinatal HIV
transmission can best be described as:

Ineffective and not recommended

Effective but not recommended

Effective and selectively recommended


Explanation: A meta-analysis of over 1,000 pregnancies
demonstrated that elective cesarean delivery decreased transmission
by 87% if used in conjunction with zidovudine therapy in the mother
and the infant. However, because these data predated the advent of
highly active antiretroviral therapy (HAART), the additional benefit of
cesarean section is probably negligible if the mother's viral load is
<500 copies/mL. (See Chapter 254 in Nelson Textbook of Pediatrics,
17th ed.)

Effective and routinely recommended

Question . 291. Factors that may affect the perinatal HIV transmission rate include:

Preterm delivery

Low maternal antenatal CD4 count

Duration of ruptured membranes

Vaginal versus cesarean delivery

All of the above


Explanation: Several risk factors influence the rate of vertical
transmission: pre-term delivery (<34 wk of gestation), a low maternal
antenatal CD4 count, and use of illicit drugs during pregnancy. The
most important variables appear to be duration of ruptured
membranes of greater than 4 hr and birthweight less than 2500 g,
each of which doubles the transmission rate. (See Chapter 254 in
Nelson Textbook of Pediatrics, 17th ed.)

Question . 292. All of the following statements regarding infection with human T-cell
lymphotrophic virus type I (HTLV-I) are true except:

It is the cause of adult T-cell leukemia/lymphoma

It is the cause of tropical spastic paraparesis (TSP)

Breast-feeding is safe because HTLV-I is not transmitted in


breast milk
Explanation: HTLV-I is prevalent in Japan and the Caribbean, where
breast-feeding is a major mode of transmission. Testing of all blood
products for HTLV-I/II antibodies was implemented in the United
States in 1997. (See Chapter 255 in Nelson Textbook of Pediatrics,
17th ed.)

HTLV-I is transmitted by sexual contact

All donated blood in the United States is tested for anti-HTLV


antibodies

Question . 293. An 8-yr-old child presents with meningitis and encephalitis of acute onset
in late summer after swimming in a pond of stagnant water. The most likely etiology is:

Pseudomonas aeruginosa infection

Herpes simplex virus infection

Mucor infection

Naegleria infection
Explanation: Naegleria organisms are found in many freshwater
sources, including ponds, lakes, and stagnant pools. Therapy is often
difficult, and morbidity and mortality are high. (See Chapter 256 in
Nelson Textbook of Pediatrics, 17th ed.)

Malaria

Question . 294. Which of the following statements concerning Naegleria infection of the
central nervous system is true?

It causes a granulomatous encephalitis

Swimming in warm freshwater ponds is a risk factor for


infection
Explanation: Naegleria organisms are found in many freshwater
sources, including ponds, lakes, and stagnant pools. It occurs most
commonly in previously healthy children and adults. Therapy is often
difficult, and morbidity and mortality are high.

Most patients are immunocompromised

It is readily treated with metronidazole and amphotericin B

It is indistinguishable from infection with Balamuthia mandrillaris

Question . 295. A 17-yr-old boy presents with symptoms suggestive of granulomatous


amebic meningoencephalitis. Which of the following helps to distinguish Acanthamoeba
from Balamuthia infection of the central nervous system?

Acanthamoeba infection manifests within days of exposure

Balamuthia is readily amenable to treatment while Acanthamoeba is


nearly always fatal

Patients with Acanthamoeba are more likely to be


immunocompromised
Explanation: The clinical manifestations and course of
Acanthamoeba and Balamuthia infection of the central nervous
system are similar. Most patients with Acanthamoeba are
immunocompromised, in contrast to Balamuthia infection. (See
Chapter 256 in Nelson Textbook of Pediatrics, 17th ed.)

Balamuthia is contracted from swimming in fresh water

Balamuthia is associated with hypereosinophilia, Acanthamoeba with


lymphocytosis

Question . 296. A 4-yr-old child presents with colicky abdominal pain, blood-stained
diarrhea, and tenesmus, all of acute onset. There is no fever. Trophozoites are seen in the
stool. The family has recently returned from a trip to Mexico. The most likely etiologic agent
is:

Giardia lamblia

Entamoeba dispar

Entamoeba histolytica
Explanation: The highest incidence of amebic colitis caused by
Entamoeba histolytica is in children 1-5 yr of age. E. dispar is
associated only with an asymptomatic carrier state. Giardia lamblia
does not cause bloody colitis. Amebiasis is uncommon in temperate
climates. (See Chapter 257 in Nelson Textbook of Pediatrics, 17th ed.)

Naegleria

Cryptosporidium

Question . 297. All of the following statements concerning the treatment of Entamoeba
histolytica amebiasis are true except:

Asymptomatic cyst excreters usually do not need to be treated


Explanation: All persons with Entamoeba histolytica trophozoites or
cysts in their stools should be treated, whether they have symptoms
or not. (See Chapter 257 in Nelson Textbook of Pediatrics, 17th ed.)

Iodoquinol is recommended for asymptomatic persons

Metronidazole is recommended for invasive intestinal amebiasis

Metronidazole is recommended for hepatic amebiasis

Metronidazole therapy should always be followed by iodoquinol


Question . 298. A 7-yr-old girl whose family recently moved from Mexico to the United
States presents with fever, abdominal pain and distention, and hepatic tenderness. A CT
scan reveals a single, 7 cm, fluid-filled cavitary lesion in the right hepatic lobe. The test
most likely to confirm the diagnosis is:

Upper gastrointestinal endoscopy and duodenal biopsy

Bone marrow aspirate and culture

Mesenteric lymph node biopsy and culture

Fungal blood culture

Serologic testing
Explanation: The clinical presentation suggests amebic abscess.
The diagnosis is established by the characteristic CT or MRI findings
and positive serologic results for antibodies to Entamoeba. (See
Chapter 257 in Nelson Textbook of Pediatrics, 17th ed.)

Question . 299. Recognized sources of Giardia lamblia include:

Swimming pools

Mountain streams

Children in daycare centers

Food

All of the above


Explanation: Contaminated water is the most common source of
Giardia organisms, especially surface water treated by faulty or
inadequate water purification systems. Other sources include
contaminated swimming pools (Giardia is resistant to chlorination),
mountain streams, and contaminated food. Giardia is very common
among children in child daycare centers and among male
homosexuals. (See Chapter 258 in Nelson Textbook of Pediatrics,
17th ed.)

question . 300. All of the following statements concerning the treatment of Giardia lamblia
infections are true except:

Asymptomatic cyst excreters usually do not need to be treated


Children with acute diarrhea require treatment

Children with chronic diarrhea require treatment

Metronidazole is the treatment of choice

Metronidazole therapy should always be followed by iodoquinol


Explanation: Symptomatic persons with G. lamblia should be
treated. Asymptomatic persons usually do not need treatment except
in specific instances such as outbreak control, prevention of
household transmission to pregnant women, and patients with
hypogammaglobulinemia or cystic fibrosis. (See Chapter 258 in
Nelson Textbook of Pediatrics, 17th ed.)

Question . 301. All of the following are spore-forming intestinal protozoans except:

Giardia lamblia
Explanation: G. lamblia is a flagellated protozoan. (See Chapter 259
in Nelson Textbook of Pediatrics, 17th ed.)

Cryptosporidium parvum

Isospora belli

Cyclospora cayetanensis

Microsporidia

Question . 302. All of the following statements concerning Cryptosporidium parvum


infections are true except:

Infection is common in children

It causes a bloody colitis


Explanation: Cryptosporidium is prevalent in developing countries
and in children younger than 2 yr of age. It causes watery, nonbloody
diarrhea that is usually self-limited but may persist for several weeks.
Because illness is self-limited in immunocompetent persons, no
specific therapy is required. Treatment of immunocompromised
persons is with paromomycin, with or without azithromycin. (See
Chapter 259 in Nelson Textbook of Pediatrics, 17th ed.)

Diarrhea may persist for several weeks

Treatment is not recommended for immunocompetent persons


Paromomycin is recommended for treatment of immunocompromised
persons

Question . 303. An 18-mo-old boy who attends out-of-home daycare develops intermittent
watery diarrhea. There is no blood present in the stool, and there is no fever. Several of the
other toddlers and adult workers in the center have also had diarrhea, but the daycare staff
report that most have had negative bacterial stool cultures. The most appropriate next step
in diagnosis would be to:

Repeat the bacterial stool culture requesting identification of E. coli


O157:H7

Request bacterial stool culture and also routine ova and parasite
testing

Request bacterial stool culture, routine ova and parasite testing,


and an acid-fast smear of the stool
Explanation: The presentation suggests parasitic infection with
Giardia or Cryptosporidium parvum. Giardia is diagnosed by
documenting trophozoites, cysts, or Giardia antigens in stool
specimens. Cryptosporidium is diagnosed by modified acid-fast
staining of the stool, appearing as small (2-6 µm), red, spherical
bodies. (See Chapter 259 in Nelson Textbook of Pediatrics, 17th ed.)

Order serologic testing

Refer the patient to the gastroenterologist for colonoscopy

Question . 304. A sexually active adolescent girl presents with copious malodorous yellow
vaginal discharge with vulvovaginal irritation. The most likely etiologic agent is:

Candida albicans

Trichomonas vaginalis
Explanation: Trichomoniasis is characterized by copious
malodorous vaginal discharge, vulvovaginal irritation, dysuria, and
dyspareunia. (See Chapter 260 in Nelson Textbook of Pediatrics, 17th
ed.)

Giardia lamblia

Haemophilus ducreyi

Enterobius vermicularis

Question . 305. Recognized presentations of leishmaniasis include:

A papular lesion on the face


Large non-ulcerating papules that resemble leprosy

Nasal mucosal involvement with recurrent epistaxis

Fever, marked splenomegaly, hepatomegaly, and cachexia

All of the above


Explanation: Leishmania causes diverse group of diseases,
including: localized cutaneous leishmaniasis; diffuse cutaneous
leishmaniasis; mucosal leishmaniasis; and visceral leishmaniasis
(kala-azar). (See Chapter 261 in Nelson Textbook of Pediatrics, 17th
ed.)

Question . 306. Which of the following Plasmodium species causes the most severe form
of malaria and is associated with the highest fatality rate?

P. falciparum
Explanation: P. falciparum malaria is the most severe form and
carries fatality rates of 25-35% in untreated persons. (See Chapter 264
in Nelson Textbook of Pediatrics, 17th ed.)

P. malariae

P. ovale

P. vivax

None of the above: all malaria species are similar in disease severity

Question . 307. The most useful method for the diagnosis of malaria is:

Serologic testing for IgM antibodies

Serologic testing for IgG antibodies

Rapid antigen testing

Thick and thin blood smears


Explanation: The diagnosis of malaria is best established by
identification of organisms on Giemsa-stained thick and thin blood
smears. (See Chapter 264 in Nelson Textbook of Pediatrics, 17th ed.)

Polymerase chain reaction (PCR) testing

Question . 308. A 12-yr-old girl who recently visited Haiti with her family presents with
signs suggestive of cerebral malaria. She did not take malaria prophylaxis. The
confirmatory procedure that should be performed immediately is:

Serologic testing for specific antibodies

CT scan of the brain

Urinalysis to detect hematuria

Temperature readings at 6-hr intervals to ascertain fever


intermittency

Microscopic examination of blood films


Explanation: Cerebral malaria can be fatal in as little as 24 hr and is
a medical emergency. The thick smear of peripheral blood should be
diagnostic, especially if repeated. Laboratory manuals help identify
the morphologic forms needed to distinguish the different malarial
species. (See Chapter 264 in Nelson Textbook of Pediatrics, 17th ed.)

Question . 309. A child who returned 2 wk ago from equatorial Africa is admitted to the
hospital in a coma, with high fever and a palpable spleen. The liver is not enlarged.
Laboratory studies reveal hypoglycemia, but the cerebrospinal fluid is normal. The most
likely diagnosis is:

Pancreatic neoplasm

Pneumococcal meningitis

Falciparum malaria
Explanation: Malaria is the first diagnosis to be considered in this
patient, who had the unfavorable prognostic features of
hypoglycemia and coma. (See Chapter 264 in Nelson Textbook of
Pediatrics, 17th ed.)

Visceral leishmaniasis

Dengue hemorrhagic fever

Question . 310. All of the following are recognized routes of malaria transmission except:

Blood transfusion

Mosquito bite

Use of contaminated needle


Transplacental, from pregnant woman to her fetus

Out-of-home daycare
Explanation: Malaria is transmitted principally via blood mosquito
bites, but has also been transmitted by blood transfusion,
contaminated needles, and vertical transmission. (See Chapter 264 in
Nelson Textbook of Pediatrics, 17th ed.)

Question . 311. A 13-yr-old adolescent boy presents with unexplained febrile illness. He
had traveled to India with his parents 6 mo previously, and both he and his parents confirm
complete adherence to the prescribed malaria prophylaxis regimen. Which of the following
is true concerning the need for investigation for malaria as the cause of his illness?

No investigation is necessary because prophylaxis was used

No investigation is necessary if mefloquine was part of the


prophylactic regimen

Investigation is necessary only if another family member was ill

Investigation is necessary only if he has had unexplained fevers


since his return

Investigation for malaria is necessary


Explanation: Any person who has fever or unexplained systemic
illness and has traveled or lived in a malarial endemic area within the
previous year should be assumed to have life-threatening malaria
until proven otherwise regardless of the use of chemoprophylaxis.
(See Chapter 264 in Nelson Textbook of Pediatrics, 17th ed.)

Question . 312. The use of quinidine gluconate should be considered in any child
experiencing malaria associated with:

Neurologic dysfunction

Pulmonary edema

Inability to retain oral fluids or medication

Parasitemia more than 5% of erythrocytes

Any of the above


Explanation: Intravenous quinidine gluconate should be
administered for patients who: cannot retain oral fluids and
medication because of vomiting; have neurologic dysfunction,
pulmonary edema or renal failure; have a peripheral asexual
parasitemia of >5% of erythrocytes; or have a peripheral asexual
parasitemia of 1-4% of erythrocytes with a severe attack. (See
Chapter 264 in Nelson Textbook of Pediatrics, 17th ed.)

Question . 313. A family is planning on visiting Africa, in an area of chloroquine-resistant


Plasmodium falciparum malaria, in 9 wk and is visiting your office for pretravel advice. You
recommend mefloquine. Appropriate guidance for timing of prophylaxis would be:

Begin prophylaxis on arrival in Africa, and discontinue on return


home (to the United States)

Begin prophylaxis 1 wk before departure, and discontinue on return


home

Begin prophylaxis 1 wk before departure, and discontinue 1 wk after


return home

Begin prophylaxis 1 wk before departure, and discontinue 4 wk


after return home
Explanation: Weekly mefloquine is the drug of choice for malaria
prophylaxis for children and adults traveling to areas with
chloroquine-resistant P. falciparum. Mefloquine is started 1 wk before
departure and continued for 4 wk after the last exposure. (See
Chapter 264 and Table 285-2 in Nelson Textbook of Pediatrics, 17th
ed.)

Begin prophylaxis with any fever while traveling, and seek medical
attention immediately

Question . 314. A 19-yr-old boy in Rhode Island presents with high fever, chills, and
myalgias. He recalls a tick bite about 2 wk previously, and you consider a diagnosis of
babesiosis. Which of the following methods is useful to establish this diagnosis?

Identification of Babesia on thin blood smears

Amplification of Babesia DNA using polymerase chain reaction


technique

Identification of Babesia after injection of hamsters with patient's


blood

Identification of Babesia antibody

Any of the above


Explanation: Any of these methods may be used to diagnose
babesiosis. (See Chapter 265 in Nelson Textbook of Pediatrics, 17th
ed.)

Question . 315. For the patient described in Question 314, the recommended treatment is:
Ceftriaxone with or without streptomycin

Ciprofloxacin

Clindamycin and quinine


Explanation: The combination of clindamycin (20-40 mg/kg/day
divided tid PO) and quinine (25 mg/kg/day divided tid PO) for 7 to 10
days is the therapy of choice for babesiosis in children. Tinnitus and
abdominal distress are common adverse reactions. (See Chapter 265
in Nelson Textbook of Pediatrics, 17th ed.)

Erythromycin

Tetracycline

Question . 316. All of the following statements concerning congenital toxoplasmosis are
true except:

Fetal infection is more common if maternal infection is acquired in


late pregnancy

Severe disease is more common if maternal infection is acquired in


early pregnancy

Almost all persons with untreated congenital infection eventually


experience symptoms or signs of infection

Chorioretinitis is common at birth but infrequently develops


afterward
Explanation: Chorioretinitis occurs in only approximately 22% of
newborns with congenital toxoplasmosis, but almost all untreated
persons will have chorioretinal lesions by adulthood and about half
will have severe visual impairment. (See Chapter 266 in Nelson
Textbook of Pediatrics, 17th ed.)

Central nervous system involvement is common

Question . 317. Which of the following causes of congenital infections is associated with
cats?

Cytomegalovirus

Rubella

Toxoplasma gondi
Explanation: Cats ingest Toxoplasma oocysts in infected meat or
from the feces of other infected cats. Cats that are strictly kept
indoors, maintained on prepared diets, and not fed fresh, uncooked
meat should not contact encysted T. gondii or shed oocysts.
Pregnant women who are seronegative for T. gondii should avoid
contact with cat feces. (See Chapter 266 in Nelson Textbook of
Pediatrics, 17th ed.)i

Syphilis

Parvovirus B19

Question . 318. Toxoplasma gondii may be acquired by all of the following means except:

Ingestion of oocysts excreted by cats

Ingestion of oocysts excreted by dogs


Explanation: Dogs do not carry T. gondii oocysts. The other sources
listed in the question are compatible with transmission of
toxoplasmosis. (See Chapter 266 in Nelson Textbook of Pediatrics,
17th ed.)

Ingestion of cysts in undercooked meat

Transplacental transmission from an acutely infected mother to her


fetus in utero

Organ transplantation of an infected to a previously uninfected


recipient

Question . 319. Which is the most common result of primary Toxoplasma gondii infection
in children?

Cellulitis

Lymphadenitis

Meningitis

Pneumonia

No specific symptoms
Explanation: Most immunologically normal children who acquire T.
gondii have no clinically recognizable disease. (See Chapter 266 in
Nelson Textbook of Pediatrics, 17th ed.)

Question . 320. Toxoplasmosis causes all of the following patterns of disease except:

Congenital infection manifested in neonates by chorioretinitis,


cerebral calcifications, and hydrocephalus due to first-trimester
infection

Congenital malformations, such as cleft palate or patent ductus


arteriosus
Explanation: Congenital toxoplasmosis is associated with organ
dysfunction, inflammation, growth retardation, thrombocytopenia,
hepatitis, retinitis, and microcephaly, but not with true congenital
anomalies. (See Chapter 266 in Nelson Textbook of Pediatrics, 17th
ed.)

Localized or generalized lymphadenopathy in previously healthy


persons

Brain abscess that results in confusion, seizures, and paralysis in


patients with acquired immunodeficiency syndrome (AIDS)

Retinal lesions involving the macula and leading to blindness

Question . 321. A pregnant woman, at 15 wk of gestation, brings her 5-yr-old son for
evaluation of fever and cervical lymphadenopathy. The lymph nodes are nontender and
have not suppurated. The child was given a new kitten 3 mo ago that frequently caught
mice in their barn. The child and the mother regularly cleaned the kitten's litter pan
together, and gardened and played in the backyard sandbox, where the kitten also
defecated. All of the following diagnostic tests to identify family members with or at risk for
toxoplasmosis are indicated except:

Sabin-Feldman [HBJ4]dye test to measure T. gondii-specific IgG in


the mother's serum

Sabin-Feldman dye test to measure T. gondii-specific IgG in the 5-yr-


old boy's serum

IgM ELISA AC/HS, and avidity tests to measure T. gondii-specific


antibodies in the mother's serum

Fetal ultrasound examination

CT scan of the head and neck for the 5-yr-old boy


Explanation: Serologic tests are the best means to confirm infection
with T. gondii. Fetal ultrasound examination, and also PCR assay of
amniotic fluid, can be used to evaluate the fetus. (See Chapter 266 in
Nelson Textbook of Pediatrics, 17th ed.)

Question . 322. An infant is born small for gestational age at 34 wk of gestation with a
"blueberry muffin" rash, hepatosplenomegaly, 100,000 platelets/mm3, 15% eosinophils,
and 10% atypical lymphocytes. His serum has a Sabin Feldman dye test titer of 1:4096. All
of the following may be helpful in establishing the diagnosis of congenital toxoplasmosis for
this infant except:

IgM ISAGA to detect T. gondii serum antibodies in his serum

IGA ELISA to detect T. gondii serum antibodies

Inoculation of a preparation of the infant's placenta into mice and


subsequent measurement of antibodies to T. gondii in the mouse's
serum 6 wk after inoculation of the placenta

CSF cell count, protein, and glucose

Serum amylase
Explanation: Serologic tests are the best means to confirm infection
with T. gondii. (See Chapter 266 in Nelson Textbook of Pediatrics,
17th ed.)

Question . 323. All of the following statements concerning Pneumocystis carinii infections
are true except:

Most persons are infected before 4 yr of age

Most primary infections are usually associated with only low-


grade fever and nonspecific symptoms
Explanation: Most persons are infected with P. carinii before 4 yr of
age. Most primary infections in immunocompetent persons are
asymptomatic. (See Chapter 267 in Nelson Textbook of Pediatrics,
17th ed.)

Cell-mediated immunity is more important than humoral immunity in


combating infection

Severe P. carinii pneumonia occurs almost exclusively in


immunocompromised persons

All of the above

Question . 323. All of the following statements concerning Pneumocystis carinii infections
are true except:

Most persons are infected before 4 yr of age

Most primary infections are usually associated with only low-


grade fever and nonspecific symptoms
Explanation: Most persons are infected with P. carinii before 4 yr of
age. Most primary infections in immunocompetent persons are
asymptomatic. (See Chapter 267 in Nelson Textbook of Pediatrics,
17th ed.)
Cell-mediated immunity is more important than humoral immunity in
combating infection

Severe P. carinii pneumonia occurs almost exclusively in


immunocompromised persons

All of the above

Question . 324. Patients who should receive prophylaxis for Pneumocystis carinii
pneumonia include those with:

X-linked agammaglobulinemia

Severe combined immunodeficiency disorder


Explanation: Severe combined immunodeficiency (SCID), with its T
lymphocyte deficiency, predisposes to P. carinii pneumonia (PCP).
(See Chapter 267 in Nelson Textbook of Pediatrics, 17th ed.)

Chronic granulomatous disease

Sickle cell disease

Congenital neutropenia

Question . 325. A 1-mo-old infant of an HIV-infected mother is classified as


"indeterminate" with respect to HIV infection status. The most appropriate next step in
management would be to:

Begin treatment with a protease inhibitor

Order strict isolation of the infant

Administer immune globulin to the infant monthly until HIV infection


can be excluded

Begin trimethoprim-sulfamethoxazole prophylaxis


Explanation: All infants between 6 wk and 1 yr of age should receive
prophylaxis for P. carinii regardless of CD4 cell count and
percentage. After 12 mo of age, prophylaxis is prescribed according
to the CD4 cell count and percentage. (See Chapters 254 and 267
and Table 254-4 in Nelson Textbook of Pediatrics, 17th ed.)

Withhold all immunizations until HIV infection can be excluded

Question . 326. A 5-yr-old boy with AIDS and who is receiving zidovudine, lamivudine, and
ritonavir presents with rapid respiratory rate and progressive dyspnea of 2 days' duration.
There is no fever. The chest radiograph reveals bilateral diffuse alveolar disease. The most
important diagnostic test would be:

CT scan of the chest

Pharyngeal swab for viral and bacterial cultures

Blood gas profile

Serum lactate dehydrogenase level

Bronchoalveolar lavage for cytology


Explanation: The presentation is characteristic of Pneumocystis
carinii pneumonia. A definitive diagnosis of requires the
demonstration of P. carinii in the lung in the presence of clinical signs
and symptoms of the infection. Methods for obtaining appropriate
specimens for detecting organisms include bronchoalveolar lavage,
tracheal aspirate, transbronchial lung biopsy, bronchial brushings,
percutaneous transthoracic needle aspiration, and open lung biopsy.
Induced sputum samples are helpful if P. carinii is found, but the
absence of the organisms in induced sputum does not exclude the
infection. The open lung biopsy is the most reliable method, although
bronchoalveolar lavage is more practical in most cases. (See Chapter
267 in Nelson Textbook of Pediatrics, 17th ed.)

Question . 327. A 10-yr-old child with Pneumocystis carinii pneumonia is allergic to


sulfonamide. Which of the following drugs should be used for treatment?

Fansidar

Pentamidine isethionate
Explanation: The recommended therapy for P. carinii pneumonia is
trimethoprim-sulfamethoxazole administered intravenously, or orally if
there is mild disease and no malabsorption or diarrhea. For patients
who cannot tolerate or fail to respond to trimethoprim-
sulfamethoxazole after 5-7 days, pentamidine isethionate may be
used. (See Chapter 267 in Nelson Textbook of Pediatrics, 17th ed.)

Trimethoprim-sulfamethoxazole

Primaquine and clindamycin

Dapsone

Question . 328. A previously healthy 7-yr-old girl living in rural South Carolina presents
with a 1-wk history of fever, cough, and shortness of breath. Her temperature is 37.5oC,
pulse 80/min, respiratory rate 30 breaths/min, and blood pressure 95/60 mm Hg. Chest film
reveals bilateral pulmonary infiltrates. The complete blood count is normal except for
eosinophilia. Which of the following organisms is the most likely cause of the patient's
illness?

Trichuris trichiura

Ascaris lumbricoides
Explanation: The presentation is characteristic of L ffler syndrome,
caused by Ascaris lumbricoides. (See Chapter 268 in Nelson Textbook
of Pediatrics, 17th ed.)

Enterobius vermicularis

Mycobacterium tuberculosis

Streptococcus pneumoniae

Question . 329. A 6-yr-old girl who was previously healthy presents with a 1-wk history of
nocturnal perianal itching. There are no other symptoms, and findings on physical
examination are normal. The most appropriate therapy is:

Bacitracin ointment to the perianal area

Diphenhydramine orally as needed for itching

Single oral dose of mebendazole repeated in 2 wk


Explanation: The recommended treatment regimen for pinworms
(Enterobius vermicularis) is mebendazole, 100 mg orally, with a
repeat dose in 2 wk. An alternative treatment is albendazole, 400 mg
orally, with a repeat dose in 2 wk. (See Chapter 271 in Nelson
Textbook of Pediatrics, 17th ed.)

A 2-wk course of amoxicillin/clavulanate

Ketoconazole, in a single dose

Question . 330. An 8-yr-old boy from Tennessee received a liver transplant 6 mo ago and
is on corticosteroids and azathioprine. He presents to the emergency department with a 2-
day history of cough and wheezing. Hemoptysis was observed within the previous 12 hr. A
stool specimen is most likely to reveal which of the following microorganisms?

Salmonella enteritidis

Histoplasma capsulatum

Toxocara canis

Pneumocystis carinii
Strongyloides stercoralis
Explanation: This is a characteristic presentation of hyperinfection
syndrome with Strongyloides stercoralis. (See Chapter 272 in Nelson
Textbook of Pediatrics, 17th ed.)

Question . 331. A 4-yr-old boy who recently moved from Puerto Rico to a northern city in
the continental United States presents with a 1-wk history of fever and malaise. Physical
examination is notable for temperature 38.5oC, pulse 130/min, respiratory rate 22
breaths/min, blood pressure 100/60 mm Hg, and hepatomegaly. Laboratory findings
include a white blood cell count of 60,000/mm3 with 66% eosinophils. His past history
includes extensive use of community playground facilities. The most likely diagnosis is:

Eosinophilic gastroenteritis

Visceral larva migrans


Explanation: This is a characteristic presentation of visceral larva
migrans, caused by Toxocara canis. (See Chapter 275 in Nelson
Textbook of Pediatrics, 17th ed.)

Angiostrongylus cantonensis infection

Ascaris lumbricoides infection

Enterobius vermicularis infection

Question . 332. A child presents with several days of increasing myalgias, especially while
chewing and with breathing, and fever. There is a history of eating undercooked meat.
Laboratory testing shows 8% eosinophilia. The most likely etiologic agent is:

Trichuris trichiura

Trichinella spiralis
Explanation: In trichinosis, migrating larvae of Trichinella spiralis
cause symptoms of fever and myalgias and elicit an eosinophilic
response. (See Chapter 276 in Nelson Textbook of Pediatrics, 17th
ed.)

Strongyloides stercoralis

Toxocara canis
Ascaris lumbricoides

Question . 333. A 13-yr-old girl presents with a 1-wk history of fever, myalgias, and
periorbital edema. She emigrated with her family from a rural area of China to the United
States 4 wk ago. A communal meal that included a locally butchered pig was part of the
family's farewell party. Her 16-yr-old brother had similar symptoms, which have now
resolved. The most likely cause of the patient's condition is:

Typhoid fever

Trichuriasis

Cysticercosis

Visceral larva migrans

Trichinosis
Explanation: This is a characteristic presentation of trichinosis,
caused by Trichinella spiralis. (See Chapter 276 in Nelson Textbook of
Pediatrics, 17th ed.)

Question . 334. The 12-yr-old son of diplomat parents presents with crampy abdominal
pain, fever, migratory arthralgias, and hepatosplenomegaly. During the past 5 yr his family
has lived in the Philippines, Kampuchea, Senegal, and Mali. Ultrasound examination of his
abdomen shows periportal fibrosis consistent with schistosomiasis. His kidneys and
bladder are normal. Which of the following schistosome species is unlikely to be the cause
of his illness?

Schistosoma mansoni

Schistosoma japonicum

Schistosoma intercalatum

Schistosoma haematobium
Explanation: Schistosoma haematobium is not endemic to the areas
visited. It also does not affect the portal circulation, preferring the
ureteral and bladder veins. (See Chapter 277 in Nelson Textbook of
Pediatrics, 17th ed.)

Schistosoma mekongi

Question . 335. The most useful test for diagnosis of neurocysticercosis is:

Serologic testing

Stool examination
Rapid antigen testing

Computed tomography (CT)


Explanation: The most useful diagnostic test for neurocysticercosis
is either computed tomography (CT) or magnetic resonance imaging
(MRI), which typically reveals a solitary cyst with or without contrast
enhancement. (See Chapter 280 in Nelson Textbook of Pediatrics,
17th ed.)

Polymerase chain reaction (PCR) testing

Question . 336. In S o Paulo, Brazil, a head CT study was performed on a 9-yr-old boy for
evaluation of headache following a fall. The CT shows a single calcified lesion in the left
hemisphere. On physical examination, the boy's headache is noted to increase on
standing, and he has poor coordination and intention. The most appropriate next step in
management would be to:

Perform MRI

Perform brain biopsy

Order stool examination for ova and parasites

Administer albendazole
Explanation: The cyst stage of T. solium infection (cysticercosis)
comes from ingestion of eggs excreted by infected humans and
cannot be directly related to pigs. (See Chapter 280 in Nelson
Textbook of Pediatrics, 17th ed.)

Measure Toxoplasma titers

Question . 337. A mother and son from a Midwestern community, who have never traveled
outside of the United States or Canada, were found to have cysticercosis. The four other
household members have normal findings on imaging studies. To establish the source of
this infection and the risk to others in the community, the single best investigation would be
to:

Test all pork in the household for cysts


Explanation: CT will show calcification, but may miss the cysts of
cysticercosis. MRI is the most sensitive test to determine if there are
fresh or active lesions. The parasite is often present in the central
nervous system while absent from the gastrointestinal tract.
Chemotherapy is usually not indicated in uncomplicated disease with
a solitary lesion, and never indicated if all of the lesions are calcified.
The symptoms could be consistent with cerebellar and/or ventricular
cysts. Ventricular cysts may require surgical intervention. (See
Chapter 280 in Nelson Textbook of Pediatrics, 17th ed.)
Ask about visits to county fairs or employment on pork farms

Check pork in the grocery and meat stores frequented by the family

Examine the stool of anyone who has prepared meals for the family.

Check the stool of the family dog

Question . 338. A 15-yr-old girl wishes to play soccer in school and needs medical
clearance. On physical examination, her liver edge is palpable and seems minimally
enlarged. Ultrasonography of the liver shows one cyst approximately 3 cm in diameter. Her
menstruation is normal. Results on serologic testing for Echinococcus and hepatitis are
negative, and liver enzyme values are normal. She has lived her whole life in Salt Lake
City, Utah, and has never traveled abroad. The most important factor in allowing her to play
this sport is:

Her travel history


Explanation: A patient who has only been in an urban area and has
not participated in agricultural activities, has never traveled to areas
endemic for hydatid transmission (e.g., in North America, areas
where there are moose), and has not had contact with sheep or dogs
is unlikely to acquire echinococcal infection. The sensitivity of an
echinococcal serologic study ranges from 60% to 90%. Healthy
hydatid cysts may not stimulate much of an immune response. The
other choices mentioned are not relevant to cystic liver disease. (See
Chapter 281 in Nelson Textbook of Pediatrics, 17th ed.)

Negative echinococcal serology

Negative hepatitis serology

Normal liver enzymes

Normal menstrual periods

Question . 339. Praziquantel is effective in the treatment of all of the following infections
except:

Fish tapeworm infection

Hydatid cysts
Explanation: Hydatid disease (echinococcosis) does not respond to
praziquantel, probably due to failure to penetrate the liver cyst.
Except for Fasciola hepatica, all other trematode and cestode
infections (flukes and tapeworms) will respond to praziquantel. (See
Chapter 281 in Nelson Textbook of Pediatrics, 17th ed.)
Cysticercosis

Schistosomiasis

Hymenolepis nana infection

Question . 340. All of the following information about vaccination is required to be


documented in the patient's medical record except:

Date of administration

Site of administration
Explanation: The National Childhood Vaccine Injury Act requires
that, for vaccines mandated in childhood, health care providers
document the date of administration, manufacturer, lot number, and
name of the health care provider administering the vaccine. (See
Chapter 282 in Nelson Textbook of Pediatrics, 17th ed.)

Manufacturer

Lot number

Name of the person administering the vaccine

Question . 341. Vaccines against the same infectious agent but produced by different
manufacturers are considered interchangeable with the exception of:

Haemophilus influenzae type b vaccines

Hepatitis A vaccines

Hepatitis B vaccines

Acellular pertussis vaccines (as DTaP)


Explanation: Vaccines made by different manufacturers but directed
against the same infections are generally considered
interchangeable, with the exception of DTaP vaccines. (See Chapter
282 in Nelson Textbook of Pediatrics, 17th ed.)

Lyme disease vaccines

Question . 342. Diphtheria and tetanus toxoids and acellular pertussis vaccine (DTaP)
should be administered to healthy children at which ages?
2 and 4 mo and 1-6 yr

2, 4, 6, and 12 mo and 10-12 yr

2, 4, 6, and 15-18 mo, and 4-6 yr


Explanation: DTaP is the preferred vaccine for diphtheria, tetanus,
and pertussis. It is recommended for all children at 2, 4, 6, and 15-18
mo, with a booster at 4-6 yr. Children younger than 7 yr of age and
adults should receive only DT or dT vaccine, as indicated. (See
Chapter 282 in Nelson Textbook of Pediatrics, 17th ed.)

2, 4, 6, and 15-18 mo, 4-6 yr, and every 5 yr thereafter

2, 4, 6, and 15-18 mo, 4-6 yr, and every 10 yr thereafter

Question . 343. Which of the following is a contraindication to administration of MMR?

History of anaphylactic reaction to eggs

Serious allergic reaction to gelatin


Explanation: Measles and mumps vaccines contain insignificant
amounts of egg proteins, and persons with hypersensitivity to eggs
are at negligible risk of anaphylactic reactions from these vaccines.
Hypersensitivity reactions to MMR that were previously attributed to
egg protein may have been caused by allergic reactions to the gelatin
stabilizer in the vaccine. (See Chapter 282 in Nelson Textbook of
Pediatrics, 17th ed.)

Otitis media with fever of 101°F

Febrile convulsion following prior DTaP immunization

History of thrombocytopenia

Question . 344. Preterm, low birthweight infants should be vaccinated at the same
chronological age as for full-term infants and in accordance with the routine childhood
immunization schedule, with the following exception:

DTaP

Pneumococcal conjugate

Hepatitis B
Explanation: Preterm infants, including those of very low birthweight,
should be vaccinated at the same chronological age as for full-term
infants and according to the routine childhood immunization
schedule. One exception to this recommendation is hepatitis B
vaccination of those born of HBsAg-negative mothers with low
birthweights (i.e., <2 kg). Initiation of vaccination in this case should
be delayed until the infant is 1 mo of age. (See Chapter 282 in Nelson
Textbook of Pediatrics, 17th ed.)

IPV

Haemophilus influenzae conjugate combination vaccine

Question . 345. For which of the following infections is an immune globulin product
available that is a monoclonal antibody?

Tetanus

Hepatitis B

Varicella

Respiratory syncytial virus


Explanation: Palivizumab is a monoclonal antibody against RSV that
is recommended for protecting high-risk children against serious
complications from RSV disease. (See Chapter 282 in Nelson
Textbook of Pediatrics, 17th ed.)

Hepatitis A

Question . 346. The only absolute contraindication to subsequent administrations of


acellular pertussis vaccine is:

History of anaphylaxis to egg or egg proteins

Collapse or shock-like state within 2 days of pertussis vaccination

Persistent, inconsolable crying lasting 3 hr or longer within 2 days of


pertussis vaccination

An anaphylactic reaction to a previous dose of whole-cell


pertussis vaccine
Explanation: Generic contraindications to vaccination are an
anaphylactic reaction to a previous dose of the same vaccine and an
anaphylactic reaction to a vaccine constituent, such as egg proteins,
gelatin, or an antibiotic. (See Chapter 282 in Nelson Textbook of
Pediatrics, 17th ed.)

Convulsions with or without fever within 3 days of pertussis


vaccination
Question . 347. Polio vaccine (IPV) should be administered to healthy children at which
ages?

2, 4, 6, and 15-18 mo and 10-12 yr

2 and 4 mo and 4-6 yr

2, 4, and 6-18 mo and 4-6 yr


Explanation: Polio vaccine, as inactivated polio vaccine (IPV),
should be administered at 2, 4, and 6-18 mo and 4-6 yr of age. (See
Chapter 282 in Nelson Textbook of Pediatrics, 17th ed.)

2, 4, 6, and 15-18 mo, 4-6 yr, and every 10 yr thereafter

2, 4, 6, and 15-18 mo, and every 10 yr thereafter

Question . 348. Which of the following is associated with development of autism?

Measles vaccination

Mumps vaccination

Rubella vaccination

Combined measles, mumps, and rubella vaccination

None of the above


Explanation: There is no association between vaccination and
autism. (See Chapter 282 and Table 282-1 in Nelson Textbook of
Pediatrics, 17th ed.)

Question . 349. All of the following are risk factors for hospital-acquired infection except:

Surgical procedures

Foley catheters

Intravascular catheters

Immunizations
Explanation: Immunizations are not a risk factor for hospital-
acquired infections. (See Chapter 283 in Nelson Textbook of
Pediatrics, 17th ed.)

Antibiotic therapy
Question . 350. The most common route of transmission of nosocomial infection is:

Hands
Explanation: Transmission of infectious agents occurs by various
routes, but by far the most common and important route is via the
hands. (See Chapter 283 in Nelson Textbook of Pediatrics, 17th ed.)

Needle stick

Small airborne droplet

Food

Water
Question . 351. You are asked to investigate an outbreak among children with profuse
watery diarrhea in a daycare center. Likely etiologic agents that you should consider
include all of the following except:

Cryptosporidium parvum

Giardia lamblia

Rotavirus

Ascaris lumbricoides
Explanation: Ascaris is not a known cause of daycare-associated
diarrheal epidemics, but choices A-C are. (See Chapter 284 in Nelson
Textbook of Pediatrics, 17th ed.)

Calicivirus

Question . 352. All of the following infections are readily transmissible among children
attending child daycare except:

Cytomegalovirus infection

Rhinoviral infections

Human immunodeficiency virus (HIV) infection


Explanation: No cases of HIV transmission in out-of-home daycare
have been reported. (See Chapter 284 in Nelson Textbook of
Pediatrics, 17th ed.)

Giardia lamblia infection

Pharyngitis
Question . 1. A blonde, blue-eyed girl was admitted at 2.5 mo of age with severe
emesis to rule out pyloric stenosis. She had poor development, eczema, and a
musty odor. The most likely diagnosis is:

Wiskott-Aldrich syndrome

Galactosemia

Cystinosis

Phenylketonuria
Explanation: This is a classic picture of phenylketonuria
(PKU). Fair complexion, eczema, emesis, and mental
retardation are common features. The musty-mousy odor is
phenylacetate, a metabolic by-product of the blocked pathway
due to deficiency of phenylalanine hydroxylase. (See Chapter
74 in Nelson Textbook of Pediatrics, 17th edition.)

Biotinidase deficiency

Question . 2. The child described in Question 1 survives to adulthood and becomes


pregnant. Risk to her fetus if her diet is poorly controlled include all of the following
except:

Microcephaly

Congenital heart disease

Mental retardation

Cataracts
Explanation: Adults who have the autosomal recessive
disease PKU are at risk for mild to moderate neurointellectual
problems if they are on an unrestricted diet. If a woman with
poorly controlled PKU becomes pregnant, her high
phenylalanine levels will be reflected in the fetus, who is at risk
for congenital heart disease, microcephaly, and developmental
delay. The baby will also be a carrier. (See Chapter 74 in
Nelson Textbook of Pediatrics, 17th edition.)

Question . 3. Features of tyrosinemia type I include all of the following except:

Hepatic carcinoma

Coagulopathy

Acute peripheral neuropathy

Hepatomegaly

Congenital heart disease


Explanation: Tyrosinemia often presents acutely with signs of
overwhelming hepatic failure. Long-term results of hepatic
disease may result in cirrhosis and hepatic carcinoma (which is
seen in [tilde ] 40% of cases). (See Chapter 74 in Nelson
Textbook of Pediatrics, 17th edition.)

Question . 4. Classic homocystinuria is characterized by all of the following except:

Mental retardation

Iridodonesis

Ectopia lentis

Short arms
Explanation: This autosomal recessive disorder, due to a
deficiency of cystathionine synthase activity, resembles Marfan
syndrome. Treatment with vitamin B6 is helpful in the subgroup
of patients who are responsive. (See Chapter 74 in Nelson
Textbook of Pediatrics, 17th edition.)

Scoliosis

Question . 5. A previously healthy 2-wk-old now has progressive lethargy. Physical


examination reveals muscle rigidity, opisthotonos posture, periods of hypertonicity,
and flaccidity. Laboratory data reveal hypoglycemia, metabolic acidosis, and
cerebral edema. Plasma levels of leucine, isoleucine, and valine are elevated. The
most likely diagnosis is:

Hartnup disease

Maple syrup urine disease


Explanation: Maple syrup urine disease (MSUD) in the severe
form presents in the first few weeks of life and is often lethal.
Peritoneal dialysis to remove the excessive metabolites of
branch-chain amino acids and means to reverse catabolism are
priorities in treating a metabolic crisis in MSUD. (See Chapter
74 in Nelson Textbook of Pediatrics, 17th edition.)

Phenylketonuria

Homocystinuria

Galactosemia

Question . 6. A 6-wk-old girl has had feeding problems, hypotonia, and emesis for 3
wk. During the last week she developed an erythematous exfoliative generalized
rash and partial alopecia. Laboratory data reveal metabolic acidosis, ketosis, and
hyperammonemia. The most likely diagnosis is:

Acrodermatitis enteropathica

Multiple carboxylase deficiency


Explanation: Multiple carboxylase deficiency in its infantile or
early form is an autosomal recessive disorder and presents
with these features in addition to tachypnea or apnea. Failure to
thrive and lethargy or coma may also develop with time. The
rash helps to distinguish this from other organic acidemias.
(See Chapter 74 in Nelson Textbook of Pediatrics, 17th edition.)

Phenylketonuria

Pellagra

Vitamin B1 deficiency

Question . 7. The treatment of the patient in Question 6 is best accomplished by


administration of:

Biotin
Explanation: Treatment with biotin will result in rapid
improvement. Biotin is an important cofactor for the four-
carboxylase enzymes in this disorder. (See Chapter 74 in
Nelson Textbook of Pediatrics, 17th edition.)

Vitamin B1

Vitamin B6

Vitamin B12

Ascorbic acid

Question . 8. A 2-day-old boy manifests poor feeding, vomiting, and lethargy leading
to coma. Laboratory data reveal respiratory alkalosis and hyperammonemia. The
urine orotic acid level is also elevated. The most likely diagnosis is:

Methylmalonic acidemia

Carbamoylphosphate synthase deficiency

Ornithine transcarbamylase deficiency


Explanation: Ornithine transcarbamylase (OTC) deficiency is
an X-linked disorder and is most severe in males. Superficially
it looks like Reye syndrome, and in older children, OTC
deficiency may mimic Reye syndrome. Unfortunately, infants
with a severe neonatal onset have a very poor prognosis. (See
Chapter 74 in Nelson Textbook of Pediatrics, 17th edition.)

Galactosemia

Reye syndrome

Question . 9. An 8-mo-old girl is brought to your office with the complaint that she
has developmental delay. Her head is enlarged, and she feeds poorly and does not
focus. You examine the baby and find her hypotonic and unable to sit. She has
considerable head lag, and her head circumference is above the 95th percentile.
Which of the following diseases would you consider in the diagnosis?

Birth asphyxia
Familial hydrocephalus

Rubella infection in utero

Canavan disease
Explanation: Answers B, D, and E are possibilities, but the
presence of visual problems and a large head suggest
Canavan disease. The poor vision is due to optic atrophy in this
autosomal recessive disorder, seen most often in Ashkenazi
Jews. (See Chapter 74 in Nelson Textbook of Pediatrics, 17th
edition.)

Hurler disease

Question . 10. At the age of 1 yr, a patient is brought to you with increased
spasticity, which was not present at birth or at 6 mo of age. The patient has a large
head circumference, above the 95th percentile, and an MRI shows diffuse white
matter disease. Which of the following tests would be most likely to be relevant in
this case?

Urinary glycosaminoglycans measurement

Blood phenylalanine levels

N-acetylaspartic acid (NAA) assay


Explanation: Canavan disease produces severe white matter
disease as noted on the MRI. N-Acetylaspartic acid is present
in large amounts in the blood and urine and in the CNS. This
lethal disease has no specific therapy. (See Chapter 74 in
Nelson Textbook of Pediatrics, 17th edition.)

Thyroid profile

Biotinidase activity in the blood

Question . 11. Medium-chain acyl-coenzyme A (CoA) dehydrogenase deficiency is


characterized by all of the following except:

Autosomal recessive inheritance

Hypoketotic hypoglycemia

Cardiomyopathy
Explanation: Muscle weakness or cardiomyopathy is not seen
in medium-chain acyl-CoA deficiency (MCAD). This disorder is
the most common disorder of fatty acid oxidation defects and is
best treated by avoiding prolonged periods of fasting or
catabolic states. (See Chapter 75 in Nelson Textbook of
th
Pediatrics, 17 edition.)

Northern European ancestry

Episodes associated with fasting


Question . 12. A 7-mo-old boy manifests muscle weakness and cardiomyopathy in
addition to hypoketotic hypoglycemia with fasting. An older female sibling also has
periods of muscle pain and rhabdomyolysis. Both have a urinary organic acid profile
with hypoketotic dicarboxylic aciduria. The most likely diagnosis is:

Medium-chain acyl-CoA dehydrogenase deficiency

Long-chain acyl-CoA dehydrogenase deficiency

Carnitine deficiency

Long-chain-very-long-chain acyl-CoA dehydrogenase


deficiency
Explanation: Long-chain acyl-CoA deficiency (LCAD)-very-
long-chain acyl-CoA deficiency (VLCAD) is one disease, which
was originally described as LCAD only. It is more severe than
MCAD, which does not have associated skeletal or cardiac
muscle involvement. Short-chain acyl-CoA deficiency (SCAD)
does not manifest hypoketotic hypoglycemia. (See Chapter 75
in Nelson Textbook of Pediatrics, 17th edition.)

Short-chain acyl-CoA dehydrogenase deficiency

Question . 13. The sister of a 40-yr-old man with adrenomyeloneuropathy seeks


genetic counseling in regard to her risk of having a son with the rapidly progressive
and fatal form of childhood cerebral adrenoleukodystrophy. The affected brother has
slowly progressive moderately severe spastic paraparesis. He is married and
manages a small business successfully. The levels of very-long-chain fatty acids
(VLCFAs) in the sister's plasma are normal. The advice is:

The risk of her having a severely affected son is low because


her plasma VLCFA levels are normal, and the phenotype in her
affected brother is mild. Prenatal studies are not required.

The VLCFA levels should be tested in the sister's cultured skin


fibroblasts. Prenatal studies are not required if VLCFA levels in
fibroblasts are also normal.

If her fetus is male, prenatal testing should be performed on


cultured amniocytes or cultured chorionic villus cells.

Mutation analysis should be performed on the sister's blood.


Prenatal testing is not required if a pathogenic mutation is not
demonstrated.

Mutation analysis should be performed on blood samples


from the sister and the affected brother. Prenatal studies
are not required if she does not carry the pathogenic
mutation demonstrated in her brother.
Explanation: Because the levels of very-long-chain fatty acids
(VLCFAs) may be normal in some carriers, this is not
reassuring. Therefore, the sister and brother should be
evaluated to determine if the sister carries the family's
mutation. This can only be done by analysis and comparison of
the brother's specific mutation with the sister's gene. (See
Chapter 75.2 in Nelson Textbook
Question . 14. A 43-yr-old woman with pigmentary degeneration of the retina and
impaired hearing since childhood has otherwise developed normally. She had good
athletic skills in adolescence and young adulthood and worked as a physical therapy
assistant. In her early 40s she has developed a dementing illness. MRI shows
symmetric demyelination in the parieto-occipital regions. Plasma VLCFA levels are
moderately elevated, and 84% of catalase activity in cultured skin fibroblasts is
located in the cytosolic fraction. The diagnosis is:

Manifesting heterozygote for X-linked adrenoleukodystrophy

Classical Refsum disease

Usher syndrome

Late-onset peroxisome biogenesis disorder


Explanation: This is a classic appearance of a late-onset
disorder. Most others present either in the neonatal period or
later during childhood. (See Chapter 75.2 in Nelson Textbook
of Pediatrics, 17th edition.)

Peroxisomal bifunctional enzyme deficiency

Question . 15. A 10-yr-old boy is found to have an elevated low-density lipoprotein


cholesterol (LDL-C) level. His sister has an elevated triglyceride level. The father has
elevated LDL-C and triglyceride levels. The paternal grandmother has a high
cholesterol level and had a heart attack at the age of 48 yr. (Note: In this case,
elevated levels refer to levels greater than 95th percentile.) The most likely diagnosis
for this child is:

Familial hypercholesterolemia

Polygenic hyperlipidemia

Familial combined hyperlipidemia


Explanation: The family demonstrates significant elevations in
cholesterol and triglycerides and a relatively benign childhood
course, but early coronary artery disease (<50 yr of age). (See
Chapter 75.3 in Nelson Textbook of Pediatrics, 17th edition.)

Unknown

Dysbetalipoproteinemia

Question . 16. A 15-yr-old girl is found to have an LDL-C level of 191 mg/dL;
triglyceride, 50 mg/dL; HDL-C, 41 mg/dL; and total cholesterol, 242 mg/dL. The
child's father had a heart attack at age 45 yr. He takes one of the so-called statin
medications (HMG-CoA reductase inhibitor), and wants his daughter to be given the
same medication. How do you respond?

The statin medications are not approved for use in potentially


childbearing women

The statin medications are not approved for use in children

No medication should be tried until the response to dietary


modification is assessed
Explanation: Although cholesterol levels are elevated, the
recommended first approach to the teenaged patient is to begin
with dietary restrictions of cholesterol and fat. (See Chapter
75.3 in Nelson Textbook of Pediatrics, 17th edition.)

Medication should be prescribed right away, because the


response to diet modification is likely to be limited in this case

Niacin is the treatment of choice

Question . 17. Dietary fat should be restricted to 20-30% of calories:

In all persons except children younger than 2 yr, as this


age group needs a higher-fat diet to support rapid growth
Explanation: Fat is an essential source of calories, essential
fatty acids, and cholesterol, which are all needed in the rapidly
growing toddler. Parents who restrict fat by using skim milk in
children <2 yr have caused failure to thrive in their children.
(See Chapter 75.3 in Nelson Textbook of Pediatrics, 17th
edition.)

In all persons except children younger than 4 yr, as this age


group needs a high fat intake to support the rapid growth of the
brain that occurs in the first 4 yr of life

In all persons except children younger than 15 yr, as a higher


fat intake is needed to support normal pubertal development

Only for individuals with hyperlipidemia

Only for individuals with hypertriglyceridemia

Question . 18. A 5-yr-old child has a paternal grandmother who had a heart attack
at the age of 45 yr. The child's father has hypercholesterolemia and takes
medication to lower his cholesterol level. How should the child be screened?

Non-fasting total cholesterol, as the total cholesterol level does


not vary significantly between

fasting and non-fasting states

Fasting total cholesterol, as the cholesterol level varies


significantly after eating

Fasting total cholesterol, triglyceride, HDL-C, and


calculated LDL-C to evaluate for elevated cholesterol or
triglycerides, or low HDL-C levels
Explanation: These laboratory tests are the recommended
approach to evaluate this 5-yr-old at risk for familial
hypercholesterolemia. (See Chapter 75.3 in Nelson Textbook
of Pediatrics, 17th edition.)

Should not be screened until older than 10 yr


Should be screened only if the child has other risk factors for
cardiovascular disease

Question . 19. GM1 gangliosidosis is characterized by all of the following except:

Psychomotor retardation

Angiokeratomata

Hepatosplenomegaly

Frontal bossing

Peripheral neuropathy
Explanation: This lethal lipidosis disorder is a progressive
neurovisceral storage disease that has some features similar to
those of some severe mucopolysaccharidoses. In addition to
dysmorphic features (coarsened facies), deafness, blindness,
seizures, and hepatosplenomegaly occur. (See Chapters 74.4
and 74.5 in Nelson Textbook of Pediatrics, 17th edition.)
Question . 20. A 4-mo-old girl presents with developmental delay, an exaggerated
startle response to loud noise, and macrocephaly. On physical examination, the
child has decreased eye contact and a cherry red spot in each retina. The most
likely diagnosis is:

GM2 gangliosidosis
Explanation: Also known as Tay-Sachs disease, this lethal
autosomal recessive lipidosis manifests with progressive
neurologic deterioration. There is no treatment, but prenatal
diagnosis is possible. (See Chapters 74.4 and 74.5 in Nelson
Textbook of Pediatrics, 17th edition.)

Gaucher disease

Fabry disease

Galactosemia

Glycogen storage disease, type I

Question . 21. A 15-yr-old presents with chronic fatigue and severe bone pain of 1
year's duration. He has hepatosplenomegaly and a normal retinal examination.
Laboratory studies reveal normocytic anemia and thrombocytopenia. Radiographs of
the distal femur reveal Erlenmeyer flask deformities. The most likely diagnosis is:

Lymphoma

Gaucher disease
Explanation: This is a classic presentation for type I Gaucher
disease, a lipidosis most commonly seen in Ashkenazi Jews.
The central nervous system is not involved, but bone pain and
hematologic problems can be severe. Treatment is effective
with recombinant enzyme replacement infusions and is quite
successful. (See Chapters 74.4 and 74.5 in Nelson Textbook of
Pediatrics, 17th edition.)

Sandhoff disease

GM3 gangliosidosis

Sickle cell anemia

Question . 22. Fabry disease is characterized by all of the following except:

Acroparesthesias

Corneal opacities

Cardiomyopathy

Mental retardation
Explanation: This X-linked disorder causes severe debilitating
pain but no mental retardation. Treatment is effective with
recombinant enzyme replacement therapy. (See Chapters 74.4
and 74.5 in Nelson Textbook of Pediatrics, 17th edition.)

Angiokeratomas

Question . 23. A 14-mo-old girl presents with irritability, poor walking, genu
recurvatum, and hypotonia.

Physical examination reveals absent deep tendon reflexes.


Laboratory data reveal an increased CSF protein level and
decreased nerve conduction velocities. The most likely
diagnosis is:

Metachromatic leukodystrophy
Explanation: This is the classic preservation of metachromatic
leukodystrophy (MLD), an autosomal recessive disorder with no
treatment. The patient's condition deteriorates progressively.
Metachromatic granules may be seen in the urine as well as
sural nerve biopsies. (See Chapters 74.4 and 74.5 in Nelson
Textbook of Pediatrics, 17th edition.)

I cell disease

GM3 gangliosidosis

Fabry disease

Neuromyopathic Gaucher disease

Question . 24. A 3-mo-old previously healthy and well infant presents to an


emergency room with a generalized seizure. The physical examination shows a
temperature of 36.5°C, respiration rate of 50/min, and marked hepatomegaly, with
the liver edge palpable down to the umbilicus. Laboratory findings show Na, 141
mEq/L; Cl, 95 mEq/L; K, 4.5 mEq/L; BUN, 12 mg/dL; Cr, 0.2 mg/dL; glucose, 25
mg/dL; CO2, 12 mEq/L; uric acid, 7.1 mg/dL; cholesterol, 270 mg/dL; and
triglyceride, 550 mg/dL. The most likely diagnosis is:

Hyperinsulinemia

Adrenal insufficiency

Growth hormone deficiency

Glycogen storage disease


Explanation: The child most likely has glycogen storage
disease, type I, also called glucose-6-phosphatase deficiency.
During fasting from a viral illness or by sleeping longer between
feeds, the metabolic defect becomes manifest as an inability to
release glucose from glycogen. Hypoglycemia, lactic acidosis,
ketosis, and hyperlipidemia are noted, in addition to high uric
acid levels. (See Chapter 76 in Nelson Textbook of Pediatrics,
17th edition.)

Mitochondrial disorder

Question . 25. A 6-day-old breast-fed infant has had poor feeding and vomiting
since birth. On physical examination, he is jaundiced, has hepatomegaly, and shows
excessive bruising. Laboratory investigations show elevated transaminase levels,
prolonged clotting time, and aminoaciduria. Galactosemia is suspected; however,
urine-reducing substance is negative. What is the most appropriate next step in
diagnosis?

Oral galactose tolerance test

Challenge with lactose-containing formula

Blood analysis to measure galactose-1-phosphate


uridyltransferase enzyme activity
Explanation: Because of perceived feeding intolerances, the
child may have been fed with a non-lactose containing formula
(soy type), or with fasting due to the metabolic
decompensation, the intake of lactose may have been
insufficient to demonstrate a reducing substance-positive urine.
Measuring the enzyme is the most reliable test to diagnose
galactosemia under any conditions, unless the patient has
received a blood transfusion. (See Chapter 76 in Nelson
Textbook of Pediatrics, 17th edition.)

Liver biopsy

Kidney biopsy
Question . 26. A 17-yr-old boy has a history of recurrent episodes of exercise
intolerance with muscle cramps. He is otherwise healthy and well. Which of the
following is most likely to develop during these episodes in this patient?

Myoglobinuria
Explanation: The history is compatible with muscle
glycogenosis. With decreased ability to release glucose from
muscle glycogen, myocyte injury occurs and places the patient
at risk for myoglobinuria and renal failure. (See Chapter 76 in
Nelson Textbook of Pediatrics, 17th edition.)
Abnormal liver function

Aminoaciduria

Cardiac arrhythmia

Metabolic acidosis

Question . 27. A patient with recurrent hypoglycemia and hepatomegaly is


diagnosed to have type I glycogen storage disease. Which of the following is unlikely
to raise the blood glucose levels during the hypoglycemia episode?

IV glucose

Oral uncooked cornstarch

Oral Polycose

Oral Pedialyte

Oral fructose
Explanation: Fructose must first be converted to glucose
within the hepatocyte. Under normal circumstances, the
intermediary metabolite, glucose-6-phosphate, will then be
released to the circulation after enzymatic conversion to
glucose by hepatic glucose-6-phosphatase. This enzyme is
deficient or missing in this form of glycogen storage disease.
(See Chapter 76 in Nelson
Question . 28. A 5-mo-old infant with gross motor delay, hypotonia, and
cardiomegaly has a creatine kinase level of 860 IU/L. Of the following, which is the
most likely diagnosis?

Spinal muscular atrophy

Hypothyroidism

Prader-Willi syndrome

Pompe disease
Explanation: Pompe disease is a glycogen storage disease
that affects the heart and skeletal muscle. Cardiac involvement
causes heart failure and a shortened P-R interval on the ECG.
Treatment with recombinant enzyme replacement has shown
promise in the management of Pompe disease. (See Chapter
76 in Nelson Textbook of Pediatrics, 17th edition.)

Down syndrome

Question . 29. Hurler disease is characterized by all of the following except:

Hepatosplenomegaly
Coarse facies

Dysostosis multiplex

X-linked inheritance
Explanation: All the mucopolysaccharidoses are inherited as
autosomal recessive disorders except Hunter syndrome (MPS
II), which is inherited as an X-linked trait. (See Chapter 77 in
Nelson Textbook of Pediatrics, 17th edition.)

Corneal clouding

Question . 30. Potential cardiac problems in patients with mucopolysaccharidoses


include all of the following except:

Coronary artery disease

Mitral regurgitation

Pericarditis
Explanation: Storage in endothelial cells can cause coronary
artery disease and even myocardial infarction. Cardiomyopathy
may be present in infancy and is associated with endocardial
fibroelastosis. Valve disease necessitating valve replacement
usually occurs in older children. (See Chapter 77 in Nelson
Textbook of Pediatrics, 17th edition.)

Aortic valve disease

Cardiomyopathy

Endocarditis

Question . 31. A 3-yr-old boy was normal at birth but developed progressive
coarsening of the face and developmental delay. In addition to coarse facies and
mental retardation, the physical examination reveals a cardiac systolic regurgitant
murmur, hepatomegaly, joint stiffness, and short stature. CT scan reveals
hydrocephalus. There is no corneal clouding. All affected family members have been
boys. The most likely diagnosis is:

Hurler syndrome

Galactosemia

Metachromatic leukodystrophy

Hunter syndrome
Explanation: This X-linked disorder affects only males, who do
not reproduce. The phenotype of severe Hunter syndrome
(MPS II) is similar to that of Hurler syndrome (MPS I), but it
does not include corneal clouding and it progresses at a slightly
slower rate. (See Chapter 77 in Nelson Textbook of Pediatrics,
17th edition.)
Mucopolysaccharidosis type III (Sanfilippo syndrome)

Question . 32. A 14-mo-old white boy presents with chorea of 2 months' duration.
Prior to this, he had failure to thrive and recurrent emesis. He was normal until age 4
mo, when hypotonia and vomiting began. Physical examination reveals hyperreflexia
and erosions on his lips and fingers. Laboratory data reveals a normal BUN and
ammonia levels and an elevated uric acid level. An older male sibling is in an
institution for severely disabled teens. The most likely diagnosis is:

Glycogen storage disease, type VI

Mucopolysaccharidosis, type XII

GM1 gangliosidosis

Familial X-linked gout

Lesch-Nyhan syndrome
Explanation: This is a classic presentation of this X-linked
genetic disorder, which is characterized by complete deficiency
of the hypoxanthine-guanine phosphoribosyltransferase
enzyme. Partial enzyme deficiency may cause some cases of
severe gout with normal cerebral function. Treatment with
allopurinol reduces the hyperuricemia but does not alter the
CNS manifestations
Question . 33. A 4-yr-old girl presents with severe failure to thrive with weight of 18
lb and height of 30 inches, a high-pitched voice, a sclerodermatous appearance to
skin overlying the abdomen, nearly total alopecia, a small jaw and face relative to
the head circumference, and normal mental and motor development. The most likely
diagnosis is:

Cystic fibrosis

Child abuse and malnutrition

Hutchinson-Gilford progeria syndrome


Explanation: This disorder of unknown etiology is associated
with rapidly progressive characteristics of aging, with normal
motor and mental development. (See Chapter 79 in Nelson
Textbook of Pediatrics, 17th edition.)

Marfan syndrome.

Hallermann-Streiff syndrome

Question . 34. An 18-yr-old girl complains of abdominal pain with menses that seem
out of proportion to the menstrual pain of her peers. She has intermittent
constipation and diarrhea. During a recent episode of pain, she was noted to have
hypertension, tachycardia, and urinary retention. Her urine was noted to have a port
wine discoloration when the urinary catheter was placed to relieve her acute urinary
retention. The most likely diagnosis is:

Acute intermittent porphyria


Explanation: Acute intermittent porphyria is one of the most
common of the porphyrias and presents with pain and
neurologic symptoms. It is rare before puberty and is
exacerbated by menstruation and other precipitating factors
such as drugs, other illness, and stress. (See Chapter 80 in
Nelson Textbook of Pediatrics, 17th edition.)

Congenital erythropoietic porphyria

Porphyria cutanea tarda

Mucopolysaccharidosis

System lupus erythematosus

Question . 35. A 3-day-old white girl was found unresponsive in the normal newborn
nursery. The mother was a 35-yr-old (gravida 3, para 2) with a normal pregnancy,
including screening tests negative for herpes simplex, group B streptococci, HIV,
diabetes (normal glucose tolerance test), and -fetoprotein. Her other children are
normal. The baby weighed 4.5 kg at birth and had Apgar scores of 9 and 9. There is
no hepatosplenomegaly or respiratory distress, but the baby has a weak cry, is
hypotonic, and responds poorly to stimuli. Immediate evaluation should include all of
the following except:

Blood magnesium level


Explanation: Hypermagnesemia, usually associated with
maternal magnesium sulfate therapy for preeclampsia, is
usually most severe on the first day of life. Hypomagnesemia
presents like hypocalcemia with neuromuscular irritability,
hyperreflexia, and clonus. (See Chapter 81 in Nelson Textbook
of Pediatrics, 17th edition.)

Blood glucose level

Blood culture

Arterial blood gas analysis

Blood electrolytes
Question . 36. The infant described in Question 35 has a blood glucose level of 22
mg/dL. The most appropriate therapy is:

Intravenous glucose infusion at 2-4 mg/kg/min

Oral administration of 5% dextrose in water (D5W)

Intravenous glucose bolus of D10W at 2 mL/kg


Explanation: A mini-bolus of glucose is indicated in all
hypoglycemic infants with symptoms. (See Chapter 81 in
Nelson Textbook of Pediatrics, 17th edition.)

Administration of prednisone

Administration of diazoxide

Question . 37. The infant described in Questions 35 and 36 initially becomes more
responsive and has normal tone. Three hours later she has a seizure, and her blood
glucose level is 18 mg/dL. The most appropriate next step in management is to:

Begin prednisone administration

Repeat a mini-bolus of D10W

Give a mini-bolus of D10W plus continuous intravenous


glucose infusions
Explanation: The first mini-bolus corrected the hypoglycemia
but was not followed by any exogenous source of glucose, and
hypoglycemia recurred. (See Chapter 81 in Nelson Textbook of
Pediatrics, 17th edition.)

Give continuous intravenous glucose infusion

Administer diazoxide

Question . 38. The infused glucose concentrations in the patient describedin


Question 35 have to be increased to 25% glucose, given by a central venous
catheter to maintain blood glucose levels of just 45 mg/dL. The most likely diagnosis
is:

Galactosemia

Glycogen storage disease

Hyperinsulinemic hypoglycemia
Explanation: This is a classic case of persistent
hyperinsulinemic hypoglycemia, which was previously called
nesidioblastosis. Defects in the regulatory KATP channel, which
stimulates insulin release from the pancreatic islet cells, is the
most common of these disorders and may be familial or
sporadic. Treatment with diazoxide or octreotide may be
successful; some patients require pancreatic resection. (See
Chapter 81 in Nelson Textbook of Pediatrics, 17th edition.)

Infant of diabetic mother

Congenital herpes simplex infection

Question . 39. A 9-day-old full-term infant is admitted to the hospital with lethargy,
fever, and increasing jaundice. Physical examination also reveals hepatomegaly.
Laboratory results reveal a blood glucose value of 10 mg/dL, total and direct bilirubin
values of 15 and 7 mg/dL, respectively, and liver enzyme test shows AST??700
units/L, and ALT, 650 units/L. The next day, the blood culture is positive for a gram-
negative rod. The most likely diagnosis is:

Necrotizing enterocolitis

Galactosemia
Explanation: The combination of hypoglycemia, jaundice,
elevated liver enzyme values, and Escherichia coli sepsis is
classic for early-onset severe galactosemia. A white blood cell
defect may predispose to E. coli sepsis, and the toxic effects of
galactose-1-phosphate explain the hepatotoxicity. In this child,
who was fed cow's milk-based formula, the urine-reducing
substances (galactose) were positive. (See Chapter 76.2 in
Nelson Textbook of Pediatrics, 17th edition.)

Neonatal hepatitis

Glycogen storage disease

Biliary atresia
Question . 40. A previously healthy 6-mo-old presents with hepatomegaly, lethargy,
increasing jaundice, and severe emesis. The child appears dehydrated; the urine
also has a positive reaction for reducing substances. The child's diet has been solely
breast milk until age 5 mo, when fruit juices and baby food were added to the diet.
The most likely diagnosis is:

Galactosemia

Gycogen storage disease

Benign fructosuria

Hereditary fructose intolerance


Explanation: The acute or subacute presentation of hereditary
fructose intolerance is very similar to that of galactosemia. The
inability to metabolize fructose can produce shock,
hypoglycemia, hepatic dysfunction, and emesis. In the child
described in the question, the urine-reducing substance was
fructose. Before the initiation of fructose (sucrose)-containing
foods (e.g., fruit juices), such a child will be asymptomatic. (See
Chapter 76.3 in Nelson Textbook of Pediatrics, 17th edition.)

Pyruvate carboxylase deficiency

Question . 41. A 2-mo-old presents with failure to thrive, emesis, alopecia, rash, and
chronic metabolic acidosis. An older sibling died at age 3 mo with hypotonia and
chronic lactic acidosis. An important diagnostic study is determination of:

Blood galactose level

Serum lactate dehydrogenase level

Serum biotinidase level


Explanation: Biotinidase levels may reflect enzymatic
deficiencies affecting carbohydrate and amino acid (organic
acid) metabolism. Both enzymatic pathways require biotinidase;
deficiencies produce manifestations, as noted in this case.
Treatment with oral biotin may overcome this defect in some
affected patients. (See Chapter 74 in Nelson Textbook of
Pediatrics, 17th edition.)

Plasma triglyceride levels

Serum calcium and magnesium levels


Question . 42. A previously healthy 4-mo-old now manifests increasing hypotonia
and poor feeding. Physical examination reveals macroglossia, a gallop rhythm,
tachycardia, and marked flaccidity, but mental status is normal. Laboratory studies
reveal a blood glucose level of 85 mg/dL and sinus tachycardia with a shortened P-R
interval on an electrocardiogram. The most helpful diagnostic study would be:

MRI of the spine

Glucagon infusion test

Muscle biopsy
Explanation: Muscle biopsy may confirm the diagnosis of
Pompe disease (glycogen storage disease, type II). Deficiency
of acid -glucosidase results in marked lysosomal glycogen
accumulation and primarily affects the heart and skeletal
muscle. Death in the infantile form is due to respiratory muscle
failure. (See Chapter 76.1 in Nelson Textbook of Pediatrics,
17th edition.)

Lumbar puncture

Question . 43. A 15-yr-old Ashkenazi Jewish girl is seen because of chronic fatigue.
On examination, she seems pale and thin and has a somewhat large abdomen. Her
spleen is felt in the iliac fossa. She is mentally alert and has a history of normal
development and normal school performance. Her blood count shows hemoglobin,
9.0 g/dL and a white blood cell count of 3,000/cu mm, with normal differential cell
count and no abnormal cells. Platelet count is 60,000/ L. The most likely diagnosis
is:

Tay-Sachs disease

Niemann-Pick disease type A

Gaucher disease type I


Explanation: All five diseases listed in the question are
prevalent among Ashkenazi Jews. Niemann-Pick disease, type
A, and Gaucher disease, type I, are the only two diseases that
present with a very large spleen. Children with Niemann-Pick,
type A, disease are likely to be retarded, but this patient is not.
Anemia, leukopenia, and thrombocytopenia are typically found
in Gaucher disease. Patients with Canavan disease, Tay-Sachs
disease, and mucolipidosis, type IV, are neurologically
impaired, but this patient is not. (See Chapter 75 in Nelson
Textbook of Pediatrics, 17th edition.)

Mucolipidosis type IV

Canavan disease

Question . 44. A 13-mo-old infant is found comatose in bed after sleeping later than
usual. On physical examination, the infant is afebrile and of normal size, and the
liver is palpated 4 cm below the costal margin. The plasma glucose level is 15
mg/dL; bicarbonate, 20 mEq/L;BUN, 35 mg/dL; ammonia, 295 mol/L; AST, 320
units/L; and ALT, 425 units/L. Bilirubin is normal. Urinalysis is negative for glucose,
ketones, protein, and reducing substances. Which of the following is the most likely
diagnosis?

Medium-chain acyl-CoA dehydrogenase deficiency


Explanation: Nonketotic hypoglycemia, with or without
hyperammonemia, in a patient of this age is often due to this
relatively common cause. In addition to hypoglycemia, medium-
chain acyl-CoA deficiency (MCAD) is associated with a Reye-
like syndrome, and some cases are initially diagnosed as
sudden infant death syndrome (SIDS). (See Chapter 73 in
Nelson Textbook of Pediatrics, 17th edition.)

Glucose-6-phosphatase deficiency (type I glycogenosis)

Congenital hyperinsulinism

Growth hormone deficiency

Isovaleric acidemia

Question . 45. Plasma very-long-chain fatty acids are elevated in all of the following
peroxisomal disorders except:

Zellweger syndrome

Rhizomelic chondrodysplasia punctata


Explanation: In rhizomelic chondrodysplasia punctata, plasma
levels of phytanic acid and erythrocyte plasmalogens are
elevated. (See Chapter 75.2 in Nelson Textbook of Pediatrics,
17th edition.)

Pseudo-Zellweger syndrome

Neonatal adrenoleukodystrophy

Infantile Refsum disease

Question . 46. Of the following physical findings noted in some patients with
hyperlipidemia, which is most likely in a 17-yr-old boy with heterozygous familial
hypercholesterolemia?

Arcus corneae

Xanthelasma

Cutaneous xanthomas over the buttocks

Tuberous xanthomas causing Achilles tendinitis


Explanation: The Achilles tendon is a first site for the
development of xanthomas, an area that most of us do not
usually examine other than to test deep tendon reflexes. (See
Chapter 75.2 in Nelson Textbook of Pediatrics, 17th edition.)

Tuberous xanthomas over the elbow


Question . 47. A 7-mo-old boy has been healthy and developing normally since
birth. His mother now reports that he has decreased eye contact with her, even
during feedings. The infant also startles very easily when there is a loud noise in the
house. Of the following, the most appropriate diagnostic test to confirm the etiology
of these findings is the measurement of:

Leukocyte -hexosaminidase A activity


Explanation: Tay-Sachs disease classically presents with
these clinical features about this time of infancy. A cherry red
spot on the fundi would also be present. (See Chapter 75 in
Nelson Textbook of Pediatrics, 17th edition.)

Serum concentration of amino acids

Serum concentration of ammonia

Urinary mucopolysaccharides

Urinary organic acids

Question . 48. A 7-mo-old white girl presents with severe developmental delay and
episodes of vomiting. Physical examination reveals blue eyes, light skin with an
eczematoid rash, and hyperactive deep tendon reflexes. There is failure to thrive
and microcephaly. The most likely diagnosis is:

Child neglect

TORCH infection

VATER syndrome

Phenylketonuria
Explanation: Phenylketonuria presents with the gradual
appearance of these symptoms. All children appear normal at
birth. (See Chapter 74 in Nelson Textbook of Pediatrics, 17th
edition.)

Galactosemia

Question . 49. Appropriate treatment of the child described in Question 49 before


the onset of serious symptoms is helpful in preventing severe retardation.
Overtreatment may result in:

Obesity and striae

A musty, mousy odor

Headache and pseudotumor cerebri

Rectal prolapse and colitis

Anorexia, lethargy, and rash


Explanation: Appropriate treatment lowers the serum level of
the essential amino acid phenylalanine. Too severe restriction
of this amino acid produces lethargy, anorexia, rash, diarrhea,
and death. (
Question . 50. Complications of tyrosinemia include all of the following except:

Cataracts
Explanation: Cataracts are unusual in tyrosinemia; they are
common in galactosemia. (See Chapter 74.2 in Nelson
Textbook of Pediatrics, 17th edition.)

Renal tubular acidosis

Peripheral neuropathy

Cirrhosis

Hepatic carcinoma
Question . 51. A 4-yr-old blue-eyed white girl manifests a malar flush, mild mental
retardation, subluxation of the ocular lens, iridodonesis, and Marfan-like features
(tall, thin, arachnodactyly). The most likely diagnosis is:

Hawkinsinuria

Alcaptonuria

Piebaldism

Homocystinuria
Explanation: Homocystinuria is due to a deficiency of
cystathionine synthase. (See Chapter 74.3 in Nelson Textbook
of Pediatrics, 17th edition.)

Angelman syndrome

Question . 52. Additional complications that may occur in the patient described in
Question 51 include all of the following except:

Seizures

Thromboembolism

Osteoporosis

Scoliosis

Cardiomyopathy
Explanation: A through D are all correct. The hypercoagulable
state may even be present if homocysteine levels are elevated
without any other signs of homocystinuria. (See Chapter 74.3 in
Nelson Textbook of Pediatrics, 17th edition.)

Question . 53. In the patient described in Questions 51 and 52, treatment is initiated
with high-dose vitamin B6, but no response is observed. The most likely explanation
is:
Folate deficiency
Explanation: After high-dose folate treatment, the patient
responds to the primary therapy of high-dose vitamin B6. (See
Chapter 74.3 in Nelson Textbook of Pediatrics, 17th edition.)

Malabsorption

Gastric hypersecretion

Vitamin B1 deficiency

Vitamin C deficiency
Question . 54. A 10-day-old child manifests profound coma 1 day after an illness
characterized by emesis, poor oral intake, and hypotonia. Laboratory findings reveal
4+ ketonuria; arterial blood gas, 6.9 mm Hg; PCO2, 19 mm Hg; PO2, 95 mm Hg;
anion gap, 37 mEq/L; and absolute neutrophil count, 400/cu mm. The most likely
diagnosis is:

Galactosemia

Glycogen storage disease type II

Methylmalonic acidemia
Explanation: Methylmalonic acidemia (MMA) presenting like
this in a newborn has a poor prognosis, even if the child
survives the acute episode. (See Chapter 74.6 in Nelson
Textbook of Pediatrics, 17th edition.)

Phenylketonuria

Primary carnitine deficiency

Question . 55. In addition to routine supportive care, what additional therapy would
be most appropriate for the child described in Question 54?

Vitamin B12
Explanation: Some cases of MMA are responsive to large
doses (1-2 mg/24 hr) of vitamin B12. (See Chapter 74.6 in
Nelson Textbook of Pediatrics, 17th edition.)

Vitamin B1

Biotin

Vitamin C

Folate

Question . 56. A 10-mo-old Ashkenazi Jewish girl manifests hypotonia and


macrocephaly. By 18 mo of age, she has hyperreflexia and optic atrophy. Additional
problems include failure to thrive and swallowing difficulties. MRI demonstrates
diffuse white matter degeneration of the cerebral cortex. The most likely diagnosis is:
Adrenal insufficiency

Cerebral palsy

Encephalomyelitis

Alexander disease

Canavan disease
Explanation: Canavan disease due to excessive amounts of
brain N-acetylaspartate acid produces leukodystrophy with
macrocephaly and loss of developmental milestones.
Alexander disease is similar but progresses at a slower rate.
(See Chapter 74.14 in Nelson Textbook of
Question . 57. A 7-yr-old boy, previously an "A" student, demonstrates progressive
hyperactivity and is diagnosed with attention deficit hyperactivity disorder. Despite
therapy, his school performance deteriorates. He has difficulty understanding people
when spoken to on the telephone. Seizures develop 6 mo later. Physical
examination reveals slight diffuse hyperpigmentation, spasticity, and reduced deep
tendon reflexes. The most likely diagnosis is:

Acute demyelinating encephalitis

Guillain-Barré syndrome

Zellweger disease

X-linked adrenoleukodystrophy
Explanation: Adrenoleukodystrophy, a peroxisomal disorder,
presents with progressive dementia, seizures, MRI cerebral
white matter lesions (symmetrical periventricular sites), and
adrenal insufficiency (as determined by a poor cortisol
response to intravenous ACTH). (See Chapter 75.2 in Nelson
Textbook of Pediatrics, 17th edition.)

Charcot-Marie-Tooth neuropathy

Question . 58. The best diagnostic test for the patient described in Question 57 is

Serum amino acid assay

Nerve conduction studies

Serum very-long-chain fatty acids determination


Explanation: Very-long-chain fatty acids are elevated in
patients with X-linked adrenoleukodystrophy. (See Chapter
75.3 in Nelson Textbook of Pediatrics, 17th edition.)

MRI angiography

Cerebrospinal fluid myelin basic protein measurement

Question . 59. A 16-yr-old boy presents with easy bruisability and chronic fatigue.
He has a 3-yr history of bone pain and poor growth. Physical examination reveals
hepatosplenomegaly. Laboratory studies reveal normal liver enzyme levels but a
hematocrit of 25% and a platelet count of 25,000/cu mm. X-ray films of the skeleton
reveal long bone lytic lesions and osteosclerosis. The most likely diagnosis is:

Leukemia

Sickle cell anemia

Tay-Sachs disease

Hunter syndrome

Gaucher disease
Explanation: Although leukemia is of concern, there are no
blasts in the blood or bone marrow, and the course is
somewhat protracted for acute leukemia. (See Chapter 75.4 in
Nelson Textbook of

Question . 60. The best diagnostic test for the patient described in Question 59 is:

Bone marrow biopsy

Assay of acid -glucosidase in leukocytes


Explanation: Gaucher disease is due to a deficiency of
enzyme with the resultant build-up of glucocerebrosides in the
reticuloendothelial system and subsequent dysfunction or
enlargement of the respective organs. (See Chapter 75.4 in
Nelson Textbook of Pediatrics, 17th edition.)

Ophthalmoscopy for cherry-red spots

Urine long-chain amino acids measurement

Urine long-chain fatty acids measurement

Question . 61. Treatment of the patient described in Questions 59 and 60 is best


achieved with:

Liver transplantation

Purified placental acid -glucosidase


Explanation: Enzyme replacement therapy has been very
effective in reducing extraskeletal symptoms. It is hoped that in
the future, recombinant enzyme replacement therapy will
eliminate the small risk of infection associated with the disease.
(See Chapter 75.4 in Nelson Textbook of Pediatrics, 17th
edition.)

Lovastatin administration

Plasmapheresis

Sphingomyelinase of lysosomal origin


SPECIAL HEALTH PROBLEMS DURING ADOLESCENCE
Question . 1. Oral contraceptive agents are associated with increased risk of
all of the following except:

Thrombophlebitis

Carbohydrate intolerance

High levels of high-density lipoproteins

Premature epiphyseal closure


Explanation: Oral contraceptive agents in the available
doses contain too little estrogen to close growth plates. In
addition, most females use oral contraceptive agents after
the adolescent growth spurt. Other complications of oral
contraceptives are quite rare in adolescent patients, and
thrombophlebitis or diabetes is very unusual. (See Chapter
108 in Nelson Textbook of Pediatrics, 17th ed.)

Question . 2. False-positive VDRL test results are most likely to occur in:

Mononucleosis

Systemic lupus erythematosus

Endocarditis

Intravenous drug misuse

Tuberculosis

All of the above


Explanation: All of the conditions listed in the question
produce a false-positive result in the VDRL or other
nontreponemal test for syphilis. This patient actually had
systemic lupus erythematosus (SLE). (See Chapter 110 in
Nelson Textbook of Pediatrics, 17th ed.)
Question . 3. A foul-smelling vaginal discharge that emits a fishy odor on
contact with 10% potassium hydroxide and that demonstrates clue cells on the
wet preparation is most likely to be due to:

Gonorrhea

Chlamydia infection

Chancroid

Gardnerella vaginalis infection


Explanation: Bacterial vaginosis due to G. vaginalis or
other pathogens typically is associated with these
biochemical and microscopic features. Treatment in
nonpregnant women is with metronidazole. (See Chapter
110 in Nelson Textbook of Pediatrics, 17th ed.)

Candida infection

pulmonary edema. He has hypertrophic lesions over the dorsum of the left
hand and the antecubital fossa. The most effective therapy is:

Intravenous glucose

Flumazenil

Disulfiram

Naloxone (Narcan)
Explanation: Narcan is indicated in this patient with
intravenous drug misuse and a heroin overdose.
Nonetheless, the patient will need the ABCs of
resuscitation addressed if he is cyanotic, and so forth.
Securing an airway while providing oxygen and artificial
ventilation is the first priority-followed by Narcan, the
antidote for opiates. (See Chapter 105 in Nelson Textbook
of Pediatrics, 17th ed.)

Intravenous calcium
Question . 5. Toluene (glue, solvents) misuse is associated with:

Hallucinations

Tolerance

Pulmonary edema

Peripheral neuropathy

Rhabdomyolysis

All of the above


Explanation: Both acute and chronic effects of organic
solvent misuse produce significant morbidity and mortality.
(See Chapter 105 in Nelson Textbook of Pediatrics, 17th
ed.)

Question . 6. Anabolic steroid use is associated with all of the following


except:

Enhanced school performance


Explanation: School performance usually deteriorates as a
result of mood changes and aggressive behavior. (See
Chapter 105 in Nelson Textbook of Pediatrics, 17th ed.)

Testicular atrophy

Aggressive behavior

Cholestasis

Increased low-density lipoprotein levels


Question . 7. A 15-yr-old girl has experienced a loss of 30 pounds during the
past 6 mo and has amenorrhea. She denies vomiting, diarrhea, and abdominal
pain and claims to feel well. Physical examination reveals cachexia and a
pulse of 40. Electrolyte determination reveals a serum potassium level of 3.0
mEq/L and bicarbonate of 30 mEq/L. Hematocrit is 30, and erythrocyte
sedimentation rate is 3 mm/hr. The most likely cause of this patient's condition
is:

Inflammatory bowel disease

Anorexia nervosa
Explanation: Such severe wasting is compatible with
anorexia nervosa. Bulimia does not produce such severe
weight loss. Additional features of anorexia nervosa include
bradycardia, hypothermia, amenorrhea, and hypokalemia.
(See Chapter 104 in Nelson Textbook of Pediatrics, 17th
ed.)

Bulimia nervosa

Addison disease

Pituitary adenoma

Question . 8. Leading causes of hospitalization for adolescents include all of


the following except:

Pregnancy

Mental disorders

Asthma

Anemia
Explanation: Anemia is rare in adolescent males and
common in females. Nonetheless, hospitalization is usually
not warranted. (See Chapters 99 and 100 in Nelson
Textbook of Pediatrics, 17th ed.)

Gastrointestinal disorders

Injuries
Question . 9. Successful preventive measures to avoid morbidity and mortality
from adolescent automobile accidents include all of the following except:

Peer pressure TV commercials


Explanation: B-E all are correct.(See Chapters 99 and 100
in Nelson Textbook of Pediatrics, 17th ed.)

Graduated licensing systems

Enforcement of drinking age laws

Night-time driving restrictions

Drivers' education classes

Question . 10. The American Academy of Pediatrics recommends annual


health visits for adolescents for all of the following except:

Learning problems
Explanation: Learning problems were probably identified
earlier than in adolescence and are not part of the AAP
guidelines. They are part of the AMA's annual screening,
however. (See Chapter 100 in Nelson Textbook of
Pediatrics, 17th ed.)

Safety practices

Scoliosis

Breast examination

Eating disorders

Question . 11. High-risk characteristics of adolescent sexuality include all of


the following except:

Young adolescents

Late-maturing boys
Explanation: Late-maturing boys are at moderate risk.
Low-risk categories are older age, stable relationships, and
regular contraceptive use. (See Chapter 100 in Nelson
Textbook of Pediatrics, 17th ed.)

Drug use

Same-sex partner
Coercive sex

Question . 12. An emancipated minor is one who:

Is able to understand health issues

Lives with stepparents

Has graduated from high school

Is a member of the U.S. military


Explanation: Children who are less than 18 years old but
who are no longer subject to parental control (married,
military, economic self-sufficiency) are considered
emancipated. (See Chapter 100 in Nelson Textbook of
Pediatrics, 17th ed.)

Travels overseas

Question . 13. Papanicolaou smears are indicated in:

All young women older than 16 yr

A 14-yr-old mother with cervical cancer

A 12-yr-old girl exposed to diethylstilbestrol

All sexually active teenage girls


Explanation: Sexual activity is an absolute indication for a
Pap smear. Examination of two successive cervical
scrapings increases the yield and avoids false-negative
results. (See Chapter 100 in Nelson Textbook of Pediatrics,
17th ed.)

None of the above

Question . 14. Depression in adolescents is characterized by all of the


following except:

Two to three times higher incidence in females

Growth spurts
Explanation: Depression is probably under-reported in the
teenager. (See Chapter 101 in Nelson Textbook of
Pediatrics, 17th ed.)

Attention deficit disorder

Substance abuse
Death of a family member

Question . 15. A 17-yr-old girl manifests diminished interest in school, poor


exercise participation, weight loss, and hypersomnia. The differential diagnosis
includes all of the following except:

Hypothyroidism

Mononucleosis

Exercise-induced asthma
Explanation: The other choices are important
considerations in the differential diagnosis. (See Chapter
101 in Nelson Textbook of Pediatrics, 17th ed.)

Chronic fatigue syndrome

Depression

Substance abuse

Question . 16. On further questioning of the patient in Question 15, you


determine that she has inappropriate guilt, poor concentration, and low self-
esteem. The most likely diagnosis is:

Pseudotumor cerebri

Depression
Explanation: Depression requires a comprehensive
treatment plan. Untreated, it may last 7-9 mo; most patients
become depressed again in the next 7 years. (See Chapter
101 in Nelson Textbook of Pediatrics, 17th ed.)

Lead poisoning

Alcohol withdrawal

Schizophrenia
Question . 17. Risk factors for adolescent suicide include all of the following
except:

Native American race

Chronic illness

Access to guns

Gender
Explanation: Males are more likely to complete suicide,
whereas females are more likely to attempt suicide. (See
Chapter 102 in Nelson Textbook of Pediatrics, 17th ed.)

Gay and bisexual activity

Question . 18. Suicide ideation:


Is a significant risk factor for completed suicide in females

Is a significant risk factor for attempted suicide in young


teens (age 12-14 yr)

Is experienced by about 25% of teens


Explanation: Ideation is relatively common, but more
serious cofactors such as an adverse event (breaking up
with a loved one, family discord) and a specific plan raise
the level of concern.

Is less of a concern if there is no specific plan

Does not require psychiatric consultation

Question . 19. Diagnostic criteria for anorexia nervosa include all of the
following except:

Bradycardia and hypothermia


Explanation: These are consequences of weight loss;
others include postural hypotension, bone marrow
hypoplasia, constipation, and dry lanugo-type skin. (See
Chapter 104 in Nelson Textbook of Pediatrics, 17th ed.)

Fear of becoming obese

A feeling of being fat despite being emaciated

Refusal to maintain body weight


Absence of three consecutive menstrual cycles

Question . 20. Bulimia is characterized by the following except:

Isocaloric nutrient malnutrition


Explanation: B-E are diagnostic of bulimia. (See Chapter
104 in Nelson Textbook of Pediatrics, 17th ed.)

Self-induced vomiting and laxatives

Fear of not being able to stop eating during a binge

At least two binge episodes per week for at least 3 mo

Binges lasting less than 2 hr

Question . 21. A 17-yr-old white boy presents with an exacerbation of acne


and with aggressive behavior and poor school performance. Physical
examination reveals gynecomastia, testes that are smaller than expected for
his sexual maturity rating, and oily hair. The most likely diagnosis is:

Anabolic steroid abuse


Explanation: Athletes of both sexes use anabolic steroids
to enhance athletic performance. This student was on the
wrestling team and, like over 500,000 high school students,
had used anabolic steroids.

Adrenal hypertrophy

Marijuana use

17-Hydroxylase deficiency

Testicular feminization syndrome

Question . 22. A 16-yr-old female has had headaches for 3 mo and visual
changes for 2 wk and now has galactorrhea. Her last normal menstrual period
was 4 mo ago. The most likely cause of her galactorrhea is:

Stress of having amenorrhea

Elevated prolactin levels


Explanation: The patient had a prolactin-secreting pituitary
adenoma detected by a CT scan. Other potential causes of
galactorrhea include oral contraceptive pills, some
antihypertensive medications, and some tranquilizers.

Elevated estrogen levels

Migraines
Adrenal insufficiency

Question . 23. All of the following may cause gynecomastia except:

Klinefelter syndrome

Phenothiazines

Anabolic steroids

Heroin

Albuterol
Explanation: All the rest plus at least a dozen other drugs
may produce gynecomastia in males. (See Chapter 106 in
Nelson Textbook of Pediatrics, 17th ed.)

Question . 24. A 16-yr-old girl has not had her first menstrual period. Her
mother had her first period at the age of 12 yr. The adolescent is short and has
poor breast development. A vaginal smear shows no estrogen effect, and
there is no withdrawal bleeding after administration of intramuscular
progesterone.Her serum FSH level is high. The most likely diagnosis is:

Primary ovarian failure


Explanation: The patient has hypergonadotropic
hypogonadism-primary ovarian failure. Her phenotype is
compatible with Turner syndrome (45,X). (See Chapter 107
in Nelson Textbook of Pediatrics, 17th ed.)

FSH pituitary tumor

Hypothyroidism

Cervical stenosis

Autoimmune endocrinopathy
Question . 25. A 14-yr-old girl presents with vaginal bleeding that is more
prolonged and profuse than her usual periods, which are irregular and first
started 13 mo ago. She has pallor and tachycardia. The next step in her
evaluation is to:

Administer medroxyprogesterone (Provera)

Administer conjugated estrogens (Premarin)

Determine the hematocrit


Explanation: The hematocrit is 27% (MCV 65), and her
tachycardia responds to the administration of 1 liter of
normal saline (intravenous)

Determine the estrogen level

Determine the platelet count

Question . 26. The most likely diagnosis for the patient described in Question
25 is:

Ovarian tumor

Fibroids

Dysfunctional uterine bleeding


Explanation: Dysfunctional uterine bleeding occurs after
anovulatory cycles and is due to lack of estrogen build-up
of the endometrium. Some affected women also have von
Willebrand disease.

Hemophilia

Leukemia
Question . 27. The most appropriate therapy for the patient described in
Questions 25 and 26 is administration of:

Premarin
Explanation: Estrogens such as Premarin or as part of a
combination oral contraceptive pill constitute the treatment
of choice. The iron-deficiency anemia must be treated with
iron.

Prednisone

17-Hydroxyprogesterone

Ibuprofen

DDAVP

Question . 28. Serious complications of exogenous estrogens in oral


contraceptive pills include all of the following except:

Thrombophlebitis

Myocardial infarction

Hepatic adenoma

Diabetes mellitus

Uterine rupture
Explanation: A-D are correct but fortunately are quite rare
events and are even rarer in nonsmokers. (See Chapter
108 in Nelson Textbook of Pediatrics, 17th ed.)
Question . 29. A 19-yr-old female patient tells you she had unprotected sex
yesterday with her steady partner. She does not want to become pregnant.
Your emergency treatment should be to give:

Morgestrel (Ovral) 2 tablets once

Spermicide now and in 12 hr

Intravenous FSH and hCG

Ovral 2 tablets now and in 12 hr


Explanation: The goal is at least 200 g of ethinyl estradiol
and 2 mg of norgestrel given twice 12 hr apart. It is also
important to provide advice about proper nonemergency
contraception. (See Chapter 108 in Nelson Textbook of
Pediatrics, 17th ed.)

Lo-Ovral 2 tablets now and in 12 hr

Question . 30. The best contraceptive for a 16-yr-old girl with a history of two
pregnancies while on combination

estrogen-progestin oral contraceptives and a steady sexual


partner is:

An intrauterine device

An all-progestin oral contraceptive

Condoms

Levo-norgestrel (Norplant)
Explanation: Norplant is a reasonable choice, but many
adolescents have trouble with its side effects (weight gain,
irregular periods) and often ask to have it removed. (See
Chapter 108 in Nelson Textbook of Pediatrics, 17th ed.)

A cervical cap
Environmental Health Hazards
Nelson Self Assessments website 17th Edition

Question . 1. A 2-yr-old boy presents with a peeling, erythematous rash on the hands
and feet. The mother reports that he has become ill tempered and refuses to walk
about, preferring to lie in bed. Physical examination reveals an irritable, pale child
with photophobia. Temperature is 98.5°F, heart rate is 80/min, and respiratory rate is
23/min. Tremor of the tongue is evident. Further history and follow-up evaluation
reveal that the child's elder brothers have been playing with liquid mercury. Which of
the following is the most likely diagnosis?

A. Measles
B. Fifth disease
C. Kawasaki disease
D. Acrodynia
Explanation: Acrodynia, characterized by erythemia, peeling, and neurologic
signs, is characteristic of mercury toxicity. (See Chapter 702 in Nelson)
E. Photosensitivity

Question . 2. Laboratory confirmation of the diagnosis in Question 1 can best be


obtained by measuring the toxic compound's concentration in:

A. Urine
Explanation: Urine collection makes this a more useful test to determine
mercury poisoning and because the differential diagnosis may include other
heavy metals or drug toxicity. (See Chapter 702 in Nelson Textbook of
Pediatrics, 17th ed.)
B. Blood
C. Hair
D. Saliva
E. Feces

Question . 3. A 3-year-old child from a suburban community presents with vomiting,


diarrhea, and blurred vision. Physical examination reveals an afebrile child with
pinpoint pupils, salivation, and muscle fasciculations. The family's lawn was treated
yesterday for insects. Which of the following tests will establish the correct
diagnosis?

A. Blood lead level


B. 24-hr urine mercury level
C. Plasma cholinesterase level
Explanation: Insecticides are commonly acetylcholinesterase inhibitors,
producing cholinergic crises. Pinpoint pupils in patients with altered mental
status are not always due to narcotics. (See Chapter 701 in Nelson Textbook of
Pediatrics, 17th ed.)
D. Urine malathion level
E. Urine morphine level

Environmental Health Hazards - Nelson Self Assessments website 17th Edition 1


Question . 4. Deteriorating insulation is found in the ceilings of a local school built in
1958. Parents and teachers are extremely concerned that it may be asbestos, and they
call on you, the local pediatrician, to give advice in an open meeting. Which of the
following approaches to the problem do you recommend?

A. Immediately remove all of the insulation


B. Confirm the presence of asbestos by laboratory evaluation, and do nothing further
C. Confirm the presence of asbestos by laboratory evaluation; if asbestos is
confirmed, call a certified engineer for expert assessment
Explanation: It is important always to be calm but to make decisions based on
accurate information. Because of the risk of asbestos, an accurate assay is
needed before ways are identified to isolate and confine or remove the hazard.
(See Chapter 701 in Nelson Textbook of Pediatrics, 17th ed.)
D. Reassure the parents that the hazards of asbestos have been overstated
E. Obtain chest films from all of the students and teachers

Question . 5. Potential sources of mercury include all of the following Except:

A. Swordfish
B. Quicksilver
C. Milk
Explanation: Milk is an unusual source of mercury. (See Chapter 702 in )
D. Pesticides
E. Latex paint
F. Folk remedies

Question . 6. The most serious manifestation of lead intoxication is:

A. Peripheral neuropathy
B. Mental retardation
C. Anemia
D. Cerebral edema
Explanation: Cerebral edema is often noted with blood lead levels exceeding 100
µg/dL. (See Chapter 703 in Nelson Textbook of Pediatrics, 17th ed.)
E. Lead lines

Question . 7. A 2-yr-old boy is noted to be drinking from a container filled with


kerosene. He immediately coughs, becomes tachypneic, and is brought to the hospital.
The best approach to his treatment is to:

A. Induce emesis
B. Perform nasogastric tube lavage
C. Instill mineral oil
D. Administer steroids
E. None of the above
Explanation: Choices A, B, and C are dangerous and may induce pulmonary
aspiration if emesis occurs. Steroids are of no value and may predispose to
infection. Supportive therapy is indicated and includes oxygen and fluids. (See
Chapter 704 in Nelson Textbook of Pediatrics, 17th ed.)

Environmental Health Hazards - Nelson Self Assessments website 17th Edition 2


Question . 8. Which of the following diseases may be associated with a severe acute
reaction to radiotherapy?

A. Chédiak-Higashi syndrome
B. Neurofibromatosis
C. Chronic mucocutaneous candidiasis
D. Ataxia-telangiectasia
Explanation: A child with ataxia-telangiectasia (AT), because of the associated
hereditary DNA repair defect, is unable to repair acute radiation damage.
Conventional doses of radiotherapy to treat lymphomas (to which AT
predisposes) have caused acute radiation sickness, and children have died. It is
important to diagnose ataxia-telangiectasia in young patients with lymphoma
to avoid radiotherapy. The other choices listed in the question are not
associated with severe acute radiation reaction. (See Chapter 700 in Nelson
Textbook of Pediatrics, 17th ed.)
E. Wiskott-Aldrich syndrome

Question . 9. About 500 children have just been exposed to radiation from a nuclear
power plant accident. The most appropriate first step in management of these children
should be to:

A. Prescribe potassium iodide to protect the thyroid


Explanation: Potassium iodide protects the thyroid against radiation, and the
sooner it is given the better. At Three Mile Island, a serious problem was the
inability to obtain eyedroppers to administer the drug by mouth, and potassium
iodide administration was the first measure taken. Thyroid function tests are
performed much later. Results of the CBC will probably be normal. Signs of
acute radiation sickness will not appear for several days and probably will not
result at all from this fallout exposure. (See Chapter 700 in Nelson Textbook
of Pediatrics, 17th ed.)
B. Order thyroid function tests
C. Order complete blood counts (CBCs)
D. Do nothing acutely; evaluate in 4-6 wk
E. Look for signs or symptoms of acute radiation sickness

Question . 10. Antivenins should be considered in the treatment of all of the


following Except:

A. Rattlesnake envenomations
B. Scorpion envenomations
C. Black widow envenomations
D. Stonefish envenomations
E. Hymenoptera envenomations
Explanation: Antivenin is not necessary for Hymenoptera bites or stings, but
immune prophylaxis may be indicated for systemic reactions. (See Chapter
708 in Nelson Textbook of Pediatrics, 17th ed.)

Environmental Health Hazards - Nelson Self Assessments website 17th Edition 3


Question . 11. A 2-yr-old child is found playing with a can of crystalline drain
cleaner. The child's mother telephones you for help. There are several crystals in the
mouth, which you have the mother wash out. The next step in treatment should be to:

A. Have the mother administer lemon juice or orange juice to neutralize the
alkaline crystals and come to your office
B. Have the mother administer water or milk and call you back in 2 hr
C. Have the mother administer water or milk and bring the child in for
esophagoscopy
Explanation: Immediately dilute the ingested drain cleaner with water or milk.
The ability to predict who will experience esophageal disease is poor; thus,
most clinicians recommend esophagoscopy within 12-24 hr of such an
ingestion. Steroids are of little value in the presence of esophageal lesions and
do not prevent stricture formation. (See Chapter 704 in Nelson Pedia, 17th ed.)
D. Simply observe the child because the crystals are so bitter that the child was
trying to spit them out when the mother called, and therefore no problems
should occur
E. Administer ipecac at home and bring the child in to see you

Question . 12. A 16-yr-old, 165-lb patient reports consuming 20-40 325-mg capsules
containing acetaminophen 1 hr ago. The most appropriate approach to treatment is to:

A. Measure the plasma level and determine potential toxicity from the level on
the nomogram
B. Wait until 4 hr after ingestion to measure the plasma level and do nothing else
C. Administer activated charcoal immediately and measure the plasma level of
acetaminophen 4 hr after ingestion
Explanation: The patient described in the question has ingested a potentially
significant amount of acetaminophen. Charcoal may reduce absorption, and
the ultimate predictor of toxicity (the 4-hr serum level) will determine the need
for N-acetylcysteine therapy. (See Chapter 704 in Nelson Textbook of
Pediatrics, 17th ed.)
D. Send the patient home because an ingestion of this magnitude is not toxic
E. Administer N-acetylcysteine at a dose of 140 mg/kg

Question . 13. A 5-yr-old child presents with a painful blister of unknown origin with
surrounding erythema on the palm of the hand and with low white cell and platelet
counts. There is also a history of mild nausea and vomiting occurring about 4-6 days
ago. The most likely diagnosis is:

A. Pemphigus
B. Stevens-Johnson syndrome
C. Thermal burn
D. Insect bite
E. Radiation burn from handling a highly radioactive source
Explanation: (See Chapter 700 in Nelson Textbook of Pediatrics, 17th ed.)

Environmental Health Hazards - Nelson Self Assessments website 17th Edition 4


Question . 14. An increase in childhood cancer after radiation exposure has been
clearly demonstrated after:

A. Multiple CT scans
B. Living in homes with a high radon level
C. Living in an area of high natural background
D. Exposure to radioactive iodine
Explanation: (See Chapter 700 in Nelson Textbook of Pediatrics, 17th ed.)
E. Paternal preconception radiation exposure

Question . 15. Compared with adults, young children are uniquely vulnerable to
chemical pollutants for all of the following reasons Except:

A. They have more future years of life


B. Their developmental processes are sensitive and easily disrupted
C. They eat less food daily
Explanation: (See Chapter 701 in Nelson Textbook of Pediatrics, 17th ed.)
D. They often play near the ground and engage in hand to mouth behavior
E. They have a greater minute ventilation

Question . 16. A 13-mo-old healthy child presents for a follow-up visit to address a
capillary blood lead level of 18 ?g/dL obtained routinely at a recent well child visit at
age 1 yr. The most appropriate next step in the evaluation and management of this
patient is to:

A. Perform a bone lead analysis


B. Take a detailed environmental history and obtain a venous blood lead level
Explanation: (See Chapter 701 in Nelson Textbook of Pediatrics, 17th ed.)
C. Obtain an x-ray film of a long bone to look for "lead lines"
D. Begin chelation therapy
E. Reassure the parents that because their child has no symptoms, this level of
lead exposure is not harmful

Question . 17. Children can be exposed to environmental pollutants through a number


of routes. All of the following are potential exposure routes to consider in an
environmental history Except:

A. Benzene exposure in a child who drinks tap water from a private well
B. Fine-particulate exposure in an asthmatic child who lives in a home heated by
a wood-burning stove
C. Organophosphate exposure in a child whose mother applies pesticides for a
termite control company
D. Asbestos exposure in a child whose home has sealed asbestos insulation
covering pipes in the ceiling of an unfinished basement used for storage
Explanation: (See Chapter 701 in Nelson Textbook of Pediatrics, 17th ed.)
E. Mercury exposure in a pregnant woman who eats tuna fish twice a day as part
of a weight loss diet

Environmental Health Hazards - Nelson Self Assessments website 17th Edition 5


Question . 18. Sources of arsenic or mercury exposure include:

A. Pesticides
B. Seafood
C. Folk remedies
D. Occupational exposures
E. All of the above
Explanation: Exposure to arsenic or mercury may occur with all of the sources
listed. (See Chapter 702 )

Question . 19. All of the following statements regarding arsenic poisoning are true
Except:

A. Inhalation of arsine gas results in hemolysis


B. Acute ingestion of inorganic arsenic salts results in hemorrhagic
gastroenteritis, cardiovascular collapse, and encephalopathy
C. A patient with chronic arsenic poisoning may present with fatigue, peripheral
sensorimotor neuropathy, leukopenia, anemia, gastroenteritis, and alopecia
D. The organic forms of arsenic found in seafood are highly toxic to the central
nervous system
Explanation: The organic forms of arsenic found in seafood (primarily
arsenobetaine) are nontoxic but may confound the results of urinary assays for
arsenic. (See Chapter 701 in Nelson Textbook of Pediatrics,17th ed.)

Question . 20. All of the following statements regarding mercury absorption are true
Except:

A. Elemental mercury liquid is rapidly and completely absorbed from the


gastrointestinal (GI) tract
Explanation: Elemental mercury liquid is poorly absorbed from the GI tract,
with less than 0.1% being absorbed. (See Chapter 702 in Nelson Textbook of
Pediatrics, 17th ed.)
B. Inorganic mercury salts are partially absorbed from the GI tract
C. Methylmercury is the most avidly absorbed of the organic mercury
compounds, with about 90% absorbed from the GI tract

Question . 21. All of the following statements regarding mercury poisoning are true
Except:

A. Acute inhalation of elemental mercury vapor may result in fever, cough, chest
pain, dyspnea, and respiratory failure
B. Acute ingestion of inorganic mercury salts may result in corrosive
gastroenteritis, severe abdominal pain, acute tubular necrosis, and
cardiovascular collapse
C. Chronic inorganic mercury intoxication produces the classic triad of tremor,
neuropsychiatric disturbances, and gingivostomatitis
D. Acrodynia is an idiosyncratic hypersensitivity reaction to mercury
characterized by generalized pain, paresthesias, irritability, hypotonia, and an
acral rash

Environmental Health Hazards - Nelson Self Assessments website 17th Edition 6


E. The delayed neurotoxicity of methyl mercury intoxication is best treated by
chelation with dimercaprol, also known as British antilewisite (BAL)
Explanation: Dimercaprol is contraindicated for chelation of methylmercury
because BAL redistributes mercury to the brain from other tissue sites,
resulting in increased neurotoxicity. (See Chapter 701 in Nelson Textbook of
Pediatrics, 17th ed.)

Question . 22. A 2-yr-old child presents in the emergency department following the
reported ingestion of a mouthful of lamp oil. The child reportedly vomited once at
home. The child has a heart rate of 160 beats/min, a respiratory rate of 48/min, and a
temperature of 37.2oC. A chest film is read as normal. The most appropriate therapy
for this child is to:

A. Administer syrup of ipecac


B. Administer activated charcoal
C. Remove any ingested lamp oil by gastric lavage
D. Admit the child for observation and supportive care
Explanation: (See Chapter 704 in Nelson Textbook of Pediatrics, 17th ed.)
E. Discharge the child home with a follow-up office visit in the morning

Question . 23. A teen-age girl presents in the emergency room with the story that she
got upset with her boyfriend and swallowed a "handful of aspirin" about 4 hr
previously. One hour afterward, after she began vomiting, she confessed to her
mother what she had done. On examination the patient has normal vital signs and is
asymptomatic except for the complaint of nausea. A serum salicylate level is ordered,
but the laboratory reports no salicylates in her blood. The most appropriate next step
in management is to:

A. Discharge the patient home


B. Order an acetaminophen level
Explanation: (See Chapter 704 in Nelson Textbook of Pediatrics, 17th ed.)
C. Request a psychiatric consultation
D. Send a second sample for salicylate determination
E. Order an abdominal x-ray study to look for pills in the stomach

Question . 24. A 15-yr-old boy is admitted to your care following the intentional
ingestion of 2 g of his own amitriptyline in a suicide gesture. He received activated
charcoal for GI decontamination. The patient is placed in the ICU for monitoring and
remains stable. He is receiving only 0.45% normal saline. You observe a change in
his cardiac monitor display with a widening of his QRS complex to 0.12 second and
occasional ectopic beats. The most appropriate next step in management is to:

A. Ignore these changes because they are still within normal limits
B. Add sodium bicarbonate to his IV fluids to try to raise his serum pH above 7.4
Explanation: (See Chapter 704 in Nelson Textbook of Pediatrics, 17th ed.)
C. Repeat a dose of activated charcoal
D. Begin a lidocaine infusion at an appropriate dose
E. Order a chest film

Environmental Health Hazards - Nelson Self Assessments website 17th Edition 7


Question . 25. A 2-year-old boy arrives in the emergency department following a
seizure. On presentation his vital signs are as follows: temperature 40.2oC, heart rate
200 beats/min, respiratory rate 52 breaths/min. His laboratory values are follows:
arterial pH 7.2, serum bicarbonate 6 mmol/L, arterial PCO2 18 mm Hg, sodium 148
mmol/L, potassium 3.1 mmol/L, WBC count 10,200/mm3. On the basis of the history
obtained and the presentation, you suspect that an accidental ingestion has occurred.
The most likely toxin is:

A. A cyclic antidepressant
B. Acetaminophen
C. Cocaine
D. An organophosphate insecticide
E. A salicylate
Explanation: (See Chapter 704 in Nelson Textbook of Pediatrics, 17th ed.)

Question . 26. All of the following statements are true Except:

A. Most bites and stings by spiders, snakes, and other venomous animals do not
require attention
B. All antivenins are animal-derived immunoglobulins and cause only delayed
hypersensitivity reactions
Explanation: (See Chapter 708 in Nelson Textbook of Pediatrics, 17th ed.)
C. Delayed hypersensitivity or serum sickness develops in up to 65% of patients
who receive equine-derived antivenin
D. In the United States, 95% of poisonous snakes are pit vipers
E. Pit vipers may be identified by their triangular heads, elliptical eyes, and
identifiable pit between the eyes and nose

Question . 27. Which of the following statements is true?

A. In the United States the only significant morbidity from spiders is caused by
the black widow spider
B. The black widow spider is glossy black and has bright red or orange markings
on the ventral surface of the abdomen
Explanation: (See Chapter 708 in Nelson Textbook of Pediatrics, 17th ed.)
C. Antivenin should be used first to control the pain of a black widow bite
D. Most scorpion bites are life-threatening
E. Sea bather's eruption is caused by stingrays

Environmental Health Hazards - Nelson Self Assessments website 17th Edition 8


Question . 1. Major causes of neonatal mortality include all of the following except:

Sudden infant death syndrome


Explanation: Sudden infant death syndrome (SIDS) usually
does not occur in the first month of life. SIDS should not be
confused with apnea of prematurity, because one is not related
to the other. Apnea of prematurity occurs in the first month of
life but does not contribute to mortality. (See Chapter 82 in
Nelson Textbook of Pediatrics, 17th edition.)

Respiratory distress syndrome

Congenital malformations

Asphyxia

Sepsis
Question . 2. All of the following regarding the low-birthweight rate are true except:

Low birthweight contributes to neonatal mortality

The low birthweight mortality has decreased between 1981


and 2000
Explanation: Indeed the low-birthweight rate has increased
during this interval. This can be explained only in part by a
rising number of multiple births following reproductive
technology advances. (See Chapter 82 in Nelson Textbook of
Pediatrics, 17th edition.)

Low birthweight may be due to prematurity

Low birthweight may be due to intrauterine growth retardation

Low birthweight mortality is higher in black infants

Question . 3. All of the following are features of postneonatal mortality except:

Death between age 28 days and 1 yr

Caused in part by sudden infant death syndrome

Caused in part by bronchopulmonary dysplasia

Caused in part by premature rupture of membranes


Explanation: Premature rupture of membranes (PROM) is
associated with risks of early-onset sepsis and premature birth.
Both should contribute to neonatal, not postnatal, mortality.
th
(See Chapter 82 in Nelson Textbook of Pediatrics, 17 edition.)

Caused in part by trauma

Question . 4. Historical family features that suggest risk in a current pregnancy


include all of the following except:
A sibling with neonatal jaundice

Maternal history of urinary tract infection

Maternal age 19-35 yr


Explanation: These are the most ideal ages for safe maternal
reproduction. Adolescents and women over the age of 40 yr
are considered high risk. (See Chapter 83 in Nelson Textbook
th
of Pediatrics, 17 edition.)

Maternal diabetes mellitus

Poor prenatal care

Question . 5. A 30-min-old term 3.4-kg baby, born after a spontaneous vaginal


delivery, is noted to have acrocyanosis. The most important next step is:

Perform a sepsis evaluation

Perform a CBC followed by a sepsis work-up if neutropenia is


found

Keep the infant warm


Explanation: Acrocyanosis is normal at this time, and if the
physical examination is also normal there is no risk of a serious
underlying disorder. Warming is all that may be needed. (See
Chapter 83 in Nelson Textbook of Pediatrics, 17th edition.)

Begin oxygen administration

Hold oral feedings until the acrocyanosis resolves

Question . 6. An African-American male is born after a normal pregnancy and


delivery with rupture of the fetal membranes for 3 hr and no fetal monitoring.
Immediately after birth, he is noted to have multiple small pustules over his chin and
neck that are on a pigmented macular base. The most likely diagnosis is:

Erythema toxicum

Pustular melanosis
Explanation: This is the classic presentation of this benign
neonatal rash. The flaccid vesicles contain neutrophils, which is
in contrast to the eosinophils seen in erythemia toxicum. (See
Chapter 83 in Nelson Textbook of Pediatrics, 17th edition.)

Congenital herpes simplex, type II

Incontinentia pigmentosa

Herpes gestationalis

Question . 7. Deformational plagiocephaly is associated with all of the following


except:
Breech presentation
Explanation: Vertex positioning with possible pressure of the
developing skull by the mother's bony pelvis contributes to this
deformation. (See Chapter 83 in Nelson Textbook of Pediatrics,
17th edition.)

Torticollis

Asymmetrical skull

Ear malalignment

Asymmetric face

th
Question . 8. Bilateral and multiple retinal hemorrhages, presenting in the 6 week
of life in a lethargic neonate without any other physical findings, are most likely due
to:

Birth trauma

Nuchal cord

Vacuum extraction

Being large for gestational age

Child abuse
Explanation: Child abuse is often associated with bilateral
retinal hemorrhages, especially in what is called the "shaken
baby syndrome." Retinal hemorrhages may be noted
immediately after birth. Nonetheless, they should all resolve by
1 mo of age. (See Chapter 83 in Nelson Textbook of Pediatrics,
17th edition.)

Question . 9. Indications for cutting the oral frenulum include:

Short frenulum

Cleft palate

Cleft lip

Bifid uvula

None of the above


Explanation: There is little evidence for the clipping of a short
frenulum. (See Chapter 83 in Nelson Textbook of Pediatrics,
17th edition.)

Question . 10. A 3.5-kg female born following repeat cesarean section is noted by
the nurses to be grunting at 10 min of age. You come to see the baby and note that
the grunting has stopped, the respiratory rate is 36/min, the pulse oximetry reading
is 99%, and the child looks vigorous. The most appropriate next step is to:
Perform a sepsis evaluation

Obtain a chest film

Observe, and if grunting returns, admit to the neonatal


intensive care unit
Explanation: Grunting is common, particularly after a
cesarean section without prior labor. If grunting persists beyond
30 min or if there are other signs of distress, the child should
be evaluated for sepsis, respiratory distress syndrome (RDS),
or congenital heart disease. (See Chapter 83 in Nelson
th
Textbook of Pediatrics, 17 edition.)

Begin surfactant therapy

Begin nasal continuous positive airway pressure

Question . 11. The best description of the Apgar score is that it:

Accurately predicts who will develop cerebral palsy

Assesses neonates in need of resuscitation


Explanation: The Apgar score helps to rapidly assess the
need to resuscitate neonates after birth. Although it has some
value in predicting neonatal mortality and cerebral palsy, it has
a poor positive predictive value. Most children with cerebral
palsy have had normal Apgar scores, while neonates with low
Apgar scores do not universally get cerebral palsy. (See
Chapter 83 in Nelson Textbook of Pediatrics, 17th edition.)

Accurately predicts a low umbilical cord pH

Is unaffected by maternal opiate pain relief

Accurately predicts neonates who will die in the neonatal


period

Question . 12. Reproductive technologies are associated with all of the following
pregnancy-related risks except a :

Higher risk for multiple fetuses

Higher risk for group B streptococcal sepsis


Explanation: Group B streptococci are not increased in these
pregnancies. In addition to the risks of low birthweight (LBW)
and very low birthweight (VLBW), as well as multiple-fetus
pregnancies, there is an increased risk of cerebral palsy,
related in part to preterm birth. (See Chapter 84 in Nelson
Textbook of Pediatrics, 17th edition.)

Higher risk for low birthweight

Higher risk for very low birthweight


Higher risk for congenital anomalies

Question . 13. Spontaneous uterine contraction monitoring is most useful to:

Accurately identify risk for very low birthweight

Determine risk of premature rupture of the membranes

Make diagnosis of abruptio placentae

Identify a short cervix

None of the above


Explanation: Although monitoring for the rate of spontaneous
uterine contractions before the onset of labor is popular, it has
a poor sensitivity and a low positive predictive value for
premature birth. (See Chapter
Question . 14. Oligohydramnios is associated with all of the following except:

Esophageal atresia
Explanation: Esophageal atresia is most often associated with
polyhydramnios, as are other upper intestinal obstructive
lesions or disorders of fetal swallowing. (See Chapter 84 in
Nelson Textbook of Pediatrics, 17th edition.)

Pulmonary hypoplasia

Potter syndrome

Posterior urethral valves

Skeletal contractures
Question . 15. Fetal ultrasonography is of use for all of the following except:

Determining multiple fetuses

Identifying congenital heart disease

Determining the lecithin-sphingomyelin (L/S) ratio


Explanation: The lecithin-sphingomyelin ratio (L/S) predicts
pulmonary maturity and is a functional test that is based on
maturation of the fetal type II pulmonary alveolar cells. Fetal
ultrasonography may estimate fetal gestational age, and thus
the potential for lung maturity, but it cannot ascertain the actual
L/S ratio. (See Chapter 85 in Nelson Textbook of Pediatrics,
17th edition.)

Screening for trisomy 21

Predicting intrauterine growth retardation

Question . 16. Fetal tachycardia may be caused by all of the following except:
Maternal lupus erythematosus
Explanation: Maternal lupus may affect the fetal cardiac
conduction system, and it produces bradycardia by the
development of an antibody immune-related process. (See
Chapter 85 in Nelson Textbook of Pediatrics, 17th edition.)

Maternal fever

Prematurity

Fetal anemia

Fetal supraventricular tachycardia

Question . 17. Immunologic diseases of the mother that may affect the fetus include
all of the following except:

Systemic lupus erythematosus

Idiopathic thrombocytopenic purpura years after splenectomy

Myasthenia gravis

Graves disease

Multiple sclerosis
Explanation: It is interesting that B might be a choice.
However, splenectomy reduces the destruction of maternal
platelet antibody complexes by the spleen, thus increasing the
mother's platelet count. Splenectomy does not reduce the IgG
antiplatelet antibodies, which can readily cross the placenta.
(See Chapter 85 in Nelson Textbook of Pediatrics, 17th edition.)

Question . 18. An important influence on the effects of cigarette smoking on fetal


growth restriction is:

Brand of cigarettes

Altitude

Polymorphisms of hydrocarbon-metabolizing enzymes


Explanation: Polyhydric aromatic hydrocarbons may alter fetal
growth. It is very possible that enzymes that metabolize these
to either toxic or-more likely-nontoxic metabolites modify the
impact of the fetal growth restriction due to cigarette smoking.
(See Chapter 85 in Nelson Textbook of Pediatrics, 17th edition.)

Filtered versus non-filtered cigarettes

Educational level of the mother

Question . 19. Cordocentesis (percutaneous umbilical blood sampling) is useful for


all of the following except:
Measuring fetal hemoglobin

Determining fetal genome

Performing fetal cardiac catheterization


Explanation: This is future shock! Although one case of fetal
valvotomy has been reported, it was not performed via PUBS
but via fetal cardiac puncture. (See Chapter 85 in Nelson
th
Textbook of Pediatrics, 17 edition.)

Measuring antibody titers

Measuring fetal pH

Question . 20. Amnion nodosum is associated with all of the following except:

Oligohydramnios

Fetal Candida infection


Explanation: Fetal-placental amniotic Candida infection may
demonstrate nodules on the umbilical cord and a neonatal
macular-papular, erythematous, disseminated rash. (See
Chapter 86 in Nelson Textbook of Pediatrics, 17th edition.)

Renal agenesis

Pulmonary hypoplasia

Flattened facies

Question . 21. The incidence of twins detected at 40 wk of gestation is


characterized by all of the following except:

Family history of monozygotic twins


Explanation: Monozygotic twinning has little or no genetic
determination. Many twins detected at 12 wk of gestation are
singletons at 40 wk owing to fetal twin demise. This process
may affect the surviving twin (e.g., incidence of cerebral palsy
is higher in the surviving twin). (See Chapter 86 in Nelson
Textbook of Pediatrics, 17th edition.)

Fetal mortality of twins noted at 12-wk of gestation

Family history of dizygotic twins

Race

Infertility treatment

Question . 22. Very-low-birthweight infants are best described as:

Predominantly growth restricted


Predominantly premature
Explanation: The very low birthweight (VLBW) rate is an
excellent marker for prematurity (<37 wk of gestation). Some
VLBW infants may have growth restriction, but almost all
infants weighing <1500 g are premature. (See Chapter 86 in
Nelson Textbook of Pediatrics, 17th edition.)

Predominantly post dates

The result of maternal illness

The result of placental infarction

Question . 23. Excessive intravenous fluid intake in very-low-birthweight infants is


associated with:

High BUN level

Bronchopulmonary dysplasia
Explanation: Both bronchopulmonary dysplasia (BPD) and a
patent ductus arteriosus (PDA) are associated with excessive
intravenous fluids in premature infants. Answers A, C, D, or E
are some reasons why fluid intakes may be increased, but at
the same time, the infant is placed at risk for BPD or a PDA.
(See Chapter 86 in Nelson Textbook of Pediatrics, 17th edition.)

Hyperbilirubinemia

Hypernatremia

Oliguria
Question . 24. Breast milk from a donor may have insufficient amounts of all of the
following nutrients except:

Protein

Calcium

Phosphorus

Vitamin D

Essential fatty acids


Explanation: Essential fatty acid deficiency has never been
reported in a human milk-fed infant in the absence of a hepatic
cholestasis malabsorption syndrome. (See Chapter 86 in
Nelson Textbook
Question . 25. Hypotension in a newborn suggests all of the following except:

Fetal-maternal hemorrhage

Hypoplastic left heart syndrome


Adrenogenital syndrome

Tension pneumothorax

Jaundice
Explanation: Answers A to D, other congenital heart defects,
postnatal hemorrhage (CNS, liver, spleen), and sepsis can all
produce hypotension. (See Chapter 87 in Nelson Textbook of
Pediatrics
Question . 26. A 900-g infant with respiratory distress syndrome has persistent
hypotension despite administration of fluid boluses and dopamine. Epinephrine also
fails to elevate the blood pressure. The next step to manage the blood pressure is to
administer:

Albumin

Hydrocortisone
Explanation: Many studies have demonstrated the value of
administering hydrocortisone (Solu-Cortef) to VLBW infants
who respond poorly to fluid boluses and dopamine. (See
Chapter 87 in Nelson Textbook of Pediatrics, 17th edition.)

Hypertonic saline

Norepinephrine

Dobutamine

Question . 27. A 1-day-old infant born to a diabetic mother is noted to have tremors
of all extremities while awake. They disappear during sleep and are not associated
with eye movements. The glucose and calcium levels are normal. The most likely
diagnosis is:

Hypoglycemia

Kernicterus

Renal vein thrombosis

Caudal regression syndrome

Jitteriness
Explanation: This is a classic problem in infants of diabetic
mothers (IDMs). All jittery children should be examined
carefully and concern for a seizure considered. If the child
appears normal and has no abnormal eye movements, and if
the motion is sensory-dependent, is only present during active
wakefulness, and is stopped by simple pressure, it is probably
not a seizure. Nonetheless, it could be associated with a
serious problem such as hypocalcemia, hypoglycemia, or
opiate withdrawal. IDMs are often more jittery than other infants
but are also at risk for hypoglycemia and hypocalcemia. (See
Chapter 87 in Nelson Textbook of Pediatrics, 17th
Question . 28. A 5-day-old, large-for-gestational-age, 4,500-g boy has a bilirubin
level of 21 mg/dL. There is no anemia or polycythemia, but on examination he has a
large cephalohematoma. The next therapeutic activity should be to:

Aspirate the hematoma

Perform an incision and drainage of the hematoma

Undertake prophylactic blood transfer

Administer phototherapy
Explanation: Phototherapy is clearly indicated. Aspiration or
incision and drainage (I + D) should not be done to manage a
cephalohematoma. (See Chapter 88 in Nelson Textbook of
Pediatrics, 17th edition.)

Perform exchange transfusion

Question . 29. Routine head ultrasonography in infants <1,500 g to detect


intracranial hemorrhage is best described as:

Performed between 7 and 14 days and at 36-40 wk


Explanation: In addition, nonroutine ultrasonography should
be performed for symptoms of intraventricular hemorrhage
(IVH) and for the follow-up of abnormalities noted on the first
ultrasound. (See Chapter 88 in Nelson Textbook of Pediatrics,
17th edition.)

Performed for anemia

Performed for seizures

Performed at birth and at 40 wk

Performed between 7 and 14 days and at 1 yr of age

Question . 30. The management of post-hemorrhagic hydrocephaly includes all of


the following except:

Serial head circumferences

Serial head ultrasound examinations

External ventricular drainage

Ventricular-peritoneal shunt

Repeated lumbar punctures


Explanation: Although repeat lumbar puncture (LP) is often
done, most physicians do not believe that they avoid the need
for a ventricular peritoneal (VP) shunt. Most cases of dilated
ventricles after IVH do not necessitate later placement of a
shunt. (See Chapter 88 in Nelson Textbook of
Question . 31. A 12-day-old, large-for-gestational-age infant is noted to have Erb
palsy. You should do all of the following except:
Refer for immediate neuroplasty
Explanation: Most Erb palsies resolve rapidly with
immobilization, rehabilitation, and positioning. If there is no
improvement between 3-6 mo, a referral for surgical evaluation
is indicated. (See Chapter 88 in Nelson Textbook of Pediatrics,
17th edition.)

Refer for physical therapy

Reassure the family

Determine if the clavicle is fractured

Look for additional nerve involvement (phrenic)

Question . 32. A term female is born by spontaneous vaginal delivery to a


primiparous woman who received two doses of meperidine 30 min and 2 hr prior to
an abrupt delivery. The baby is apneic and limp. The most important, immediate
management is to:

Administer naloxone in the umbilical vein

Perform bag-mask ventilation


Explanation: Apnea from any cause is treated with securing a
patent airway and instituting ventilation. If bag mask ventilation
is ineffective, endotracheal intubation should be performed.
Naloxone (Narcan) can be given only after the baby is
ventilated. (See Chapter 89 in Nelson Textbook of Pediatrics,
17th edition.)

Administer naloxone in the endotracheal tube

Begin chest compressions

Obtain a cord pH

Question . 33. Successful ventilation is determined by all of the following except:

Zero reading of end-tidal CO2 measurement


Explanation: Indeed, one expects that with a successful
intubation and ventilation, the end-tidal (exhaled) CO2 will rise
dramatically. This is now used in many neonatal resuscitations.
Complete cardiac onset with poor perfusion may also cause a
low end-tidal CO2. (See Chapter 89 in Nelson Textbook of
Pediatrics, 17th edition.)

Pink color

Rising heart rate

Symmetric breath sounds

Good chest rise


Question . 34. A 3-wk-old, 32-wk-gestation, premature infant has sudden onset of
apnea and bradycardia that occurs repeatedly. On examination, the infant appears
to be asleep, and mild abdominal distention is noted. The approach to the patient's
management is best performed by:

Considering this a case of apnea of prematurity

Being concerned about a potential medical condition


Explanation: A previously healthy "gainer and groaner" should
not develop apnea of prematurity at 3 wk of age and at a
corrected age of 35 wk. Sudden onset and repeated episodes
strongly suggest an underlying illness. The abdominal
distention also suggests an ileus or necrotizing enterocolitis.
(See Chapter 90 in Nelson Textbook of Pediatrics, 17th edition.)

Obtaining a head ultrasound study

Doing an echocardiogram

Starting caffeine
Question . 35. A 32-wk gestational age infant develops grunting, flaring, and
retraction after birth. He requires 50% oxygen (O2) by hood to keep his oxygen
saturation above 95%. The next step in management, if he requires more O2, is to:

Institute inhalation of nitric oxide

Perform a blood transfusion

Begin nasal continuous positive airway pressure (CPAP)


Explanation: Nasal CPAP is often quite effective when larger
premature infants develop respiratory distress syndrome
(RDS). (See Chapter 90 in Nelson Textbook of Pediatrics, 17th
edition.)

Begin penicillin infusion

Begin dexamethasone infusion

Question . 36. In addition to administration of CPAP, immediate approaches to the


patient described in Question 35, would include all of the following except:

Head ultrasonography
Explanation: In a neonate with no signs of intraventricular
hemorrhage (IVH) and on the first day of life, one does not
need a head ultrasound. The others (B-E) are important to look
for other causes of respiratory distress and prepare to treat
empirically for the possibility of early-onset group B
streptococcal pneumonia. (See Chapter 90 in Nelson Textbook
of Pediatrics, 17th edition.)

Blood culture

Chest x-ray
Complete blood count

Blood gas analysis

Question . 37. A 3-day-old, 790-g female infant had been ventilated for respiratory
distress syndrome and was being weaned effectively from the ventilator. Today she
is noted to have an active precordium, bounding pulses, and hypoxia with
hypercarbia. All of the following should be done to evaluate and manage her except:

Administer prostaglandin E2
Explanation: In a neonate with no signs of intraventricular
hemorrhage (IVH) and on the first day of life, one does not
need a head ultrasound. The others (B-E) are important to look
for other causes of respiratory distress and prepare to treat
empirically for the possibility of early-onset group B
streptococcal pneumonia. (See Chapter 90 in Nelson Textbook
of Pediatrics, 17th edition.)

Obtain a chest film

Obtain an echocardiogram

Restrict fluids

Treat the hypoxia

Question . 38. The immediate treatment of choice for the patient described in
Question 37 is administration of:

Digoxin

Dobutamine

Dexamethasone

Indomethacin
Explanation: Indomethacin, and perhaps ibuprofen when
approved by the FDA, inhibits prostaglandin synthesis,
facilitating medical closure of a PDA. If medical treatment is
unsuccessful, surgical ligation is performed. (See Chapter 90 in
Nelson Textbook of Pediatrics, 17th edition.)

Vitamin E

Question . 39. The differential diagnosis of persistent pulmonary hypertension of


the newborn includes all of the following except:

Polycythemia

Total anomalous pulmonary venous return

Meconium aspiration pneumonia


Hypoglycemia

Renovascular hypertension
Explanation: Systemic hypertension may be a complication of
treating persistent pulmonary hypertension of the newborn
(PPHN) by ECMO or from umbilical artery catheterization. (See
Chapter
Question . 40. A term baby girl has two episodes of bile-stained emesis at 24 hr
after birth. There is a history of excessive amniotic fluid volume. The most
appropriate diagnostic test is:

Blood culture

Barium swallow and upper gastrointestinal tract x-ray


series
Explanation: All infants with bile stained emesis should be
evaluated radiologically. A plain film or KUB film is not sufficient
to detect all abnormalities but should be done before a barium
intestinal series. (See Chapter 91 in Nelson Textbook of
Pediatrics, 17th edition.)

Barium enema

CT scan

Head ultrasound study

Question . 41. Meconium plug is associated with all of the following except:

Hypermagnesemia

Infants of diabetic mothers

Cystic fibrosis

Hirschsprung disease

Prematurity
Explanation: Although prematurity may cause delayed
passage of meconium (NPO, immature colonic function), it is
not associated with a higher incidence of meconium plugs.
(See Chapter 91 in Nelson Textbook of Pediatrics, 17th edition.)

Question . 42. All of the following are true about necrotizing enterocolitis except:

Epidemics occur

Prematurity is the greatest risk factor

Human milk reduces the incidence

Surgeons need not be involved until there is evidence of


intestinal perforation
Explanation: Surgeons should be involved early to help in the
management and to discuss indications for surgery, including
evidence of perforation and failure to respond to medical
management. Placement of an abdominal drain with peritoneal
lavage may also help in the management of a sick VLBW
infant. (See Chapter 91 in Nelson Textbook of Pediatrics, 17th
edition.)

Pneumatosis intestinalis is an important diagnostic radiologic


sign

Question . 43. High risk factors for neonatal jaundice include all of the following
except:

Neonatal polycythemia

A sibling with jaundice

Poor enteral intake

Asian heritage

Post dates
Explanation: Postmature infants have a lower incidence of
jaundice unless polycythemia is present. Other risk factors for
jaundice include hemolysis, Gilbert disease, breast-feeding,
prematurity, diabetic mother, bruising, intestinal obstruction,
hypothyroidism, and diseases producing cholestatic disorders.
(See Chapter 91 in Nelson Textbook of Pediatrics, 17th edition.)

Question . 44. A 3-wk-old breast-fed infant has deepening jaundice. On physical


examination, the liver is 3 cm below the right costal margin. The most important
laboratory test in this child at this time is:

Serum ceruloplasmin determination

Direct and total bilirubin level


Explanation: Until this test is done, it is unknown whether the
infant has cholestatic or indirect hyperbilirubinemia. In this
child, the total bilirubin was 20 mg/dL and the direct was 10
mg/dL. She had biliary atresia. (See Chapter 91 in Nelson
Textbook of Pediatrics, 17th edition.)

Hepatic ultrasonography

Complete blood count

Urine urobilinogen determination

Question . 45. The hour-specific bilirubin assay (see accompanying figure) best
predicts:

Hemolysis

Exaggerated (indirect) hyperbilirubinemia


Explanation: This test helps determine who is at risk for
exaggerated indirect hyperbilirubinemia, which may predispose
to kernicterus if untreated. But the graphic risk designation
based on hour-specific bilirubin levels (Fig. XI-1) does not
predict, nor was it designed to predict, kernicterus.
Nonetheless, it is a very useful graph. (See Chapter 91 in
Nelson Textbook of Pediatrics, 17th edition.)

Kernicterus

Sepsis

Biliary atresia

Question . 46. All of the following are true about breast milk jaundice except:

It is associated with a risk of kernicterus

It peaks on the 3rd day of life


Explanation: Breast-feeding jaundice (poor intake,
dehydration) may be present this early, but breast milk jaundice
traditionally appears during the second week of life (and of
nursing). Kernicterus has been reported with breast-feeding
and bilirubin levels between 21 and 50 mg/dL. (See Chapter 91
in Nelson Textbook of Pediatrics, 17th edition.)

It resolves within 24 hr of temporarily stopping breast-feeding

It is most common in the second week of life

The cause is unknown

Question . 47. All of the following are true about tin protoporphyrin except:

It may have value for Jehovah's Witnesses

It may have value for patients with glucose-6-phosphate


dehydrogenase deficiency

It may avoid the use of phototherapy

It may cause hemolysis


Explanation: Although not extensively tested, this single dose
inhibition of heme oxygenase may be an important method to
prevent extreme jaundice in specific patient populations. (See
Chapter 91 in Nelson Textbook of Pediatrics, 17th edition.)

It may cause erythema with concurrent administration of


phototherapy

Question . 48. A term female infant is born with Apgar scores of 9 and 9. At 15 hr of
age, she is noted to be pale. The vital signs reveal tachycardia; there is no
hepatosplenomegaly or jaundice. The family history is not contributory, and the
review of the labor and delivery do not reveal any sources of blood loss. Her
hematocrit at 16 hr of age is 30%. The reticulocyte count is 15%, whereas the
platelet and WBC counts are normal, as is the blood smear. The bilirubin is 2 mg/dL.
The next important step in her evaluation is to do:

Red blood cell fragility test

Coombs test

Kleihauer-Betke test
Explanation: The Kleihauer-Betke test is performed on
maternal blood and tests for the presence of fetal hemoglobin
containing erythrocytes from a fetal-to-maternal transfusion. A
low bilirubin suggests that there is no hemolysis, and a normal
examination, except for tachycardia, suggests no internal blood
loss. Fetal-to-maternal bleeding can be chronic or acute. (See
Chapter 92 in Nelson Textbook of Pediatrics, 17th edition.)

Apt test

Serum ferritin determination

Question . 49. The role of recombinant human erythropoietin in the management of


very low birthweight is best defined as:

Providing prophylaxis in jaundiced infants

Providing prophylaxis in infants only <750 g

Preventing of bronchopulmonary dysplasia

Preventing of retinopathy of prematurity

None of the above?it has no routine role


Explanation: Recombinant human erythropoietin (rHuEpo) has
not consistently demonstrated benefit. Indeed, in older patients,
antibodies to erythropoietin may develop, which will bind to
endogenous erythropoietin, thus producing a severe anemia.
At this time, the risks exceed the benefits. There may be a role
in special circumstances, such as when a patient is a
Jehovah's Witness. (
Question . 50. Matching: ABO vs. Rh hemolytic disease.
A. ABO
B. Rh
C. Both
D. Neither

1. Kernicterus
Explanation: C. Both

2. Jaundice on day 1
Explanation: C. Both

3. Increasing severity with subsequent pregnancies


Explanation: B. Rh

4. Anemia
Explanation: C. Both

5. Intracranial hemorrhage
Explanation: D. Neither, unless associated with premature
death

6. In utero treatment
Explanation: B. Rh

7. Preventable with immunoglobulin


Explanation: B. Rh

8. Iron deficiency
Explanation: D. Neither, unless exchange transfusion

9. Hydrops fetalis
Explanation: B. Rh

10. Hepatosplenomegaly
Explanation: B. Rh, is seen occasionally with ABO
incompatibility (See chapter 92 in Nelson Textbook of
pediatrics, 17th edition.)

Question . 51. Polycythemia is associated with all of the following except:

Trisomy 21

Donor twin in a twin-twin transfusion


Explanation: The recipient twin develops polycythemia.
Additional risks for polycythemia include intrauterine growth
retardation, postmature infants, Beckwith-Wiedemann
syndrome, and being born at high altitude. (See Chapter 92 in
Nelson Textbook of Pediatrics, 17th edition.)

Neonatal Graves disease

Adrenogenital syndrome

Infant of diabetic mother

Question . 52. A term fetus is noted to have bilateral dilation of the renal pelvis and
ureter. Amniotic fluid volume is adequate, and the bladder size looks normal. After
birth, the child voids spontaneously within 24 hr. The most appropriate diagnostic
approach to this child is:

Renal ultrasound study at birth and at 1 mo


Explanation: Because fluid intake and urine flow may be
reduced in the first day or two after birth, an obstruction may be
missed if only one ultrasound is performed. Therefore, it is
recommended to repeat the ultrasound examination at 1 mo of
age. (See Chapter 93 in Nelson Textbook of Pediatrics, 17th
edition.)

Renal ultrasound study at birth


Renal ultrasound study at 1 mo

Intravenous pyelogram

None of the above?no diagnostic evaluation is needed


because he voided spontaneously within 1 day of birth

Question . 53. A 2-mo-old African-American term male has a seizure at home. In


the emergency department he seems awake but has tremors and feels rigid. He is
being breast-fed by a mother who is a strict vegetarian. Prior to this, his growth and
development were normal. He feeds often and well compared with her other children
who are 18 mo, 5 yr, and 10 yr old. The most likely diagnosis is:

Nesidioblastosis

Intracranial hemorrhage

Hypoglycemia

Pyridoxine deficiency

Hypocalcemia
Explanation: This infant has late neonatal onset tetany, which
is usually due to ingesting cow's milk with a very high
phosphate content. However, some infants breast-fed by a
mother at nutritional risk may develop hypocalcemia or, later in
infancy, rickets. The patient responded to intravenous calcium
and was sent home on supplemental vitamin D. Infants with
hypocalcemia often have only peripheral neuromuscular signs
and appear alert during what appears to be a seizure. (See
Chapter 95 in Nelson Textbook of Pediatrics, 17th edition.)

Question . 54. The best method to identify prenatal drug misuse is:

Careful maternal history

Meconium testing
Explanation: Meconium drug testing has become the standard
and reflects long-term drug misuse at a site of drug excretion
and storage. Urine test results may vary, depending on the
timing between drug intake and testing and the dose. (See
Chapter 95 in Nelson Textbook of Pediatrics, 17th edition.)

Testing of neonatal urine

Abstinence withdrawal score

Maternal urine testing

Question . 55. The best approach to prevent congenital anomalies in infants of


diabetic mothers is to:

Discontinue insulin and begin glyburide


Provide continuous insulin infusion during labor and delivery

Switch from an oral hypoglycemic agent to insulin until 36-wk of


gestation

Do serial glucose tolerance tests

Maintain periconceptional tight control of maternal blood


glucose levels
Explanation: Periconception control and control during the
early period of organogenesis help reduce the risk of
congenital malformations. Common anomalies affect the heart
and musculoskeletal system. (See Chapter 96 in Nelson
Textbook of Pediatrics, 17th edition.)

Question . 56. A 4.5-kg baby girl of a mother with classic diabetes develops
grunting after cesarean delivery. At 40 hr of age, the baby has tachycardia,
tachypnea, cyanosis, and a normal blood glucose level. A chest x-ray reveals
cardiomegaly, and you believe that the infant is in heart failure. A fetal
echocardiogram does not reveal any evidence of congenital heart anomalies. The
therapeutic agent of choice for this infant is:

Digoxin

Dobutamine

Propranolol
Explanation: This infant has cardiomyopathy associated with
infants of diabetic mothers. This is a form of ventricular septal
hypertrophy that can obstruct the left ventricular outflow tract
and cause heart failure and poor cardiac output. Inotropic
agents that increase the force of ventricular muscle contraction
exacerbate the obstruction and are contraindicated. (See
Chapter 96 in Nelson Textbook of Pediatrics, 17th edition.)

Epinephrine

Dopamine
Question . 57. A newborn boy is noted on physical examination to have a unilateral
cleft lip, bilateral postaxial polydactyly, microcephaly, intrauterine growth retardation,
and micropenis. You are unable to make a specific overall diagnosis. What is the
most appropriate initial diagnostic test for this patient?

Quantitative urinary organic acid measurement

Plasma amino acid determination

Karyotype
Explanation: This constellation of findings suggests a
chromosomal abnormality. Multiple malformations however
may be of unknown etiology. Metabolic disorders rarely cause
multiple malformations but may result in normal formation with
subsequent changes, such as coarsening of the facial features
or microcephaly or macrocephaly. (See Chapter 97 in Nelson
Textbook of Pediatrics, 17th edition.)
17-hydroxyprogesterone assay

Urinary mucopolysaccharide screen

Question . 58. A 2-day-old female infant is noted to have a bilateral cleft lip and cleft
palate. Based on a careful physical examination, you determine that she is
otherwise completely normal from a structural standpoint. Neither parent has a cleft
lip or cleft palate. Upon careful questioning of the mother, you determine that she
was not taking any medications during her pregnancy. Based on a careful family
history, you further determine that no other family member is affected. Which of the
following recurrence risk figures would you give this baby's parents?

25%

0%

50%

3-5%
Explanation: Recurrence risks of what is traditionally thought
to be a polygenetic disorder are between 3-5%. This is seen in
congenital heart disease and other single-organ malformations
without a positive family history. (See Chapter 97 in Nelson
Textbook of Pediatrics, 17th edition.)

75%

Question . 59. A 1-day-old male infant, born to an 18-yr-old primigravida female, is


noted to have multiple joint contractures and to be microcephalic. The pregnancy
was complicated by polyhydramnios. Which of the following is the most likely
explanation for these clinical findings?

A serious defect in CNS development


Explanation: Polyhydramnios suggests obstruction of the
gastrointestinal tract or disorders of swallowing due to poor
CNS function in utero. Contractions suggest poor motor activity
in the fetus or being fixed in one position (this is unlikely in the
presence of polyhydramnios). (See Chapter 97 in Nelson
Textbook of Pediatrics, 17th edition.)

Deformational defects associated with a primigravida mother

Uterine fibroids leading to deformational defects

A chromosomal disorder associated with deformational defects

Renal agenesis
Question . 60. The most effective way to prevent early onset of neonatal group B
streptococcal sepsis is:

Screen all pregnant women for group B streptococci


Explanation: Most studies suggest that screening for group B
Streptococcus in the mother and then selectively treating
affected women will reduce the risk of neonatal sepsis. (See
Chapter 98 in Nelson Textbook of Pediatrics, 17th edition.)

Employ higher risk criteria

Give intramuscular penicillin to all newborns

Institute tocolysis to prevent preterm birth

There is no effective prevention

Question . 61. A 30-wk-old average-for-gestational-age male infant is born after 24


hr of ruptured membranes and another 10 hr of labor. After birth, he develops
grunting, flaring, and restrictions, as well as cyanosis. He requires intubation,
ventilation, and surfactant therapy. Despite these measures he is still hypoxic and
hypotensive. You should now:

Do a blood culture and await results

Do a Gram stain of the tracheal fluid

Do a blood culture and begin intravenous antibiotics


Explanation: Premature infants with respiratory distress
syndrome (RDS) may look indistinguishable from infants with
early-onset sepsis. Waiting for culture results in this situation
would be dangerous, as the infant will die without therapy
before the culture is reported positive. A lumbar puncture is not
wise in this case, as the infant is hypotensive and hypoxic; the
blood culture is the only needed initial culture. If the blood
culture is positive, when the patient's condition becomes more
stable, a lumbar puncture can be performed. (See Chapter 98
th
in Nelson Textbook of Pediatrics, 17 edition.)

Begin intravenous antibiotics

Obtain blood and CSF for culture and begin intravenous


antibiotics

Question . 62. Matching: Malformations


A. Rubinstein-Taybi syndrome
B. Trisomy 21
C. Williams syndrome
D. Pierre Robin syndrome
E. Prader-Willi syndrome
F. VATER syndrome
G. None of the above

1. Deletion of elastin allele


Explanation: C. Williams syndrome

2. Microdeletion 16p13.3
Explanation: A. Rubinstein-Taybi syndrome

3. Deletion of band q11-12


Explanation: E. Prader-Willi syndrome
4. Endocardial Cushing effects
Explanation: B. Trisomy 21

5. Mandibular hypoplasia
Explanation: D. Pierre Robin syndrome

6. Tracheoesophageal fistula
Explanation: F. VATER syndrome

7. Anal atresia
Explanation: F. VATER syndrome

8. Radial hypoplasia
Explanation: F. VATER syndrome, (See Chapter 97 in Nelson
Textbook of pediatrics, 17th edition.)

Question . 63. An infant has the following findings at 5 min of life: pulse, 130/min;
cyanotic hands and feet; good muscle tone; and a strong cry and grimace. This
infant's Apgar score is:

9
Explanation: One point in the Apgar score is taken off for
color. (See Chapter 83 in Nelson Textbook of Pediatrics, 17th
edition.)

10
Question . 64. Which of the following is most appropriate for treating
hyperbilirubinemia (11.2 mg/dL) in a 3-wk-old, breast-fed infant with normal growth
and development?

Phototherapy

Exchange transfusion

Phenobarbital

None of the above


Explanation: No treatment is necessary for the infant
described in the question, assuming normal growth and
development. (See Chapter 91 in Nelson Textbook of
Pediatrics, 17th edition.)

Question . 65. A 2-wk-old infant is brought to the emergency room in coma with
retinal hemorrhages and severe pallor. He was born at home and was first seen by
a physician at 10 days of age and placed on amoxicillin for otitis media. His diet is
breast milk. The day before admission, his parents took him in a four-wheel-drive
vehicle on a hot day over a rough road in the mountains. Seizures began 8 hr later,
and his condition steadily deteriorated for the next 16 hr. He oozes blood from all
venipuncture sites. Diagnostic tests should include all of the following except:
Coagulation studies

Skeletal survey

CT scan

Complete blood count

Lumbar puncture
Explanation: (See Chapter 92.4 in Nelson Textbook of
Pediatrics, 17th edition.)

Question . 66. Immediate therapy for the infant described in Question 63 should
include administration of:

Vitamin A

Vitamin B6 (pyridoxine)

Vitamin C

Vitamin E

Vitamin K
Question . 67. The most likely diagnosis for the child described in Question 63 is:

Pyridoxine deficiency

Severe scurvy

Hemorrhagic disease of the newborn


Explanation: (See Chapter 92.4 in Nelson Textbook of
Pediatrics, 17th edition.)

Child abuse

Hypervitaminosis A

Question . 68. The death of the infant described in Question 63 could have been
prevented by which one of the following measures:
Administration of AquaMEPHYTON (vitamin K) at birth
Explanation: This infant has a case of severe vitamin K
deficiency-hemorrhagic disease of the newborn. The next most
likely diagnosis is child abuse; most infants in coma with retinal
hemorrhages have been shaken, and a skeletal survey thus is
appropriate. The combination of home delivery (no
AquaMEPHYTON administered), breast-feeding (low vitamin K
content), and the amoxicillin treatment that eliminated normal
intestinal bacterial synthesis of vitamin K led to the tragic
demise of an otherwise normal infant. Of all possible preventive
measures, administration of vitamin K at birth would have been
the most effective. (See Chapter 92.4 in Nelson Textbook of
Pediatrics, 17th edition.)

Home visitor services

Discontinuance of antibiotics

Proper use of an infant seat

An air conditioner

Question . 69. A 3,600-g, breast-fed, white girl with a gestational age of 42 wk is


noted to have persistent hyperbilirubinemia at 2 wk of age. On physical examination,
the infant is found to have not gained weight since birth; she has decreased tone, an
umbilical hernia, and an anterior fontanel measuring 4 x 6 cm. The most likely
diagnosis is:

Crigler-Najjar syndrome

Gilbert disease

Biliary atresia

Hypothyroidism
Explanation: Hypothyroidism was confirmed by the late arrival
of the newborn screening results, indicating high TSH and low
T4 levels. Treatment with thyroxine improved the jaundice and
the other signs. The hyperbilirubinemia is indirect
(unconjugated). Crigler-Najjar syndrome is a possibility and is
either autosomal dominant or recessive (check the family
history). However, there are signs other than jaundice that
suggest another disease. Biliary atresia is always a concern in
infants with delayed clearance of jaundice or worsening
jaundice after 2 wk of life. The hyperbilirubinemia is
predominantly direct (conjugated). Galactosemia should be
considered, especially in the presence of hypoglycemia, direct-
reacting jaundice, hepatomegaly, or ascites. (See Chapter 91.3
in Nelson Textbook of Pediatrics, 17th edition.)

Galactosemia

Question . 70. A 4-wk-old, A-positive, African-American infant (former 40-wk-


gestational-age) was born to an O-positive mother and experienced
hyperbilirubinemia, which required 2 days of phototherapy in the newborn nursery
after birth. The infant appears apathetic and demonstrates pallor, a grade 2/6
systolic ejection murmur, and a heart rate of 175 beats/min. The most likely
diagnosis is:

Anemia of chronic disease

Cholestasis secondary to neonatal hepatitis

Hereditary spherocytosis

Sickle cell anemia hemolytic crisis

ABO incompatibility with continued hemolysis


Explanation: Jaundice usually resolves in all infants with
hyperbilirubinemia due to ABO incompatibility in the first week
of life. Nonetheless, the hemolysis continues without evidence
of jaundice because the liver can now excrete the bilirubin load.
Late-onset anemia must be watched for and treated with a
packed red blood cell transfusion if the infant is symptomatic.
Hereditary spherocytosis is a possibility but is relatively rare. A
thorough family history and examination of the child's and
parents' blood smear are helpful (because most cases of
spherocytosis are inherited as an autosomal dominant trait).
Sickle cell anemia hemolytic crisis is not encountered this early
in life because a considerable amount of fetal hemoglobin
remains; thus, there are few -chains to sickle. (See Chapter
92.2 in Nelson Textbook of Pediatrics, 17th edition.)

Question . 71. Matching: Congenital infections


A. CMV
B. Varicella
C. Enteroviruses
D. Rubella
E. Parvovirus
F. Toxoplasmosis
G. Trypanosoma cruzi

1. Megaesophagus
Explanation: G. Trypanosoma cruzi

2. Patent ductus arteriosus


Explanation: D. Rubella

3. Cerebral periventricular calcifications


Explanation: A. CMV

4. Limb hypoplasia
Explanation: B. Varicella

5. Myocarditis
Explanation: C. Enteroviruses

6. Fetal anemia
Explanation: E. Parvovirus

7. Hydrocephalus
Explanation: F. Toxoplasmosis, (See Chapter 98 in Nelson
Textbook of Pediatrics, 17th edition)

Question . 72. Reasons to avoid the early discharge of a normal term infant include
all of the following except:

Jaundice evident on day 1

Positive VDRL

Bleeding after circumcision

Two successful feedings


Explanation: At least two successful feedings are reassuring,
whereas these other issues raise concern and need further
evaluation or therapy. (See Chapter 83 in Nelson Textbook of
Pediatrics, 17th edition.)

No stools
Question . 73. Matching:Maternal drugs and the fetus and newborn
A. Neonatal heart failure
B. Oligohydramnios
C. Spina bifida
D. Arthrogryposis
E. Facial-ear anomalies
F. Goiter

1. Accutane
Explanation: E. Facial-ear anomalies

2. Blue cohosh herbal tea


Explanation: A. Neonatal heart failure

3. Iodide
Explanation: F. Goiter

4. Ibuprofen
Explanation: B. Oligohydramnios

5. Valproate
Explanation: C. Spina bifida

6. Misoprostol
Explanation: D. Arthrogryposis, (See Chapter 85 in Nelson
Textbook of Pediatrics, 17th edition

Question . 74. A term baby of an uncomplicated pregnancy is born limp, cyanotic,


and apneic after a difficult vaginal delivery. Possible considerations in the differential
diagnosis include all of the following except:

Prolapsed umbilical cord

Central nervous system trauma


Administration of morphine to the mother

Klumpke paralysis
Explanation: Klumpke paralysis involves injury to the 7th and
8th cervical nerves and the 1st thoracic nerve. It is usually
unilateral, due to traction injury of the brachial plexus. (See
Chapter 88 in Nelson Textbook of Pediatrics, 17th edition.)

Administration of local anesthetic into the fetal scalp

Question . 75. After intubation and resuscitation, the patient in Question 70 remains
limp but appears aware and looks around, although the baby does not cry when the
toes are pinched. The most likely diagnosis is:

Congenital botulism

Narcotic overdose

Transection of the spinal cord


Explanation: Transection of the spinal cord may occur in
vertex and breech positions and may be noted with normal
vertebral body anatomy. It would manifest as in this patient,
and also with shock, hypothermia, and bowel and bladder
dysfunction. With time, hypotonia resolves into hypertonia and
hyperreflexia. (See Chapter 88 in Nelson Textbook of
Pediatrics, 17th edition.)46

Congenital myasthenia gravis

Neurosyphilis

Question . 76. A 2,100-g infant of a diabetic mother experiences seizures on the


first day of life. Pregnancy was complicated by severe diabetic vasculopathy and
placental insufficiency, with the development of late decelerations and the need for
emergency caesarean section. The Apgar scores were 8 and 9 at 1 and 5 minutes,
respectively. During the first 6 hr of life, the child was well and tolerated formula
feedings well. Jitteriness developed at 10 hr of age and progressed to tonic-clonic
seizures at 18 hr of age. Laboratory studies revealed blood glucose of 80 mg/dL and
calcium of 6.0 mg/dL. Thereafter, calcium gluconate (10%), 2 mL/kg, was given
repeatedly without affecting the frequency of seizures. The most appropriate step to
manage this infant's condition is to:

Administer glucose

Administer pyridoxine

Administer 1,35-dihydroxyvitamin D

Administer magnesium sulfate


Explanation: Infants of diabetic mothers (IDM) often
experience hypocalcemia within 24-48 hr after birth, due in part
to an attenuated parathyroid gland response to reduced
calcium levels. The IDM also develops hypomagnesemia,
which, if uncorrected, results in persistent hypocalcemia due to
the dependency of parathyroid function on magnesium. (See
Chapter 95 in Nelson Textbook of Pediatrics, 17th edition.)

Administer lorazepam
The Respiratory System Question
Nelson Self Assessments website 17th Edition

Question . 1. A 3-yr-old boy is undergoing mechanical ventilation 12 hr after repair of an


atrial septal defect. Opioids and benzodiazepines are being used for analgesia and
sedation. The mandatory ventilatory rate has been decreased from 20 to 10 breaths/min in
preparation for removal of the endotracheal tube. The arterial PO2 is 120 mm Hg and the
arterial PCO2 is 75 mm Hg. The arterial pH is 7.13. The child has no spontaneous
respirations. Auscultation of the chest demonstrates that the breath sounds are slightly
reduced on the left side. Occasional crackles can be heard over both bases.
The most likely reason for this child's acidosis is:
A. Pulmonary edema
B. Pneumonia
C. Cardiogenic shock
D. Respiratory depression
Explanation: As a result of sedation and analgesia, he is hypoventilating, which is
manifested by an acute respiratory acidosis and hypercarbia.Oxygenation may not
be affected if he is breathing enriched oxygen.The reduced breath sounds and
crackles could be due to atelectasis. (See Chapter 357 in Nelson, 17th ed.)
E. Pneumothorax

Question . 2. A 3-mo-old infant is brought to the emergency room because of lethargy


and poor feeding. She appears pale. Other findings include subcostal retractions and use
ofthe abdominal muscles during expiration. Her breathing rate is 30/min. Breath sounds
are decreased bilaterally. She has a prolonged expiratory phase. No wheezing or crackles
are audible.An abnormality in which of the following components of the respiratory
system is most likely to be involved in the genesis of these manifestations?
A. Alveolar surfactant
B. Lung interstitium
C. Intrathoracic airways
Explanation: Airway edema or inflammation will produce this constellation of
findings.With smaller airway involvement, this patient may eventually
demonstrate wheezing. (See Chapter 357 in Nelson Text. Pediatrics, 17th ed.)
D. Diaphragm
E. Medullary respiratory neurons

Question . 3. Which of the following functional findings is most likely in a 12-yr-old girl
who has developed acute rheumatic carditis with severe mitral insufficiency?
A. Increased peak expiratory flow
B. Increased vital capacity
C. Increased residual volume
D. Decreased functional residual capacity
Explanation: Cardiac failure produces interstitial and alveolar edema, which will
reduce the FRC. (See Chapter 357 in Nelson Textbook of Pediatrics, 17th ed.)
E. Increased CO diffusion capacity

The Respiratory System Nelson Self Assessments website 17th Edition 1


Question . 4. A 2-wk-old infant begins to experience episodes of acute respiratory
distress after undergoing repair of esophageal atresia. The episodes appear to be triggered
by crying. The infant becomes agitated and demonstrates decreased breath sounds
bilaterally, with the development of cyanosis and bradycardia. Physical examination
conducted when he is calm reveals mild subcostal retractions with a respiratory rate of 45
breaths/min, bilateral rhonchi, and a prolonged expiratory phase. Which of the following
is most likely to be the cause of the respiratory distress episodes?

A. Patent ductus arteriosus


B. Recurrent laryngeal nerve injury
C. Choanal atresia
D. Pulmonary hypertension
E. Tracheomalacia
Explanation: Tracheomalacia is quite common after repair of esophageal
atresia.Weakness of both the extrathoracic and intrathoracic trachea can produce
episodes of cyanosis and respiratory distress often triggered by crying, anxiety, or
pain.(See Chapter 357 in Nelson Textbook of Pediatrics, 17th ed.)

Question . 5. Intercostal retractions are caused by:

A. Direct traction applied by the diaphragm on the ribs


B. Contraction of the internal intercostal muscles
C. Contraction of the external intercostal muscles
D. Decreased pleural pressure
Explanation: When the compliance of the chest is greater than the negative
intrathoracic pressure generated during inspiration, retractions will develop.The
intercostal space is even more compliant than the chest wall. (See Chapter 357 in
Nelson Textbook of Pediatrics, 17th ed.)
E. Recruitment of the scalene and sternocleidomastoid muscles

Question . 6. A 1-mo-old infant is breathing supplemental oxygen from a hood at a


measured concentration of 45% after developing respiratory distress. A PO2 of 60 mm
Hg, a PCO2 of 50 mm Hg, and a pH of 7.30 are measured in a blood sample obtained
from the left radial artery. Which of the following interpretations is most consistent with
these findings?

A. The blood sample is venous


B. The infant has a right-to-left shunt via the ductus arteriosus
C. Oxygen diffusion across the alveolar-capillary membrane is impaired
D. The patient is hypoventilating
E. The blood gas anomalies are caused by ventilation-perfusion inequality
Explanation: The patient has both hypercarbia and hypoxia.Assuming the FIO2 in
the hood is 45%, one would expect a PaO2 much higher than 60 mmHg.The most
common cause of hypoxia in children with acute respiratory disorders is a
ventilation/perfusion mismatch. (See Chapter 357 in Nelson, 17th ed.)

The Respiratory System Nelson Self Assessments website 17th Edition 2


Question . 7. A premature infant is undergoing mechanical ventilation for respiratory
distress syndrome. Peak inspiratory pressure is 32 cm H2O, positive end-expiratory
pressure (PEEP) is 5 cm H2O, and ventilatory rate is 30 breaths/min. The infant has
decreased peripheral perfusion, manifested as a prolonged capillary refill time and weak
arterial pulses. The central venous pressure measured at the right atrium with an
umbilical venous catheter is 2 mm Hg (or approximately 3 cm H2O). Arterial PO2 is 80
mm Hg, and arterial PCO2 is 38 mm Hg.Which of the following measures is most likely
to result in an improvement in this infant's perfusion?

A. Reduce PEEP to 3 cm H2O


B. Reduce peak inspiratory pressure to 28 cm H2O
C. Reduce ventilatory rate to 26 breaths/min
D. Administer 10 mL/kg of normal saline
Explanation: The PaO2 and PCO2 are quite appropriate and in the target range for
appropriate therapy.Poor peripheral perfusion and weak pulses (and presumably
low blood pressure) in this setting should respond to expansion of the
intravascular volume with normal saline.The poor perfusion may have preceded
the initiation of PEEP, but may have also been exacerbated by the PEEP. (See
Chapter 357 in Nelson Textbook of Pediatrics, 17th ed.)
E. Begin an infusion of dopamine at 5 µg/kg/min

Question . 8. A 5-mo-old infant develops signs of respiratory distress after coughing and
sneezing for 3 days. He has marked subcostal and intercostal retractions and a respiratory
rate of 80 breaths/min. Breath sounds are markedly diminished on both sides. Diffuse
crackles can be heard bilaterally. There is no stridor. Arterial oxygen saturation in 100%
oxygen by non-rebreather mask is 80%. The skin is pale and peripheral arterial pulses are
weak.Which of the following is the most appropriate immediate course of action?

A. Administration of corticosteroids
B. Intubation of the trachea and mechanical ventilation
Explanation: This child is in respiratory failure.An arterial blood gas
determination may be helpful, but persistent hypoxia (80% saturation on pulse
oximetry) while the patient is on 100% FIO2 is an indication for intubation and
mechanical ventilation.CPAP may be tried under very controlled circumstances
but rarely avoids intubation.(See Chapter 357 in Nelson Textbook of Pediatrics,
17th ed.)
C. Sampling of arterial blood and measurement of arterial pH and blood gases
D. Administration of normal saline
E. Administration of diuretics

The Respiratory System Nelson Self Assessments website 17th Edition 3


Question . 9. A 16-yr-old boy who has been diagnosed with a yet uncharacterized form
of muscular dystrophy develops increased somnolence. He responds only to painful
stimuli. His respiratory rate is 40 breaths/min. His arterial oxygen saturation in room air
is 87%. After administration of supplemental oxygen, the arterial PO2 is 150 mm Hg, the
arterial PCO2 is 70 mm Hg, and the arterial pH is 7.30. Which of the following
statements defines this situation most accurately?

A. Decreased hypoxic drive after correction of the hypoxemia has resulted in acute
hypercapnia
B. Pulmonary hypertension caused by chronic hypoxemia has produced increased
ventilation-perfusion inequality
C. Renal tubular compensation of prolonged hypercapnia has resulted in an elevation
of serum bicarbonate levels
Explanation: Owing to poor ventilatory muscular effort, the patient has been
hypoventilating for a long enough time to allow renal tubular reabsorption of
bicarbonate to compensate for the prolonged hypercarbia (respiratory
acidosis).Compensation cannot totally correct the pH to normal.If the drive for
ventilation was inhibited by the hyperoxia, the PCO2 would be even higher and
the pH lower. (See Chapter 357 in Nelson Textbook of Pediatrics, 17th ed.)
D. The patient has become dehydrated
E. A fixed intrapulmonary right-to-left shunt is responsible for the limited response
to administration of oxygen

Question . 10. A 4-mo-old African-American infant was found unresponsive in his crib
by his mother in the early morning and could not be resuscitated. He had been placed for
sleep on his back but was found on his stomach. At a well-child examination the previous
day, he had been found to be in good health and received his routine immunizations. He
was born at 36 wk of gestation and weighed 2,420 g. His medical history was otherwise
unremarkable. After a thorough scene investigation, autopsy, and review of the medical
history, the cause of death was determined to be sudden infant death syndrome (SIDS).
Which of the following factors has not been found to be associated with greater risk of
SIDS?

A. Prematurity
B. Movement to a prone position after having been placed supine to sleep
C. Immunizations
Explanation: Multiple studies have looked at the potential associations between
immunizations and SIDS.None has ever demonstrated a relationship with SIDS.
(See Chapter 360 in Nelson Textbook of Pediatrics, 17th ed.)
D. African-American heritage
E. Low birth weight

The Respiratory System Nelson Self Assessments website 17th Edition 4


Question . 11. In the clinical scenario described in Question 10, which of the following
physiologic abnormalities is most likely to be related to the child's sudden, unexpected
death due to SIDS?

A. Increased susceptibility to bacterial infection


B. Prolonged Q-T interval
C. Medium-chain fatty acid metabolic abnormality
D. Arousal responsiveness from sleep
Explanation: Arousal responsiveness from sleep is thought to be the most
common mechanism for SIDS.This together with rebreathing in the prone
position may explain many cases. B has been associated with SIDS but is
uncommon. C has also been associated with SIDS but its incidence is
unknown.Both B and C should be suspected when more than one case of SIDS
occurs in a family or if there are non-healthy affected family members.(See
Chapter 360 in Nelson Textbook of Pediatrics, 17th ed.)
E. Brainstem autonomic control of heart rate and blood pressure

Question . 12. Of the following, the strongest risk factor associated with SIDS is:

A. Smoking by the mother in the prenatal period


Explanation: This is the epidemiologically correct answer.Although C is also
important, the prenatal exposure is more dominant. (See Chapter 360 in Nelson
Textbook of Pediatrics, 17th ed.)
B. Smoking by the father in the prenatal period
C. Exposure of the infant to environmental tobacco smoke after he or she is born
D. Smoking by the mother prenatally only in association with alcohol use
E. There is no association between smoking and SIDS

Question . 13. All of the following measures are recommended by the American
Academy of Pediatrics to reduce the risk of SIDS except:

A. Placing babies on their back to sleep


B. Avoiding waterbeds, sofas and other soft surfaces for sleep
C. Avoiding overheating during sleep
D. Using a pacifier if the infant is not breast feeding
Explanation: Pacifier use is interesting, as some believe that it reduces the risk of
SIDS.It is controversial and is not consistently agreed on as a protective factor
and is not recommended by the AAP.(See Chapter 360 in Nelson Textbook of
Pediatrics, 17th ed.)
E. Avoiding pillows in the infant's sleep environment

The Respiratory System Nelson Self Assessments website 17th Edition 5


Question . 14. All of the following statements are true except:

A. Most episodes of acute pharyngotonsillitis are viral


B. Rapid enlargement of one tonsil is typical of pharyngotonsillitis
Explanation: Enlargement of one tonsil, which occurs acutely, is typical of a
peritonsillar abscess and not routine pharyngotonsillitis.Peritonsillar abscesses
may obstruct the airway and are treated with intravenous antibiotics (penicillin is
OK) and incision and drainage or aspiration.(See Chapter 368 in Nelson, 17th ed.)
C. With cryptic tonsillitis, a frequent clinical presentation is halitosis, chronic sore
throat, or a history of expelling foul-tasting and foul-smelling cheesy lumps
D. In many children, the diagnosis of airway obstruction is made by history and
physical examination
E. Tonsillectomy alone is usually performed for recurrent or chronic
pharyngotonsillitis

Question . 15. Which of the following is not an indication for adenoidectomy alone?

A. Chronic nasal infection (chronic adenoiditis)


B. Chronic sinus infections that have failed medical management
C. Recurrent bouts of acute otitis media
D. Recurrent otorrhea in children with tympanostomy tubes
E. Recurrent pharyngotonsillitis
Explanation: In this situation tonsillectomy alone is effective treatment.(See
Chapter 368 in Nelson Textbook of Pediatrics, 17th ed.)

Question . 16. A 7-yr-old African-American boy is brought to your office by his parents,
who describe loud snoring, difficulty breathing, and obstructed breathing at night. His
teacher has complained that he seems inattentive and hyperactive, but his parents think he
is just a "high-energy" child. The father is obese and on CPAP for obstructive sleep
apnea, and his BMI is 20 kg/m2. Findings on physical examination are completely
unremarkable except for 2+ tonsillar hypertrophy and some mouth breathing.What is the
most appropriate next step in diagnosis?

A. Neuropsychological testing
B. Lateral soft tissue radiograph of the neck
C. CT study of the upper airway
D. Diagnostic testing for obstructive sleep apnea
Explanation: Although all of these are useful, at some point it is most wise to
perform dynamic testing for obstructive sleep apnea in a sleep laboratory.(See
Chapter 369 in Nelson Textbook of Pediatrics, 17th ed.)
E. ECG and echocardiogram

The Respiratory System Nelson Self Assessments website 17th Edition 6


Question . 17. A 10-yr-old African-American boy is referred to you by the school
psychologist for medical management of suspected ADHD because of inattention,
behavior problems, and poor school performance. He goes to bed at 10:30 P.M. and falls
asleep immediately. His brother will no longer share a room with him because of loud
snoring. It is difficult for his parents to wake him for school in the morning at 7:00 A.M.
He reports that he struggles to stay awake and pay attention during the day. He usually
naps for 1-2 hr after school, but not in school. On the weekends, he stays up until
midnight and sleeps until 9:00 A.M. He denies sudden losses of muscle tone, dreaming
during the day, or sleep paralysis. Findings on physical examination are remarkable for a
BMI of 30 kg/m2 and 3+ tonsillar hypertrophy. Which of the following is the most likely
diagnosis?

A. Insufficient sleep
B. Delayed sleep phase syndrome
C. Obstructive sleep apnea syndrome
Explanation: His snoring, BMI, and tonsillar hypertrophy strongly suggest
obstructive sleep apnea.It would be interesting if in addition to snoring the family
noted pauses between his noisy sleep-related breathing.(See Chapter 369 in
Nelson Textbook of Pediatrics, 17th ed.)
D. Narcolepsy
E. Idiopathic hypersomnia

Question . 18. You receive a follow-up note from your community cardiology colleague
that a mutual patient, a 7-yr-old boy with trisomy 21, has new findings of pulmonary
hypertension. The child had a VSD that spontaneously closed by age 2 yr. His room air
pulse oximetry value is 98%. The second heart sound is loud and the intensity of the P2
component is increased. ECG shows normal sinus rhythm and right ventricular
hypertrophy. Echocardiogram shows normal intracardiac anatomy with no evidence of a
PDA, but there is mild tricuspid regurgitation with an increased jet velocity across the
tricuspid valve. The heart size was normal on x-ray study. Your colleague is planning a
cardiac catheterization to assess the nature and severity of the pulmonary hypertension.
He also orders thyroid function studies.You see the child in your office for a pre-
catheterization general health assessment. The child, who is usually very cooperative, is
sleepy and irritable. You note prominent mouth breathing, 3+ tonsillar hypertrophy, and a
prominent pectus deformity.What is the most appropriate next step in management?

A. Check pre-catheterization hemoglobin level, hematocrit, electrolytes, and clotting


factors
B. Request diagnostic studies for obstructive sleep apnea
Explanation: Children with trisomy 21 have an increased risk for obstructive sleep
apnea.In this case, the pulmonary hypertension is not due to cardiac problems but
is most likely to be due to prolonged hypoxia and hypercarbia during sleep.(See
Chapter 362 in Nelson Textbook of Pediatrics, 17th ed.)
C. Order a lateral soft tissue radiograph of the neck
D. Order an MRI study of the upper airway
E. Start the child on steroids to shrink the enlarged tonsils

The Respiratory System Nelson Self Assessments website 17th Edition 7


Question . 19. Nosebleeds are commonly associated with all of the following except:

A. Digital trauma
B. Menstruation
Explanation: The five most common causes of epistaxis are on your hand (the
fingers!).Menstruation-related epistaxis is extremely uncommon. With severe
recurrent non-infection-related epistaxis, the child may have a coagulopathy such
as von Willebrand disease.(See Chapter 362 in Nelson Pediatrics, 17th ed.)
C. Family history of epistaxis
D. Sinus infections
E. Dry winter air

Question . 20. Nosebleeds in children most commonly arise from

A. Turbinates
B. Nasopharynx
C. Posterior septum
D. Kiesselbach's plexus (anterior septum)
Explanation: This is an easily reachable area that is easily irritated by picking or
inflammation.(See Chapter 362 in Nelson Textbook of Pediatrics, 17th ed.)
E. Maxillary sinus

Question . 21. When a disk battery is seen as a foreign body in the nose of a child, which
of the following is the most important consideration in management?

A. The patient should be referred electively to a specialist for removal


B. It may leak and cause local tissue damage
Explanation: These are particularly dangerous because of the risk of a chemical
burn or pressure necrosis in a small space and because attempts to remove them
could actually push them from the anterior space to the more distal posterior
space.Immediate removal is indicated.(See Chapter 362 in Nelson, 17th ed.)
C. The parents should remove it immediately
D. Nose drops should be given until it can be removed
E. Removal may be simply done in the office

Question . 22. Nasal polyps in children are:

A. Never found to arise in the ethmoid sinus


B. Common in infancy
C. Seen only in children with cystic fibrosis
D. Associated with allergic rhinitis
Explanation: Although cystic fibrosis is a common cause of nasal polyps,
especially in children younger than 12 yr, it is also seen with other conditions
such as allergies.(See Chapter 363 in Nelson Textbook of Pediatrics, 17th ed.)
E. Hard to distinguish from nasal turbinates

The Respiratory System Nelson Self Assessments website 17th Edition 8


Question . 23. The paranasal sinuses in children:

A. Are all present at birth


B. Develop during the teenage years
C. Grow and develop during the first seven years of life
Explanation: Some sinuses are present at birth, whereas others develop after
birth.Aeration as seen on x-ray may occur after sinus formation.(See Chapter 365
in Nelson Textbook of Pediatrics, 17th ed.)
D. Easily visualized on plain radiographs
E. Are unlikely to be infected before the age of 12 yr

Question . 24. An 18-mo-old girl has a 2-day history of rhinorrhea, pharyngitis, and low-
grade fever. During the night, she wakes with a barky cough, hoarseness, and inspiratory
stridor. Which of the following is the most likely etiologic agent?

A. Influenza virus type A


B. Respiratory syncytial virus
C. Parainfluenza virus
Explanation: This is the classic presentation of croup.Involvement of the vocal
cords (laryngitis in adults) is most often due to parainfluenza virus but may also
be due to any of these pathogens.(See Chapter 371 in Nelson Textbook of
Pediatrics, 17th ed.)
D. Adenovirus
E. Mycoplasma pneumoniae

Question . 25. A 2-yr-old boy is presented to the emergency department at 3 A.M. with a
chief complaint of fever and cough. His respiratory rate is 36/min, his temperature is
39oC, and his pulse oximetry reading is 96%. On physical examination he has a barky
cough and stridor only with crying. He is well hydrated, able to drink, and consolable.
What is the appropriate next step in patient management?

A. Nasal washing for influenza virus and respiratory syncytial virus


B. Lateral radiograph of the neck
C. Nebulized racemic epinephrine
D. Complete blood count and blood culture
E. Dose of dexamethasone
Explanation: In this patient with croup and manifesting stridor only with crying,
dexamethasone is indicated.If there were stridor at rest, racemic epinephrine and
dexamethasone would be indicated. (See Chapter 371 in Nelson Textbook of
Pediatrics, 17th ed.)

The Respiratory System Nelson Self Assessments website 17th Edition 9


Question . 26. You are the pediatric consultant for a community emergency department.
The department's physician calls to ask advice about a 3-yr-old boy with fever and a
cough. He thinks the patient has croup but is also concerned about epiglottitis. Which of
the following physical findings is most helpful in attempting to differentiate croup from
epiglottitis?
A. Fever
B. Barky cough
Explanation: In an unimmunized child, epiglottitis usually manifests with high
fever, toxicity, air hunger, and drooling but without a barking cough.Epiglottitis is
uncommon in children immunized against H. influenzae type b, and if it occurs, it
does so in unimmunized children or those with an unusual bacterial etiology. (See
Chapter 370 in Nelson Textbook of Pediatrics, 17th ed.)
C. Stridor
D. Drooling
E. Respiratory distress

Question . 27. You are a primary pediatrician in an office where a third-year medical
student is doing her clerkship. You have just seen the fifth patient that day with a classic
clinical presentation for croup: barky cough, fever, and stridor when agitated. You once
again prescribe dexamethasone. The third-year medical student asks about the data for
the use of steroids in croup. Which of the following has not been demonstrated in studies
of the use of steroids in croup?

A. Shorter hospitalization
B. Decreased need for subsequent medical interventions
C. Oral dexamethasone is as effective as intramuscular administration
D. Decreased need for oxygen
Explanation: Dexamethasone has been quite effective in the management of
children with mild to moderate croup.Its efficacy in reducing the need for oxygen
in more severely affected children has not been demonstrated.(See Chapter 371)
E. Reduced hospitalization

Question . 28. A 4-yr-old boy presents with sore throat and fever of sudden onset. He has
difficulty swallowing and his breathing is labored. He is drooling and sitting upright and
leaning forward in a tripod position. What is the appropriate next step in patient
management?
A. Complete blood count and blood culture followed by immediate prophylactic
intravenous antibiotics
B. Lateral radiograph of the neck
C. Dose of oral dexamethasone
D. Direct laryngoscopy in the operating room
Explanation: This is the classic presentation for epiglottitis. Although this
disorder is uncommon in the era of immunization against H. influenzae type b,
physicians must be aware of this dangerous disease with its requirement for
immediate airway protection.(See Chapter 371 in NelsonPediatrics, 17th ed.)
E. Complete physical examination including inspection of the oral cavity

The Respiratory System Nelson Self Assessments website 17th Edition 10


Question . 29. A 2-yr-old girl had an upper respiratory tract infection approximately 5-7
days ago. She had a low-grade fever, cough, and rhinorrhea. She did not require any
medical intervention. She appeared to be improving; now, however, she has a high fever
and brassy cough. On physical examination she is toxic-appearing with a fever of 39.8oC.
She can lie flat; she does not drool and has no dysphagia, but does have some evidence of
respiratory distress with increased work of breathing and retractions. On auscultation her
lungs are clear bilaterally. Which of the following is the most appropriate antibiotic for
this condition?

A. Erythromycin
B. Ciprofloxacin
C. Ampicillin
D. Gentamicin
E. Nafcillin
Explanation: This child has bacterial tracheitis as a complication of a previous
viral respiratory tract infection.The most likely bacterial organism is
Staphylococcus aureus, although other organisms may be responsible.Some
physicians might use ceftriaxone to cover these pathogens.(See Chapter 371 in
Nelson Textbook of Pediatrics, 17th ed.)

Question . 30. A 2½-yr-old girl has had symptoms of an upper respiratory infection for 1
week. Over the past 24 hours, fever and tachypnea developed, with worsening cough and
increased work of breathing. She has a temperature of 39.1o C, a respiratory rate of
40/min, and mild to moderate intercostal retractions. Oxygen saturation is 94-95%. Her
examination reveals diffuse wheezing, inspiratory rhonchi, and crackles in the right
anterolateral chest. Chest film shows a shaggy right heart border, generalized
hyperinflation, and peribronchial cuffing. White blood cell count is 18,000/mm3with
70% granulocytes.Of the following, which is the most appropriate next step in diagnosis?

A. Sputum culture
B. Lung puncture
C. Blood culture
Explanation: The child has bacterial pneumonia.A child this young usually does
not produce sputum or require a lung puncture.A blood culture is quite
appropriate.Viral PCR assay may be better than a viral culture. (See Chapter 379
in Nelson Textbook of Pediatrics, 17th ed.)
D. Viral culture
E. Cold agglutinins titer

The Respiratory System Nelson Self Assessments website 17th Edition 11


Question . 31. Outpatient management of the patient described in Question 30 is
considered in view of the patient's clinical status. Clinical features suggest a bacterial
pneumonia. Which of the following is the most appropriate treatment option?

A. Penicillin PO
B. Cefixime PO
C. Erythromycin PO
D. Cephalexin PO
E. Amoxicillin PO
Explanation: High-dose oral amoxicillin will be effective against most
pneumococci.With highly resistant pneumococci, treatment with intravenous
vancomycin is necessary.(See Chapter 379 in Nelson Pediatrics, 17th ed.)

Question . 32. A previously healthy 12-yr-old boy presents with upper respiratory
symptoms of 8 days' duration, with worsening cough and fever. The season is autumn.
Examination reveals a temperature of 39°C, a respiratory rate of 24/min, and inspiratory
crackles in both lung fields. Oxygen saturation is 93-94%. Chest film shows scattered
infiltrates in multiple lung fields, more focal consolidation in the right lower lobe, and
blunting of the right costophrenic angle. White blood cell count is 11,000/mm3 with a
normal differential.Which of the following is the most appropriate next step in diagnosis?

A. Sputum culture
B. Tuberculin skin testing
C. Throat culture
D. Cold agglutinins titer
Explanation: It is likely that this patient has Mycoplasma pneumonia.More
effective diagnostic tests include Mycoplasma PCR and IgM assays.(See Chapter
379 in Nelson Textbook of Pediatrics, 17th ed.)
E. Erythrocyte sedimentation rate

Question . 33. Which of the following is the most appropriate next step in the
management of the patient described in Question 32?

A. Erythromycin PO
Explanation: Erythromycin or azithromycin is quite effective in improving the
clinical course of Mycoplasma pneumonia. (See Chapter 379 in Nelson Textbook
of Pediatrics, 17th ed.)
B. Amoxicillin PO
C. No antibiotic therapy
D. Ceftriaxone IM
E. Cefuroxime IV

The Respiratory System Nelson Self Assessments website 17th Edition 12


Question . 34. A 4-yr-old boy has had rhinorrhea and cough for 4 days, with fever,
worsening cough, and chest discomfort over the past 2 days. His temperature is 40.1oC,
respiratory rate is 28/min, and oxygen saturation is 88%. Examination reveals splinting,
decreased breath sounds, and dullness to percussion over the right posterior chest, with
crackles heard over the right upper posterior chest. His white blood cell count is 30,000/mm3
with a predominance of granulocytes. Chest film shows opacification of the right hemithorax.
A right lateral decubitus film of the chest reveals significant pleural fluid.Which of the
following is the most appropriate next step in diagnosis and management?

A. Tube thoracostomy drainage


Explanation: Thoracentesis is of value both as a diagnostic aid and as a
therapeutic procedure.This patient obviously had a symptomatic effusion and
improved dramatically after withdrawal of 700 mL of cloudy fluid.(See Chapter
379 in Nelson Textbook of Pediatrics, 17th ed.)
B. Bronchoscopy
C. Sputum culture
D. Nasopharyngeal swabs for viral antigens
E. Cold agglutinins titer

Question . 35. Gram stain of the pleural fluid from the patient described in Question 34
reveals gram-positive cocci in clusters.Which of the following is the most appropriate
treatment?

A. Ampicillin IV
B. Cefuroxime IV
C. Erythromycin IV
D. Erythromycin and ampicillin IV
E. Cefotaxime and vancomycin IV
Explanation: The child probably has pneumococcal pneumonia.It could be
pneumonia due to S. aureus, but there are no pneumatoceles.The pneumococcus is
becoming resistant to penicillins and even to cephalosporins.If it is a life-
threatening illness, vancomycin should be added. (See Chapter 379 in Nelson)

Question . 36. A 7-yr-old child with a 3-yr history of cough, intermittent wheezing, and
poor growth has two sweat chloride values of 36 and 41 mEq/liter. Additional diagnostic
testing to rule out cystic fibrosis should include:

A. CT imaging of the chest


B. Nasal potential difference measurement
Explanation: This is a useful test that has abnormal results in CF. Today's DNA
testing for the many (in the hundreds) of mutations in the CFTR gene is available
and is of great value in equivocal test results.(See Chapter 402 in)
C. Fat balance measurement (72-hr stool collection)
D. DNA analysis for the F508 mutation
E. Sweat chloride analysis in siblings

The Respiratory System Nelson Self Assessments website 17th Edition 13


Question . 37. A 12-yr-old child with confirmed cystic fibrosis has experienced
cramping abdominal pain intermittently for the past 8 days. The pain is diffuse and
unrelated to eating and is not attended by guarding or rebound. The patient denies emesis
or diarrhea. The most likely cause of the abdominal pain is:

A. Intussusception
B. Peritonitis
C. Pancreatitis
D. Distal intestinal obstruction syndrome (DIOS)
Explanation: DIOS, also called meconium ileus equivalent, is an obstruction due
to impacted stool.If the patient has been receiving high-dose pancreatic enzymes,
a fibrosing colonopathy must also be considered. (See Chapter 402 in Nelson)
E. Cholecystitis

Question . 38. A newborn infant fails to pass meconium for the first 48 hr. Abdominal
distention and emesis have occurred overnight. The next diagnostic steps would include:

A. Sweat chloride assay


B. Genotyping the child for CF
C. Contrast imaging of the lower gastrointestinal tract
Explanation: The examination will look for a meconium plug or a small left colon
(which suggests proximal intestinal obstruction as in atresias).In both examples,
CF must be considered.If Hirschsprung disease is considered, a suction biopsy
should also be performed.(See Chapter 402 in Nelson Pediatrics, 17th ed.)
D. Manometry
E. Serum immunoreactive trypsin assay

Question . 39. A 3.5-kg male infant born at term after an uncomplicated pregnancy and
delivery develops respiratory distress shortly after birth and requires mechanical
ventilation. The chest radiograph reveals a normal cardiothymic silhouette but a diffuse
ground-glass appearance to the lung fields. Surfactant replacement fails to improve gas
exchange. Over the first week of life, the hypoxemia worsens. Results of routine cultures
and echocardiographic findings are negative. A term female sibling died at 1 mo of age
with "respiratory distress."Which of the following is the most likely diagnosis?

A. Total anomalous pulmonary venous return


B. Meconium aspiration
C. Neonatal pulmonary alveolar proteinosis
Explanation: RDS in a term infant not responding to surfactant replacement
therapy is most likely to represent neonatal pulmonary alveolar proteinosis.(See
Chapter 389 in Nelson Textbook of Pediatrics, 17th ed.)
D. Disseminated herpes simplex infection
E. Medium-chain acyl-dehydrogenase deficiency

The Respiratory System Nelson Self Assessments website 17th Edition 14


Question . 40. Which of the following laboratory evaluations should be obtained in the
case in Question 39?

A. Blood and tracheal cultures for virus and yeast


B. Repeat echocardiogram
C. RFLP analysis of the SP-B gene
Explanation: Neonatal alveolar proteinosis is due to a genetic mutation causing a
deficiency of surfactant protein B.(See Chapter 389 in Nelson Pediatrics, 17th ed.)
D. RFLP analysis of the MCAD gene
E. Urine organic acid screen

Question . 41. If the laboratory analysis confirms SP-B deficiency, what is the most
appropriate next step in management of the patient described in Question 39?

A. Obtain a lung biopsy


B. Begin inhaled nitric oxide
C. Discuss lung transplantation with the family
Explanation: ECMO is a temporizing bridge to lung transplantation, which
potentially can cure this disorder. (See Chapter 389 in Nelson Pediatrics, 17th ed.)
D. Administer surfactant
E. Begin corticosteroids

Question . 42. A 3.2-kg full-term female infant is delivered by vaginal delivery. She is
initially cyanotic and is in significant respiratory distress. Auscultation of the chest
reveals diminished breath sounds in the left hemithorax and a scaphoid abdomen. After
bag and mask ventilation, an endotracheal tube is placed. The point of maximal impulse
(PMI) is shifted to the right side of the chest.The most important initial intervention is:

A. Immediate bronchoscopy
B. Placement of a nasogastric tube
Explanation: This patient potentially has a diaphragmatic hernia and needs gas to
be removed or prevented from entering the bowel, which acts as a space-
occupying lesion in the chest.(See Chapter 370 in Nelson Textbook of Pediatrics,
17th ed.)
C. A chest radiograph to assess placement of endotracheal tube
D. Immediate surgery
E. Administration of epinephrine

The Respiratory System Nelson Self Assessments website 17th Edition 15


Question . 43. A 14-yr-old boy with a pectus excavatum deformity presents for
evaluation. He denies any exercise intolerance or cough but does experience intermittent
wheezing on exertion. He states he is not concerned about how his chest appears.
Physical examination reveals a mild pectus deformity. Which of the following
abnormalities on diagnostic work-up suggests the need for surgical correction?

A. FEV1/FVC ratio of 0.60 on spirometry


B. A Wolff-Parkinson-White pattern on ECG
C. Low ventilatory reserves during a maximal exercise test
D. A total lung capacity of 80% of predicted
E. A peak work capacity of 60% of predicted
Explanation: Many children with a pectus excavatum do not need surgery
according to results of work capacity studies.(See Chapter 410 in Nelson 17th ed.)

Question . 44. A 15-yr-old boy with thoracic scoliosis undergoes evaluation for surgery.
History is unremarkable, and findings on physical examination are normal except for a
mild thoracic scoliosis. A Cobb angle of 25 degrees is noted on the chest radiograph. His
vital capacity is 80% of predicted, and his exercise tolerance is minimally reduced.What
is the most appropriate next step in treatment of this patient?

A. Repeat assessment in 6 mo
Explanation: Depending on where he is in his puberty growth spurt, the curve
may not change, or if he continues to grow, the curve may worsen.(See Chapter
410 in Nelson Textbook of Pediatrics, 17th ed.)
B. Reassurance that surgery will not be required
C. Spinal fusion surgery
D. Bone density assessment (DEA scan)
E. Physical therapy to correct scoliosis

Question . 45. A 7½-yr-old girl presents with a history of low-grade fever, nonproductive
cough, and mild dyspnea. After treatment with an oral antibiotic, the child began to show
some signs of improvement. However, the child subsequently experienced increasing
dyspnea, a productive cough, and wheezing. A chest radiograph demonstrates
hyperlucency. Spirometry shows a severe obstructive pattern. The most likely diagnosis is:

A. Pulmonary alveolar microlithiasis


B. Wilson-Mikity syndrome
C. Follicular bronchitis
D. Bronchiolitis obliterans
Explanation: Bronchiolitis obliterans may follow a viral bronchitis or
pneumonia.Measles virus and adenovirus may be potential agents.(See Chapter
378 in Nelson Textbook of Pediatrics, 17th ed.)
E. Postviral syndrome

The Respiratory System Nelson Self Assessments website 17th Edition 16


Question . 46. A 3-yr-old boy has been coughing daily for 2 mo. The cough is
nonproductive and occurs during sleep in the early morning hours as well as during the
day, particularly when the child is active. On physical examination both height and
weight are in the 50-75th percentile, and chest examination is unremarkable. There is no
evidence of digital clubbing. A chest roentgenogram is interpreted as normal. The
diagnostic procedure most likely to ascertain the cause is:

A. Sputum cytology and culture


B. Sweat chloride testing
C. Bronchoscopy
D. Complete blood count
E. Trial of bronchodilator therapy
Explanation: This is a common pattern seen in some children with
asthma.Bronchitis in children is often an incorrect diagnosis, as many children are
later found to have asthma.(See Chapter 376 in Nelson Pediatrics, 17th ed.)

Question . 47. A previously healthy 2-yr-old girl is given oral antibiotic therapy for a
cough, fever, and patchy consolidation of the right lower lobe. No crackles are heard on
chest auscultation. The fever abates, the cough improves, but a follow-up chest film at 8
wk demonstrates even more dense consolidation involving the right lower lobe. Next
steps in the evaluation should include:

A. Bronchoscopy
Explanation: This child could have many problems such as a foreign body, but the
clinical picture is highly suggestive of a sequestration.If a pulmonary
sequestration is found, Doppler flow studies of the artery supplying the
sequestration will show the artery coming from the aorta.(See Chapter 370 in
Nelson Textbook of Pediatrics, 17th ed.)
B. Bacterial culture of the nasopharynx
C. Barium esophagram
D. Allergy skin testing
E. Lung biopsy

Question . 48. For the mechanically ventilated child, which medical condition is most
often associated with successful wean off all ventilatory support?

A. Central hypoventilation
B. Bronchopulmonary dysplasia
Explanation: Children with BPD usually are successfully weaned.The others in
choices A and C often become dependent on their ventilator.(See Chapter 411 in
Nelson Textbook of Pediatrics, 17th ed.)
C. Spinal muscular atrophy
D. None of the above

The Respiratory System Nelson Self Assessments website 17th Edition 17


Question . 49. All of the following may present a barrier to home discharge on a
ventilator except:

A. Lack of appropriate housing


B. Lack of committed caregivers
C. Lack of private insurance
Explanation: Communication and being able to provide constant care are keys to
home ventilator management.Insurance status could be private or public.(See
Chapter 411 in Nelson Textbook of Pediatrics, 17th ed.)
D. Lack of telephone

Question . 50. A 2-yr-old boy undergoing outpatient surgery for tonsillectomy vomits
while recovering from general anesthesia in the postoperative recovery area. Within 1-2
min he develops tachypnea, chest retractions, and hypoxemia. Which of the following is
the most likely explanation for these findings?

A. Acute blood loss from postoperative bleeding


B. Anesthetic reaction
C. Bacterial infection and toxin release
D. Acute airway obstruction from aspirated material
Explanation: Acutely after an aspiration there is a mechanical obstruction with
irritation, which can cause bronchospasm.Later a chemical pneumonia and
possibly bacterial infection may supervene.(See Chapter 380 in Nelson Textbook
of Pediatrics, 17th ed.)
E. Atelectasis

Question . 51. Of the following diagnostic tests, the most sensitive for detecting recurrent
airway aspiration is:

A. Upper gastrointestinal series


B. Modified barium swallow with video fluoroscopy
Explanation: This dynamic study will often demonstrate aspiration during
swallowing but may not show aspiration from emesis or reflux.(See Chapter 380
in Nelson Textbook of Pediatrics, 17th ed.)
C. A gastroesophageal radionuclide scintiscan
D. Chest CT scan
E. Bronchoscopy

The Respiratory System Nelson Self Assessments website 17th Edition 18


Question . 52. A 4-wk-old healthy-appearing term infant is evaluated in the office for
stridor, which has persisted since birth. The noisy breathing is accompanied by moderate
signs of inspiratory obstruction including suprasternal and subcostal retractions. He feeds
adequately and is gaining weight but frequently spits up. The most likely cause of his
symptoms is:

A. Tracheomalacia
B. Vascular ring
C. Laryngomalacia
Explanation: Laryngomalacia is common and often produces noisy breathing that
worsens with viral upper respiratory tract infections or in the supine position.(See
Chapter 370 in Nelson Textbook of Pediatrics, 17th ed.)
D. Tonsil and adenoid hypertrophy
E. Subglottic hemangioma

Question . 53. A 3-mo-old infant has had progressively worsening biphasic stridor. Her
parents report two brief episodes of croup. A 1-cm-diameter hemangioma is present on
her right thigh. Airway radiographs are most likely to show:

A. Asymmetric subglottic narrowing


Explanation: Airway hemangiomas may produce stridor and crouplike symptoms
with viral upper respiratory tract infections.Facial hemangiomas distributed in a
"beard pattern" carry the highest risk for an associated airway
hemangioma.Airway lesions may also occur in the absence of any cutaneous
lesions.(See Chapter 375 in Nelson Textbook of Pediatrics, 17th ed.)
B. Gastroesophageal reflux
C. Enlarged adenoids
D. Tracheomalacia
E. Laryngeal cyst

Question . 54. The most common bronchial foreign body is:

A. Raw carrot fragments


B. Popcorn
C. Nut fragments, particularly peanuts
Explanation: Anything small enough can get into the bronchus."Small" is relative
to the size of the bronchus and is thus age dependent.Nuts, sunflower seeds, and
the like should not be given to small children.(See Chapter 372 in Nelson
Textbook of Pediatrics, 17th ed.)
D. Nails
E. Coins

The Respiratory System Nelson Self Assessments website 17th Edition 19


Question . 55. A 6-mo-old boy presents with biphasic stridor and a recent episode of
croup. He has had minimal response to bronchodilator therapy. His past history reveals
that he was a premature infant who was intubated and ventilated for 6 wk. The most
likely cause of his respiratory distress is:

A. Acquired subglottic stenosis


Explanation: Subglottic stenosis may be congenital or acquired. Direct
laryngoscopy will confirm the diagnosis.(See Chapter 373 in Nelson Textbook of
Pediatrics, 17th ed.)
B. Vascular ring
C. Viral laryngotracheobronchitis
D. Reactive airways disease/chronic lung disease
E. Reflux laryngitis

Question . 56. A 4-yr-old boy with a history of tracheoesophageal fistula (TEF) repair at
birth is evaluated for a chronic cough. The cough has persisted since he was discharged
from the hospital after his TEF repair. The cough is dry and barking and occasionally
associated with expiratory wheezing. The most likely cause of the chronic cough is:

A. Cough-variant asthma
B. Sinusitis
C. Tracheomalacia
Explanation: Tracheomalacia is very common after a TEF repair.Some patients
also develop reactive airways and reflux.(See Chapter 370 in Nelson Textbook of
Pediatrics, 17th ed.)
D. Gastroesophageal reflux
E. Subglottic stenosis

Question . 57. A 4-yr-old boy with two older sisters is evaluated for symptoms of
chronic hoarseness and strained voice, which has been present for several months. The
hoarseness is worse in the evening and lessens in the morning. He has no symptoms of
airway obstruction. The most likely cause of his symptoms is:

A. Laryngopharyngeal reflux with reflux laryngitis


B. Recurrent respiratory papillomatosis
C. A malignant laryngeal neoplasm
D. A congenital laryngeal cyst
E. Vocal nodules (screamer's nodes)
Explanation: Chronic hoarseness or deepness of the voice that is exacerbated with
talking, singing, or crying is common.Treatment is symptomatic; no surgery is
needed.(See Chapter 375 in Nelson Textbook of Pediatrics, 17th ed.)

The Respiratory System Nelson Self Assessments website 17th Edition 20


Question . 58. A 23-mo-old male infant with mild eczema is presented for evaluation of
recurrent wheezing episodes. His mother reports that he developed recurrent wheezing
with colds following acute RSV infection at age 6 mo for which he was hospitalized and
received oxygen by nasal cannula and nebulizer treatments. This was his only
hospitalization, but he has been seen on several occasions in the emergency department
of their local hospital. She adds that there are no pets in the home and that his father does
smoke, but always outside. She herself has moderate asthma and mild seasonal allergies
and would like to know if this means her son will have asthma also. You inform the
child's mother that her son may be at risk for persistent wheezing.Which of the following
is not a risk factor for the persistent wheezing phenotype in this infant?

A. Passive smoke exposure


B. Maternal history of asthma
C. Maternal history of allergies
Explanation: Drug or food allergies in themselves are not a risk factor for
asthma.(See Chapter 376 in Nelson Textbook of Pediatrics, 17th ed.)
D. Mild eczema
E. Recurrent episodes of wheezing

Question . 59. An 11-mo-old infant is admitted to the hospital in December because of


failure to thrive and recurrent pneumonia. Within 24 hr of his hospitalization, he develops
progressive clear nasal drainage, mild respiratory distress with a respiratory rate of
48/min, intercostal retractions, and diffuse wheezing throughout the lung fields. His
oxygen saturation is 88% on room air. His parents state that his 4-yr-old sibling has had
cold symptoms for about 4 days.What is the most appropriate next step in the acute
treatment of this patient?

A. Trial of nebulized albuterol


B. Oxygen supplementation via nasal cannula
Explanation: Any previously well patient who becomes hypoxic and is in
respiratory distress must be given warmed, humidified oxygen regardless of the
diagnosis.(See Chapter 359 in Nelson Textbook of Pediatrics, 17th ed.)
C. Nasopharyngeal wash for RSV antigen and viral culture
D. A sweat chloride test
E. A 2-mg/kg load of oral corticosteroids, then 1 mg/kg twice daily for 5 days

Question . 60. An 8-yr-old boy presents with a 24-hr history of fever, chills, cough,
dyspnea, and malaise. Examination reveals an ill-appearing child who is mildly
tachypneic with bibasilar crackles. There are no ill contacts; however, he did help his
grandmother clean her pigeon coop the day before presentation.The diagnosis of
hypersensitivity pneumonitis would be based primarily on:

A. Chest radiograph and complete blood count


B. Pulmonary function tests
C. A high index of suspicion based on the clinical presentation in association with a
suspected exposure

The Respiratory System Nelson Self Assessments website 17th Edition 21


Explanation: The history and clinical course are the key, especially if the patient
gets better every time he or she is removed from the antigen (e.g., on weekends
for factory workers).(See Chapter 383 in Nelson Textbook of Pediatrics, 17th ed.)
D. Flexible fiberoptic bronchoscopy with bronchoalveolar lavage
E. Skin testing and serum precipitins to the suspected antigen

Question . 61. A 17-yr-old boy previously well presents with a history of cough and
dyspnea 24 hr after working in a corn silo. Examination reveals a normal-appearing
young man who is mildly tachypneic with normal findings on lung auscultation. There
are no ill contacts. Silo filler's disease is typically a result of:

A. Carbon monoxide poisoning


B. Hydrocarbon toxicity
C. Ammonia toxicity
D. Nitrogen dioxide toxicity
Explanation: Inhaled chemical or gaseous agents produce airway
inflammation.Silo filler's disease results from fermentation and gas production in
a closed space.(See Chapter 383 in Nelson Textbook of Pediatrics, 17th ed.)
E. Cyanide toxicity

Question . 62. A 16-yr-old boy attempted suicide by ingesting the herbicide


paraquat.Which of the following is the most ominous prognostic indicator?

A. Ingestion of dilute solution


B. Ingestion of 40 mg of paraquat per kg of body weight
Explanation: The dose is the most important.Smoking this agent when it is used as
a herbicide is also toxic.(See Chapter 384 in Nelson Text Pediatrics, 17th ed.)
C. Complaint of vomiting and diarrhea
D. Oxygen saturation of 91% on room air
E. Restrictive pattern on pulmonary function tests

Question . 63. A 6-yr-old girl presents with a 1-mo history of mild cough, intermittent
low-grade fever, intermittent wheezing, and an episode of hemoptysis. Pulse rate is
100/min; respiratory rate is 25/min; lung ausculation reveals diffuse end-expiratory
wheeze. Chest film shows nonspecific bilateral diffuse infiltrates. There is marked
peripheral eosinophilia on complete blood count.The most likely diagnosis is:

A. Pulmonary infiltrates with eosinophilia (L?ffler syndrome)


Explanation: This disorder has many potential causes, or it may be a primary
disease.Eosinophilia is a major clue!(See Chapter 384 in Nelson Textbook of
Pediatrics, 17th ed.)
B. Asthma
C. Cystic fibrosis
D. Mycoplasma pneumonia
E. Pulmonary lymphoma

The Respiratory System Nelson Self Assessments website 17th Edition 22


Question . 64. A 4-yr-old previously well girl presents with fever (temperature of
103°F), nonproductive cough, dyspnea, and left-sided chest pain. Initial examination
reveals an ill-appearing child; pulse rate is 125/min, respiratory rate is 40/min, and room
air oxygen saturation is 89%. Auscultation demonstrates decreased air movement on the
left side, with crackles. Chest radiograph shows a left lower lobe consolidation.CBC
reveals marked leukocytosis.The child was hospitalized and provided with supplemental
oxygen and started on high-dose IV cefuroxime.Two days later the child is still febrile
with a temperature of 102oF; respiratory rate is 35/min and room air oxygen saturation is
90%. Findings on lung examination and chest film are unchanged from admission.Which
of the following is the most appropriate next step?

A. Change antibiotics to IV ampicillin/sulbactam


B. Obtain chest CT scan
C. Add oral macrolide antibiotic
D. Send blood for an immune work-up
E. Begin IV vancomycin
Explanation: This child probably has a pneumococcal infection with resistant
organisms that has not responded to cefuroxime.(See Chapter 379 in )

Question . 65. A 15-yr-old boy presents with a history of chronic cough productive of
yellow-green sputum, dyspnea on exercise, digital clubbing, and poor weight gain. He
has a history of recurrent sinopulmonary infections. The patient is a thin-appearing young
man; his pulse rate is 95/min and respiratory rate is 24/min; auscultation of the lungs
demonstrates diffuse fine crackles and end-expiratory wheezing.The gold standard
technique for demonstrating bronchiectasis is:

A. Bronchoscopy with bronchoalveolar lavage


B. Ventilation-perfusion scan
C. Thin-section high-resolution CT scan of the chest
Explanation: CT scanning of the chest is the diagnostic test of choice to
demonstrate bronchiectasis.(See Chapter 396 in Nelson Pediatrics, 17th ed.)
D. Bronchography
E. Chest MRI study

Question . 66. A 6-yr-old girl presents 72 hr after undergoing a


tonsillectomy/adenoidectomy with fever, cough, dyspnea, and right-sided chest pain.
Examination reveals an ill-appearing child; her temperature is 104°F, pulse rate 110/min,
and respiratory rate 40/min; her lungs have decreased air movement on the right, with
dullness to percussion. Complete blood count demonstrates marked leukocytosis with left
shift, and appearance on chest radiograph is consistent with a pulmonary abscess.Which
of the following is the most appropriate initial management?

A. Admit for IV antibiotics providing aerobic and anaerobic coverage


Explanation: Most lung abscesses are the result of aspiration and contain
anaerobic bacteria.They can usually be managed with high-dose antibiotics, with
no need for drainage.(See Chapter 397 in Nelson Textbook of Pediatrics, 17th ed.)

The Respiratory System Nelson Self Assessments website 17th Edition 23


B. Arrange for percutaneous drainage in interventional radiology department
C. Prescribe oral macrolide antibiotic and follow-up evaluation within 24 hr
D. Arrange for thoracotomy and decortication
E. Arrange for bronchoscopic drainage of the abscess

Question . 67. A 3-yr-old child does well for the first 6 hr following surgery. Then
dyspnea and tachycardia develop, with rapid shallow respirations. On physical
examination the patient has decreased breath sounds and coarse rales on the right.Which
of the following is the most appropriate first step in treatment?

A. Chest tube placement


B. Intravenous antibiotics
C. Positive pressure ventilation
D. Fiberoptic bronchoscopy
E. Cough, deep breathing, and percussion
Explanation: This patient has atelectases and needs assistance to achieve good
inspiratory efforts, etc.Careful use of pain medications to control chest or
abdominal pain and to avoid oversedation is also valuable.(See Chapter 392 in
Nelson Textbook of Pediatrics, 17th ed.)

Question . 68. An 18-yr-old female patient presents with chest pain, tachypnea, and
cyanosis of sudden onset 1 wk after the birth of her first child. Her chest radiograph is
nondiagnostic, but her PaO2 is 60 mm Hg on 40% oxygen. The most likely diagnosis is:

A. Preeclampsia
B. Legionella pneumonia
C. A fractured rib
D. A pulmonary embolism
Explanation: Pulmonary embolism must be considered with the sudden onset of
chest pain, dyspnea, and cyanosis. A normal-appearing chest radiograph with
significant hypoxia is classic for pulmonary embolism. A spiral CT study is a
useful test to determine the presence of a pulmonary embolism. (See Chapter 395)
E. Hysterical hyperventilation

Question . 69. All of the following are gastrointestinal manifestations of cystic fibrosis
except:
A. Intussusception
B. Appendicitis
C. Colonic mucosal thickening
D. Gastric outlet obstruction
Explanation: Other causes of intestinal obstruction are possible and include
neonatal meconium ileus, congenital ileal atresia, neonatal mucus plus syndrome,
meconium ileus equivalent (from insufficient use of pancreatic enzyme
replacement), intussusception, inguinal hernia, and appendiceal obstruction. (See
Chapter 402 in Nelson Textbook of Pediatrics, 17th ed.)
E. Inguinal hernias

The Respiratory System Nelson Self Assessments website 17th Edition 24


Question . 70. All of the following statements regarding cystic fibrosis are true except:

A. Incidence of 1 case per 3,500 black and 1 case per 17,000 white infants
Explanation: The incidence of cystic fibrosis is highest in northern European
whites (1 in 3500) and lowest in Asian infants in Hawaii (1 in 90,000). (See
Chapter 402 in Nelson Textbook of Pediatrics, 17th ed.)
B. Autosomal recessive inheritance
C. More than 700 gene mutations
D. F508 as the dominant mutation
E. WI282X mutation in 60% of Ashkenazi Jews
F. Severity of lung disease is not predictable by the gene mutation

Question . 71. The most common manifestation of preliminary involvement in children


with cystic fibrosis is:

A. Cyanosis
B. Clubbing
C. Cough
Explanation: The cough begins as dry and nonproductive but progresses to loose
with production of purulent sputum. (See Chapter 402 in Nelson Textbook of
Pediatrics, 17th ed.)
D. Wheezing
E. Nasal polyps

The Respiratory System Nelson Self Assessments website 17th Edition 25


The Cardiovascular System
Question .1. A 1-mo-old female infant is presented with a chief complaint of poor
feeding and lethargy. Parents report that the child was well until 3 days earlier, when
poor feeding began. Pulse rate is 280 beats/min, respiratory rate is 50/min, and blood
pressure is 80/50 mm Hg. Physical examination shows a gallop rhythm and an
enlarged liver palpable 2-3 cm below the right costal margin. The most appropriate
diagnostic test for this patient would be:

A. Chest radiograph
B. CBC
C. ECG
Explanation: This is a very high heart rate for a simple sinus rhythm. The child
is in heart failure, but the excessively high heart rate may be the cause, not the
result, of heart failure. (See Chapter 428 in Nelson Pediatrics, 17th ed.)
D. ABG analysis
E. Blood culture

Question . 2. What would be the most likely diagnosis in the case in Question 1?

A. Reentrant SVT
Explanation: This is a classic presentation of supraventricular tachycardia in
infancy. It is usually idiopathic and transient but does require rapid diagnosis
and treatment. It may also be present in utero. Always be sure the parents are
not giving the child over-the-counter cold remedies that contain
sympathomimetic agents, which may include "herbal" remedies that could
contain ephedra. (See Chapter 428 in Nelson Textbook of Pediatrics, 17th ed.)
B. Ventricular tachycardia
C. Sepsis
D. Anemia
E. Pneumonia

Question . 3. What would be the most appropriate next step in treatment for the case
in Question 1?

A. Electrical defibrillation
B. IV verapamil
C. Adenosine therapy
Explanation: Intravenous rapid push adenosine is the drug treatment of choice
for all cases of SVT, in infants and in patients of all ages. Verapamil was once
used in older patients, but in infants in heart failure, verapamil has serious
adverse effects (cardiac arrest) owing to its negative inotropic effects on an
already failing myocardium. (See Chapter 428 in Nelson Textbook of
Pediatrics, 17th ed.)
D. Antibiotic therapy
Blood transfusion

The CVS System Nelson Self Assessments website 17th Edition 1


Question . 4. Chest pain at rest in a 10-yr-old boy is most likely to be due to all of the
following Except:

A. Asthma
B. Overuse injury
C. Musculoskeletal trauma
D. Coronary atherosclerosis
Explanation: Unless there is a strong family history of premature myocardial
infarction or sudden death, or unless symptoms are exacerbated by exercise,
chest pain in children is often due to musculoskeletal or pulmonary disorders.
Children may complain of chest pain after an adult family member has a
myocardial infarction. This is less likely to be coronary vessel disease and
more likely to be anxiety. Nonetheless, the clinician should be aware of more
serious causes in children with chest pain, dyspnea, or syncope with exercise.
(See Chapter 415 in Nelson Textbook of Pediatrics, 17th ed.)
E. Anxiety

Question . 5. The best management approach in a 10-yr-old with an ostium secundum


atrial septal defect with a 3:1 shunt is:

A. Observation until heart failure is present


B. Prophylaxis for endocarditis each month
C. Transvenous device closure
Explanation: Although surgical closure is highly successful, experience with
device closure in the cardiac catheterization laboratory has been quite
successful, and this procedure is now approved by the FDA. (See Chapter 419
in Nelson Textbook of Pediatrics, 17th ed.)
D. Observation until pulmonary hypertension develops
E. Phlebotomies for recurrent polycythemia

Question . 6. Pulmonary stenosis is associated with all of the following Except:

A. Noonan syndrome
B. Tetralogy of Fallot
C. Rheumatic fever
Explanation: Pulmonary valve lesions in rheumatic fever are very unusual.
This lesion is common as a dysplastic valve in Noonan syndrome and in
Alagille syndrome, which is due to mutations in Jagged 1. Isolated PS is also
noted to have an increased frequency of Jagged 1 mutations without Alagille
syndrome. (See Chapter 420 in Nelson Textbook of Pediatrics, 17th ed.)
D. Alagille syndrome
E. Mutations of Jagged 1 gene

The CVS System Nelson Self Assessments website 17th Edition 2


Question . 7. Complications of vascular rings include all of the following Except:

A. Recurrent emesis
B. Chronic wheezing
C. Tracheomalacia
D. Heart failure
Explanation: The vessels involved in vascular rings do not produce a shunt or
excessive blood flow. The main problems are mechanical impingement on the
trachea and/or esophagus and the symptoms resulting from these points of
obstruction. (See Chapter 425 in Nelson Textbook of Pediatrics, 17th ed.)
E. Cough

Question . 8. A 10-yr-old white girl has a history of increasingly severe exercise


intolerance and fatigability. The day of admission she had a syncopal event. On
examination, she has a systolic ejection click and a loud, narrowly split second heart
sound. This is a soft systolic murmur. The chest radiograph demonstrates prominent
pulmonary arteries and an enlarged right ventricle. The peripheral pulmonary vascular
markings are greatly decreased. The most likely diagnosis is:

A. Tetralogy of Fallot
B. Stills murmur
C. Rheumatic fever
D. Cor pulmonale
E. Primary pulmonary hypertension
Explanation: Primary pulmonary hypertension is an idiopathic disorder. Some
familial cases have mutations in the gene for bone morphogenetic protein
receptor II. (See Chapter 426 in Nelson Textbook of Pediatrics, 17th ed.)

Question . 9. Wolff-Parkinson-White syndrome is associated with all of the following


Except:

A. Short P-R interval


B. Slow upstroke of QRS complex
C. Renal tubular acidosis
Explanation: There are rarely any extracardiac problems in patients with
WPW. The ECG features of the delta wave (slow upstroke of the QRS
complex, also known as preexcitation) are not seen during the SVT and are
visible only in sinus rhythm. (See Chapter 428 in Nelson 17th ed.)
D. Ebstein anomaly
E. Episodes of supraventricular tachycardia

Question . 10. The initial management of a hemodynamically stable 2-mo-old infant


with supraventricular tachycardia should include:

A. Vagal stimulation
Explanation: In infants, the vagal maneuver of choice is placing a plastic bag
containing iced saline completely over the nose and mouth. If this is
unsuccessful, intravenous adenosine is the next step. Vagotonic maneuvers in
older children include doing a Valsalva maneuver, straining, breath holding,

The CVS System Nelson Self Assessments website 17th Edition 3


squatting, drinking iced water, coughing, vomiting, and gagging. (See Chapter
428 in Nelson, 17th ed.)
B. Cardioversion
C. Defibrillation
D. Digitalization
E. Intravenous verapamil

Question . 11. Disorders associated with complete heart block include all of the
following Except:

A. Maternal systemic lupus erythematosus


B. Kearns-Sayre syndrome
C. Rheumatoid arthritis
Explanation: Rheumatoid arthritis primarily involves the pericardium and not
the conduction system. Another cause of complete heart block is injury to the
conduction system during reparative surgery for congenital heart disorders.
(See Chapter 428 in Nelson Textbook of Pediatrics, 17th ed.)
D. Cardiac rhabdomyoma
E. Endocarditis

Question . 12. A previously healthy 7-yr-old girl has a 3-wk history of fever,
myalgias, and a positive blood culture for Staphylococcus aureus. The past medical
history is negative including a normal camp physical examination 6 mo ago, which
did not reveal any heart murmur. After repeating a blood culture, the next step in her
evaluation should be:

A. Chest radiograph
B. Complete blood count
C. Echocardiogram
Explanation: This patient has bacterial endocarditis of a previously normal
valve. S. aureus is a common pathogen producing endocarditis of a native
valve. The duration of the illness is too long for a simple viral illness, and in
the absence of bone or soft tissue findings, any person with a positive blood
culture for S. aureus should be considered as having endocarditis until proven
otherwise. (See Chapter 429 in Nelson Textbook of Pediatrics, 17th ed.)
D. Bone scan
E. Dental clinic appointment

Question . 13. Major findings in the Duke criteria for the diagnosis of endocarditis
include all of the following Except:

A. Two separate positive blood cultures for common bacteria


B. Intracardiac mass on a valve seen with echocardiography
C. Dehiscence of a prostatic valve
D. Osler nodes
Explanation: Immune complex phenomena and embolic events are minor
criteria. Two major or one major and 3-5 minor criteria suggest definite
endocarditis. (See Chapter 429 in Nelson Textbook of Pediatrics, 17th ed.)
E. More than two positive blood cultures for unusual bacteria

The CVS System Nelson Self Assessments website 17th Edition 4


Question . 14. An 8-yr-old boy presents with fever of 2 weeks' duration, shortness of
breath, ankle edema, intermittent painful swelling of the wrists, elbows, and knees
unrelated to edema, and a new systolic murmur. One month ago he had a sore throat
that lasted for 5 days, which resolved spontaneously. The most likely diagnosis is:

A. Endocarditis
B. Rheumatoid arthritis
C. Meningococcal sepsis
D. Glomerulonephritis
E. Rheumatic fever
Explanation: The sore throat represented an untreated episode of group A
streptococcal pharyngitis. Rheumatic fever presents acutely as a migrating
polyarthritis, pancarditis (valves, myocardium, pericardium), and erythema
marginatum (not in this patient). Delayed or chronic manifestations include
chorea or subcutaneous nodules. These all represent major criteria for the
diagnosis, but documentation of a previous streptococcal infection is also
required. (See Chapter 430 in Nelson Textbook of Pediatrics, 17th ed.)

Question . 15. A 3-mo-old female infant has a history of poor feeding, shortness of
breath during feedings, failure to thrive, and chronic cough. Physical examination
reveals tachycardia and a gallop rhythm but no murmur. There is hepatomegaly but no
cyanosis. A chest radiograph reveals cardiomegaly. The most appropriate diagnostic
test is:

A. Echocardiogram
Explanation: The differential diagnosis includes myocarditis,
cardiomyopathies, anomalous coronary arteries, and arteriovenous
malformations in the liver or brain. The echocardiogram reveals poor
contractibility and a dilated cardiomyopathy. (See Chapter 431 in Nelson)
B. Electrocardiogram
C. Blood culture
D. Serum amino acids
E. Urine pH

Question . 16. After further evaluation and treatment for heart failure, the child
described in Question 15 should also be started on:

A. Selenium
B. Vitamin B1
C. Thyroid hormone
D. Carnitine
Explanation: Although deficiencies of all of these nutrients (or hormone) may
produce heart failure, the clinical picture and history are not compatible with
A-C. Many believe that all infants with cardiomyopathy should receive a trial
of carnitine after being evaluated for metabolic inborn errors of metabolism
associated with cardiomyopathy. (See Chapter 431 in Nelson Textbook of
Pediatrics, 17th ed.)
E. None of the above

The CVS System Nelson Self Assessments website 17th Edition 5


Question . 17. A 20-day-old previously well full-term infant is presented with fever,
tachypnea, tachycardia of severity out of proportion to the fever, a gallop rhythm, and
hepatomegaly. Important steps in the evaluation of this neonate include all of the
following Except:

A. Head ultrasonography
Explanation: This neonate has a febrile illness with heart failure. (See Chapter
431 in Nelson Textbook of Pediatrics, 17th ed.)
B. Echocardiography
C. Viral cultures and PCR assay
D. ECG
E. Liver function tests
F. Blood culture

Question . 18. The echocardiogram for the patient in Question 17 reveals


myocarditis, and the viral cultures reveal an enterovirus. In addition to medications to
treat the heart failure, which additional therapy is now indicated?

A. Heart transplantation
B. Pleconaril
Explanation: Pleconaril is a potent antiviral agent that is very effective against
enteroviruses. It is the agent of choice for treatment of severe enteroviral
infections. (See Chapter 431 in Nelson Textbook of Pediatrics, 17th ed.)
C. Ribavirin
D. Intravenous immunoglobulins
E. Oseltamivir

Question . 19. A 1-day-old infant is noted to be cyanotic. Physical examination


reveals a grade 2-3/6 systolic murmur and a single loud second heart sound. The chest
radiograph reveals a normal-sized heart and decreased pulmonary vascular markings.
The electrocardiogram (ECG) reveals left ventricular dominance. The next step in the
management of this neonate is to administer:

A. Sodium bicarbonate
B. Morphine
C. Prostaglandin E1
Explanation: The murmur may represent a patent ductus arteriosus (PDA). If
the PDA closes, marked cyanosis would supervene, resulting in acidosis,
shock, and death. Prostaglandin E1 (PGE1) maintains patency of the ductus
arterious between the pulmonary artery and the aorta. (See Chapter 423 in
Nelson Textbook of Pediatrics, 17th ed.)
D. Digoxin
E. Positive pressure ventilation

The CVS System Nelson Self Assessments website 17th Edition 6


Question . 20. The most likely diagnosis for the patient described in Question 19 is:

A. Persistent pulmonary hypertension


B. Transposition of the great arteries
C. Truncus arteriosus
D. Pulmonary atresia
Explanation: Pulmonary atresia is manifested by a small right ventricle,
decreased pulmonary vascular markings, early and marked cyanosis without
heart failure, and ductal dependence to maintain some pulmonary blood flow.
(See Chapter 423 in Nelson Textbook of Pediatrics, 17th ed.)
E. Total anomalous venous return

Question . 21. An 18-mo-old child is noted to assume a squatting position frequently


during playtime at the daycare center. The mother also notices occasional episodes of
perioral cyanosis during some of these squatting periods. The day of admission, the
child becomes restless, hyperpneic, and deeply cyanotic. Within 10 min, the child
becomes unresponsive. The most likely underlying lesion is:

A. Cardiomyopathy
B. Anomalous coronary artery
C. Tetralogy of Fallot
Explanation: The child described has tetralogy of Fallot with exercise-induced
cyanosis. The more serious episode is a cyanotic, blue, or "tet" spell and may
be due to decreased systemic vascular resistance, increased pulmonary artery
pressure, or right ventricular outflow tract obstruction. The murmur of
tetralogy (the pulmonary stenosis) often disappears or lessens during a spell.
(See Chapter 423 in Nelson Textbook of Pediatrics, 17th ed.)
D. Constipation
E. Breath-holding spell

Question . 22. Therapy of a "blue" or "tet" spell could include all of the following
Except:

A. Epinephrine
Explanation: Epinephrine is potentially dangerous because it may exacerbate
inotropy and contractile forces, which may obstruct the right ventricular
infundibulum. Indeed, propranolol has been used to treat "tet" spells. (See
Chapter 423 in Nelson Textbook of Pediatrics, 17th ed.)
B. Knee-chest position
C. Oxygen
D. Morphine
E. Sodium bicarbonate
F. Phenylephrine

The CVS System Nelson Self Assessments website 17th Edition 7


Question . 23. A 2-day-old infant experiences cyanosis, hypotension, and metabolic
acidosis. On examination, the infant is lethargic, tachycardic, and gray-blue, with
hepatomegaly, a grade 2-3/6 systolic murmur, and poor radial and femoral pulses. A
chest radiograph reveals cardiomegaly, and an ECG demonstrates right ventricular
dominance with markedly reduced R waves in V5 and V6. The most likely diagnosis:

A. Myocarditis
B. Hypoplastic left heart syndrome
Explanation: Poor pulses, reduced left ventricular forces on ECG, cardiogenic
shock, and severe cyanosis are typical of hypoplastic left heart syndrome. (See
Chapter 424 in Nelson Textbook of Pediatrics, 17th ed.)
C. Anomalous coronary arteries
D. Total anomalous venous return
E. Tetralogy of Fallot

Question . 24. A previously well 3½-mo-old presents with poor feeding, diaphoresis
during feeding, and poor growth. Vital signs reveal a respiratory rate of 70/min, pulse
of 175/min, and blood pressure of 90/65 mm Hg in the upper and lower extremities.
The cardiac examination reveals a palpable parasternal lift and a systolic thrill. A
grade 4 holosystolic murmur and a mid-diastolic rumble are noted. The chest
radiograph reveals cardiomegaly. The most likely diagnosis is:

A. Cardiomyopathy
B. Myocarditis
C. VSD
Explanation: A large VSD with a large left-to-right shunt produces significant
heart failure. The age at onset usually corresponds to the time when the
normally high fetal pulmonary vascular resistance declines in the first 1-3 mo
of life. With decreasing pulmonary artery pressure, the left-to-right shunt
increases. (See Chapter 419.6 in Nelson Textbook of Pediatrics, 17th ed.)
D. Coarctation of the aorta
E. Transposition of the great arteries

Question . 25. The initial treatment of choice for a symptomatic patient with isolated
pulmonic stenosis is:

A. Closed surgical blade valvotomy


B. Open surgical valvotomy
C. Balloon catheter valvuloplasty
Explanation: Balloon valvuloplasty has greatly improved the management of
stenotic lesions of the pulmonic and aortic valves. (See Chapter 420 in Nelson
Textbook of Pediatrics, 17th ed.)
D. Blalock-Taussig shunt
E. Valve replacement

The CVS System Nelson Self Assessments website 17th Edition 8


Question . 26. Pulsus paradoxus is associated with:

A. Pericarditis
Explanation: Paradoxic pulse is also noted in asthma. (See Chapter 432 in
Nelson Textbook of Pediatrics, 17th ed.)
B. Endocarditis
C. Rheumatic fever
D. Myocarditis
E. Postperfusion syndrome

Question . 27. The radiographic finding of notching of the ribs is associated with:

A. Pulmonary hypertension
B. Anomalous pulmonary venous return above the diaphragm
C. Coarctation of the aorta
Explanation: Rib notching is caused by increased collateral arteries trying to
supply the lower trunk and extremities and bypass the aortic coarctation. (See
Chapter 420.6 in Nelson Textbook of Pediatrics, 17th ed.)
D. Systemic hypertension
E. Aortic insufficiency

Question . 28. An atrioventricular septal defect is different from an ostium secundum


ASD because the AV septal defect:

A. Does not manifest heart failure


B. Does not create volume overload
C. Has the same ECG findings
D. Produces an early tendency for pulmonary hypertension
Explanation: The atrial and the atrial-ventricular defects both are associated
with a higher tendency for an earlier and more rapid onset of pulmonary
hypertension. The ECG of an A-V septal defect is characteristic because of the
superior left deviation of the QRS axis. (See Chapter 419 in Nelson, 17th ed.)
E. Creates an atrial-level shunt

Question . 29. A 5-mo-old previously well infant is found to have a loud holosystolic
murmur (4/6) at the left sternal border. The first and second heart sounds are normal;
there is no tachycardia, rumble, or gallop; and hepatomegaly is not noted. The child
feeds well and has grown adequately. You suspect:

A. A restrictive VSD
Explanation: A small VSD (<0.5 cm2) produces a loud murmur, with perhaps
a thrill but no other abnormalities, owing to the limited left-to-right shunt.
Many of these defects will close spontaneously. (See Chapter 419 in Nelson
Textbook of Pediatrics, 17th ed.)
B. Anomalous left coronary artery
C. A VSD with a 4:1 shunt
D. Tetralogy of Fallot
E. Single ventricle

The CVS System Nelson Self Assessments website 17th Edition 9


Question . 30. A 6-mo-old is presented with tachycardia, tachypnea, and poor feeding
for 3 mo. Physical examination reveals a continuous machinery murmur and a wide
pulse pressure with a prominent apical impulse. The most likely diagnosis is:

A. Pulmonic stenosis
B. Aortic stenosis
C. Ventricular septal defect
D. Patent ductus arteriosus
Explanation: A PDA often presents like a large VSD except that there is a
continuous murmur as well as the diastolic runoff, which produces the wide
pulse pressure and bounding pulses. (See Chapter 419 in Nelson, 17th ed.)
E. Anomalous coronary artery

Question . 31. The differential diagnosis for the lesion in Question 30 includes all of
the following Except:

A. Truncus arteriosus
B. Aorticopulmonary window
C. Sinus of Valsalva aneurysm rupture
D. Aortic valve insufficiency
E. Critical aortic stenosis from a bicuspid valve
Explanation: Aortic stenosis without significant aortic regurgitation
(insufficiency) produces an inactive precordium, normal or weak pulses, and
pressure overload of the left ventricle. (See Chapter 419 in Nelson, 17th ed.)

Question . 32. For the patient described in Questions 30 and 31, treatment is best
performed by which of the following method(s)?

A. Intravenous indomethacin
B. Surgical closure
Explanation: A PDA manifesting in any patient other than a premature infant
does not close spontaneously. Surgery carries low morbidity and even lower
mortality. However, the availability of transcatheter closure with coils or
umbrellas has reduced the need for surgery in most patients. (See Chapter 419)
C. Catheter coil closure
D. Digoxin until the PDA closes spontaneously
E. Angiotensin-converting enzyme inhibitor

Question . 33. A neonate manifests cyanosis and hepatomegaly. There is a grade 4/6
systolic ejection murmur without an audible ejection click. The ECG reveals tall,
spiked P waves and right ventricular hypertrophy. The best method to evaluate this
patient is to perform:
A. Chest x-ray examination
B. Vector cardiography
C. Immediate cardiac catheterization
D. MRI
E. Echocardiogram
Explanation: Echocardiology has replaced most methods of visualizing the
anatomy of congenital and other heart lesions. In many circumstances, the

The CVS System Nelson Self Assessments website 17th Edition 10


echocardiographic diagnosis is all that is needed before surgery or other
therapies. (See Chapter 416 in Nelson Textbook of Pediatrics, 17th ed.)

Question . 34. The most likely diagnosis in the patient described in Question 33 is:

A. Patent ductus arteriosus


B. Critical aortic stenosis
C. Critical pulmonic stenosis
Explanation: Critical pulmonic stenosis often presents in the neonate.
Cyanosis is due to elevated right-sided pressures and right-to-left shunting at
the patent foramen orale. (See Chapter 420 in Nelson of Pediatrics, 17th ed.)
D. Tetralogy of Fallot
E. Truncus arteriosus

Question . 35. The treatment of choice for the lesion in the neonate described in
Questions 33 and 34 is:

A. Digoxin
B. Propranolol (Inderal)
C. Surgical shunt
D. Balloon valvuloplasty
Explanation: Balloon pulmonary valvuloplasty is a remarkable nonsurgical
method to dilate the valve and partially relieve the obstruction to right
ventricular outflow. (See Chapter 420 in Nelson of Pediatrics, 17th ed.)
E. Valve replacement

Question . 36. A 12-yr-old boy tries out for a middle school hockey team. He has a
history of a heart murmur as an infant, but the doctor thought it would go away.
During the tryout, he experiences severe dyspnea and becomes light-headed. At your
office, he has a normal rhythm, pulse, and blood pressure and is no longer dizzy.
There is a grade 4/6 systolic ejection murmur that radiates to the neck. There is also
an ejection click. An ECG reveals left ventricular hypertrophy. The next step in his
management should be:

A. Chest x-ray examination


B. Exercise test
C. Digitalization
D. Echocardiography
Explanation: Echocardiography is essential to identify valve disease, estimate
gradients, and detect signs of endocardial fibroelastosis. (See Chapter 420 in
Nelson Textbook of Pediatrics, 17th ed.)
E. Tilt table testing

The CVS System Nelson Self Assessments website 17th Edition 11


Question . 37. The patient described in Question 36 most probably has:

A. Pulmonic stenosis
B. Mild aortic stenosis
C. Severe aortic stenosis
Explanation: Severe aortic stenosis may develop gradually, progressing over
5-10 yr. (See Chapter 420 in Nelson Textbook of Pediatrics, 17th ed.)
D. Patent ductus arteriosus
E. Williams syndrome

Question . 38. For the patient described in Question 36, treatment is best
accomplished with

A. Immediate valve replacement


B. A shunt
C. Digoxin
D. Propranolol
E. Balloon valvuloplasty
Explanation: Balloon dilatation valvotomy and surgically splitting the valve
are treatments of choice. (See Chapter 420 in Nelson of Pediatrics, 17th ed.)

Question . 39. An 8-yr-old boy from the former Soviet Union is brought to your
office because of a heart murmur and dusky blue skin. He has marked clubbing of the
fingers and toes, and his hematocrit is 70%. He has a grade 4/6 holosystolic murmur.
His chest film reveals a normal pulmonary blood flow as well as a right-sided aortic
arch. The most likely diagnosis is:

A. Total anomalous venous return


B. Tetralogy of Fallot
Explanation: TOF includes a VSD, pulmonary stenosis, right ventricular
hypertrophy, and an overriding aorta between the two ventricles and above the
VSD. In addition, 25% of patients have a right-sided aortic arch. (See Chapter
423 in Nelson Textbook of Pediatrics, 17th ed.)
C. Eisenmenger syndrome
D. Transposition of the great vessels
E. Single ventricle

Question . 40. The reason that the hematocrit is so high in the patient described in
Question 39 is:
A. Chronic hypoxia
Explanation: Chronic hypoxia produces both the clubbing and the
polycythemia. High hematocrits (usually >65% on a central blood sample)
usually produce hyperviscosity, which impairs tissue perfusion. Therefore, up
to a point, the raised hematocrit improves oxygen delivery; once
hyperviscosity develops, the tissue microcirculation is compromised. (See
Chapter 427 in Nelson Textbook of Pediatrics, 17th ed.)
B. It was taken in a low cardiac output state
C. It was a peripheral finger stick
D. Polycythemia improves flow through VSD
E. Hyperviscosity aids flow to the pulmonary artery

The CVS System Nelson Self Assessments website 17th Edition 12


Question . 41. The patient described in Question 39 experiences a headache and is
unable to move the left side of his body. The most likely cause is:

A. Embolic stroke
B. Cerebral thrombosis
C. Cerebral abscess
Explanation: Cerebrovascular accidents result from a combination of
polycythemia and the right-to-left shunting in patients with TOF. Strokes are
more common in children younger than 2 yr and in those with iron-deficiency
anemia. Cerebral abscess formation is more common in children older than 2
yr. (See Chapter 427 in Nelson Textbook of Pediatrics, 17th ed.)
D. Migraine
E. Moyamoya disease

Question . 42. The most likely cause of sudden death in an athlete immediately after
forceful blunt trauma to the chest is:

A. Commotio cordis
Explanation: This unusual event places the heart in asystole, which is
unfortunately refractory to almost all resuscitative efforts. At the scene,
defibrillation may improve the outcome if done within minutes of the episode.
(See Chapter 428 in Nelson Textbook of Pediatrics, 17th ed.)
B. Myocardial infarction
C. Lacerated coronary artery
D. Ruptured cardiac tendineae
E. Hemorrhagic pericarditis

Question . 43. An 8-yr-old girl presents with chest pain of 1 day's duration. Five days
before onset of the pain, she had fever, chills, and myalgias. Physical examination
reveals an uncomfortable, anxious, afebrile patient with tachycardia, no murmur, and
distant heart sounds. The chest radiograph shows cardiomegaly without pulmonary
edema. There is a paradoxical pulse of 22 mm Hg. The most likely diagnosis is:

A. Myocarditis
B. Cardiomyopathy
C. Kawasaki disease
D. Pericarditis
Explanation: Pericarditis, in this child due to coxsackievirus infection, often
presents as chest pain, tachycardia, narrow pulse pressure, and a pulsus
paradoxus. The chest radiograph demonstrates cardiomegaly, which can be
enlargement due to myocardial dilatation or expansion of the pericardial space
from fluid. (See Chapter 432 in Nelson Textbook of Pediatrics, 17th ed.)
E. Sepsis

The CVS System Nelson Self Assessments website 17th Edition 13


Question . 44. The differential diagnosis for the patient described in Question 43
includes all of the following Except:

A. Rheumatic fever
B. SLE
C. Juvenile rheumatoid arthritis
D. Post-meningococcal immune complexes
E. Chronic atrial fibrillation
Explanation: Immune complex and other immune mechanisms may produce a
pericardial effusion, pericarditis, and rarely cardiac tamponade. (See Chapter
432 in Nelson Textbook of Pediatrics, 17th ed.)

The CVS System Nelson Self Assessments website 17th Edition 14


The Digestive System
Nelson Self Assessments website 17th Edition.

Question . 1. An 8-yr-old child presents with enamel defects on the permanent central
and lateral incisors. At what time would a metabolic disturbance affecting the ameloblast
have occurred?

A. First trimester of pregnancy


B. Last trimester of pregnancy
C. Age 2 yr
Explanation: This is the critical time for enamel defects on permanent incisors.
Prenatal disturbances often affect primary teeth. (See Chapter 288 in Nelson
Textbook of Pediatrics, 17th edition.)
D. Age 5 yr
E. Age 7 yr

Question . 2. Tetracycline administration during tooth formation may lead to:

A. Enamel defects
B. Tooth shape abnormalities
C. Enamel hypoplasia
D. Gingival enlargement
E. Tooth discoloration
Explanation: Yellow-brown discoloration or even color changes that are visible
with a fluorescent lamp have been reported with long periods of tetracycline use
before the age of 8 yr. (See Chapter 288 in Nelson Textbook of Pediatrics, 17th
edition.)

Question . 3. A persistent thumb sucking habit in a 7-yr-old child will:

A. Not affect the dentition because the child is continuing to grow


B. Cause the maxillary permanent incisor teeth to protrude
Explanation: Prolonged thumb-sucking after the eruption of permanent teeth may
produce maxillary tooth protrusion. (See Chapter 290 in Nelson Textbook of
Pediatrics, 17th edition.)
C. Cause the maxillary primary incisors to protrude
D. Necessitate future orthodontic correction
E. Necessitate immediate orthodontic correction

The Digestive System Nelson Self Assessments website 17th Edition 1


Question . 4. Clinical manifestations of a cleft lip or palate may include:

A. Deformed or absent teeth


B. Deficiencies of the nasal alar cartilage
C. Protrusion of the premaxillary process
D. Notching of the vermilion border
E. All of the above
Explanation: Cleft lip combined with cleft palate is a major developmental
anomaly of the mid-face and produces all of these manifestations. Surgical repair
must take each of these into consideration in addition to long-term sequelae. (See
Chapter 291 in Nelson Textbook of Pediatrics, 17th edition.)

Question . 5. Syndrome(s) with significant reduction in the number of teeth is (are):

A. Osteogenesis imperfecta
B. Cleidocranial dysostosis
C. Ectodermal dysplasia
Explanation: Variations in size or shape, but not in number, of teeth are noted in
the other syndromes. (See Chapter 292 in Nelson Textbook of Pediatrics, 17th
edition.)
D. Pierre Robin syndrome
E. All of the above

Question . 6. Dental caries (cavities) in children are due to:

A. Metabolism of Streptococcus mutans


Explanation: A. This oral flora ferments carbohydrates and produces the local
metabolic environment necessary to produce and expand a cavity. (See Chapter
293 in Nelson Textbook of Pediatrics, 17th edition.)
B. Enamel defects in teeth
C. Hereditary factors
D. Failure to brush teeth
E. All of the above

Question . 7. A 2-yr-old presents to your office with black spots and missing enamel on
the maxillary anterior teeth. The most likely etiology of this condition is:

A. Hereditary enamel hypoplasia


B. Enamel defects
C. High sucrose consumption and Streptococcus mutans infection
Explanation: This is classic for a child who consumes excessive carbohydrate
drinks (soda, juices, milks) and has cariogenic bacteria. These teeth may need to

The Digestive System Nelson Self Assessments website 17th Edition 2


be protected with sealants. (See Chapter 293 in Nelson Textbook of Pediatrics,
17th edition.)

D. Medications, such as tetracycline, that become absorbed in the enamel


E. Oral trauma from falling

Question . 8. A 5-yr-old child presents to your practice with pain from a tooth. The
parent says that for the last week the child has been waking up at night, crying from tooth
pain. Your plan includes:

A. Referral to a dentist
B. Prescription for an antibiotic
C. Prescription for an analgesic
D. All of the above
Explanation: This child most likely has an apical tooth abscess. Caries are usually
painless unless an abscess develops. Penicillin is an appropriate antibiotic. (See
Chapter 293 in Nelson Textbook of Pediatrics, 17th edition.)

Question . 9. Oral complications associated with seizures in children include:

A. Malocclusion
B. Gingival enlargement
Explanation: Gingival hyperplasia is classically seen in patients receiving
phenytoin (Dilantin). Not all patients get this complication. (See Chapter 294 in
Nelson Textbook of Pediatrics, 17th edition.)
C. Increased rate of dental caries
D. Notching of the vermilion border
E. All of the above

Question . 10. A football coach calls you about a 15-yr-old who just had a central incisor
knocked out on the field. You tell the coach to:

A. Put the tooth in milk and bring the child to your office
B. Try to reimplant the tooth and take the child to the dentist
Explanation: The best place to put a dislodged tooth is back in its socket.
Reimplantation of the tooth (rinse it first, but don't scrub it) is the best approach in
a cooperative patient. (See Chapter 295 in Nelson Textbook of Pediatrics, 17th
edition.)
C. Wrap the tooth in a damp paper towel and bring the tooth and child to your office
D. Do not worry about finding the tooth, but take the child immediately to the
emergency department

The Digestive System Nelson Self Assessments website 17th Edition 3


Question . 11. Oral candidiasis can be found in:

A. Infants
B. Children with AIDS
C. Children receiving antibiotics
D. Children with nutritional deficiencies
E. All of the above
Explanation: Oral candidiasis is quite common in all infants and should be a
concern if it doesn't respond to topical therapy or if it occurs in the presence of
other opportunistic infections, recurrent otitis media, or chronic diarrhea and
failure to thrive. (See Chapter 296 in Nelson Textbook of Pediatrics, 17th
edition.)

Question . 12. Long-term xerostomia is a high risk factor for dental caries. It can be
associated with:

A. Dehydration
B. Anticholinergic drugs
C. Sj gren syndrome
D. Tumoricidal dose of radiation in the area of the salivary glands
E. All of the above
Explanation: Dry mouth is uncommon in children but must be considered in high-
risk populations as noted in this question. (See Chapter 297 in Nelson Textbook
of Pediatrics, 17th edition.)

Question . 13. The most common developmental abnormality of the exocrine pancreas is:

A. Cystic fibrosis
B. Pancreas divisum
Explanation: Pancreas divisum is a common finding now that CT, MRI, and
ultrasonography readily identify this anatomic abnormality. It may or may not
predispose to the development of pancreatitis. Most people with pancreatitis do
not have pancreas divisum. Whether most people with pancreas divisum develop
pancreatitis is unknown, but it is unlikely. (See Chapter 328 in Nelson Textbook
of Pediatrics, 17th edition.)
C. Annular pancreas
D. Pancreatic pseudocyst
E. Choledochal cyst

The Digestive System Nelson Self Assessments website 17th Edition 4


Question . 14. The most common cause of pancreatic insufficiency in childhood is:

A. Chronic pancreatitis
B. Congenital lipase deficiency
C. Cystic fibrosis
Explanation: In the developed countries, cystic fibrosis is the most common
etiology of pancreatic insufficiency. This is present in more than 90% of children
with cystic fibrosis and is usually present at birth. In developing countries,
malnutrition is the most common cause of pancreatic insufficiency. (See Chapter
330 in Nelson Textbook of Pediatrics, 17th edition.)
D. Shwachman-Diamond syndrome
E. Pearson syndrome

Question . 15. All of the following tests or determinations may be useful in the diagnosis
of acute pancreatitis Except:

A. Serum amylase
B. Serum lipase
C. Abdominal ultrasound examination
D. Fecal fat
Explanation: Determination of fecal fat content is useful in the diagnosis of fat
malabsorption from hepatobiliary or pancreatic disease. Acute pancreatitis is not
associated with significant steatorrhea. Chronic pancreatitis may cause fat
malabsorption. (See Chapter 332 in Nelson Textbook of Pediatrics, 17th edition.)
E. Serum amylase isoenzymes

Question . 16. A 10-yr-old boy develops pancreatitis following a blow to the abdomen.
During a 5-day hospitalization, he is treated with intravenous fluids and analgesics and is
discharged home on a low-fat diet. Four weeks later he has recurrence of abdominal pain
and vomiting. The most likely diagnosis is:

A. Functional abdominal pain


B. Duodenal ulcer due to Helicobacter pylori
C. Chronic pancreatitis
D. Hemorrhagic pancreatitis
E. Pancreatic pseudocyst
Explanation: A pancreatic pseudocyst is a common sequela to acute pancreatitis.
Symptomatic pseudocysts require drainage. If symptoms develop within a week
of blunt trauma, one must also consider a duodenal hematoma. (See Chapter 333
in Nelson Textbook of Pediatrics, 17th edition.)

The Digestive System Nelson Self Assessments website 17th Edition 5


Question . 17. A 15-yr-old girl with cystic fibrosis and difficult-to-manage pancreatic
insufficiency is treated with pancreatic enzyme supplementation containing lipase,
10,000 IU/kg, and omeprazole, 20 mg/day, with improvement in her stools for 11 mo.
She then develops severe constipation, abdominal pain, hematochezia, and vomiting. The
most likely diagnosis is:

A. Fibrosing colonopathy
Explanation: Excessive quantities of oral pancreatic enzyme replacements can
produce fibrosing colonopathy. If insufficient enzymes were used, a meconium
ileus-like picture could develop. (See Chapter 331 in Nelson Textbook of
Pediatrics, 17th edition.)
B. Crohn disease
C. Distal intestinal obstruction syndrome (meconium ileus equivalent)
D. Acute pancreatitis
E. Intussusception

Question . 18. A 5-mo-old boy was clinically diagnosed with gastroesophageal reflux at
3 mo of age. Treatment with cimetidine resolved his constant crying and even seemed to
improve his spitting. His weight continues in the 50th percentile. On a follow-up visit,
however, his parents report that for the past 2 wk he once again has begun frequent
regurgitation, hiccups, and irritability. He is breast-fed and is given occasional
supplements of cow's milk-based formula. His examination is normal. Which of the
following is the most appropriate intervention?

A. Reassurance
B. Fundoplication
C. Upper endoscopy with biopsies
D. Adjustment of the dose of cimetidine
Explanation: This child responded well to the initial therapy with an H2-blocking
agent. With his continued growth along the 50th percentile, he has outgrown his
dose of cimetidine. He still has reflux, and more efficient acid suppression is
required to heal the esophagitis and improve the child's condition. (See Chapter
304 in Nelson Textbook of Pediatrics, 17th edition.)
E. Daily use of antacids

Question . 19. A 2-mo-old Bangladeshi boy, born in the United States, has been coughing
since birth. The coughing episodes are particularly pronounced during feeding. His
mother also mentions occasional choking and gagging episodes. His immunizations are
up-to-date and there is no history of travel. Which of the following will be the most
appropriate diagnostic test?

A. Extended pH monitoring
B. Tuberculin skin test
C. Pressure-injection barium swallow

The Digestive System Nelson Self Assessments website 17th Edition 6


Explanation: This is a classic history for the most uncommon type of
tracheoesophageal fistula: the H type. With each feeding, milk goes through the
fistula to the trachea, producing a gag-cough response. (See Chapter 300 in
Nelson Textbook of Pediatrics, 17th edition.)
D. Modified barium swallow ("cookie swallow")
E. Endoscopy of esophagus and stomach

Question . 20. A 6-yr-old girl with documented ?1-antitrypsin deficiency presents to the
emergency department for assessment of large-volume hematemesis. Physical
examination is remarkable for clear lung fields, pallor, and splenomegaly. Hepatomegaly
and petechiae are absent. Which of the following is the most likely source of the
hematemesis?

A. Peptic ulcer disease


B. Thrombocytopenia secondary to hypersplenism
C. Swallowed blood from pulmonary hemorrhage
D. Esophageal varices
Explanation: Severe ?1-antitrypsin deficiency presenting in early childhood often
involves the liver and could lead to cirrhosis, portal hypertension, and esophageal
varices. (See Chapter 337 in Nelson Textbook of Pediatrics, 17th edition.)
E. Erosive reflux esophagitis

Question . 21. A 3-mo-old infant presents for a well-child evaluation. Which of the
following complaints would be a cause for concern?

A. Regurgitation of 15-30 mL of formula three times a day


B. One bowel movement every other day
C. Three bowel movements per day
D. Liver edge palpable 10 cm below the right costal margin
E. Hemoccult-positive stool
Explanation: A through D are physiologic variations of normal infants. Blood in
the stool, even in microscopically detectable quantities, is abnormal and suggests
gastrointestinal bleeding. Inflammation due to milk protein allergy is a common
cause of heme-positive stools in formula-fed infants. (See Chapter 287 in Nelson
Textbook of Pediatrics, 17th edition.)

Question . 22. An 8-yr-old boy presents with a recurrent history of vomiting. Vomiting
episodes last 24 to 48 hr and usually end abruptly. There have been four episodes over
the last 8 mo. Growth is normal, and the child is well between episodes. Which of the
following is most likely to develop in this patient?

A. Pancreatitis
B. Appendicitis

The Digestive System Nelson Self Assessments website 17th Edition 7


C. Migraine equivalent
Explanation: Migraine equivalent manifesting as cyclic episodes of vomiting
("cyclic vomiting") is the most likely diagnosis. The differential diagnosis of
episodic recurrent emesis includes metabolic and mitochondrial disorders,
intestinal obstruction, overfeeding, and intracranial pathology. Treatment includes
intravenous fluids acutely and antimigraine medication if long-term treatment is
needed. (See Chapter 287 in Nelson Textbook of Pediatrics, 17th edition.)
D. Gastroesophageal reflux
E. Cholecystitis

Question . 23. A 6-wk-old infant has a thoracotomy for transposition of the great vessels.
Following surgery, the infant develops loose stools. Stool cultures are negative and the
stool is Hemoccult-negative. Fecal pH is 6.0. The infant fails to gain weight. Chest x-ray
shows a pleural effusion. Which of the following would be the most appropriate dietary
intervention?

A. Medium chain tryglycerides-predominant formula


Explanation: Inadvertent injury to the thoracic duct during cardiac surgery may
produce a chylous effusion and diarrhea. Medium-chain triglycerides are absorbed
through the venous plexus and not the lymphatics and are the traditional
nutritional approach to this problem. (See Chapter 332 in Nelson Textbook of
Pediatrics, 17th edition.)
B. Soy formula
C. Lactose-free formula
D. Carbohydrate-free formula
E. Long chain fat-predominant formula

Question . 24. A 6-wk-old male infant presents with a 3-wk history of intermittent
vomiting and a weight loss of 300 g. Serum electrolytes are as follows: sodium, 147
mEq/L; potassium, 2.9 mEq/L; HCO3 -, 32 mEq/L; and chloride, 89 mEq/L. The most
likely diagnosis is:

A. Gastric volvulus
B. Adrenogenital syndrome
C. Hypertrophic pyloric stenosis
Explanation: This is the classic history of pyloric stenosis in a child. The
hypochloremic hypokalemic metabolic alkalosis helps in the diagnosis. Physical
examination immediately after feeding may reveal the olive-shaped pyloric
obstruction, while ultrasonography or an upper gastrointestinal contrast study will
demonstrate the hypertrophied pylorus as the site of obstruction. Adrenogenital
syndrome often presents with emesis and diarrhea with acidosis, hyponatremia,
and hyperkalemia. (See Chapter 310 in Nelson Textbook of Pediatrics, 17th
edition.)
D. Malrotation with volvulus
E. Cyclic vomiting

The Digestive System Nelson Self Assessments website 17th Edition 8


Question . 25. A 6-mo-old infant has a 6-wk history of intermittent vomiting and poor
weight gain. The prenatal history is unremarkable. Meconium was passed on the second
day of life. Which of the following findings are diagnostic studies most likely to show?

A. Calcifications on KUB
B. Inversion of the superior mesenteric artery and vein noted on abdominal
ultrasound examination
Explanation: This child has a malrotation and may have obstruction due to a
volvulus or Ladd bands. An upper gastrointestinal contrast study will reveal an
abnormal position of the duodenum and any obstructing lesions. An associated
volvulus may produce bowel infarction and is a surgical emergency. (See Chapter
311 in Nelson Textbook of Pediatrics, 17th edition.)
C. Dilated distal small bowel loops on KUB
D. Target sign on abdominal ultrasound examination
E. Dilation of the rectum and colon on barium enema

Question . 26. A 4-yr-old girl presents with a 1-day history of a painless, maroon-colored
stool. She appears comfortable and in no distress or pain but is pale. Pulse is 110/min.
Hemoglobin level is 9.8 g/dL. The most appropriate step to establish the diagnosis is:

A. Abdominal ultrasound examination


B. Air-contrast enema
C. Angiography
D. Colonoscopy
E. Meckel scan
Explanation: A Meckel diverticulum often presents with painless rectal bleeding
in young children. The presence of gastric tissue in the diverticulum predisposes
the patient to ulceration and painless bleeding. The gastric tissue is identified by
the scan as it picks up the technetium. (See Chapter 312 in Nelson Textbook of
Pediatrics, 17th edition.)

Question . 27. A 2-mo-old infant has a history of infrequent bowel movements and poor
weight gain. Physical examination reveals a thin, poorly nourished child with abdominal
distention. Rectal examination reveals normal tone, and the infant subsequently passes an
explosive, foul-smelling stool. All the following may be useful in establishing the
diagnosis Except:
A. Colonoscopy
Explanation: Colonoscopy is not indicated in children suspected of having
Hirschsprung disease. This is a classic presentation, especially if it was known
that he needed a glycerin suppository and rectal stimulation to stool in the nursery
after 72 hours of not passing meconium. The disease is more common in males.
(See Chapter 313 in Nelson Textbook of Pediatrics, 17th edition.)
B. Un-prepped barium enema
C. Rectal suction biopsy
D. Full-thickness rectal biopsy
E. Rectal motility

The Digestive System Nelson Self Assessments website 17th Edition 9


Question . 28. A previously healthy 6-mo-old child develops paroxysmal colicky
abdominal pain. The infant has occasional vomiting. Over the next 12 hr the infant passes
stool containing blood and mucus and becomes progressively lethargic. Following fluid
resuscitation, the most appropriate next step in management is:

A. Colonoscopy with polypectomy


B. Stool culture
C. Meckel scan
D. Air-contrast enema
Explanation: This child most likely has intussusception. It is unknown why
children with this disorder become lethargic, but many are incorrectly thought to
have central nervous system pathology and wind up getting a head CT rather than
a radiologic enema. Air or barium enema is the diagnostic tool of choice and, with
hydrostatic pressure, is also the treatment of choice. (See Chapter 314 in Nelson
Textbook of Pediatrics, 17th edition.)
E. Empiric antibiotic therapy

Question . 29. A 3-yr-old hearing-impaired child is found playing with his hearing aid.
No battery can be found, and the child is noted to be drooling. The most appropriate next
step in treatment would be:

A. Ensuring consumption of fluids


B. Ipecac administration
C. Activated charcoal administration
D. Chest x-ray study
Explanation: Hearing aid batteries are quite dangerous and if ingested they should
be removed as soon as possible. Observation is not a correct choice. (See Chapter
315 in Nelson Textbook of Pediatrics, 17th edition.)
E. Observation and follow-up in 24 hr

Question . 30. A 14-yr-old girl has a 6-mo history of recurrent abdominal pain. The pain
is characterized as periumbilical to epigastric, lasting 20 to 30 min, with a frequency of
two or three times per week. The pain is relieved following the passage of stool. Her
mother has had similar symptoms intermittently since the age of 12 yr. There has been no
fever, rash, joint symptoms, or weight loss. No blood has been noted in the stool. The
most appropriate next step in the evaluation or management of this patient would be:

A. Abdominal CT scan
B. Endoscopic retrograde cholangiopancreatography (ERCP)
C. Trial of dietary fiber
Explanation: Both mom and daughter probably have irritable bowel syndrome.
The disease is of unknown etiology but is not "psychological." Dietary fiber is a
good first line of therapy. (See Chapter 323 in Nelson Textbook of Pediatrics,
17th edition.)
D. Colonoscopy
E. Gluten-free diet

The Digestive System Nelson Self Assessments website 17th Edition 10


Question . 31. In a newborn baby with an anorectal malformation, which of the
following diagnostic tests is most important during the first 24 hr of life?

A. MRI of the entire body


B. CT of the pelvis
C. Electroencephalogram
D. Ultrasound examination of the abdomen
Explanation: Associated internal anomalies in children with anorectal
malformations often include the genitourinary system, especially the kidneys.
Sacral anomalies may also be present, but it is critical to know if there are
associated renal anomalies prior to surgery. (See Chapter 325 in Nelson Textbook
of Pediatrics, 17th edition.)
E. Radiograph of the sacrum

Question . 32. A female newborn has a single perineal orifice. The chance that this baby
has associated urologic problems is:

A. <10%
B. 30%
C. 60%
D. 90%
Explanation: This is a high-risk anomaly and is associated with a major risk for
urologic problems of the bladder and kidneys. (See Chapter 325 in Nelson
Textbook of Pediatrics, 17th edition.)
E. None of the above

Question . 33. A baby is born with no anal opening and obvious features of Down
syndrome. The most likely type of anorectal malformation in this baby is:

A. Recto-bladder neck fistula


B. Rectourethral fistula
C. Rectoprostatic fistula
D. Rectoperineal fistula
E. Imperforate anus with no fistula
Explanation: Imperforate anus without a fistula is a common pattern of anal
anomalies in children with trisomy 21. (See Chapter 325 in Nelson Textbook of
Pediatrics, 17th edition.)

The Digestive System Nelson Self Assessments website 17th Edition 11


Question . 34. The most common type of anorectal malformation seen in females is:

A. Cloacal defect
B. Rectovaginal fistula
C. Rectovestibular fistula
Explanation: This fistula may not be completely obstructed. The child may "pass"
meconium as well. Without direct observation of the perineum, the imperforate
anus may be missed. (See Chapter 325 in Nelson Textbook of Pediatrics, 17th
edition.)
D. Rectoperineal fistula
E. None of the above

Question . 35. A 9-yr-old boy complains of lower gastrointestinal bleeding and mucous
drainage. He appears well and denies nausea or vomiting, but notes occasional crampy
abdominal pain. Heart rate and blood pressure are normal, and rectal examination reveals
velvety polyps. The boy's father and grandfather have undergone colectomies as young
adults. The next appropriate step is:

A. Colonoscopy
Explanation: This child most likely has familial adenomatous polyposis coli and
is at risk for developing multiple polyps for the rest of his life; more serious is the
risk of developing colonic cancer within one or more of the polyps. The incidence
of familial adenomatous polyps is 1:8000 people. Patients require annual
colonoscopy. (See Chapter 326 in Nelson Textbook of Pediatrics, 17th edition.)
B. Proctocolectomy
C. COX-2 inhibitor therapy
D. CT scan
E. Clinical follow-up until age 25 yr

Question . 36. A 10-yr-old girl is brought for evaluation because her mother was found
to have familial adenomatous polyposis coli (APC). Genetic screening identified identical
APC gene mutations in mother, aunt, and grandfather, but no mutations were identified in
the girl's APC gene. The girl requires:

A. Colonoscopy
B. Proctocolectomy
C. Annual physical examination
Explanation: This autosomal dominant gene is responsible for familial
adenomatous polyps with a high risk of malignant transformation. If the gene is
known in the family and the patient does not have that gene, her risk is no greater
than that of the general population. (See Chapter 326 in Nelson Textbook of
Pediatrics, 17th edition.)
D. CT scan
E. Repeat genetic testing in 1 yr

The Digestive System Nelson Self Assessments website 17th Edition 12


Question . 37. A 5-yr-old girl with a history of heart transplant presents with nausea and
severe intermittent abdominal pain. CT scan is consistent with ileocolic intussusception.
The next step in her care is:

A. Hydrostatic reduction in radiology


B. Surgical exploration
Explanation: This child is at increased risk of intussusception from a specific
pathological lead point. Partly because of her age, but more so because she is at
increased risk for lymphoma or a lymphoproliferative disease, a hydrostatic
reduction may not be safe or successful and will not be diagnostic of malignancy.
(See Chapter 326 in Nelson Textbook of Pediatrics, 17th edition.)
C. Pain medication and observation in the hospital
D. Enteroclysis
E. Colonoscopy

Question . 38. A 4-yr-old boy presents with diarrhea and headaches. He appears to have
a limp, which favors the left leg. On examination, he has a poorly defined mass in the left
flank and a blood pressure of 170/100 mm Hg. Abdominal CT scan demonstrates a large
paraspinous mass on the left. The next step in the care of this boy is to:

A. Start therapy with a long-acting somatostatin analogue


B. Refer for immediate surgical exploration
C. Start -adrenergic blockade and obtain MRI scan
Explanation: This child has a catecholamine-secreting or other endocrinologically
active tumor. At his age and given the size of the tumor, it is most likely a
neuroblastoma. Pheochromocytomas are usually too small to be palpable. (See
Chapter 326 in Nelson Textbook of Pediatrics, 17th edition.)
D. Start -adrenergic blockade and obtain spine MRI scan
E. Refer to physical therapy

Question . 39. A 12-yr-old girl presents with severe crampy abdominal pain. The pain
resolves after 30 min in the emergency department. On examination, she is found to have
freckles on the lips, buccal mucosa, and gums. The abdomen is not tender on
examination. The next most appropriate step is to:

A. Discharge home with a diagnosis of constipation


B. Obtain upper and lower gastrointestinal contrast studies to rule out
intussusception
Explanation: The clinical picture suggests a complicated intussusception, perhaps
secondary to Peutz-Jeghers syndrome. This is an autosomal dominant disorder
that carries a high risk of eventual development of colonic cancer. (See Chapter
326 in Nelson Textbook of Pediatrics, 17th edition.)
C. Remove all polyps endoscopically to prevent cancer
D. Perform genetic screening
E. Perform proctocolectomy

The Digestive System Nelson Self Assessments website 17th Edition 13


Question . 40. A 15-yr-old boy presents with frequent episodes of flushing and watery
diarrhea. The laboratory test that would suggest the presence of a carcinoid tumor is
assay of:

A. Urinary catecholamines
B. Urinary 5-hydroxyindoleacetic acid (5-HIAA)
Explanation: Carcinoid is uncommon in children and may present as a
nonsecreting tumor or one with identical features to those in adults, as noted in
this case. (See Chapter 326 in Nelson Textbook of Pediatrics, 17th edition.)
C. Serum ACTH
D. Serum serotonin
E. Serum parathyroid hormone

Question . 41. A 4-yr-old boy in good health presents with a 3-day history of a tender
mass, 2 cm in diameter, midway between the umbilicus and the xyphoid process. He is
afebrile, is eating normally, and has an otherwise normal physical examination. The most
likely diagnosis is:

A. Metastatic neuroblastoma
B. Rhabdomyosarcoma of the abdominal wall
C. Incarcerated epigastric hernia
Explanation: This is a typical site for an epigastric hernia, which usually does not
contain bowel and is thus usually identified as an asymptomatic mass. (See
Chapter 354 in Nelson Textbook of Pediatrics, 17th edition.)
D. Lipoma
E. Infected dermoid cyst

Question . 42. Although the toxic dose of fluoride is many times greater than the dose
needed to cause mild fluorosis (mottling of dental enamel), the dose associated with
mottling in young infants is how many times the dose from fluoridated water?

A. 2-5 times
Explanation: The therapeutic window for mild mottling of teeth from fluoride is
narrow. (See Chapter 293 in Nelson Textbook of Pediatrics, 17th edition.)
B. 10-20 times
C. 100-200 times
D. 1000 times

The Digestive System Nelson Self Assessments website 17th Edition 14


Question . 43. A chronic or recurrent course of abdominal pain, weight loss, and anemia
in an adolescent girl suggests:

A. Henoch-Sch nlein purpura


B. Inflammatory bowel disease
Explanation: Inflammatory bowel disease is one of the most common causes of
significant chronic or recurrent abdominal pain in adolescents. Functional
abdominal pain (irritable bowel), which is part of the differential diagnosis, is
excluded by the presence of anemia and weight loss. (See Chapter 317 in Nelson
Textbook of Pediatrics, 17th edition.)
C. Anorexia nervos
D. Pelvic inflammatory disease
E. Irritable bowel disease

Question . 44. A 16-yr-old girl with a past history of hypothyroidism, which developed
at age 10 yr, now manifests fever, anorexia, amenorrhea, and jaundice of 4 months'
duration. The direct bilirubin level is 6 mg/dL, and the total bilirubin value is 11 mg/dL.
Results of the tests for hepatitis A, B, C, D, and E are negative, and the serum IgG level
is 16.5 g/L. The most likely diagnosis is:

A. Mononucleosis
B. Chronic active hepatitis
Explanation: Choices A, B, and C must be considered because each may produce
similar hepatic and extrahepatic manifestations. Chronic active hepatitis of the
autoimmune type (lupoid hepatitis) is often associated with other autoimmune
diseases (the patient described in the question had Hashimoto thyroiditis) and is
more prevalent in adolescent females. High titers of liver-kidney microsomal
antibodies are present. The antinuclear antibody response is also positive in many
patients. (See Chapter 343 in Nelson Textbook of Pediatrics, 17th edition.)
C. 1-Antitrypsin deficiency
D. Hepatoblastoma
E. Wilson disease

Question . 45. An 18-yr-old boy complains of right upper respiratory quadrant pain and
fever for 2 wk. Physical examination reveals hepatomegaly, no icterus, and right lower
quadrant fullness. Four weeks before admission, he returned from Mexico, where he
received an over-the-counter medication for an illness characterized by abdominal pain,
nausea, and emesis. The most likely diagnosis is:

A. Amebiasis
B. Hepatitis
C. Hepatic abscess (bacterial)
Explanation: In the patient described in the question, the hepatic abscess
developed as a complication of a partially treated ruptured appendix after acute

The Digestive System Nelson Self Assessments website 17th Edition 15


appendicitis. The over-the-counter medication was an oral antibiotic that
suppressed some of the signs of appendicitis. The right lower quadrant fullness is
a walled-off appendiceal abscess. This is treated with antibiotics and resected at a
later date. The hepatic abscess developed after septic embolization into the portal
vein (during the acute appendicitis) and, subsequently, the liver. The organisms
are usually enteric anaerobes in this form of liver abscess. Treatment includes
antibiotics (including metronidazole or clindamycin) and percutaneous drainage
under ultrasonographic or computed tomographic guidance. Amebiasis is in the
differential diagnosis. (See Chapter 334 in Nelson Textbook of Pediatrics, 17th
edition.)
D. Cholangitis
E. Crohn disease

Question . 46. A 10-yr-old girl who had biliary atresia treated with the Kasai procedure
in infancy now manifests increasing clumsiness, reduced deep tendon reflexes, and
ataxia. The most likely diagnosis is:

A. Hepatic encephalopathy
B. Vitamin A deficiency
C. Encephalitis
D. Vitamin E deficiency
Explanation: Vitamin E deficiency has a long latency (years) before it eventually
produces ataxia, posterior (spinal cord) column signs, and peripheral neuropathy.
Early treatment with water-soluble vitamin E may prevent and reverse these
neurologic processes. (See Chapter 337 in Nelson Textbook of Pediatrics, 17th
edition.)
E. Kernicterus

Question . 47. An 11-yr-old girl, Tanner stage 2, experiences intermittent periumbilical


abdominal pain 2 days before an emergency visit to her pediatrician. Six hours later, she
is nauseated and has one or two episodes of emesis. She also has had two soft bowel
movements without blood or relief of symptoms. She walks cautiously into your office
and lies still on your examining table. When you begin your examination, she is
apprehensive and watches every move of your examining hand. You notice guarding and
tenderness throughout her abdomen; the most tender area is the right lower quadrant. The
most likely diagnosis is:

A. Pelvic inflammatory disease


B. Ruptured ectopic pregnancy
C. Crohn disease
D. Appendicitis
Explanation: All of the choices in the question must be included in the differential
diagnosis. Pelvic inflammatory disease often presents with lower abdominal
suprapubic pain, vaginal discharge, and signs of peritoneal irritation. A ruptured
ectopic pregnancy would be unlikely, but not unheard of, in a girl at Tanner stage
2. Mesenteric adenitis may present in a manner similar to appendicitis

The Digestive System Nelson Self Assessments website 17th Edition 16


(pseudoappendicitis) and can be detected by abdominal ultrasonography or CT
scan. This patient has the characteristic appendicitis sequence of periumbilical
pain, followed more often by nausea and less often by emesis, followed by right
lower quadrant pain. The 2-day history is typical of appendicitis, as is the
apprehension about the examiner's hand during the abdominal palpation. (See
Chapter 324 in Nelson Textbook of Pediatrics, 17th edition.)
E. Mesenteric adenitis

Question . 48. A 5-yr-old girl with cirrhosis and portal hypertension experiences
increasing abdominal distention and fever. Shifting dullness and a puddle sign, as well as
abdominal tenderness, are noted on physical examination. Paracentesis reveals cloudy
fluid. Culture of the ascitic fluid is most likely to reveal:

A. Pseudomonas
B. Candida albicans
C. Pneumococci
Explanation: The pneumococcus is a common pathogen producing peritonitis
(primary) in any condition causing ascites (nephrosis, cirrhosis). Next in
frequency is Escherichia coli. (See Chapter 352 in Nelson Textbook of Pediatrics,
17th edition.)
D. Serratia
E. Haemophilus influenzae

Question . 49. Dysphagia is best described as:

A. Pain on swallowing
B. Emesis without nausea
C. Drooling due to obstruction
D. Difficulty swallowing
Explanation: Dysphagia is a sensation that swallowing is difficult because of
something being "stuck." Odynophagia is pain on swallowing. (See Chapter 287
in Nelson Textbook of Pediatrics, 17th edition.)
E. Reflux dyspepsia

Question . 50. All of the following statements regarding cyclic vomiting are true Except:

A. Onset occurs between ages 3 and 5 yr


B. Episodes last 2-3 days
C. There are four or more emesis episodes per hour
D. It is a migraine equivalent
E. It is associated with nystagmus
Explanation: A-D are all correct. Nystagmus should suggest serious central
nervous system disease or vestibular disease. (See Chapter 287 in Nelson
Textbook of Pediatrics, 17th edition.)

The Digestive System Nelson Self Assessments website 17th Edition 17


Question . 51. Palatopharyngeal incompetence is characterized by all of the following
Except:

A. Hypernasal speech
B. Presence of a submucosal cleft
C. Difficulty in pronouncing p, b, d, t, h, v, f, s
D. Improvement after adenoidectomy
Explanation: This is quite false. Indeed, palatopharyngeal incompetence may first
become evident or be exacerbated by adenoid removal. (See Chapter 291 in
Nelson Textbook of Pediatrics, 17th edition.)
E. Difficulty whistling

Question . 52. After blunt facial trauma, a 12-yr-old appears to have a missing incisor.
There is no evidence of the tooth at the scene, and the child does not remember
swallowing the tooth. The next step in evaluating the child is to:

A. Plan for a bridge


B. Obtain an abdominal x-ray study to search for the avulsed tooth
C. Obtain a dental film to look for intrusion
Explanation: Intrusion of a tooth into the maxillary bone with the appearance of
avulsion may be a common problem in children. (See Chapter 295 in Nelson
Textbook of Pediatrics, 17th edition.)
D. Placement of a ceramic tooth
E. Administration of penicillin to prevent facial cellulitis

Question . 53. Acquired causes of gastric outlet obstruction include all of the following
Except:

A. Cystic fibrosis
Explanation: B-E are all correct. (See Chapter 310 in Nelson Textbook of
Pediatrics, 17th edition.)
B. Epidermolysis bullosa
C. Prostaglandin E infusions
D. Eosinophilic gastritis
E. Crohn disease

The Digestive System Nelson Self Assessments website 17th Edition 18


Question . 54. A 5-yr-old boy has a history of chronic recurrent abdominal pain and
emesis. A diagnosis of cyclic vomiting was made at 3 yr of age. Therapy with antimigraine
medications was unsuccessful. During each episode, the serum ammonia, pH, glucose, and
bicarbonate levels were normal. He now manifests bile-stained emesis, a tender distended
abdomen, and bright-red blood per rectum. The most likely diagnosis is:

A. Stress ulcers
B. Intussusception
C. Malrotation
Explanation: Malrotation, and in this patient volvulus (a bowel and life-
threatening complication), usually presents with bowel obstruction before 1 yr of
age. The oldest reported patient, however, was over 80 yr. Most cases present
before age 3 yr. Superior mesenteric artery obstruction of the intestines may also
produce bile-stained emesis; however, intestinal infarction is not observed. (See
Chapters 311 and 313 in Nelson Textbook of Pediatrics, 17th edition.)
D. Superior mesenteric artery syndrome
E. Meconium ileus equivalent

Question . 55. All of the following statements regarding Hirschsprung disease are true
Except:

A. It is uncommon in preterm infants


B. Males are affected more commonly than females
C. It is associated with VATER syndrome
Explanation: Imperforate anus is associated with VATER syndrome.
Hirschsprung disease is the most common anatomic cause of lower intestinal
obstruction in neonates. (See Chapter 313 in Nelson Textbook of Pediatrics, 17th
edition.)
D. It is associated with trisomy 21
E. Breast-fed infants may appear less ill than formula-fed infants

Question . 56. An 18-mo-old white boy manifests failure to thrive, poor appetite,
abdominal distention, diarrhea, and irritability. He had been well until 9 mo of age.
Thereafter, he was weaned from breast milk to regular foods. His growth curve is
flattening. The most likely diagnosis is:

A. Giardiasis
B. Celiac disease
Explanation: Imperforate anus is associated with VATER syndrome.
Hirschsprung disease is the most common anatomic cause of lower intestinal
obstruction in neonates. (See Chapter 313 in Nelson Textbook of Pediatrics, 17th
edition.)
C. Cystic fibrosis
D. Pancreatitis
E. Mitochondrial myopathy

The Digestive System Nelson Self Assessments website 17th Edition 19


Question . 57. The disease in Question 56 is also noted with higher prevalence in
patients with all of the following conditions Except:

A. IgA deficiency
B. Down syndrome
C. Juvenile rheumatoid arthritis
D. Diabetes mellitus
E. Systemic lupus erythematosus
Explanation: SLE is not a predisposing condition for celiac disease. (See Chapter
320 in Nelson Textbook of Pediatrics, 17th edition.)

Question . 58. The best diagnostic serum test for the disease in Question 56 is:

A. Antinuclear antibody (ANA) assay


B. IgA endomysial antibody assay
Explanation: This is the serum diagnostic test of choice and is almost 100%
sensitive and specific, except in IgA-deficient patients. (See Chapter 320 in
Nelson Textbook of Pediatrics, 17th edition.)
C. IgA antigluten antibody assay
D. IgA antigliadin antibody assay
E. Antineutrophil cytoplasmic antibody assay

Question . 59. Wilson disease is associated with all of the following Except:

A. Cardiomyopathy
Explanation: B-F represent the hepatic, neurologic, and renal (plus hematologic)
findings. Ocular disease is detected by Kayser-Fleischer corneal rings, which may
be absent in younger patients, who usually manifest hepatic disease. (See Chapter
338 in Nelson Textbook of Pediatrics, 17th edition.)
B. Hepatomegaly
C. Ascites and portal hypertension
D. Dystonia and tremor
E. Hemolysis
F. Fanconi syndrome

Question . 60. An 18-mo-old is discovered with his mouth over a storage bottle
containing a strong alkali. The parents remove the bottle, and the boy seems well. Some
fluid is missing from the bottle, but no external signs are found on the child's clothing,
and the child has no burns on his face or his lips. The most appropriate advice to give the
parents, who are on their way to the hospital, is to:
A. Administer ipecac
B. Administer milk
Explanation: Administration of milk helps calm a child and helps dilute the alkali
in the esophagus and stomach. (See Chapter 308)
C. Administer toast
D. Administer acetaminophen

The Digestive System Nelson Self Assessments website 17th Edition 20


Question . 61. After the child described in Question 60 arrives in the emergency
department, results of his physical examination, including an examination of his posterior
pharynx, are found to be unremarkable.
The most appropriate next step in management is to:

A. Administer prednisone to decrease stricture formation


B. Administer penicillin to prevent infection
C. Administer an acidic fluid to neutralize the alkali
D. Perform endoscopy to assess the severity of the ingestion
Explanation: Endoscopy is indicated to assess the severity of inflammation and
necrosis. Even without oral lesions, significant esophageal involvement is
possible. Prednisone is of no value in preventing strictures. (See Chapter 308 in
Nelson Textbook of Pediatrics, 17th edition.)
E. Place a nasogastric tube to feed the child

Question . 62. A 4-yr-old boy has had vomiting and diarrhea for 4 days. He now has had
24 hr of abdominal pain that appears maximal in the lower abdomen. His temperature is
38.4°C; WBC count, 9,400/cu mm with 75 PMNs and 20 lymphocytes; hematocrit, 36;
and platelet count, 160,000/cu mm. Urinalysis is normal, with the exception of 3+
ketonuria. He has mild tenderness in both lower quadrants, normal rectal examination,
and mild muscular guarding in the lower quadrants. The most likely diagnosis is:

A. Intussusception
B. Viral gastroenteritis
C. Appendicitis
Explanation: Appendicitis does not always follow the classic pattern, particularly
in younger children and in those with an appendix in an unusual place. When in
doubt, abdominal imaging should be performed. Imaging is most useful in
identifying other causes of pain, such as mesenteric adenitis and gynecologic
pathology in females. (See Chapter 325 in Nelson Pediatrics, 17th edition.)
D. Bacterial enteritis
E. Henoch-Sch nlein purpura

Question . 63. For the patient described in Question 62, the most appropriate next step in
management would be:

A. CT scan
Explanation: Abdominal CT scanning is the imaging study of choice but is not
necessary for all patients with appendicitis. Abdominal ultrasonography has been
useful, but CT has shown superior results. (See Chapter 325 in Nelson Textbook
of Pediatrics, 17th edition.)
B. Barium enema
C. Laparotomy
D. Stool culture
E. Peripheral smear

The Digestive System Nelson Self Assessments website 17th Edition 21


Question . 64. A 5-yr-old boy underwent open appendectomy for perforated appendicitis
4 days ago. He continues to be febrile with temperatures to 38.8°C but is tolerating a
regular diet and has two loose stools per day. The WBC count is 9,400/cu mm, with 75
PMN and 20 lymphocytes; hematocrit, 36; and platelet count, 160,000/cu mm. The most
appropriate next step in patient management would be to:

A. Continue intravenous antibiotics


Explanation: At this point, he may have local peritonitis or may be developing an
intra-abdominal abscess. If fever continues despite administration of antibiotics,
or if localizing signs develop, a CT scan is definitely indicated to look for an
abscess. (See Chapter 335 in Nelson Textbook of Pediatrics, 17th edition.)
B. Perform fecal leukocyte examination
C. Order a CT scan
D. Order an ultrasound examination
E. Begin oral antibiotics

Question . 65. A 3.5-kg girl is delivered by spontaneous vaginal delivery, with Apgar
scores of 2 and 3. Respiratory distress is apparent in the delivery room, with diminished
breath sounds on the left and heart tones displaced to the right. The abdomen is scaphoid.
The most appropriate next step in treatment is to:

A. Administer sodium bicarbonate, 3.5 mEq


B. Perform left tube thoracostomy
C. Perform emergency laparotomy
D. Perform endotracheal intubation
Explanation: This is a classic presentation for a left-sided diaphragmatic hernia.
This infant needs to be resuscitated immediately and have the airway protected
and ventilation begun. Mask-bag ventilation would increase gas in the intestines,
which could increase the mass effect in the left thorax. Therefore, immediate
endotracheal intubation is indicated. (See Chapter 353 in Nelson, 17th edition.)
E. Perform umbilical artery catheterization

Question . 66. A 2.9-kg girl is born at term after having carried a prenatal diagnosis of
left congenital diaphragmatic hernia since 19 wk of gestation. Delivery and the early
postnatal period are uneventful, with mild tachypnea and retractions developing at 24 hr
of age. Chest film reveals a normal abdominal gas pattern and multiple lucent areas in the
left lower thorax. The most likely diagnosis is:

A. Congenital diaphragmatic hernia (Bochdalek)


B. Congenital diaphragmatic hernia (Morgagni)
C. Congential cystic adenomatoid malformation
Explanation: Congenital cystic adenomatoid malformation (CCAM) is in the
differential diagnosis of congenital diaphragmatic hernia. (See Chapter 353 in
Nelson Textbook of Pediatrics, 17th edition.)
D. Congenital diaphragmatic hernia (hiatal)
E. Cystic fibrosis

The Digestive System Nelson Self Assessments website 17th Edition 22


Question . 67. A 4-yr-old boy has a history of intermittent non-bilious vomiting since
infancy. He has remained below the 5th percentile in weight, but has been at the 50th
percentile for height and head circumference. He was evaluated for suspected pneumonia
with a chest film, which demonstrated a single gas bubble in the medial aspect of the left
lower hemithorax. The most likely diagnosis is:
A. Congenital diaphragmatic hernia
B. Congenital diaphragmatic hernia (Morgagni)
C. Congential cystic adenomatoid malformation
D. Congenital diaphragmatic hernia (hiatal)
Explanation: This anomaly may be confused with reflux or gastritis. It requires
surgical repair. (See Chapter 353 in Nelson Textbook of Pediatrics, 17th edition.)
E. Cystic fibrosis

Question . 68. A 29-yr-old woman is identified as carrying a fetus with a right congenital
diaphragmatic hernia. The gestation is 23 wk, and an ultrasound study at 16 wk did not
reveal the lesion. There is no evidence of fetal hydrops, the liver and stomach appear to
be in the abdomen, and the lung-to-head ratio is reported as 1.6. Which of the following
is most likely to occur in this situation?

A. Fetal demise
B. Premature delivery
C. Successful repair of fetal hernia
Explanation: In children with congenital diaphragmatic hernia (CDH) and no
other anomalies, fetal survival is quite good. Fetuses with other anomalies have a
high intrauterine fetal demise rate, the hidden mortality of CDH. Group B
streptococcal sepsis is, for some unknown reason, associated with right-sided
CDH. (See Chapter 353)
D. Severe pulmonary hypertension in fetus
E. Group B sepsis in fetus

Question . 69. A 16-yr-old boy is being evaluated for an incidentally documented


increase in alanine aminotransferase (ALT). Over the past 6 mo the ALT level has
fluctuated between 100 and 200 IU/L. Diagnostic evaluations to date include positive
results on hepatitis A IgG assay and hepatitis C ELISA and an anti-liver-kidney
microsomal antibody titer of 1:80. The most appropriate next diagnostic test would be:

A. Hepatitis A IgM assay


B. Hepatitis C reverse transcriptase-polymerase chain reaction (RT/PCR) assay
Explanation: Testing for chronic active hepatitis with LKM antibodies in patients
may result in a false-positive test result, incorrectly suggesting a diagnosis of an
autoimmune process. RT/PCR is the best method to confirm the diagnosis of
hepatitis C. It is important to document persistent hepatitis C infection, as
chronicity is associated with the risk of cirrhosis. (See Chapter 343)
C. Quantitative immunoglobulin level
D. Prothrombin time
E. Serum creatinine level

The Digestive System Nelson Self Assessments website 17th Edition 23


Question . 70. A 12-yr-old girl presented with nonspecific abdominal pain and
hepatosplenomegaly and was found to have an ALT of 185 IU/L; total bilirubin, 2.4
mg/dL; alkaline phosphatase, 640 IU/L; and an anti-smooth muscle antibody titer, 1:80.
Liver biopsy revealed a lymphoplasmacytic infiltrate of the portal tracts. She has been
treated for 4 mo with prednisone without significant improvement. The next most
appropriate diagnostic study would be:

A. Hepatitis C reverse transcriptase-polymerase chain reaction (RT/PCR) assay


B. Repeat liver biopsy
C. Abdominal ultrasound examination with Doppler interrogation
D. Endoscopic retrograde cholangiopancreatography (ERCP)
Explanation: There are clinical similarities between autoimmune hepatitis and
sclerosing cholangitis in children. This child could also have chronic viral
infection, which does not respond to prednisone but requires interferon and
antiviral drugs. (See Chapter 343 in Nelson Textbook of Pediatrics, 17th edition.)
E. Slit-lamp examination

Question . 71. On screening, a 6-yr-old adopted child has the following biochemical and
serologic findings: ALT, 335 IU/L; positive HBsAg; positive HBcAb; and positive
HBeAg. Which of the following statements is true about interferon therapy?

A. A realistic goal of therapy is to reduce the risk of the development of


hepatocellular carcinoma
Explanation: This child has evidence of being actively infected with hepatitis B
virus, which has a high risk of subsequent hepatocellular cancer. Specific
treatment reduces this risk. (See Chapter 343 in Nelson, 17th edition.)
B. A realistic goal of therapy is to induce hepatitis B surface antibodies
C. Therapy is not approved and is not indicated
D. Duration of therapy is typically 4 wk
E. Therapy is typically not associated with significant side effects

Question . 72. A 9-yr-old girl presented with persistent fatigue and hepatomegaly and
was found to have an ALT of 275 IU/L; total bilirubin, 1.2 mg/dL; alkaline phosphatase,
265 IU/L: and antinuclear antibody titer, 1:80. Liver biopsy revealed a
lymphoplasmacytic infiltrate of the portal tracts. The most appropriate initial therapy
would be:

A. 50 mg of azathioprine (Imuran) daily


B. 50 mg of azathioprine (Imuran) on alternate days
C. 40 mg of prednisone daily
Explanation: Daily therapy with corticosteroids is essential for initial management
of autoimmune hepatitis. Patients with chronic active autoimmune hepatitis
respond quite well to daily prednisone. This is also called lupoid hepatitis. (See
Chapter 343 in Nelson Textbook of Pediatrics, 17th edition.)
D. 40 mg of prednisone on alternate days
E. 40 mg of prednisone and 50 mg of azathioprine (Imuran), both on alternate days

The Digestive System Nelson Self Assessments website 17th Edition 24


Question . 73. A 5-wk-old white boy was noted to be jaundiced. The patient had been
breast-fed since birth. The stools were described by the mother to be yellow in color. The
patient had a normal physical examination. Total bilirubin was 11 mg/dL. The mother
was concerned that the baby might have biliary atresia. The most reasonable initial step is
to obtain the following diagnostic test:

A. Abdominal ultrasound examination


B. Hepatobiliary scintigraphy
C. Percutaneous liver biopsy
D. Urine culture
E. Total and direct bilirubin
Explanation: First and foremost, the hyperbilirubinemia must be characterized as
prehepatic (indirect) (e.g., hemolytic, breast milk, physiologic, Gilbert disease) or
cholestatic. Biliary atresia usually causes pale stools and hepatomegaly and is
always accompanied by cholestatic jaundice. (See Chapter 337 in Nelson
Textbook of Pediatrics, 17th edition.)

Question . 74. A 3 yr-old white girl was referred to a pediatric gastroenterologist because
of elevated levels of liver enzymes and total and direct bilirubin. The patient has a history
of renal tubular acidosis and peripheral pulmonic stenosis. On examination, the patient
was noted to have a broad forehead and deep-set, widely spaced eyes. Scratch marks
were seen on the skin secondary to pruritus. A liver biopsy will most likely show:

A. Bile duct proliferation


B. Giant cell transformation
C. Bile duct paucity
Explanation: This is a classic presentation of a bile duct paucity syndrome, such
as Alagille syndrome. Pruritus is secondary to poor excretion of bile salts, which
induce itching when deposited in the skin. (See Chapter 337 in Nelson Textbook
of Pediatrics, 17th edition.)
D. PAS-positive diastase-resistant globules
E. No changes

The Digestive System Nelson Self Assessments website 17th Edition 25


Diseases of the Blood
Nelson Self Assessments website 17th Edition

Question . 1. All of the following statements are true Except:

A. The anatomic site of hematopoiesis changes during gestation and the


population of cells generated at those sites are distinct
B. Few neutrophils are found in the fetal circulation until the third trimester
C. Thrombopoietin is the physiologic regulator of platelet production but does
not act as a potent stimulator of all stages of megakaryocyte growth and
development
Explanation: (See Chapter 438 in Nelson Textbook of Pediatrics, 17th ed.)
D. Erythropoiesis in utero is controlled by erythroid growth factors produced
solely by the fetus
E. Some HbA can be detected in even the smallest embryos

Question . 2. All of the following statements about anemia are true Except:

A. In anemia with an appropriate reticulocyte response, the cause usually is a


consequence of bleeding or ongoing hemolysis
B. Transient erythroblastopenia of childhood is the most common acquired red
cell aplasia occurring in children
C. Recovery from severe anemia due to parvovirus B19 infection is usually
spontaneous
D. The major component of anemia in chronic renal disease is decreased
erythropoietin production due to damage of renal tubular cells
Explanation: (See Chapter 439 and 443 in Nelson Text. Pediatrics, 17th ed.)
E. In congenital dyserythropoietic anemia type 1, the onset of anemia and/or
jaundice may occur at any age.

Question . 3. All of the following statements regarding iron deficiency are true
Except:

A. Because absorption of dietary iron is assumed to be about 10%, a diet


containing 80-100 mg of iron daily is necessary for optimal nutrition
Explanation: (See Chapter 447 in Nelson Textbook of Pediatrics, 17th ed.)
B. Intense exercise conditioning may result in iron depletion in adolescent girls
C. C.Iron deficiency may have effects on neurologic and intellectual function
D. The level of serum ferritin provides a relatively accurate estimate of body iron
stores in the absence of inflammatory disease
E. The red cell distribution width (RDW) is elevated in iron deficiency but not in
and thalassemia trait

The Diseases of the Blood Nelson Self Assessments website 17th Edition 1
Question . 4. All of the following statements are true Except:

A. Approximately 10% of RBCs are normally removed each day and replaced by
the marrow to maintain the RBC count
Explanation: (See Chapter 449 in Nelson Textbook of Pediatrics, 17th ed.)
B. When the capacity of the heme-binding proteins in the plasma is exceeded,
free hemoglobin appears in the plasma
C. The most common cause of aplastic crisis is parvovirus B19 infection
D. The marrow can increase its output of RBCs two- to threefold acutely
E. The usual marrow response to a chronic hemolytic anemia is reflected by a
reticulocyte index of 3-4

Question . 5. All of the following statements regarding hereditary spherocytosis are


true Except:

A. Individuals with hereditary spherocytosis may be asymptomatic without


anemia
B. The newborn with hereditary spherocytosis may present with anemia and
hyperbilirubinemia severe enough to require phototherapy and exchange
transfusions
C. Isoimmune hemolytic anemia due to ABO incompatibility may mimic
hereditary spherocytosis
D. Thermal injury can cause spherocytosis
E. Splenectomy does not eliminate most of the hemolysis of hereditary
spherocytosis
Explanation: (See Chapter 450 in Nelson Textbook of Pediatrics, 17th ed.)

Question . 6. All of the following statements regarding elliptocytosis are true Except:

A. Hereditary elliptocytosis is inherited as a dominant disorder.


B. Hereditary elliptocytosis may produce neonatal jaundice even though
characteristic elliptocytosis may not be evident at the time
C. The most severe form of hereditary elliptocytosis, hereditary
pyropoikilocytosis (HPP), is characterized by macrocytosis
Explanation: (See Chapter 451 in Nelson Textbook of Pediatrics, 17th ed.)
D. No treatment is necessary unless hemolysis is present
E. Patients with chronic hemolysis should receive folic acid to prevent secondary
folic acid deficiency

Question . 7. All of the following statements about paroxysmal nocturnal


hemoglobinuria (PNH) are true Except:

A. Bone marrow failure is a rare presentation of PNH


Explanation: (See Chapter 453 in Nelson Textbook of Pediatrics, 17th ed.)
B. Chronic hemolysis is more common than nocturnal and morning
hemoglobinuria in patients with PNH
C. Thrombosis and thromboembolic phenomena are serious complications of
PNH
D. Flow cytometry is the best diagnostic test for PNH
E. Splenectomy is not indicated for PNH

The Diseases of the Blood Nelson Self Assessments website 17th Edition 2
Question . 8. All of the following statements are true Except:

A. In patients who have Hb SS electrophoresis pattern and concomitant


microcytosis, iron deficiency or a combination of Hb S with or thalassemia
must be considered a possible diagnosis
B. Febrile infants with sickle cell disease should be managed in the hospital
C. The iron in hemoglobin is normally in the ferric state, which is essential for
oxygen transport
Explanation: (See Chapter 454 in Nelson Textbook of Pediatrics, 17th ed.)
D. Children with homozygous 0 thalassemia usually become symptomatic from
progressive hemolytic anemia during the second 6 mo of life if not treated
E. The thalassemia traits present as microcytic anemia, which can be mistaken
for iron-deficiency anemia

Question . 9. All of the following statements regarding pyruvate kinase (PK)


deficiency are true Except:

A. PK deficiency is the most common glycolytic enzyme deficiency


B. Severe jaundice and anemia may occur during the neonatal period
C. Splenectomy for severe PK deficiency is curative
Explanation: (See Chapter 455 in Nelson Textbook of Pediatrics, 17th ed.)
D. Nonincubated osmotic fragility is normal in PK deficiency
E. Spherocytes are common in PK deficiency

Question . 10. All of the following statements regarding G6PD deficiency are true
Except:

A. Symptoms usually develop in patients with G6PD deficiency 24-48 hr after


ingesting a substance with oxidative properties
B. Infection may result in hemolysis in patients with G6PD deficiency
C. A pregnant woman who ingests oxidative drugs may cause hemolytic anemia
in a fetus with G6PD deficiency
D. Enzyme activity in affected persons is 10% of normal or less
E. The usual dose of aspirin causes clinically relevant hemolysis in the A variety
of G6PD deficiency
Explanation: (See Chapter 455 in Nelson Textbook of Pediatrics, 17th ed.)

Question . 11. All of the following may cause autoimmune hemolytic anemia Except:

A. Epstein-Barr virus
B. Systemic lupus erythematosus
C. Agammaglobulinemia
D. Methyldopa
E. Penicillin
Explanation: (See Chapter 456 in Nelson Textbook of Pediatrics, 17th ed.)

The Diseases of the Blood Nelson Self Assessments website 17th Edition 3
Question . 12. All of the following statements are true Except:

A. In most instances of warm antibody hemolysis, no underlying cause is found


B. The prognosis for patients with Evans syndrome (immune thrombocytopenic
purpura) is good after the acute episode
Explanation: (See Chapter 456 in Nelson Textbook of Pediatrics, 17th ed.)
C. The hallmark of the autoimmune hemolytic anemias is a positive result on the
direct Coombs test
D. Cold antibodies are primarily of the IgM class and require complement for
activity
E. Paroxysmal cold hemoglobinuria may account for 30% of immune hemolytic
episodes among children

Question . 13. Hemolytic anemia may be caused by any of the following Except:

A. Extensive burns
B. Renal disease
C. Liver disease
D. Wilson disease
E. Hypopituitarism
Explanation: (See Chapter 457 in Nelson Textbook of Pediatrics, 17th ed.)

Question . 14. All of the following may cause secondary polycythemia Except:

A. Twin-twin hemorrhage
B. High altitude
C. Plasma volume decrease
Explanation: (See Chapter 459 in Nelson Textbook of Pediatrics, 17th ed.)
D. Methemoglobin reductase deficiency
E. Anabolic steroid therapy

Question . 15. All of the following may cause splenomegaly Except:

A. Endocarditis
B. Malaria
C. Gaucher disease
D. Polycythemia vera
E. Progeria
Explanation: (See Chapter 478 in Nelson Textbook of Pediatrics, 17th ed.)

Question . 16. Splenic hypofunction is a usual finding in all of the following except

A. Premature infants
B. Sickle cell disease
C. Congenital polyspenia
Explanation: (See Chapter 479 in Nelson Textbook of Pediatrics, 17th ed.)
D. Severe hemolytic anemia
E. Metabolic storage diseases with splenomegaly

The Diseases of the Blood Nelson Self Assessments website 17th Edition 4
Question . 17. Common causes of generalized lymphadenopathy incude all of the
following Except:

A. Mononucleosis
B. Niemann-Pick disease
C. Leukemia
D. Cat scratch disease
Explanation: (See Chapter 482 in Nelson Textbook of Pediatrics, 17th ed.)
E. Serum sickness

The Diseases of the Blood Nelson Self Assessments website 17th Edition 5
Bone and Joint Disorders
Nelson Self Assessments website 17th Edition

Question . 1. Metatarsus adductus is associated with all of the following Except:

A. Out-toeing
Explanation: This condition gives the appearance of intoeing. It may be due to
deformational forces in utero hence the association with being firstborn and
with hip dysplasias. (See Chapter 664 in Nelson Textbook Pediatrics, 17th ed.)
B. Bilaterality in 50%
C. More common in firstborn
D. Hip dysplasia in 10%
E. Nonoperative treatment

Question . 2. The risk of congenital talipes equinovarus (clubfoot) in the offspring of


a parent so affected is:

A. 100%
B. 3-5%
C. 20-30%
Explanation: The congenital clubfoot is defined as (1) absence of other
congenital anomalies, (2) variable rigidity of the foot, (3) mild calf atrophy,
and (4) mild hypoplasia of the tibia, fibula, and foot bones. Recurrence in
siblings is 3%; in a family with an affected parent it is 20-30%. (See Chapter
664 in Nelson Textbook of Pediatrics, 17th ed.)
D. 0
E. 50% if the involved parent is female

Question . 3. The cavus foot is associated with all of the following Except:

A. Hindfoot varus position


B. Cardiomyopathy
Explanation: Idiopathic or neuromuscular disorders are the common causes of
cavus feet. (See Chapter 664 in Nelson Textbook of Pediatrics, 17th ed.)
C. Charcot-Marie-Tooth disease
D. Spinal cord disorders
E. Peripheral neuropathy

Question . 4. A 10-yr-old boy stubs his toe while walking barefoot. The toe is tender,
with bleeding from the nail fold. This child is at risk for:
A. Endocarditis
B. Osteomyelitis
Explanation: Bleeding from the nail bed suggests an open fracture (Salter-
Harris type 1) and places the patient at risk for osteomyelitis. In the absence of
infection, the toe should heal quite well without deformity or limitation of
growth. (See Chapter 664 in Nelson Textbook of Pediatrics, 17th ed.)
C. Poor growth of the toe
D. Anemia
E. Exostosis

Bone and Joint Disorders - Nelson Self Assessments website 17th Edition 1
Question . 5. Internal femoral torsion is characterized by all of the following Except:

A. Most common cause of intoeing in children older than 2 yr of age


B. More common in boys
Explanation: This disorder is more common in girls with ligamentous laxity
who sit on the floor watching television in the "W" position (hips drawn up
with legs underneath the buttocks). With time and correct sitting, the condition
will resolve. (See Chapter 665 in Nelson Textbook of Pediatrics, 17th ed.)
C. Associated with ligamentous laxity
D. Preference for sitting in a "W" position among affected children
E. Limited external rotation to 0-10 degrees

Question . 6. Associated conditions in infants with developmental dysplasia of the hip


include all of the following Except:

A. Breech position
B. Metatarsus adductus
C. Torticollis
D. Ligamentous laxity
E. Muscular dystrophy in males
Explanation: DDH is most often seen in females and may be due to in utero
position and laxity of ligaments. (See Chapter 668 in Nelson Textbook of
Pediatrics, 17th ed.)

Question . 7. Legg-Calvé-Perthes disease is associated with all of the following


Except:

A. Female gender
Explanation: The male to female ratio is approximately 5:1. (See Chapter 668
in Nelson Textbook of Pediatrics, 17th ed.)
B. Thrombophilia
C. Bilateral involvement in 20%
D. Age at diagnosis of 7 yr
E. A painless limp

Question . 8. A 14-yr-old tall afebrile girl presents with hip pain and an extremely
rotated right leg. On examination, internal rotation is limited. The most likely
diagnosis is:

A. Legg-Calvé-Perthes disease
B. Diskitis
C. Slipped capital femoral epiphysis
Explanation: External rotation, severe pain, and tall stature in an adolescent
are important clues. In addition to pain, limitation of internal rotation of the
leg is classic. (See Chapter 668 in Nelson Textbook of Pediatrics, 17th ed.)
D. Osgood-Schlatter disease
E. Toxic synovitis

Bone and Joint Disorders - Nelson Self Assessments website 17th Edition 2
Question . 9. A 15-yr-old obese girl presents with a history of nontraumatic knee
pain. On examination, there is full range of motion of the knee without swelling or
tenderness. There is decreased hip rotation. The most likely diagnosis is:

A. Osteogenic sarcoma
B. Ewing sarcoma
C. Osteonecrosis of the femoral head
D. Slipped capital femoral epiphysis
Explanation: Be wary of referred knee pain from hip disease. Always examine
the entire extremity and especially the hip in patients who complain of knee
pain but have no demonstrable knee tenderness. (See Chapter 668 in Nelson
Textbook of Pediatrics, 17th ed.)
E. Blount disease

Question . 10. Congenital scoliosis is associated with all of the following Except:

A. Genitourinary anomalies
B. Congenital heart disease
C. Extradural lipomas
D. Tethered spinal cords
E. Alagille syndrome
Explanation: Spinal dysraphism is very common in children with congenital
scoliosis and must be investigated with MRI. (See Chapter 669 in Nelson
Textbook of Pediatrics, 17th ed.)

Question . 11. A 12-yr-old gymnast presents with back pain, reduced lumbar lordosis,
and sacral kyphosis. The buttocks are flattened, and a step-off is felt in the
lumbosacral region. Findings on neurologic examination are normal. The most likely
diagnosis is:

A. Spondylolisthesis with spondylolysis


Explanation: This patient had a severe grade of slippage and required spinal
fusion to prevent progressive deformity (See Chapter 669 in Nelson 17th ed.)
B. Spondylolysis
C. Tethered cord
D. Spina bifida occulta
E. Diskitis

Question . 12. Sports participation is permitted for patients with all of the following
conditions Except:

A. Cerebral palsy
B. One functional eye
C. One testis
D. Fever
Explanation: An acute febrile illness is a risk for dehydration and
hyperthermia. Relative contraindications to sports participation include acute
hepatomegaly or splenomegaly, certain dysrhythmias, myocarditis, and
bleeding disorders. (See Chapter 675 in Nelson Pediatrics, 17th ed.)
E. One kidney

Bone and Joint Disorders - Nelson Self Assessments website 17th Edition 3
Question . 13. The best approach to management of chronic tennis elbow is:

A. Rest
B. Use of wrist splints
C. Steroid injections
D. Physiotherapy
Explanation: In the long run, physiotherapy is better than steroid injections.
(See Chapter 676 in Nelson Textbook of Pediatrics, 17th ed.)
E. Nonsteroidal anti-inflammatory agents

Question . 14. The patellofemoral pain syndrome is characterized by all of the


following Except:

A. Worsening of pain on going up stairs


B. "Giving way" and then falling
Explanation: Giving way of this magnitude is most often seen with meniscal
or ligament tears and not with patellofemoral pain syndromes. (See Chapter
676 in Nelson Textbook of Pediatrics, 17th ed.)
C. Chronic anterior knee pain
D. Worsening of pain with prolonged sitting
E. Presence of peripatellar tenderness

Question . 15. Rehabilitation of an ankle injury includes all of the following Except:

A. Prolonged immobilization
Explanation: Immobilization may produce further atrophy and weakness.
Taping the ankle has little value. The 5-hop test demonstrates that the athlete
can hop as high on the previously injured side as on the well side. When this
happens, the athlete is ready to participate in sports. (See Chapter 676)
B. The five-hop test
C. Restoration of peroneal muscle strength
D. Reduce vulnerability to re-injury
E. Use of ankle braces

Question . 16. Mutations of cartilage mature proteins cause disease by:

A. Autosomal recessive inheritance


B. Dominant negative mechanisms
Explanation: Type II collagen is a triple helix of 3 collagen chains produced by
the two inherited type II collagen genes. In this autosomal dominant pattern of
inheritance, if one of the three collagen chains is abnormal, the net result will be a
functionally abnormal triple helix: one abnormal chain causes dysfunction of the
entire collagen triple helix. (See Chapter 682 in Nelson Textbook of Pediatrics,
17th ed.)
C. Mitochondrial inheritance
D. Imprinting
E. Paternal disomy

Bone and Joint Disorders - Nelson Self Assessments website 17th Edition 4
Question . 17. Osteogenesis imperfecta is characterized by all of the following Except:

A. Defects in type I collagen


B. Absence of lethal variants
Explanation: With OI type II, the affected child may be stillborn or may die in
the first year of life. There is severe rib involvement, which contributes to
respiratory failure. (See Chapter 689 in Nelson Pediatrics, 17th ed.)
C. Autosomal dominant inheritance
D. Vertebral compression fractures
E. Long bone fractures

Question . 18. Treatment of osteogenesis imperfecta is not curative. Nonetheless,


improvement with decreased bone fractures and pain has been reported with:

A. Vitamin D
B. Calcium
C. Pamidronate
Explanation: The bisphosphates (pamidronate and alendronate) have shown
promising results in children with OI. (See Chapter 689 in Nelson, 17th ed.)
D. Calcitonin
E. Fluoride

Question . 19. Major criteria in the diagnosis of Marfan syndrome include all of the
following Except:

A. Apical blebs or pneumothorax


Explanation: This is a minor criterion. Additional minor criteria include joint
hypermotility, pectus excavatum of mild to moderate severity, high-arched
palate, myopia, and mitral valve prolapse. (See Chapter 690 in Nelson
Textbook of Pediatrics, 17th ed.)
B. Pectus carinatum requiring surgery
C. Pectus excavatum requiring surgery
D. Ectopia lentis
E. Aortic dilation
F. Dural ectasia in lumbosacral spine detected by MRI or CT

Question . 20. A 20-mo-old female infant presents with disproportionately short


limbs, long trunk, and large head. An examination shows midface hypoplasia, mild
hypotonia of the lower extremities, and exaggerated lumbar lordosis. Which of the
following is the most appropriate diagnostic test for this patient?

A. MRI of lumbar spine


B. Plain films of the skeleton
Explanation: The initial evaluation of a child with a skeletal dysplasia is to
define the skeletal abnormalities and the degree of involvement of trunk or
distal versus proximal shortening or bowing. (See Chapter 682 in Nelson
Textbook of Pediatrics, 17th ed.)
C. Analysis of DNA for FGFR3 mutations
D. Iliac crest growth plate biopsy
E. Growth hormone provocative testing

Bone and Joint Disorders - Nelson Self Assessments website 17th Edition 5
Question . 21. An infant dies from respiratory distress shortly after birth. Physical
findings include short, deformed extremities, a short neck, and a small thorax.
Skeletal radiographs indicate a skeletal dyplasia but do not reveal a specific diagnosis.
You prepare to provide genetic counseling to the parents. Although you know that
these disorders usually have a genetic basis, a careful family history is negative for a
similar occurrence. Which of the following is the least likely explanation for the
negative family history?

A. The occurrence in this infant represents a new mutation for normal parents
B. The disorder is a recessive condition and the parents are carriers
(heterozygotes)
C. One of the parents has germ line mosaicism for the mutation
D. The disorder varies in severity, and the clinical manifestations are too mild to
be recognized in one patient who harbors the mutant gene
E. The mutant gene was transmitted but suppressed in previous generations,
reaching the threshold for expression only in this infant
Explanation: Explanations A-D all are important considerations in counseling
parents. (See Chapter 682 in Nelson Textbook of Pediatrics, 17th ed.)

Question . 22. The daughter of healthy, normal-stature parents demonstrated


deviation from the normal growth curve at age 2 yr, with growth substantially below
the 3rd percentile by age 5 yr, when malabsorption is noted. The growth impairment
involves both trunk and limbs. She has bowed legs and very short hands and feet, with
marked joint laxity. Her hair is blond and thin, and she has a light complexion.
Radiographs show flaring of the metaphyses of tubular bones. The family history is
negative for a similar occurrence. Which of the following is the most likely diagnosis?

A. Spondyloepiphyseal dysplasia tarda


B. Diastrophic dysplasia
C. Cartilage-hair hypoplasia
Explanation: This is a classic picture of a late-appearing skeletal dysplasia.
The differential diagnosis includes other causes of malabsorption. However,
these should not disproportionately affect the skeleton. (See Chapter 682)
D. Achondroplasia
E. Schmid metaphyseal dysplasia

Question . 23. A 1-wk-old infant is diagnosed with achondroplasia on the basis of a


combination of clinical and x-ray findings. The parents are being counseled on
potential serious complications to be anticipated during infancy and childhood. Which
of the following is the most likely potential complication?

A. Retinal detachment
B. Spinal cord compression
Explanation: Spinal compression may occur at the level of the foramen
magnum or in the lumbar spine and sites of spinal canal stenosis. (See Chapter
684 in Nelson Textbook of Pediatrics, 17th ed.)
C. Precocious osteoarthritis of hips and knees
D. Malignancy
E. Repeated bone fractures

Bone and Joint Disorders - Nelson Self Assessments website 17th Edition 6
Question . 24. An adolescent girl who is a cheerleader comes to you with a painful
bump below her right knee. She denies fever or trauma. Which of the following is the
most likely diagnosis?

A. Legg-Calvé-Perthes disease
B. Osteoid osteoma
C. Osgood-Schlatter disease
Explanation: (See Chapter 667.4 in Nelson Textbook of Pediatrics, 17th ed.)
D. Osteochondritis dissecans
E. Osteomyelitis of the tibial tubercle

Question . 25. The best treatment for the patient described in Question 24 is:

A. Decreased activity of the knee


Explanation: The patient described in Question 24 has a classic history of
Osgood-Schlatter disease, best managed by decreased activity of the involved
joint. (See Chapter 667.1 in Nelson Textbook of Pediatrics, 17th ed.)
B. Anti-inflammatory drugs
C. Antibiotics
D. Excisional biopsy
E. Casting for 6-8 wk

Question . 26. An overweight adolescent boy complains of pain in the medial aspect
of his knee. He denies trauma, and he has not had a fever. The most likely diagnosis
is:

A. Toxic synovitis
B. Legg-Calvé-Perthes disease
C. Medial collateral ligament (knee) strain
D. Slipped capital femoral epiphysis
Explanation: Legg-Calvé-Perthes disease occurs at a younger age than is
typical for slipped capital femoral epiphysis. The pain is referred from the hip
to the knee. (See Chapter 668.3 in Nelson Textbook of Pediatrics, 17th ed.)
E. Avulsion of the gastrocnemius muscle

Question . 27. A 2-yr-old child is brought to you because he refuses to use his right
arm. Any attempt to touch it is met with a cry, and the child will not hold objects in
his right hand. The mother denies trauma, but she did pull the child by the arm
recently when he refused to go into an elevator. The most likely diagnosis is:

A. Nonaccidental trauma (child abuse)


B. Fracture of the radius
C. Muscle strain of the right pronator
D. Dislocated radial head
Explanation: (See Chapter 671.3 in Nelson Textbook of Pediatrics, 17th ed.)
E. Osteomyelitis

Bone and Joint Disorders - Nelson Self Assessments website 17th Edition 7
Question . 28. From the following list, choose the most appropriate measures for
management of the child described in Question 27.
1. Radiograph of the arm with thin casting or splinting
2. Supination of the forearm
3. Antibiotics
4. Alerting the parents to the cause of the problem
5. Reporting the case to a child welfare agency

A. 1 and 4
B. 1 and 3
C. 2 and 3
D. 2 and 4
Explanation: This is a classic history with dislocation of the radial head.
Supination of the forearm is curative, and counseling parents not to pull small
children by the arm is important. (See Chapter 671.3 in Nelson, 17th ed.)
E. 1, 3, and 5

Question . 29. A 12-yr-old boy sustains a nail puncture of the right foot through an
old sneaker. Two days later, he limps and complains of pain and swelling in that area.
The most likely diagnosis is:

A. Tetanus
B. Osteochondritis
Explanation: (See Chapter 664.9 in Nelson Textbook of Pediatrics, 17th ed.)
C. Foreign body reaction
D. Toxic shock syndrome
E. Ecthyma gangrenosum

Question . 30. From the following list, choose the organism(s) most likely to cause
the problem described in Question 29.
1. Clostridium perfringens
2. Staphylococcus aureus
3. Staphylococcus epidermidis
4. Pseudomonas aeruginosa
5. Serratia marcescens

A. 1 only
B. 1 and 3
C. 2 and 3
D. 2 and 4
Explanation: Pseudomonas probably came from the sneaker, and S. aureus
from the skin. New sneakers (running shoes) do not have Pseudomonas
present until they are worn for some time. (See Chapter 664.9 in Nelson
Textbook of Pediatrics, 17th ed.)
E. 3 and 5

Bone and Joint Disorders - Nelson Self Assessments website 17th Edition 8
Question . 31. The most appropriate first step in management of the boy described in
Questions 29 and 30 is:

A. Piperacillin-tazobactam
B. Ciprofloxacin
C. Incision, drainage, debridement
Explanation: Incision and drainage with debridement of necrotic infected
material constitute one of the most important aspects of treatment. After the
material is cultured and Gram stained, the patient is started on a combination
of intravenous nafcillin and gentamicin. Treatment with antibiotics is usually
for 7-14 days if debridement is successful. (See Chapter 664.9 )
D. Tetanus toxoid
E. Warm soaks

Question . 32. The Barlow test is used to diagnose:

A. Patellar subluxation
B. Legg-Calvé-Perthes disease
C. Developmental dysplasia of the hip
Explanation: The Barlow test is the most important maneuver in examination
of the newborn hip to detect developmental dysplasia. This provocative test to
dislocate an unstable hip is performed by stabilizing the pelvis with one hand
and then flexing and adducting the opposite hip and applying a posterior force.
If the hip is dislocatable, it is usually readily felt. After release of the posterior
force, the hip usually relocates spontaneously. The Ortolani test is a maneuver
to reduce a recently dislocated hip; if reduction is possible, the relocation will
be felt as a "clunk," not as an audible "click." It is most likely to be positive in
infants of age 1-2 mo because adequate time must have passed for the true
dislocation to occur. (See Chapter 668.1 in Nelson Pediatrics, 17th ed.)
D. Slipped capital femoral epiphysis
E. Scoliosis

Question . 33. The recommended management of a newborn with an unstable hip


examination indicating developmental dysplasia of the hip is:

A. Observation alone
B. Pavlik harness or use of double- or triple-diapering
Explanation: Methods to maintain the unstable newborn hip in the position of
flexion and abduction include the Pavlik harness, the Frejka splint, and a
variety of abduction orthoses. Double and triple diapers, although
controversial, are commonly used in newborns with dislocatable hips for 2-3
wk because initially the splints and harnesses usually do not fit satisfactorily.
Treatment is continued until the hip is clinically stable and ultrasonographic or
radiographic measurements of the hip are normal. (See Chapter 668.1 in
Nelson Textbook of Pediatrics, 17th ed.)
C. Serial spica casts
D. Surgical closed reduction
E. Open surgical reduction with pelvic or femoral osteotomy

Bone and Joint Disorders - Nelson Self Assessments website 17th Edition 9
Question . 34. The most serious complication of developmental dysplasia of the hip is:

A. Femoral shortening
B. Avascular necrosis of the capital femoral epiphysis
Explanation: The most important and severe complication of developmental
dysplasia of the hip is avascular necrosis of the capital femoral epiphysis. This
is an iatrogenic complication that results from reduction of the femoral head
under pressure, producing cartilaginous compression, which can result in
occlusion of the intra-articular, extraosseous epiphyseal vessels, leading to
partial or complete infarction. (See Chapter 668.1 in Nelson, 17th ed.)
C. Joint instability
D. Fat embolism
E. Myelokathexis

Question . 35. Which of the following is a suspected contributing factor for


development of slipped capital femoral epiphysis?

A. Hypocalcemia
B. Rapid alterations of growth hormone and sex hormones
Explanation: An endocrine basis of slipped capital femoral epiphysis has been
postulated because it is frequently associated with abnormalities of growth.
Sex hormones, growth hormone, and other hormones alter the rate of growth
in the capital femoral epiphysis and the rate of skeletal growth. In obese
adolescents, a low level of sex hormones has been postulated, whereas in tall,
thin patients, an overabundance of growth hormone is implicated. Slipped
capital femoral epiphysis occurs in adolescents who are obese and have
delayed skeletal maturation or who are tall and thin and have had a recent
growth spurt. (See Chapter 668.4 in Nelson Textbook of Pediatrics, 17th ed.)
C. Eating disorders (e.g., bulimia)
D. Vitamin C deficiency
E. Repeated, minor trauma associated with strenuous physical exercise

Question . 36. All of the following statements regarding scoliosis are true Except:

A. The most common cause is idiopathic, but there appears to be a genetic


component
B. The incidence is much higher in girls than in boys (4:1 ratio)
Explanation: Idiopathic scoliosis is the most common form of scoliosis and
occurs in healthy, neurologically normal children. The incidence is only
slightly greater in girls than in boys, but scoliosis is more likely to progress in
girls than in boys. There appears to be a genetic component. Daughters of
affected mothers are more likely than other children to have scoliosis, but
identical twins are not uniformly affected. The magnitude of curvature in an
affected person is not related to the magnitude of curvature in affected
relatives. (See Chapter 669.1 in Nelson Textbook of Pediatrics, 17th ed.)
C. Scoliosis is more likely to progress and require treatment in girls than in boys
D. The age at onset is most commonly in adolescence.

Bone and Joint Disorders - Nelson Self Assessments website 17th Edition 10
Question . 37. Which of the following features defines toddler fractures?

A. Subluxation of the radial head


B. Salter-Harris type 3 fracture of the distal fibular epiphysis
C. Fracture of the distal radius
D. Spiral fracture of the tibia
Explanation: Toddler fractures represent a spiral fracture of the distal third of
the tibia. They are usually the result of simple falls during running or playing.
These fractures occur in children 2-4 yr of age and occasionally up to 6 yr of
age. Clinical features include pain, refusal to walk, minimal soft tissue
swelling, a slight increase in warmth to palpation over the fracture, and pain
with palpation. (See Chapter 673.6 in Nelson Textbook of Pediatrics, 17th ed.)
E. Any fracture occurring in a toddler

Question . 38. Types of injury requiring immediate attention and orthopedic


consultation include:

A. Fracture with vascular or nerve compromise


B. Open fracture
C. Deep laceration over a joint
D. Grade III (complete) tear of a muscle-tendon unit
E. All of the above
Explanation: Blood flow may be obstructed by a dislocated structure, which
should be reduced. Peripheral nerve damage can be repaired after vascular and
skeletal stability has been achieved. An open fracture should not be
immediately reduced because of the risk of further contamination. (See
Chapter 673.10 in Nelson Textbook of Pediatrics, 17th ed.)

Question . 39. Principal features of osteogenesis imperfecta include all of the


following Except:
A. Fragile bones
B. Blue sclerae
C. Deafness
D. Microcephaly
Explanation: The classic triad of findings in osteogenesis imperfecta is
composed of fragile bones, blue sclerae, and early deafness. Infants have
shortened bowed extremities and relative macrocephaly. (See Chapter 689 in
Nelson Textbook of Pediatrics, 17th ed.)

Question . 40. All of the following statements regarding rickets are true Except:

A. Rickets results from poor mineralization at the growth plate


B. All patients with rickets have osteomalacia
C. All patients with osteomalacia have rickets
Explanation: Mineral deficiency of either calcium or phosphate in growing
children before fusion of the epiphyses results in rickets. All patients with
rickets have osteomalacia (poor mineralization of trabecular bone), but not all
patients with osteomalacia have rickets. (See Chapter 697 in Nelson, 17th ed.)
D. Rickets is found only in growing children before fusion of the epiphyses
E. Rickets may result from calcium deficiency or phosphate deficiency

Bone and Joint Disorders - Nelson Self Assessments website 17th Edition 11
RHEUMATOLOGY
Question . 1. Erythema nodosum, an erythematous, nodular, often pretibial
rash, is associated with all of the following except:

Inflammatory bowel disease

Pseudotumor cerebri
Explanation: Erythema nodosum in itself is not a disease
but is associated with many infections (e.g., group A
streptococcal infection, tuberculosis, Yersinia infection,
histoplasmosis, coccidioidomycosis) and inflammatory
states (e.g., inflammatory bowel disease, systemic lupus
erythematosus, juvenile rheumatoid arthritis, sarcoidosis)
and may occur after administration of drugs (e.g.,
sulfonamides, phenytoin, oral contraceptives). (See
Chapter 143 in Nelson Textbook of Pediatrics, 17th ed.)

Sarcoidosis

Yersinia

Birth control pills

Question . 2. A positive result on an ANA assay (titer 1:80) is associated with


all of the following except:

Systemic lupus erythematosus

Juvenile rheumatoid arthritis

Chronic autoimmune hepatitis

Salicylate intoxication
Explanation: A positive result on an ANA assay is a
nonspecific sign of increased lymphocyte activity and is
noted in many inflammatory states. (See Chapter 143 in
Nelson Textbook of Pediatrics, 16th ed.)

Phenytoin
Question . 3. Low levels of complement may be associated with all of the
following except:

Nephrotic syndrome
Explanation: Nephrotic syndrome is associated with
normal levels of complement. Immune complex-mediated
diseases reduce total hemolytic complement levels.

Vasculitis

Nephritis

Serum sickness

Systemic lupus erythematosus

Question . 4. Pseudoporphyria is best described as a complication of:

Systemic lupus erythematosus

Naproxen therapy
Explanation: Naproxen, more often than other
nonsteroidal anti-inflammatory drugs, can produce this skin
lesion, characterized by small hypopigmented flat scars
after a blister formation.

Oral prednisone therapy

Topical steroid therapy

Topical chloroquine therapy

Question . 5. Which of the following is the preferred agent for initial therapy of
most rheumatic diseases?

Nonsteroidal anti-inflammatory drugs


Explanation: Nonsteroidal anti-inflammatory drugs are the
cornerstone of drug treatment of most rheumatic diseases
in children. They are the only drugs needed for at least half
of the patients, and they provide significant relief for many
of the remaining patients.

Cyclophosphamide

Methotrexate

Sulfasalazine
Azathioprine

Question . 6. A 6-yr-old girl is diagnosed with juvenile rheumatoid arthritis. In


advising her parents about the probability of persistence of active disease into
adulthood, you cite which of the following?

All symptoms will resolve by age 21

5-10%

50%
Explanation: Approximately 45% of children with juvenile
rheumatoid arthritis will have active disease that persists
into early adulthood. (See Chapter 145 in Nelson Textbook
of Pediatrics, 17th ed.)

90%

99%

Question . 7. A 7-yr-old white boy presents with malaise, chest pain, high
spiking fevers, and chills, with onset of his illness 3 wk previously. He has had
no ill contacts, and he has missed school during the last week. Physical
examination reveals an acutely ill child with a heart rate of 125/min, a
temperature of 40.5°C, a fine but faint macular red-pink rash on the trunk and
proximal extremities, lymphadenopathy, liver edge palpable 4 cm below the
right costal margin, and a palpable spleen tip. Laboratory studies reveal a
hemoglobin of 9.7 g/dL, a total white blood cell count of 26,000/mm3, and a
platelet count of 650,000/mm3. The most important step in evaluating this
patient would be:

Erythrocyte sedimentation rate (ESR)

Lyme disease titer

Chest radiograph

Echocardiogram
Explanation: An echocardiogram reveals a moderate-
sized pericardial effusion, the most probable cause of this
boy's chest pain. An ESR is not diagnostic and occasionally
shows low values in serious inflammatory, infectious, or
oncologic diseases. A chest radiograph may reveal
cardiomegaly due to pericardial effusion but does not
distinguish cardiomegaly from heart failure or effusion. A
bone marrow aspirate may be informative if leukemia is a
consideration. (See Chapter 145 in Nelson Textbook of
Pediatrics, 17th ed.)

Bone marrow aspiration


Question . 8. On further evaluation, the patient described in Question 7 has no
evidence of pericardial tamponade or reduced cardiac function. His pulse
normalizes with defervescence. The most appropriate next step in approach
management would be to:

Perform pericardiocentesis

Begin inotropic drugs

Improve preload with fluids

Begin an oral nonsteroidal anti-inflammatory drug


Explanation: Administration of an oral anti-inflammatory
agent or a corticosteroid is the correct choice for treating
the pericardial effusion. (See Chapter 145 in Nelson
Textbook of Pediatrics, 17th ed.)

Begin methotrexate

Question . 9. The most likely diagnosis for the patient described in Question 8
is:

Systemic-onset juvenile rheumatoid arthritis


Explanation: Systemic-onset JRA often manifests with
prolonged fevers, a salmon-pink macular rash,
asymptomatic pericarditis, leukocytosis, anemia of chronic
inflammatory disease, hepatosplenomegaly, and
lymphadenopathy. Arthralgia or myalgia may be present,
but arthritis does not usually develop until later in the
course of the illness. SLE and rheumatic fever may also
produce rheumatologic symptoms and pericardial effusion.
(See Chapter 145 in Nelson Textbook of Pediatrics, 17th
ed.)

Uremia

Systemic lupus erythematosus

Scleroderma

Rheumatic fever
Question . 10. A 4-yr-old white girl has had joint swelling in multiple joints for
over 6 mo. She is slow to move in the morning and moves as if stiff for the first
hours of the day. Thereafter, she is a very active child. She has no rash and
very little limitation of range of motion. Her erythrocyte sedimentation rate is 4.
The most likely diagnosis is:

Hypermobility syndrome

Dermatomyositis

SLE

JRA
Explanation: The ESR may be normal in patients with
active JRA. The C reactive protein may be elevated, as well
as the platelet count. Nonetheless, the ESR is not always
elevated. (See Chapter 145 in Nelson Textbook of
Pediatrics, 17th ed.)

Henoch-Sch nlein purpura

Question . 11. The enthesitis-related arthritides are characterized by


involvement of the axial skeleton and the presence of enthesitis (inflammation
at the attachment of tendons to bone). These disorders include all of the
following except:

Rheumatoid factor-negative JRA


Explanation: The enthesitis-related arthritides include what
has traditionally been called juvenile ankylosing spondylitis.
Psoriatic arthritis is considered a distinct entity. These are
associated with axial arthritis and are nearly universally
associated with HLA-B27. (See Chapter 146 in Nelson
Textbook of Pediatrics, 17th ed.)

Psoriatic arthritis

Arthritis with inflammatory bowel disease

reactive arthritis secondary to diarrhea

Reactive arthritis secondary to genitourinary infection


Question . 12. Organisms associated with reactive arthritis include:

Shigella

Chlamydia trachomatis

Yersinia enterocolitica

Campylobacter jejuni

All of the above


Explanation: Reactive arthritis may follow enteric infection
(with Shigella, Salmonella, Yersinia enterocolitica, or
Campylobacter jejuni) or genitourinary tract infection with
Chlamydia trachomatis. (See Chapter 147 in Nelson
Textbook of Pediatrics, 17th ed.)

Question . 13. An 18-yr-old boy has a swollen right wrist and left ankle with
bilateral pain over both Achilles tendons. He was treated 2 wk ago with an
antibiotic for a urethral discharge; his girlfriend was also treated. Physical
examination reveals tenderness over both Achilles tendons, swollen painful
joints (right wrist and left ankle), and limited forward bending at the waist. The
most appropriate therapeutic agent would be:

Ceftriaxone

Doxycycline

Prednisone

A nonsteroidal anti-inflammatory agent


Explanation: Reactive arthritis may follow enteric infection
with non-typhoidal Salmonella, Shigella, Yersinia
enterocolitica, Campylobacter jejuni, Cryptosporidium
parvum, Giardia intestinalis, or genitourinary tract infection
with Chlamydia trachomatis. No specific treatment is
necessary for reactive arthritis, except for relief of pain and
the functional limitations of arthritis with nonsteroidal anti-
inflammatory agents. (See Chapter 147 in Nelson Textbook
of Pediatrics, 17th ed.)

Intravenous immunoglobulin
Question . 14. Infectious agents associated with arthralgia or arthritis include
all of the following except:

Rubella vaccine

Respiratory syncytial virus


Explanation: Agents may produce arthritis by immune
complex deposition, cross-reactivity to shared antigenic
epitopes between the synovium and the agent, or direct
infection. (See Chapter 147 in Nelson Textbook of
Pediatrics, 17th ed.)

Parvovirus

Yersinia

Campylobacter

Question . 15. All of the following are diagnostic criteria for the diagnosis of
SLE except:

Malar rash

Seizures

Raynaud phenomenon
Explanation: Diagnosis of SLE requires 4 or more of the
11 diagnostic criteria. (See Table 148-2 and Chapter 148 in
Nelson Textbook of Pediatrics, 17th ed.)

Thrombocytopenia

Pericarditis
Question . 16. A 12-yr-old white girl presents with arthralgias of the knees and
elbow and swollen hands of 6 months' duration. She has had intermittent fever
and has lost 15 lb. Other than swollen joints, findings on physical examination
are normal. Three years earlier, she was found to have thrombocytopenia and
was diagnosed with idiopathic thrombocytopenic purpura (ITP). In addition,
one summer she had severe sunburn, and 2 yr ago she had mouth sores.
Today she has a hematocrit of 25% and a positive result on a Coombs test,
and the urinalysis shows multiple red blood cells. The most likely diagnosis is:

JRA

ITP

Evans syndrome

Periarteritis

SLE
Explanation: The diagnosis of SLE is confirmed by the
presence of arthritis, mouth ulcers, photosensitivity,
thrombocytopenia, hemolytic anemia, and hematuria. All
manifestations need not be present at the same time. (See
Chapter 148 in Nelson Textbook of Pediatrics, 17th ed.)

Question . 17. Laboratory testing of the patient described in Question 16


reveals positive results on assays for antibody to double-stranded DNA and
antinuclear antibody (ANA) and undetectable complement levels. The best
approach to treatment is administration of:

Intravenous immune globulin (IVIG)

Plasmapheresis

Cyclosporine

Prednisone
Explanation: Prednisone is the treatment of choice for SLE
exacerbations. (See Chapter 148 in Nelson Textbook of
Pediatrics, 17th ed.)

Ibuprofen
Question . 18. Which of the following is a component of the diagnostic criteria
for systemic lupus erythematosus?

Leukocytosis (>15,000 WBCs/mm3)

Seizures
Explanation: The diagnosis of SLE is confirmed by the
combination of 4 of 11 criteria, including neurologic
disorders such as seizures and psychosis in the absence of
other identified causes. Leukopenia, malar rash, false-
positive serologic results for syphilis, and nonerosive
arthritis are also components of the diagnostic criteria. (See
Table 148-2 and Chapter 148 in Nelson Textbook of
Pediatrics, 17th ed.)

Subcutaneous nodules

False-positive heterophile test

Erosive arthritis involving two or more peripheral joints

Question . 19. A newborn is found to have congenital heart block. Which of


the following is the most likely etiology?

Group B streptococcal infection

Neonatal echovirus infection

Gestational diabetes

Maternal lupus
Explanation: Congenital heart block is a prominent feature
of neonatal lupus, which results from maternal transfer of
IgG autoantibodies, usually anti-Ro, between the 12th and
16th wk of gestation. The heart block is permanent and
often requires cardiac pacing. (See Chapter 148.1 in
Nelson Textbook of Pediatrics, 17th ed.)

Gray baby syndrome


Question . 20. A 9-yr-old girl reports that she has had difficulty combing her
hair and walking up stairs for approximately 1 mo. Physical examination
reveals a positive Gowers sign and a faint maculopapular rash over the
metacarpophalangeal joints. The most appropriate laboratory study to order is:

Determination of erythrocyte sedimentation rate

Measurement of serum creatine kinase level


Explanation: The creatine kinase level was 7,500 IU/mL.
(See Chapter 149 in Nelson Textbook of Pediatrics, 17th
ed.)

Rheumatoid factor

Motor nerve conduction study

Assay for antinuclear antibodies

Question . 21. The most likely diagnosis for the patient described in Question
20 is:

Muscular dystrophy

Dermatomyositis
Explanation: Juvenile dermatomyositis classically affects
preadolescent girls with an insidious onset of muscle
weakness. Vasculitic rashes may be present over knuckles,
the malar area, or the eyelids (which are violet-tinged).
(See Chapter 149 in Nelson Textbook of Pediatrics, 17th
ed.)

Periarteritis nodosa

Systemic lupus erythematosus

Myotonic dystrophy
Question . 22. A 11-yr-old girl has had difficulty in getting out of chairs and
combing her hair for 3 mo. Physical examination reveals tenderness over the
quadriceps muscles and 4/5 strength. In addition, there is a faint erythematous
rash over both upper eyelids. The most appropriate next step in the diagnosis
is:

Serum ANA assay

Measurement of serum creatine kinase level


Explanation: The creatine kinase level was 2000 IU/mL,
approximately 40 times normal. An ANA assay result was
positive but was not considered diagnostic. (See Chapter
149 in Nelson Textbook of Pediatrics, 17th ed.)

Complement asay

Muscle biopsy

Nerve conduction study

Question . 23. The patient described in Question 22 begins to have difficulty


swallowing solids and starts to drool. The most likely cause of her problems is:

Palatopharyngeal muscle weakness


Explanation: Striated muscle weakness of the oropharynx
produces dysphagia and possible airway compromise. (See
Chapter 149 in Nelson Textbook of Pediatrics, 17th ed.)

Bulbar neuropathy

Anterior horn cell disease

Botulism

Trichinosis
Question . 24. The most likely diagnosis for the patient described in Questions
22 and 23 is:

Polio

JRA

Juvenile dermatomyositis
Explanation: Juvenile dermatomyositis, an autoimmune
inflammatory disease, is characterized by involvement of
skeletal muscle and, at times, the cardiac muscle.
Treatment with prednisone is usually effective. (See
Chapter 149 in Nelson Textbook of Pediatrics, 17th ed.)

Scleroderma

Viral myositis

Question . 25. An 8-yr-old boy presents with complaints of fatigue and


arthralgias and a rash that is prominent in sun-exposed areas. Physical
examination shows periorbital violaceous discoloration, hypertrophic and
reddish pink skin over the metacarpal and proximal interphalangeal joints, and
proximal muscle weakness. The most likely diagnosis is:

Contact dermatitis

Juvenile rheumatoid arthritis

Juvenile dermatomyositis
Explanation: The cutaneous findings and proximal muscle
weakness are the cardinal features of juvenile
dermatomyositis. (See Chapter 149 in Nelson Textbook of
Pediatrics, 17th ed.)

Mercury ingestion

Lyme disease
Question . 26. On exposure to cold, a 14-yr-old girl characteristically develops
pallor, then cyanosis, and finally erythema of the fingers and toes. She is at
increaded risk of developing:

Systemic-onset juvenile rheumatoid arthritis

Scleroderma
Explanation: Raynaud phenomenon may precede
extensive skin and internal organ involvement of
scleroderma by months or years. The peak age at onset of
scleroderma is 30-50 yr; children represent fewer than 10%
of cases. (See Chapter 150 in Nelson Textbook of
Pediatrics, 17th ed.)

Leukemia

Histiocytosis

Diabetes mellitus

Question . 27. Which of the following organ or body systems is least likely to
be involved in patients with Behçet disease?

Central nervous system

Joints

Blood vessels

Liver
Explanation: Hepatic involvement is not characteristic of
Behçet disease. (See Chapter 151 in Nelson Textbook of
Pediatrics, 17th ed.)

Eyes
Question . 28. A 15-yr-old girl complains of burning and itching eyes. Physical
examination shows bilateral, painless enlargement of the parotid glands. She
denies a sense of dry mouth. The most likely diagnosis is:

Scleroderma

Sj gren syndrome
Explanation: Sj gren syndrome is very uncommon in the
pediatric population. Subjective symptoms of xerostomia
are less common among younger patients. (See Chapter
152 in Nelson Textbook of Pediatrics, 17th ed.)

Acute HIV infection

Mumps

Allergic parotitis

Question . 29. All of the following are features of Sj gren syndrome except:

Arthritis
Explanation: Arthritis is not a feature of Sj gren syndrome.
(See Chapter 152 in Nelson Textbook of Pediatrics, 17th
ed.)

Keratoconjunctivitis sicca

Parotid gland enlargement

Xerostomia

Dysphagia

Question . 30. A 4-yr-old Middle Eastern boy presents with a history of brief
acute episodes of fever and abdominal pain. The most likely diagnosis is:

Behçet syndrome

Sj gren syndrome

Juvenile dermatomyositis

Familial Mediterranean fever


Explanation: The onset of familial Mediterranean fever is
usually before age 5 yr and is characterized by brief, acute,
self-limited episodes of fever and polyserositis recurring at
irregular intervals.
Amyloidosis

Question . 31. Which of the following statements regarding familial


Mediterranean fever is true?

It is inherited as an autosomal dominant disorder

The onset of clinical manifestations is usually before age 5


yr

Colchicine is the mainstay of treatment


Explanation: Colchicine is the mainstay of treatment for
familial Mediterranean fever. (See Chapter 153 in Nelson
Textbook of Pediatrics, 17th ed.)

The disease is common among Native Americans

Pulmonary insufficiency is the most important complication


and determines the prognosis

Question . 32. All of the following diseases are known to be complicated by


reactive amyloidosis (AA type) except:

Familial Mediterranean fever

Juvenile rheumatoid arthritis

Henoch-Sch nlein purpura


Explanation: Henoch-Sch nlein purpura is not associated
with risk for developing amyloidosis. (See Chapter 154 in
Nelson Textbook of Pediatrics, 17th ed.)

Inflammatory bowel disease

Ankylosing spondylitis
Question . 33. A 13-yr-old boy has had chronic cough, fever, and fatigue for
the past 4 wk and also has experienced weight loss. Physical examination
shows slender habitus, mild tachypnea but no crackles or wheezes, and
enlarged lymph nodes, most notably the axillary and epitrochlear nodes. Chest
radiograph shows hilar lymphadenopathy. A biopsy of the epitrochlear node
shows noncaseating granulomas. The most likely diagnosis is:

Tuberculosis

Non-Hodgkin lymphoma

Infectious mononucleosis

Sarcoidosis
Explanation: There are no specific diagnostic tests for
sarcoidosis. An elevated erythrocyte sedimentation rate,
hyperproteinemia, hypercalcemia, hypercalciuria,
eosinophilia, and an elevated angiotensin-converting
enzyme level are common. Definitive diagnosis requires
demonstration of the characteristic noncaseating
granulomatous lesions in a biopsy of tissue from an
affected area. (See Chapter 155 in Nelson Textbook of
Pediatrics, 17th ed.)

Rheumatoid arthritis

Question . 34. A 10-yr-old boy has been diagnosed with sarcoidosis on the
basis of clinical features of fever, rash and arthritis and characteristic features
of noncaseating granuloma on skin biopsy. Chest radiograph shows enlarged
hilar lymph nodes. Which of the following studies should be performed to
monitor for irreversible damage from sarcoidosis?

Bronchoscopy with transbronchial biopsy

Slit-lamp examination of the eyes now and on a yearly


basis
Explanation: Eye involvement may lead to blindness.
Serum angiotensin-converting enzyme levels correlate with
disease activity. (See Chapter 155 in Nelson Textbook of
Pediatrics, 17th ed.)

Abdominal CT scan now and on yearly basis

Serum angiotensin-converting enzyme (ACE) assay now


and on a monthly basis

Head MRI study now and on a yearly basis


Question . 35. All of the following statements regarding Kawasaki disease are
true except:

It occurs in outbreaks

Exposure to an affected patient greatly increases the


risk of the disease
Explanation: Although outbreaks are common, it is
uncommon to catch Kawasaki disease from an affected
person. (See Chapter 156 in Nelson Textbook of Pediatrics,
17th ed.)

Asian children have the highest risk

80% of patients are younger than 5 yr of age

It is the leading cause of acquired heart disease in children


in the United States

Question . 36. Possible complications of Kawasaki disease include all of the


following except:

Arthritis

Splenic rupture
Explanation: Splenic rupture and splenomegaly are not
associated with Kawasaki disease.

Septic meningitis

Urethritis

Hydrops of the gallbladder

Question . 37. The most common cause of death from Kawasaki disease in
the early phase of disease is:

Myocardial infarction

Rupture of an aneurysm

Stroke

Myocarditis
Explanation: In the early stage of disease, before
aneurysm of the coronary arteries occurs, severe
myocarditis may produce cardiogenic shock.
Pericarditis

Question . 38. The differential diagnosis of Kawasaki disease includes all of


the following except:

Dermatomyositis
Explanation: Juvenile dermatomyositis is characterized by
marked skeletal muscle weakness, which is not
characteristic of Kawasaki disease or the other illnesses.
(See Chapter 156 in Nelson Textbook of Pediatrics, 17th
ed.)

Toxic shock syndrome

Scarlet fever

Stevens-Johnson syndrome

Measles

Rocky Mountain spotted fever

Question . 39. A 2-yr-old girl presents with high, remittent fever of 9 days'
duration associated with marked irritability, nonpurulent conjunctival injection,
red cracked lips, swollen and erythematous hands and feet, and a
maculopapular erythematous rash on the trunk and extremities. Laboratory
testing at presentation would most likely show evidence of:

Aseptic meningitis
Explanation: The platelet count in Kawasaki disease is
generally normal initially but rapidly rises by wk 2 or 3 of the
illness; counts may exceed 1,000,000/mm3. (See Chapter
156 in Nelson Textbook of Pediatrics, 17th ed.)

Thrombocytosis

Hyponatremia

Lymphocytosis

Hemolytic anemia
Question . 40. An 11-mo-old boy is treated for acute Kawasaki disease with 2
g/kg of intravenous immune globulin. No cases of measles have reported in his
hometown in the last 8 yr. The first measles-mumps-rubella vaccine should be
administered to this child at the age of:

12 mo

4 yr

16 mo

3 yr

22 mo
Explanation: With the high dose (2 g/kg) of IVIG used for
Kawasaki disease, live virus vaccines should be deferred
until at least 11 mo following IVIG administration. The
antibodies in the IVIG may interfere with the replication of
the live virus vaccines, thereby impairing immunogenicity.
(See Chapter 156 in Nelson Textbook of Pediatrics, 17th
ed.)

Question . 41. A 4-yr-old white girl has had a low-grade fever, intermittent
crampy abdominal pain with emesis, and swollen knees for 3 days. There is a
petechial rash on the lower extremity. The most likely diagnosis is:

Meningococcemia

Idiopathic thrombocytopenia purpura

Henoch-Sch nlein purpura


Explanation: Henoch-Sch nlein purpura is a common
vasculitis among children and manifests with the classic
findings of abdominal pain with or without rectal bleeding,
vasculitic rash, arthritis, and nephritis. The platelet count is
normal. The gastrointestinal involvement may progress to
intussusception. (See Chapter 157 in Nelson Textbook of
Pediatrics, 17th ed.)

SLE

Rocky Mountain spotted fever


Question . 42. A 5-yr-old boy presents with the complaint of severe abdominal
pain that started 3 days previously. He is unable to eat and has occasional
emesis. Physical examination reveals an anxious, acutely ill child with normal
vital signs, generalized abdominal tenderness, voluntary guarding of the
anterior abdominal muscles, and normal findings on rectal examination. A
surgical consultant believes the child has an acute abdomen, possibly
appendicitis. Before the child is sent to the operating room, the urinalysis
reveals 4+ hematuria and 1+ proteinuria. The most appropriate next step in
management would be to:

Perform coagulation studies

Obtain a complete blood count

Perform renal ultrasonography

Perform an electrocardiogram

Cancel the operation


Explanation: You should cancel the operation until the
patient is evaluated further and the results of the urinalysis
are considered. (See Chapter 157.1 in Nelson Textbook of
Pediatrics, 17th ed.)

Question . 43. On repeat physical examination, the patient described in


Question 42 now has petechiae on the dorsal surfaces of the feet and hands
and over the buttocks. His platelet count is 350,000/mm3. The most likely
diagnosis is:

Kawasaki syndrome

Henoch-Sch nlein purpura


Explanation: Henoch-Sch nlein purpura is a common form
of vasculitis in childhood and manifests with involvement of
the skin (nonthrombocytopenic petechiae), joints (arthritis),
kidneys (nephritis), and intestine (vasculitis, mucosal
hemorrhage, intussusception). (See Chapter 157.1 in
Nelson Textbook of Pediatrics, 17th ed.)

Rocky Mountain spotted fever

Meningococcemia

Appendicitis with gram-negative sepsis


Question . 44. All of the following are potential complications of Henoch-
Sch nlein purpura except:

Seizures

Coronary aneurysm
Explanation: Coronary aneurysms occur most often in the
other common childhood vasculitis-Kawasaki disease. (See
Chapter 157.1 in Nelson Textbook of Pediatrics, 17th ed.)

Pancreatitis

Pulmonary hemorrhage

Neuropathy

Question . 45. The percent of children with Henoch-Sch nlein purpura in


whom serious persistent renal disease occurs is:

0%

<0.1%
Explanation: End-stage renal disease is an uncommon
sequela of Henoch-Sch nlein purpura. Nonetheless, 1% of
patients do have persistent renal abnormalities.

<10%

20% of males

50% of females

Question . 46. A 12-yr-old black girl has a 2-yr history of chronic sinusitis.
Today she has had an episode of hemoptysis and is experiencing respiratory
distress. Her urinalysis reveals hematuria. The most helpful laboratory test is:

Sm antibody assay

AntiPR3 ANCA assay


Explanation: Assay for antineutrophil cytoplasmic
antibodies (ANCA) that are directed to protease-3 (PR3) of
the neutrophil -granule is the most appropriate test, and
results are markedly positive.

Measurement of angiotensin-converting enzyme (ACE)

ANA assay
Erythrocyte sedimentation rate (ESR)

Question . 47. The most likely diagnosis for the patient described in Question
46 is:

SLE

Goodpasture disease

Wegener granulomatosis
Explanation: This vascular pulmonary renal syndrome is
not that rare and fortunately responds to therapy. (See
Chapter 157.4 in Nelson Textbook of Pediatrics, 17th ed.)

Sarcoidosis

Tuberculosis
ENDOCRINE
Question . 1. A previously healthy 10-mo-old female infant is found
unresponsive in her crib. In the emergency department, she is noted to be well
developed and well nourished with normal blood pressure and appearance of the
genitalia but with increased pigmentation of her skin. Blood glucose level is 30
mg/dL. The most likely diagnosis is:

Congenital adrenal hyperplasia due to 21-hydroxylase


deficiency

Familial glucocorticoid deficiency


Explanation: Glucocorticord deficiency results in increased
pituitary production of ACTH, which has melanocyte-like
stimulatory hormone properties. The combination of normal
blood pressure and normal-appearing genitalia in a female
patient helps exclude adrenal hyperplasias, which would
present much earlier in life (usually in the first month). This
familial autosomal recessive disorder does not have salt
wasting as a feature and is due in some patients to defects in
the ACTH receptor. (See Chapter 569 in Nelson Textbook of
Pediatrics, 17th ed.)

Cushing syndrome

Hyperinsulinemia

Congenital adrenal hyperplasia due to 11-hydroxylase


deficiency

Question . 2. A 4-wk-old female infant is presented to the emergency


department with hyponatremia and hyperkalemia. Other than the cardiovascular
abnormalities and dehydration, the physical findings are normal. The most
informative laboratory examination would be:

17-Hydroxyprogesterone

Renin

Aldosterone
Explanation: Aldosterone is most likely to be deficient in this
child. This occurs as a rare autosomal recessive disorder. If
she had virilization, one would also consider congenital
adrenal hyperplasia. In that case, 17-hyroxyprogesterone
assay is the most important diagnostic test. (See Chapter 569
in Nelson Textbook of Pediatrics, 17th ed.)

Cortisol

DAX-1
Question . 3. An 11-yr-old boy is followed for chronic mucocutaneous
candidiasis and hypoparathyroidism. His mother reports increasing fatigue,
cutaneous pigmentation, and chronic abdominal pain. The most likely cause is:

New-onset diabetes mellitus type 2

Hypothyroidism

Hepatitis

Adrenal insufficiency
Explanation: Polyendocrine autoimmune-induced
hypofunction is common in chronic cutaneous
mucocutaneous candidiasis. Adrenal involvement is highly
suggestive in this case. (See Chapter 569 in Nelson
Textbook of Pediatrics, 17th ed.)

Celiac disease

Question . 4. A 10-yr-old boy is under medical management for adrenal


insufficiency. His mother reports he has complained that he is not as good at
playing sports as previously and has trouble reading. The first step in the care of
this boy is to:

Measure very long-chain fatty acids


Explanation: This patient probably has a form of adrenal
leukodystrophy, which is an X-linked peroxisomal disorder. In
the absence of antiadrenal antibodies, this X-linked disorder
may be responsible for about 50% of cases of adrenal
insufficiency in males. (See Chapter 569 in Nelson Textbook
of Pediatrics, 17th ed.)

Measure thyroid hormone levels

Increase his replacement dose of hydrocortisone

Order an MRI study of his brain and pituitary

Refer him to an ophthalmologist


Question . 5. A 6-yr-old girl underwent removal of a craniopharyngioma 3 mo
previously. She is on a thyroid replacement regimen, but her mother reports she
is very fatigued. The next step would be to:

Increase the dose of her thyroid medication.

Schedule a growth hormone stimulation test

Measure ACTH and cortisol


Explanation: If she already has one endocrine disorder
following removal of a pituitary tumor, she is at high risk for
another, and in this case adrenal function must be evaluated.
If she has adrenal insufficiency from a pituitary hormone
deficiency, hyperpigmentation will be absent because ACTH
and cortisol levels will both be very low. (See Chapter 569 in
Nelson Textbook of Pediatrics, 17th ed.)

Obtain an MRI study of her head

Measure IGF-1 and IGFBP-3

Question . 6. A 2-mo-old infant is presented with failure to thrive, recurrent


emesis, hepatosplenomegaly, and adrenal insufficiency. Adrenal calcification is
noted radiologically. The most likely diagnosis is:

Adrenal hemorrhage

Tuberculosis

Neuroblastoma

Pheochromocytoma

Wolman disease
Explanation: This is a classic presentation of this cholesterol
storage disease. In the absence of these significant clinical
findings, adrenal calcifications may be an incidental finding.
But in this case, it is an important clue to the diagnosis. (See
Chapter 75 in Nelson Textbook of Pediatrics, 17th ed.)
Question . 7. A 2-wk-old male infant is eating poorly, has repeated emesis, and
has not regained his birth weight. On physical examination he appears
dehydrated and is noted to have increased pigmentation of skin creases and
genitalia. In addition to measuring serum electrolytes, assay of which of the
following hormones would be most informative?

17-Hydroxyprogesterone
Explanation: This is a classic presentation and timing for the
presentation of a salt-losing congenital adrenal hyperplasia in
a genotypic male. 17-Hydroxyprogesterone assay is the initial
diagnostic test. Determination of electrolytes would reveal
hyponatremia and hyperkalemia. The patient should be
examined to confirm the presence of gonads in the scrotum.
If there are not, this patient could be a virilized female with
complete fusion of the labial-scrotal folds. (See Chapter 570
in Nelson Textbook of Pediatrics, 17th ed.)

Cortisol

17-Hydroxypregnenolone

Renin

Aldosterone

Question . 8. An 8-yr-old girl treated from infancy for adrenal insufficiency is


noted to have inguinal gonads. Karyotype is 46,XY. The most likely diagnosis is:

True hermaphroditism

Testicular feminization

DAX-1 mutation

Lipoid adrenal hyperplasia


Explanation: Adrenal insufficiency rules out testicular
feminization and true hermaphrodism, which does not involve
the adrenals. This is a classic example of lipoid adrenal
hyperplasia. The gonads are probably testes, which will need
to be removed in the future. (See Chapter 570 in Nelson
Textbook of Pediatrics, 17th ed.)

17-Hydroxylase deficiency
Question . 9. A newborn infant is noted to have increased pigmentation of his
skin and genitalia, perineal hypospadias with bifid scrotum, and a 2-cm phallus
with chordee. Gonads are palpable in the inguinal canal, and no uterus is
visualized on ultrasound examination. The most likely diagnosis is:

Incomplete testicular feminization

21-Hydroxylase deficiency

3 -HSD deficiency
Explanation: This incompletely virilized male (no uterus but
partial male genitalia) probably has 3 -HSD deficiency. (See
Chapter 570 in Nelson Textbook of Pediatrics, 17th ed.)

Lipoid adrenal hyperplasia

11-Hydroxylase deficiency

Question . 10. A 5-yr-old boy presents with pubic hair development. He is tall
and has increased pigmentation of his genitalia and phallic enlargement. Blood
pressure is 130/90 mm Hg. Measurement of which of the following hormones
would be most likely to be diagnostic?

Testosterone

17-Hydroxyprogesterone

11-Deoxycortisol
Explanation: This male has a non-salt-losing form of
congenital adrenal hyperplasia due to 11-hydroxylase
deficiencies. The metabolic defect produces salt-retaining
steroids (deoxycorticosteroids) and is thus associated with
hypertension and usually presents later in life than is typical
for salt-wasting forms of 21-hydroxylase deficiencies. (See
Chapter 570 in Nelson Textbook of Pediatrics, 17th ed.)

Aldosterone

Deoxycorticosterone
Question . 11. A 2-yr-old boy presents with glandular breast development and
pubic hair development. His testes are prepubertal. The most appropriate next
step in his evaluation is:

ACTH stimulation test

LHRH stimulation test

Ultrasound examination of his abdomen


Explanation: These findings suggest increased androgen
production (pubic hair) and androgen conversion to estrogen.
An adrenal source is a strong possibility and must be
excluded before other testing is planned. (See Chapter 556 in
Nelson Textbook of Pediatrics, 17th ed.)

MRI of brain and pituitary

CT of brain and pituitary

Question . 12. A 4-yr-old previously normal girl has developed acne and pubic
hair. On physical examination she has clitoromegaly and mild increase in her
blood pressure. Serum dehydroepiandrosterone sulfate is markedly elevated.
The most likely diagnosis is:

Malignant adrenal tumor


Explanation: This is a dramatic presentation of endogenous
androgen production, most likely of an adrenal origin as
indicated by the elevation of the adrenal-derived hormone
dehydroepiandrosterone sulfate. (See Chapter 556 in Nelson
Textbook of Pediatrics, 17th ed.)

CAH due to 11-hydroxylase deficiency

CAH due to 21-hydroxylase deficiency

Malignant ovarian tumor

McCune-Albright syndrome
Question . 13. A 6-yr-old girl with extensive café-au-lait lesions and polyostotic
fibrous dysplasia is at risk for:

Gonadotropin-independent precocious puberty

Cushing syndrome

Thyroid nodules

Pituitary tumor

All of the above


Explanation: McCune-Albright syndrome is a
polyendocrinopathy with excessive hormone production.
Precocious puberty that is independent of central
gonadotrophic hormones is classic. Additional features
include fibrous dysplasia and cutaneous hyperpigmentation.
(See Chapter 556 in Nelson Textbook of Pediatrics, 17th ed.)

Question . 14. A 13-yr-old boy is undergoing evaluation for Cushing syndrome.


His cortisol level is unchanged following dexamethasone 30 µg/kg/day in 4
divided doses but decreases to 4 µg/dL following 120 µg/kg/day in 4 divided
doses. The next step(s) in his evaluation should include:

MRI of his head with and without contrast


Explanation: It is very possible that the suppressibility from
higher dose dexamethasone represents ACTH production
from a central origin, which requires higher doses of
dexamethasone to reduce cortisol levels. (See Chapter 571
in Nelson Textbook of Pediatrics, 17th ed.)

Petrosal sinus sampling before and after CRH determination

MRI of his abdomen

Adrenal vein catheterization

All of the above


Question . 15. Carney complex includes which of the following components?

Large cell calcifying Sertoli cell tumors

Cardiac myxomas

Primary pigmented adrenocortical disease

Autosomal dominant transmission

All of the above


Explanation: This autosomal dominant disorder is mapped
to chromosome 2p16 and may be due to a gain-of-function
mutation. The adrenal disorder is a pigmented nodular
adrenocortical disease with associated features such as
lentigines, blue nevi, cardiac and skin myxomas, and sexual
precocity in boys with large cell calcifying Sertoli tumors and
melanotic schwannomas. (See Chapter 571 in Nelson
Textbook of Pediatrics, 17th ed.)

Question . 16. A 10-yr-old girl has grown 4 cm and gained 7 kg in the past year.
She is diffusely obese with violaceous striae on her trunk and extremities. Her
24-hour urinary free cortisol is increased. The next step in her evaluation is:

MRI of her brain and pituitary

MRI of her abdomen

Measurement of morning cortisol

A 2-step dexamethasone suppression test


Explanation: This two-step test will help determine if ACTH
is of central origin. This is a classic clinical picture of Cushing
disease, which in children is most often due to a primary
adrenal disorder. (See Chapter 571 in Nelson Textbook of
Pediatrics, 17th ed.)

An ACTH stimulation test


Question . 17. A 12-yr-old boy is hypertensive with suppressed plasma renin
activity and elevated aldosterone. The next step in the evaluation is:

Dexamethasone suppression test


Explanation: Glucocorticoid-suppressible hypertension is a
primary adrenal disorder that must be evaluated with the
dexamethasone suppression test. (See Chapter 572 in
Nelson Textbook of Pediatrics, 17th ed.)

CT study of his abdomen

Adrenal vein catheterization

ACTH stimulation test

Sodium loading test

Question . 18. A 6-yr-old girl is hypertensive with hypokalemic alkalosis. Renin


activity is suppressed. The ratio of tetrahydrocortisol to tetrahydrocortisone is
increased in her urine. The most likely diagnosis is:

Glucocorticoid-remediable hyperaldosteronism

Apparent mineralocorticoid excess (11 -HSD deficiency)


Explanation: The hypokalemic alkalosis in a hypertensive
patient without abnormal genitalia or sexual precocity is
highly suggestive of a primary disorder of mineralocorticoid
excess. (See Chapter 572 in Nelson Textbook of Pediatrics,
17th ed.)

Liddle syndrome

Aldosterone-secreting adenoma

Bilateral micronodular adrenocortical hyperplasia

Question . 19. Glucocorticoid-remediable aldosterone is caused by:

A mutation in the gene for 11-hydroxylation

A mutation in the gene for 11 -HSD

A mutation in the gene for 17-hydroxylation

Formation of a hybrid gene between CYPI IB1 and CYPI


IB2
Explanation: This unique mechanism explains the
suppressibility of the hyperaldosteronism, with one gene
regulating the other.
A mutation in the gene for the mineralocorticoid receptor

Question . 20. A 10-yr-old girl has hyperaldosteronism that is unresponsive to


dexamethasone. No adrenal tumor is visualized on CT. The next step is:

Adrenal vein catheterization


Explanation: Local production of aldosterone can be
determined only by selective adrenal vein catheterization. A
small tumor may not be seen on CT. (See Chapter 572 in
Nelson Textbook of Pediatrics, 17th ed.)

Metyrapone administration

Spironolactone (Aldactone) administration

Exploratory laparotomy

Furosemide (Lasix) administration

Question . 21. Pheochromocytomas may be associated with:

Neurofibromatosis

Von Hippel-Lindau disease

MEN IIA and MEN IIB

Tuberous sclerosis

All of the above


Explanation: Pheochromocytomas may be isolated tumors,
but a good family history as well as physical examination may
reveal associated syndromes. (See Chapter 574 in Nelson
Textbook of Pediatrics, 17th ed.)

Question . 22. Presenting manifestations of pheochromocytoma may include:

Sustained hypertension

Abdominal pain

Polyuria and polydipsia

Weight loss

All of the above


Explanation: In addition, we have seen some patients
present with symptoms initiated only during exercise. (See
Chapter 574 in Nelson Textbook of Pediatrics, 17th ed.)
Question . 23. An 11-yr-old boy complains of increasing headaches and recent
visual changes affecting his lateral fields of vision. MRI of the brain reveals a
midline mass measuring 2 cm in diameter in the region of the anterior pituitary
gland. He undergoes transsphenoidal resection of the mass and is transferred to
the pediatric intensive care unit for postoperative care. Six hours later he is noted
to have a brisk urine output of 4-6 mL/kg/hr. Vital signs are stable: heart rate
120/min, blood pressure 120/75 mm Hg. He appears well hydrated. His serum
osmolality is 310 mOsm/kg and urine osmolality is 100 mOsm/kg. Which of the
following is the most likely diagnosis?

SIADH

Cerebral salt wasting

Normal postoperative diuresis

Central diabetes insipidus


Explanation: Interference with the transport or production of
hypothalamically synthesized arginine vasopression may be
a temporary or permanent defect following surgery in this
area of the brain. (See Chapter 553 in Nelson Textbook of
Pediatrics, 17th ed.)

Nephrogenic diabetes insipidus

Question . 24. A 10-yr-old boy receiving vincristine for treatment of a malignancy


has developed the syndrome of inappropriate antidiuretic hormone secretion
(SIADH), a known complication of this therapy. All of the following parameters
would be decreased in this patient except:

Urine production rate

Serum osmolality

Intravascular volume
Explanation: The pathophysiologic mechanism is excessive
free water retention and dilutional hyponatremia. Other
causes of dilutional hyponatremia such as heart failure or
cirrhosis are characterized by a reduction in effective renal
blood flow. This is not a problem in SIADH. (See Chapter 553
in Nelson Textbook of Pediatrics, 17th ed.)

Uric acid

Serum sodium
Question . 25. A 14-yr-old boy with bipolar disorder is seen for a health
maintenance visit. During the review of systems questioning, you note increased
thirst and frequency of urination. Current medications include risperidone
(Risperdal) and lithium. A random serum sodium is reported to be 148 mmol/L.
The most likely diagnosis in this patient is:

SIADH

Primary polydipsia

Adrenal insufficiency

Central diabetes insipidus

Nephrogenic diabetes insipidus


Explanation: Nephrogenic diabetes insipidus may be
congenital or acquired. In the acquired form, drugs (lithium)
and electrolyte disturbances (hypokalemia, hypercalcemia)
are common etiologic factors. (See Chapter 553 in Nelson
Textbook of Pediatrics, 17th ed.)

Question . 26. A 5-yr-old girl with a 6-mo history of excessive urination and thirst
is seen for an evaluation. She has no history of trauma, recent illness, or
medication use. Physical examination demonstrates a well-appearing child with
normal vital signs, stable weight, and normal findings on the neurologic
examination.You suspect diabetes insipidus.The most appropriate next step in
the management of this patient is:

Serum and urine electrolytes

MRI scan of the brain

Renal ultrasound study

Quantification of daily fluid input and output


Explanation: This is essential to confirm the history but
should not be done alone if there are signs of hypovolemia
that need immediate treatment. If the history is suggestive of
diabetes insipidus, a water deprivation test is performed,
followed by parenteral administration of arginine vasopressin
(DDAVP) if the deprivation test does not demonstrate
effective urine-concentrating ability. A response to DDAVP
suggests a central origin for the DI. (See Chapter 552 in
Nelson Textbook of Pediatrics, 17th ed.)

Water deprivation test


Question . 27. A 1-mo-old child is brought to the emergency department during a
heat wave after she has had a generalized seizure. A CT scan of the brain is
reportedly normal. The family notes having recently diluted the child's formula to
provide additional water. The most likely explanation for this child's seizures is:

Hyperthermia

Child abuse

Hypernatremia

Hyponatremia
Explanation: Excessive free water intake in children younger
than 6 mo can produce hyponatremia, which may manifest
with seizures, lethargy, and hypothermia. The hyponatremia
often corrects spontaneously, but if symptoms are present
when the hyponatremia is reported, hypertonic (3%) saline
may be used. (See Chapter 553 in Nelson Textbook of
Pediatrics, 17th ed.)

Febrile seizure

Question . 28. All of the following are important in the evaluation of tall patients
except:

Family history

IGF-1

Serum homocystine

Chromosomal analysis

Long-chain fatty acids


Explanation: Tall stature may be familial or due to pituitary,
chromosomal (XYY, XXY), or metabolic (Marfan syndrome,
homocysteinuria) disorders, obesity, or hyperthyroidism. (See
Chapter 556 in Nelson Textbook of Pediatrics, ed.)
Question . 29. A 3.5-kg male infant has a seizure at 7 days of age. The seizure
is generalized and lasts approximately 8 minutes. Measurement of serum
calcium in the emergency department reveals a level of 6.4 mg/dL. On
examination, the infant appears well but has a grade III/VI systolic murmur. The
most appropriate next step in diagnosis is:

EEG

Measurement of intact PTH after the serum calcium is


corrected

Measurement of intact PTH while the calcium is low


followed by a fluorescence in situ hybridization (FISH)
assay for DiGeorge syndrome
Explanation: This child has hypoparathyroidism and
congenital heart disease, a combination of manifestations
highly suggestive of DiGeorge/velocardiofacial syndrome.
This is caused by a deletion of chromosome 22q11-12, which
is readily detected with FISH techniques. (See Chapter 565 in
Nelson Textbook of Pediatrics, 17th ed.)

Measurement of serum phosphorus level

Question . 30. A 12-yr-old Hispanic boy with a history of a seizure disorder


presents to his pediatrician with a complaint of leg cramps. He is of normal
stature. His physical examination is unremarkable except for a Chvostek sign.
His serum calcium is 6.1 mg/dL with a serum phosphorus level of 9.4 mg/dL.
Serum level of intact PTH is 312 pg/mL (normal 10-60 pg/mL). The most likely
diagnosis is:

Primary hyperparathyroidism

Pseudohypoparathyroidism type 1B
Explanation: Pseudohypoparathyroidism (elevated PTH, low
calcium) of this type is not associated with phenotypic
morphologic features. In contrast, patients with type 1A have
brachydactyly, cataracts, mild mental retardation, and
calcification of basal ganglia. (See Chapter 566 in Nelson
Textbook of Pediatrics, 17th ed.)

Pseudohypoparathyroidism type 1A

A calcium-sensing receptor-activating mutation


Question . 31. A 14-yr-old boy has unilateral gynecomastia and is Tanner stage
3 in pubertal development. He complains of occasional episodes of breast
tenderness, which last less than 30 minutes and occur once a month. His serum
estradiol and prolactin levels are normal. The most appropriate next step in the
treatment of this patient is:

Reassurance and explaining the transient nature of the


breast findings
Explanation: The problem described in the question is
transient and compatible with the physiology of puberty in
normal males. Indeed, as many as 60% of boys have some
degree of pubertal gynecomastia. Some cases are familial.
(See Chapter 556 in Nelson Textbook of Pediatrics, 17th ed.)

Mammography

Abdominal CT including the pelvis

Head CT focusing on the pituitary

Karyotype

Question . 32. A 12-yr-old girl experiences muscle cramps and tingling of her
hands and feet unrelated to exertion. When she grabs a door handle to open the
door, she is unable to release her grasp because her hand is in spasm. The most
important laboratory test is:

Serum glucose determination

Serum calcium determination


Explanation: For the patient described, the serum total
calcium is 6.0 mg/dL, the phosphorus level is 8.5 mg/dL, and
the albumin is normal. (See Chapter 565 in Nelson Textbook
of Pediatrics, 17th ed.)

Electromyography (EMG)

Nerve conduction velocity testing

Arterial blood gas determination


Question . 33. Physical findings in Graves disease include all of the following
except:

Motor hyperactivity

Cold intolerance
Explanation: Patients with hyperthyroidism have heat
intolerance because of their hypermetabolism.

Tremor

Weight loss

Tachycardia

Smooth, flushed, warm skin

Question . 34. A mother and her 14½-yr-old daughter come to you because the
girl has not begun to menstruate. Her medical history and findings on the
complete physical examination are normal. Breast development and pubic hair
have been present for 18 months and are normal. Which of the following would
be the most appropriate next step in the management of this patient?

Reassurance that she probably will begin menstruating


within the year
Explanation: Reassurance about lack of menarche is
appropriate at age 14. Workup should begiin after age 16.

Laboratory evaluation for systemic disease

Urinary estriol determination

Buccal smear

Referral for psychologic counseling

Question . 35. Conditions associated with growth hormone deficiency include all
of the following except:

Cleft palate

Midfacial anomalies

Solitary maxillary central incisor

Optic nerve hypoplasia

VATER syndrome
Explanation: Facial anomalies, especially if short stature is
present, suggest pituitary and thus growth hormone
deficiency.
Question . 36. Children with growth hormone deficiency are best characterized
by all of the following except:

Low birthweight
Explanation: Most children with isolated GH deficiency are
of normal length and weight at birth. Those with multiple
pituitary hormone deficiencies may demonstrate a shorter
length. By age 1 yr, the children appear short but are not thin
or wasted, as occurs in severe caloric deprivation.

Delayed epiphyseal closure

Short stature by 1 yr of age

Round-shaped head

Hypoglycemia

Small hands and feet

Question . 37. Sotos syndrome is characterized by all of the following except:

Macrocrania

Large size at birth

Tall stature

Clumsiness

Normal intelligence
Explanation: Most patients have some degree of mental
retardation. Perceptual deficits are also common.

Question . 38. A 6-yr-old girl presents with breast enlargement and pubic hair
development. Otherwise, she is asymptomatic. Her serum LH level is elevated.
The most likely diagnosis is:

Hypothyroidism

Exposure to exogenous estrogen

An estrogen-secreting tumor

An adrenal tumor

Central precocious puberty


Explanation: Central precocious puberty involves idiopathic
release of LH and a normal, albeit early, progression of
puberty. (See Chapter 556 in Nelson Textbook of Pediatrics,
17th ed.)

Question . 39. The most appropriate next step in the evaluation of the child in
Question 38 is:

Serum prolactin level determination

Head MRI
Explanation: C or B. In the absence of neurologic or visual
signs, a pituitary or hypothalamic lesion is highly unlikely. In
girls, the risk of such a lesion is low; in boys, the risk of
identifying a CNS lesion is 25-75%. In addition, the lesions
are seldom malignant and rarely require neurosurgical
intervention. (See Chapter 556 in Nelson Textbook of
Pediatrics, 17th ed.)

No further testing
Explanation: C or B. In the absence of neurologic or visual
signs, a pituitary or hypothalamic lesion is highly unlikely. In
girls, the risk of such a lesion is low; in boys, the risk of
identifying a CNS lesion is 25-75%. In addition, the lesions
are seldom malignant and rarely require neurosurgical
intervention. (See Chapter 556 in Nelson Textbook of
Pediatrics, 17th ed.)

Serum testosterone test

Adrenal ultrasound study

Question . 40. Optimal treatment for the girl described in Questions 38 and 39 is:

Leuprolide
Explanation: Long-acting GnRH analogs in depot form help
prevent gonadotropic cell release of their hormones, thus
abating the early onset of puberty. (See Chapter 556 in
Nelson Textbook of Pediatrics, 17th ed.)

Growth hormone

Progesterone

Prednisone

Cranial irradiation
Question . 41. All of the following are characteristics of congenital
hypothyroidism except:

Higher incidence in females

Incidence of 1:4,000

Lower incidence in African-Americans

Central role of thyroid peroxidase antibodies


Explanation: These autoantibodies have a minor role, if any,
in the etiology of this disorder. However, thyroid gland growth
blocking and cytotoxic antibodies may have a role in the
etiology of congenital hypothyroidism. (See Chapter 559 in
Nelson Textbook of Pediatrics, 17th ed.)

Absence of symptoms at birth

Question . 42. A 6-wk-old infant has gained no weight since birth. Her skin
appears mottled, and an indirect bilirubin level is measured at 24 mg/dL. Her
extremities are cold, and her temperature is 35oC. The most likely diagnosis is:

Kernicterus

Sepsis

Galactosemia

Hypothermia

Hypothyroidism
Explanation: Congenital hypothyroidism may also
demonstrate large fontanels, feeding intolerance, distended
abdomen, constipation, prolonged sleep, and a poor cry.
(See Chapter 556 in Nelson Textbook of Pediatrics, 17th ed.)
Question . 43. The evaluation of the child described in Question 42 is best
accomplished by:

Serum TSH determination

Serum T4 assay
Explanation: The serum T4 is markedly depressed. Most
cases of congenital hypothyroidism are due to dysgenesis of
the thyroid gland, and therefore the TSH is elevated and a
thyroid scan shows no uptake or ectopic tissue. Nonetheless,
the serum T4 is the best of these tests. Central hypothalamic
pituitary causes have a low TSH. Therapy with T4 should be
initiated immediately. Fortunately, state screening for
congenital hypothyroidism has prevented this type of late
presentation with the inherent risks of mental retardation.
(See Chapter 559 in Nelson Textbook of Pediatrics, 17th ed.)

Bone age determination

Head CT

Thyroid scan

Question . 44. A 14-yr-old girl presents with poor growth and delayed puberty.
She denies headaches or poor school performance. She is physically sluggish
and has a small goiter, and her serum cholesterol is 500 mg/dL. The most likely
diagnosis is:

Hashimoto disease
Explanation: Hypothyroidism that is acquired is insidious in
onset and primarily affects growth. Schoolwork is not as
severely affected, as one would expect. (See Chapter 559 in
Nelson Textbook of Pediatrics, 17th ed.)

Graves disease

Congenital hypothyroidism

Familial type II hyperlipidemia

Pituitary prolactinoma
Question . 45. Common features of Graves disease include all of the following
except:

A 5:1 male:female ratio


Explanation: The female:male sex ratio is actually 5:1.

A 6-mo to 1-yr delay in diagnosis

Emotional disturbances

Poor school work

Tremors

Voracious appetite

Exophthalmos

Question . 46. Type 1 diabetes mellitus is most often associated with:

Mumps infection

Coxsackievirus infection

Antibodies to glutamic acid dehydrogenase


Explanation: Anti-GAD antibodies, also known as anti-islet
antibodies, are present in at least 90% of children with
insulin-dependent diabetes.

Cow's milk

Mitochondrial DNA deletions

Question . 47. Hyperglycemia during diabetic ketoacidosis may be associated


with:

Hypocalcemia

Hypernatremia

Hyponatremia
Explanation: Hyponatremia may be due to measurement
artifacts of serum glucose levels. Failure of the serum sodium
level to rise during therapy places the patient at risk for
cerebral edema, as the serum osmolarity drops below that in
the brain, resulting in shift of fluid to the CNS.
Hypomagnesemia

Hypocholesterolemia

Question . 48. Hyperkalemia in severe diabetic ketoacidosis is due to:

Renal failure

Hemolysis

Hyperglycemia

Artifact

Acidosis
Explanation: Transcellular shifts of hydrogen into the cell
with potassium leaving the cell during acidosis produce
transient hyperkalemia, which is usually reversed with
improvement in metabolism by insulin and improved tissue
perfusion from isotonic fluids. Hypokalemia may develop
during therapy with insulin; placing potassium salts in the
intravenous solution given to the patient may reduce this risk.
(See Chapter 583 in Nelson Textbook of Pediatrics, 17th ed.)
Nephrology
Question . 1. A 10-yr-old girl (body surface area of 1.0 m2) with chronic renal
insufficiency is seen in your clinic and undergoes a 24-hr urine collection for
measurement of creatinine clearance. The results are as follows: urine
creatinine 144 mg/dL; serum creatinine 1.7 mg/dL; urine volume 700 mL.
Based on these measurements, this patient's standard creatinine clearance
(mL/min/1.73 m2) is:
2
35 mL/min/1.73 m

60 mL/min/1.73 m2

40 mL/min/1.73 m2

2
70 mL/min/1.73 m
Explanation: (See Chapter 500 in Nelson Textbook of
Pediatrics, 17th ed.)

25 mL/min/1.73 m2

Question . 2. A 3-yr-old boy presents to your office with sudden onset of cola-
colored urine, progressive facial swelling over the past 3 days, and decreased
urine volume over the past day. His examination is notable for blood pressure
130/80 mm Hg, periorbital edema, bibasilar rales, and ankle swelling. His
urinalysis is remarkable for 3+ hematuria, 1+ proteinuria, 100 red blood cells
per high-power field, and red blood cell casts. His serum electrolytes are
normal and the serum albumin is 3.2 g/liter. This clinical presentation is most
consistent with:

Acute renal failure

Acute pyelonephritis

Nephrotic syndrome

Acute glomerulonephritis
Explanation: (See Chapters 500- 514 in Nelson Textbook of
Pediatrics, 17th ed.)

Chronic renal failure

Question . 3. In the case described in Question 2, which of the following


laboratory studies would be the most helpful in determining the cause of this
patient's hematuria and proteinuria?

Complete blood cell count

Serum cholesterol determination

Antistreptolysin O antibody level


24-hr urine collection for measurement of protein and
creatinine clearance

Complement C3 and C4 assays


Explanation: (See Chapters 500- 514 in Nelson Textbook of
th
Pediatrics, 17 ed.)

Question . 4. A 10-yr-old boy is noted to have hematuria and proteinuria on a


routine physical examination. He is without complaints, and examination
findings are normal. Results of blood chemistry studies are also normal, but
analysis of the 24-hr urine specimen reveals 2 g of protein and a normal
creatinine clearance. A renal biopsy is performed, which reveals mesangial
proliferative glomerulonephritis with very bright immunoglobulin A deposits
in the mesangium on immunofluorescence. Which of the following statements
is true regarding this child's form of glomerulonephritis?

This disease is more common in females

The primary treatment is blood pressure control


Explanation: (See Chapters 500- 514 in Nelson Textbook of
Pediatrics, 17th ed.)

Progressive kidney disease occurs in a majority of


children

The complement C3 value is usually low

Children with this disease rarely present with gross


hematuria

Question . 5. A 15-yr-old boy with a 12-yr history of microscopic hematuria is


noted to have bilateral high-frequency sensorineural hearing loss, blood
pressure of 140/90 mm Hg, serum creatinine of 1.5 mg/dL, and urinary protein
of 2,000 mg/24 hr. This patient's mother also has microscopic hematuria. The
most likely mode of inheritance for this child's glomerular disease is:

Autosomal dominant with incomplete penetrance

X-linked dominant
Explanation: (See Chapters 500- 514 in Nelson Textbook of
Pediatrics, 17th ed.)

Autosomal recessive

X-linked recessive

Autosomal dominant
Question . 6. A 5-yr-old girl presents with cola-colored urine, oliguria, and
body edema 2 wk after being treated for group A -hemolytic streptococcal
pharyngitis. Her complement C3 is noted to be very low at 15 mg/dL. When
should this patient's complement C3 level be repeated in order to confirm
your suspected diagnosis?

In 1 week

In 2 wk

In 3 wk

In 4 wk

In 8 wk
Explanation: (See Chapters 500- 514 in Nelson Textbook of
Pediatrics,

Question . 7. The most common clinical presentation for membranous


nephropathy in children is:

Asymptomatic microscopic hematuria

Acute nephritic syndrome

Nephrotic syndrome
Explanation: (See Chapters 500- 514 in Nelson Textbook of
Pediatrics, 17th ed.)

Complete absence of symptoms with normal results on


urinalysis

Acute renal failure


Question . 8. Which of the following statements about membranoproliferative
glomerulonephritis in children is true?

It occurs most commonly in the first decade of life

Hypocomplementemia usually resolves within 2 mo of


presentation

Alternate-day glucocorticoid therapy may be beneficial in


stabilizing the clinical course
Explanation: (See Chapters 500- 514 in Nelson Textbook of
Pediatrics, 17th ed.)

Progression to end-stage renal disease is rare

It is an uncommon cause of chronic glomerulonephritis


Question . 9. A 14-yr-old girl presents to your clinic with a 1-mo history of
fatigue. Over the past week, she has developed low-grade fevers with
temperatures to 100oF, bilateral knee pain, and chest pain with deep
inspiration. On examination, she is seen to be a tired-appearing adolescent in
no acute distress. Blood pressure is 130/80 mm Hg. Breath sounds are
diminished over the right lung base. Cardiac examination findings are normal.
Abdominal auscultation/palpation reveals no abnormalities. A urinalysis
reveals 3+ hematuria and 3+ proteinuria. Which of the following laboratory
studies is the most appropriate next step in confirming this patient's
diagnosis?

Sedimentation rate

Rheumatoid factor

Antinuclear antibody
Explanation: (See Chapters 500- 514 in Nelson Textbook of
th
Pediatrics, 17 ed.)

Anti-neutrophil cytoplasmic antibody

Kidney biopsy

Question . 10. The most appropriate initial treatment for the patient described
in Question 9 is:

Oral chorambucil

Oral prednisone
Explanation: (See Chapters 500- 514 in Nelson Textbook of
th
Pediatrics, 17 ed.)

Intravenous monthly cyclophosphamide infusions

Plasmapheresis

Conservative management by continued follow-up


evaluation in your clinic before initiation of further
treatment

Question . 11. A 3-yr-old boy presents to an urgent care clinic with a 3-day
history of abdominal pain and difficulty walking. Abnormal findings include
blood pressure of 120/80 mm Hg, diffuse abdominal tenderness, purpuric rash
of the hands and ankles, and diffuse periarticular tenderness and swelling of
the ankles. The most likely diagnosis is:

Systemic lupus erythematosus

Kawasaki's disease
Juvenile rheumatoid arthritis

Henoch-Sch nlein purpura


Explanation: (See Chapters 500- 514 in Nelson Textbook of
Pediatrics, 17th ed.)

Stevens-Johnson syndrome

Question . 12. All of the following glomerular diseases often manifest with
rapidly progressive glomerulonephritis except:

Wegener's granulomatosis

Systemic lupus erythematosus

Membranoproliferative glomerulonephritis

Goodpasture syndrome

Focal segmental glomerulosclerosis


Explanation: (See Chapters 500- 514 in Nelson Textbook of
Pediatrics, 17th ed.)

Question . 13. A 3-yr-old girl presents to your office with acute onset of
lethargy and pallor. The child's mother reports that the child had bloody
diarrhea for 5 days that cleared one day prior to presenting to your office. She
also notes acute onset of cola-colored urine. On examination, the patient is
pale and lethargic. Blood pressure is 120/80 mm Hg. The most appropriate
next step in diagnosis would be:

Urinalysis

X-ray examination of the abdomen

Urine culture

Complete blood cell count


Explanation: (See Chapters 500- 514 in Nelson Textbook of
Pediatrics, 17th ed.)

Prothrombin time
Question . 14. A 3-yr-old girl develops bloody diarrhea and pallor of acute
+
onset. A stool culture reveals E. coli O157:H7. Laboratory values include Na
+ -
130 mg/dL, K 5.5 mEq/L, Cl 90 mg/dL, total CO2 18 mEq/L, BUN 100 mg/dL,
and creatinine 4.0 mg/dL. All of the following are accepted treatments for this
patient except:

Antihypertensive pharmacotherapy to maintain blood


pressure below the 90th percentile for age and height

Fluid replacement at rate to cover insensible losses plus


urine output

Institution of antibiotic treatment against E. coli bacteria


Explanation: (See Chapters 500- 514 in Nelson Textbook of
Pediatrics, 17th ed.)

Early institution of dialysis

Aggressive nutrition

Question . 15. A full-term male newborn is noted to have gross hematuria of


acute onset associated with new bilateral abdominal flank masses at 24 hr of
life. All of the following may be contributing factors in this clinical scenario
except:

Dehydration

Perinatal asphyxia

Hypertension
Explanation: (See Chapters 500- 514 in Nelson Textbook of
Pediatrics, 17th ed.)

Sepsis

Maternal diabetes

Question . 16. An 8-yr-old girl presents with dysuria, abdominal pain, and
intermittent pink urine. A urinalysis reveals specific gravity of 1.020, pH of 6.0,
2+ hematuria, no protein, and 50 red blood cells per high-power field. A 24-hr
urine specimen reveals 6 mg/kg body weight of calcium. Which of the
following is an acceptable treatment for this patient's problem?

Increased intake of sodium-containing fluids

Dietary calcium restriction

Single daily dose of hydrochlorothiazide


Explanation: (See Chapters 500- 514 in Nelson Textbook of
Pediatrics, 17th ed.)

Vitamin D supplementation

Vitamin C supplementation

Question . 17. A full-term newborn male is noted to have bilateral flank


masses, hepatomegaly, and blood pressure of 120/80 mm Hg. A renal
ultrasound study reveals enlarged hyperechoic kidneys bilaterally and an
echogenic liver. Which of the following is correct regarding this patient's
diagnosis?

Hypertension is uncommon

This condition is inherited in an autosomal recessive


pattern
Explanation: (See Chapters 500- 514 in Nelson Textbook of
Pediatrics, 17th ed.)

This condition is associated with pancreatic cysts

Cerebrovascular hemorrhage is often seen in these


patients

Respiratory distress is uncommon in the neonatal period

Question . 18. A 15-yr-old boy is noted to have enlarged kidneys with


macrocysts seen bilaterally on a renal ultrasound study after developing
gross hematuria while playing hockey. His mother is 40 yr of age and
demonstrates similar findings on renal ultrasound examination. What is the
most appropriate next step in this patient's management?

Evaluate the patient for kidney transplantation

Check the blood pressure


Explanation: (See Chapters 500- 514 in Nelson Textbook of
Pediatrics, 17th ed.)

Obtain a cystogram to evaluate for vesicoureteral reflux

Start the patient on an antibiotic for prevention of urinary


tract infection

Obtain a renal ultrasound study of all siblings


Question . 19. A 15-yr-old boy is seen in your office for dysuria and is noted to
have 4+ hematuria and >100 red blood cells per high-power field on urinalysis.
All of the following organisms may cause infection leading to these symptoms
except:

Ureaplasma

Chlamydia

E. coli

Adenovirus

Enterovirus
Explanation: (See Chapter 500-514 in Nelson Textbook of
Pediatrics, 17th ed.)

Question . 20. All of the following statements are true except:

In humans, formation of nephrons is complete at birth but


functional maturation continues during the first decade of
life

The plasma filtered through the glomerular capillary walls


is cell free but contains all the substances in the plasma
Explanation: (See Chapters 500- 514 in Nelson Textbook of
Pediatrics, 17th ed.)

Fetal kidney function is not necessary for normal


intrauterine homeostasis

After birth, the glomerular filtration (GF) increases until


kidney growth ceases toward the end of the second
decade of life

Serum creatinine level does not rise above normal until the
GF rate falls by 30-40%

Question . 21. All of the following statements are true except:

Hematuria is defined as the presence of at least 5 red


blood cells (RBCs) in the urine

Urethrorrhagia refers to urethral bleeding in the presence


of urine
Explanation: (See Chapters 500- 514 in Nelson Textbook of
Pediatrics, 17th ed.)

Heme-positive urine without RBCs is due to either


hemoglobin or myoglobin.
False-negative results on Chemstrip testing may be due to
a urine preservative such as formalin

Screening urinalysis should be performed at well child


visits at 5 yr of age

Question . 22. All of the following statements are true except:

IgA nephropathy commonly manifests with gross


hematuria 1-2 days after the onset of an apparent viral
upper respiratory tract infection

Gross hematuria does not occur in patients with thin


glomerular basement membrane disease
Explanation: (See Chapters 500- 514 in Nelson Textbook of
Pediatrics, 17th ed.)

Gross hematuria occurs in Alport syndrome

The primary treatment of IgA nephropathy is proper blood


pressure control

The presence of anterior lenticonus is pathognomonic for


Alport syndrome

Question . 23. All of the following statements regarding poststreptococcal


glomerulonephritis (PSGN) are true except:

PSGN is common in children 2 to 5 yr of age


Explanation: (See Chapters 500- 514 in Nelson Textbook of
Pediatrics, 17th ed.)

The acute phase of PSGN usually resolves in 6-8 wk

Microscopic hematuria may persist for 1-2 yr following the


initial presentation of PSGN

The serum C3 level is usually reduced in the acute phase


of PSGN

The best single antibody titer to document cutaneous


streptococcal infection is the deoxyribonuclease (DNase)
B antigen
Question . 24. All of the following statements about hemolytic-uremic
syndrome (HUS) are true except:

HUS is the most common cause of acute renal failure in


young children

Verotoxin elaborated by Escherichia coli O157:H7 initiates


endothelial cell injury in HUS

HUS always presents after an enteritis with diarrhea


Explanation: (See Chapters 500- 514 in Nelson Textbook of
Pediatrics, 17th ed.)

The diagnosis of HUS requires microangiopathic


hemolytic anemia, thrombocytopenia, and acute renal
failure

Mortality from HUS is less than 10

Question . 25. All of the following statements regarding autosomal recessive


and autosomal dominant polycystic kidney disease (ARPKD and ADPKD) are
true except:

ARPKD typically presents in the 4th or 5th decade of life


Explanation: (See Chapters 500- 514 in Nelson Textbook of
Pediatrics, 17th ed.)

ADPKD is a systemic disorder affecting many organ


systems

The treatment of ARPKD and ADPKD is primarily


supportive

The presentation of ADPKD in older children has a


favorable prognosis

In about 85% of patients with ADPKD, the trait maps to the


PKD1 gene on the short arm of chromosome 16

Question . 26. Young infants are particularly susceptible to fluid, electrolyte,


or acid-base abnormalities during acute illnesses. All of the following are
characteristic of renal physiology in normal young infants except:

Renal tubular immaturity

Reduced glomerular filtration rate

Decreased urinary concentrating ability

Decreased nephron number


Explanation: (See Chapter 520 in Nelson Textbook of
Pediatrics, 17th ed.)

Diminished responsiveness to antidiuretic hormone (ADH)

Question . 27. A 3-mo-old patient hospitalized for bronchiolitis is noted on


admission laboratory studies to have a serum bicarbonate of 14 mmol/L. Of
the following, which is the most appropriate next step in the patient's
management?

Perform renal ultrasonography

Perform ammonium chloride loading test

Repeat serum electrolyte determination via venipuncture


Explanation: (See Chapter 521 in Nelson Textbook of
th
Pediatrics, 17 ed.)

Measure urine electrolytes, with calculation of urine anion


gap

Initiate treatment with oral bicarbonate supplementation

Question . 28. A 4-mo-old boy is noted to have poor growth at a routine well
child visit. Results of laboratory studies include serum sodium 140 mmol/L,
potassium 3.5 mmol/L, chloride 116 mmol/L, and bicarbonate 13 mmol/L. All of
the following should be considered in the differential diagnosis except:

Distal renal tubular acidosis

Chronic diarrhea

Proximal renal tubular acidosis

Lactic acidosis
Explanation: (See Chapter 521 in Nelson Textbook of
Pediatrics, 17th ed.)

Renal Fanconi syndrome

Question . 29. A 15-yr-old-girl is admitted with a 5-day history of worsening


fever, left flank pain, and vomiting. Physical examination reveals an ill-
appearing, dehydrated adolescent. Pulse rate is 110/min; temperature is 40°C.
Left costovertebral angle tenderness is present. Serum sodium is 131 mmol/L,
potassium 6.7 mmol/L, chloride 108 mmol/L, and bicarbonate 15 mmol/L. The
most likely diagnosis is:

Addison's disease
Ingestion of high potassium-containing foods

Distal renal tubular acidosis

Acute pyelonephritis
Explanation: (See Chapter 521 in Nelson Textbook of
Pediatrics, 17th ed.)

Congenital adrenal hyperplasia

Question . 30. A patient develops renal Fanconi syndrome after receiving


ifosfamide for treatment of Wilms tumor. All of the following are features of
this condition except:

Phosphaturia

Metabolic alkalosis
Explanation: (See Chapter 521 in Nelson Textbook of
Pediatrics, 17th ed.)

Rickets

Polyuria

Growth retardation

Question . 31. A 9-mo-old male infant is noted to have failure to thrive. At


birth, his height and weight were at the 25th percentile, but by 9 mo of age,
height and weight are at less than the 5th percentile. Laboratory tests reveal a
non-anion gap metabolic acidosis, with a serum potassium of 3.5 mmol/L.
Serum creatinine is 0.3 mg/dL. Urinalysis shows a urine pH of 8.0, with no
abnormalities. A diagnosis of distal renal tubular acidosis is made. Renal
ultrasonography is most likely to show:

Unilateral renal agenesis

Polycystic kidneys

Nephrocalcinosis
Explanation: (See Chapter 521 in Nelson Textbook of
Pediatrics, 17th ed.)

Small echogenic kidneys bilaterally

Enlarged kidneys
Question . 32. A 1-wk-old full-term male infant presents with irritability and
low-grade fever. His parents report that his urine output has been very high,
despite a decrease in oral intake. Physical examination reveals a moderately
to severely dehydrated infant. Serum sodium is 170 mmol/L. Serum osmolarity
is 340 mmol/kg. Urinalysis reveals a specific gravity of 1.000, with no protein,
blood, or leukocytes. Urine osmolarity is 240 mmol/kg. The patient is given
intravenous fluids. Massive polyuria is noted. Vasopressin is administered,
but no change in urine output or urine osmolarity is seen. The genetic defects
that cause this congenital condition result in:

Inability to respond to aldosterone

Inability to produce antidiuretic hormone

Overproduction of atrial natriuretic factor

Tubular unresponsiveness to antidiuretic hormone


Explanation: (See Chapter 522 in Nelson Textbook of
Pediatrics, 17th ed.)

Abnormal regulation of osmoreceptors in the


hypothalamus

Question . 33. A 16-yr-old girl has a 3-yr history of bipolar disorder and
seizures. In the last several months she has noted new onset of polyuria and
excessive thirst. Her current medications include lithium and valproic acid.
Findings on physical examination are unremarkable. All of the following are
potential causes of her recent symptoms except:

Psychogenic polydipsia

Lithium toxicity

Central diabetes insipidus

Congential nephrogenic diabetes insipidus


Explanation: (See Chapter 522 in Nelson Textbook of
th
Pediatrics, 17 ed.)

Diabetes mellitus

Question . 34. A 1-mo-old infant presents with lethargy. Physical examination


reveals a mildly to moderately dehydrated infant. Results of laboratory tests
include serum sodium 137 mmol/L, potassium 3.2 mmol/L, chloride 90
mmol/L, and bicarbonate 38 mmol/L. All of the following are potential causes
of this patient's laboratory findings except:

Pyloric stenosis

Surreptitious loop diuretic administration


Chronic respiratory insufficiency

Chronic diarrhea
Explanation: (See Chapter 523 in Nelson Textbook of
Pediatrics, 17th ed.)

Bartter syndrome

Question . 35. A 13-yr-old girl develops a sore throat and low-grade fever. A
throat culture is positive for group A streptococcal infection, for which she is
given oral penicillin. Seven days later, she develops a rash and fever. Urine
output is normal. Her pulse is 90/min; blood pressure is 110/60 mm Hg. Serum
3
creatinine is 2.4 mg/dL. WBC count is 12,000 per mm with 60% neutrophils,
25% lymphocytes, and 15% eosinophils. C3 level is normal. Urinalysis
demonstrates specific gravity of 1.010, small amount of blood, no protein, 5-10
WBCs per high-power field, 5-10 RBCs per high-power field, and no RBC
casts. The most likely diagnosis is:

Acute poststreptococcal glomerulonephritis

Toxic shock syndrome

Acute interstitial nephritis


Explanation: (See Chapter 524 in Nelson Textbook of
Pediatrics, 17th ed.)

Minimal-change nephrotic syndrome

Acute tubular necrosis

Question . 36. A 12-yr-old boy presents with a 1-yr history of worsening


polyuria and a 2- to 3-wk history of nausea, fatigue, and malaise. Serum
creatinine is 4.0 mg/dL; bicarbonate is 15 mg/dL. Urinalysis shows specific
gravity of 1.004, trace leukocytes, trace blood, and no protein, with 3-5 WBCs
per high-power field, 3-5 RBCs per high-power field, and no RBC casts. Which
of the following is the most likely diagnosis?

Acute poststreptococcal glomerulonephritis

Chronic interstitial nephritis


Explanation: (See Chapter 524 in Nelson Textbook of
Pediatrics, 17th ed.)

Minimal-change nephrotic syndrome

Acute interstitial nephritis

Chronic glomerulonephritis
Question . 37. A 3-yr-old boy presents with severe abdominal pain and
vomiting of acute onset, as well as fever with temperatures to 105°F, and
diarrhea that initially is watery but becomes grossly bloody. After stool
cultures are obtained and oral rehydration is begun, management should
include:

Loperamide

Empirical ceftriaxone administered parenterally


Explanation: (See Chapter 510 in Nelson Textbook of
Pediatrics, 17th ed.) Shigellosis is the most likely treatable
etiology for this acute dysenteric syndrome. Loperamide
may prolong illness. TMP/SMX is a poor choice for
empirical management because of the frequency of
resistant Shigella. A no-antibiotics approach is
inappropriate because waiting for culture results will leave
the child ill for many days and because stool culture,
although the best clinically available tool to confirm
shigellosis, has poor sensitivity.

Empirical trimethoprim-sulfamethoxazole

No antibiotics pending culture results

Question . 38. A 2-yr-old girl has an acute afebrile diarrheal syndrome


characterized by abdominal pain, vomiting, and grossly bloody stools. A stool
culture on MacConkey sorbitol media suggests E. coli O157:H7; a fecal toxin
electroimmunoassay suggests that a Shiga toxin is present. Appropriate care
includes:

Loperamide

An oral antibiotic (choice based on susceptibility of the


Shiga toxin-producing E. coli)

A parenteral antibiotic (choice based on susceptibility of


the Shiga toxin-producing E. coli)

Careful follow-up evaluation for development of


thrombocytopenia, anemia, and/or renal failure
Explanation: (See Chapter 510 in Nelson Textbook of
Pediatrics, 17th ed.) Both loperamide and antibiotic therapy
appear to increase the risk of HUS in E. coli O157
infection.

Question . 39. A 1-mo-old infant develops bloody diarrhea associated with


fever. The most likely agent causing this illness is:

Nontyphoidal Salmonella
Explanation: (See Chapter 510 in Nelson Textbook of
Pediatrics, 17th ed.) Salmonella is by far the most common
cause of febrile gastroenteritis in early infancy.
Campylobacter is second in frequency to Salmonella as a
bacterial cause of enteritis in infancy. Yersinia is a rare
cause. Shigellosis is a disease that is rare in infancy but
common in the 1- to 3-yr-old child. Rotavirus rarely (or
never) causes bloody diarrhea.

Shigella spp.

Campylobacter spp.

Yersinia spp.

Rotavirus

Question . 40. Isolation of S. ser. marina from an infant suggests:

The child is in contact with an iguana


Explanation: (See Chapter 510 in Nelson Textbook of
th
Pediatrics, 17 ed.) S. ser. marina is usually acquired by
contact with an iguana.

The child lives near a marina

The child lives near Marina, California

The child has been exposed to raw seafood

None of the above?the isolation of this serotype has no


special epidemiologic significance

Question . 41. Hemolytic-uremic syndrome is typically a complication of


infection with E. coli O157:H7 or other Shiga toxin-producing strain of E. coli.
The same process, microangiopathic hemolytic anemia with renal failure, can
also follow infection with:

Salmonella typhi or Campylobacter jejuni

Shigella fIexneri

Shigella sonnei

Shigella dysenteriae serotype 1


Explanation: (See Chapter 510 in Nelson Textbook of
Pediatrics, 17th ed.) Only Shigella dysenteriae serotype 1
commonly produces Shiga toxin and causes HUS.

Any species or serotype of Shigella


Question . 42. A 14-yr-old healthy girl has a urinalysis as part of her well child
visit. Urine dipstick testing shows specific gravity 1.014, pH 6.0, and 2+
proteinuria and is negative for blood. Microscopic examination of the urine is
unrevealing. The most appropriate next step in diagnosis is to:

Collect a 24-hr urine specimen for measurement of protein


and creatinine

Draw blood for a serum chemistry panel

Measure serum complement levels (C3, C4)

Perform urine dipstick testing on a first morning voided


sample
Explanation: (See Chapters 515-519 in Nelson Textbook of
Pediatrics, 17th ed.)

Measure the urine protein to creatinine ratio

Question . 43. Urine dipstick testing of a specimen obtained from a febrile 4-


yr-old child with acute viral gastroenteritis shows specific gravity 1.030, pH
5.0, 2+ proteinuria, and no blood cells. The most likely cause of the patient's
proteinuria is:

Transient proteinuria
Explanation: (See Chapters 515-519 in Nelson Textbook of
Pediatrics, 17th ed.)

Nephrotic syndrome

Orthostatic proteinuria

Acute glomerulonephritis

Chronic glomerulonephritis

Question . 44. A 3-yr-old boy presents with periorbital and pedal edema. The
differential diagnosis includes all of the following except:

Protein-losing enteropathy

Acute glomerulonephritis

Nephrotic syndrome

Hepatic failure

Orthostatic proteinuria
Explanation: (See Chapters 515-519 in Nelson Textbook of
Pediatrics, 17th ed.)
Question . 45. An asymptomatic 16-yr-old African-American girl with
hypertension is found to have 3+ proteinuria by dipstick testing on mid-day
and first morning voided urine samples. The microscopic analysis shows 0-2
red blood cells per high-power field. The most likely diagnosis is:

Postinfectious glomerulonephritis

Diabetic nephropathy

Minimal-change disease

Focal segmental glomerulosclerosis


Explanation: (See Chapters 515-519 in Nelson Textbook of
Pediatrics, 17th ed.)

Lupus nephritis

Question . 46. Initial evaluation of a child with fixed proteinuria should include
all of the following except:

Serum creatinine level

Complement level (C3)

Renal biopsy
Explanation: (See Chapters 515-519 in Nelson Textbook of
Pediatrics, 17th ed.)

24-hr urine collection for measurement of protein and


creatinine

Serum albumin level

Question . 47. A 12-yr-old girl with a history of vesicoureteral reflux and


recurrent pyelonephritis is seen for routine follow-up examination. Her blood
pressure is 140/90 mm Hg, and a first morning urinalysis reveals specific
gravity 1.015, pH 6.5, and 2+ proteinuria, and no blood. Which of the following
statements is appropriate regarding the patient's proteinuria?

The proteinuria is the consequence of reflux nephropathy


Explanation: (See Chapters 515-519 in Nelson Textbook of
Pediatrics, 17th ed.)

A 24-hr urine collection would probably show >1000 mg of


protein

The proteinuria is likely to be transient in nature

The major protein in this patient's urine is albumin

The primary renal pathology is in the glomerular


compartment
Question . 48. A 2-yr-old boy presents with a 2-wk history of gradually
increasing periorbital and pedal edema. Urine dipstick testing reveals 4+
proteinuria and is negative for blood. Diagnostic studies are likely to show all
of the following except:

Serum albumin 1.5 mg/dL

Serum cholesterol 130 mg/Dl


Explanation: (See Chapters 515-519 in Nelson Textbook of
Pediatrics, 17th ed.)

Serum creatinine 0.5 mg/dL

Complement C3 100 mg/dL

Urine protein to creatinine ratio 4.0

Question . 49. A 3-yr-old girl presents to the emergency department with


anasarca. The urinalysis shows 4+ proteinuria and is negative for blood.
Serum albumin is 1.2 mg/dL and serum creatinine is 0.4 mg/dL. The most
likely diagnosis is:

Postinfectious glomerulonephritis

Minimal-change disease
Explanation: (See Chapters 515-519 in Nelson Textbook of
Pediatrics, 17th ed.)

IgA nephropathy

Lupus nephritis

Focal segmental glomerulosclerosis

Question . 50. A 13-yr-old boy with newly diagnosed nephrotic syndrome is


seen in an outpatient clinic. Complications that may occur in this patient
include all of the following except:

Deep vein thrombosis

Spontaneous bacterial peritonitis

Weight gain

Behavioral change

Paresthesias
Explanation: (See Chapters 515-519 in Nelson Textbook of
Question . 51. An 8-yr-old boy with newly diagnosed nephrotic syndrome
enters remission 7 days after beginning treatment with high-dose prednisone
therapy (30 mg PO bid). The most appropriate next step in therapy is to:

Discontinue prednisone and monitor closely for relapse

Continue high-dose prednisone therapy for a total of 4-6


wk
Explanation: (See Chapters 515-519 in Nelson Textbook of
Pediatrics, 17th ed.)

Change to alternate-day prednisone dosing for the next 2


mo

Add cyclophosphamide therapy

Add cyclosporine therapy

Question . 52. A 5-yr-old boy presents with new-onset nephrotic syndrome.


Which of the following clinical findings is most likely?

Gross hematuria

Periorbital edema
Explanation: (See Chapters 515-519 in Nelson Textbook of
Pediatrics, 17th ed.)

Hypertension

Pleural effusions

Fever

Question . 53. A 4-yr-old girl is being treated for nephrotic syndrome relapse.
All of the following are appropriate treatment recommendations except:

Low-sodium diet

High-dose prednisone therapy

Adherence to schedule for all routine childhood


immunizations
Explanation: (See Chapters 515-519 in Nelson Textbook of
Pediatrics, 17th ed.)

Daily monitoring of urinary protein by parents

Participation in school and physical activities as tolerated


Question . 54. A 10-yr-old girl presents with edema and gross hematuria. Her
evaluation reveals serum creatinine 1.4 mg/dL, serum albumin 2.3 mg/dL, 24-
hr urine protein excretion 5.5 g, and C3 12 mg/dL. The most likely cause of her
nephrotic syndrome is:

Membranoproliferative glomerulonephritis
Explanation: (See Chapters 515-519 in Nelson Textbook of
Pediatrics, 17th ed.)

Minimal-change disease

IgA nephropathy

Membranous nephropathy

Focal segmental glomerulosclerosis

Question . 55. A newborn infant develops anasarca and poor urine output
during the first week of life. Serum creatinine is 0.3 mg/dL and serum albumin
is 1.0 mg/dL. Which of the following clinical findings is least likely?

Enlarged placenta

Prematurity

Rapid response to steroid therapy


Explanation: (See Chapters 515-519 in Nelson Textbook of
Pediatrics, 17th ed.)

Elevated maternal serum -fetoprotein

Massive proteinuria

Question . 56. A newborn infant develops nephrotic syndrome within the first
2 wk of life. Which of the following is the most likely cause of this patient's
nephrotic syndrome?

Congenital toxoplasmosis

Abnormality in the nephrin gene


Explanation: (See Chapters 515-519 in Nelson Textbook of
Pediatrics, 17th ed.)

Congenital syphilis

Abnormality in the polycystin gene

Maternal exposure to ACE inhibitors


Question . 57. A 16-yr-old girl is admitted to the intensive care unit with sepsis
following bone marrow transplantation. All of the following are appropriate
recommendations to prevent nephrotoxic injury except:

Substitute ultrasonography or MRI for contrast-enhanced


CT scanning

Substitute cephalosporin for aminoglycoside therapy

Adjust medication doses and intervals according to renal


function

Administer continuous infusion of renal-dose dopamine


Explanation: (See Chapters 515-519 in Nelson Textbook of
Pediatrics, 17th ed.)

Avoid nonsteroidal anti-inflammatory agents

Question . 58. A newborn boy is delivered by emergency cesarean section for


severe fetal distress and placental abruption. Resuscitation is required in the
delivery room. The infant develops gross hematuria and oliguric acute renal
failure with a peak serum creatinine level of 8.0 mg/dL at 8 days of age. The
most likely cause of the infant's acute renal failure is:

Renal dysplasia

Cortical necrosis
Explanation: (See Chapters 515-519 in Nelson Textbook of
Pediatrics, 17th ed.)

Obstructive uropathy

Acute glomerulonephritis

Prerenal azotemia

Question . 59. An 18-mo-old toddler develops severe hemolytic-uremic


syndrome from E. coli O157:H7 colitis. She remains anuric and dialysis-
dependent for 3 mo. Follow-up renal ultrasonography reveals a significant
decrease in renal size with increased renal echogenicity. The most
appropriate statement about this patient's condition is:

She probably had pre-existing chronic renal insufficiency

With continued dialysis support, she can be expected to


regain normal renal function within the next 3 mo

She has sustained irreversible renal injury from cortical


necrosis
Explanation: (See Chapters 515-519 in Nelson Textbook of
th
Pediatrics, 17 ed.)
Chronic dialysis is the preferred long-term treatment
option

This is a common clinical course for children with


hemolytic-uremic syndrome

Question . 60. A 2-year-old boy is admitted to the intensive care unit with
severe dehydration from bacterial gastroenteritis. Initial laboratory work
reveals BUN of 80 mg/dL and serum creatinine of 2.5 mg/dL. With rehydration,
his BUN and creatinine fall to 20 mg/dL and 0.5 mg/dL in 2 days. The most
likely diagnosis is:

Acute tubular necrosis

Hemolytic-uremic syndrome

Rhabdomyolysis syndrome

Prerenal acute renal failure


Explanation: (See Chapters 515-519 in Nelson Textbook of
Pediatrics, 17th ed.)

Acute glomerulonephritis

Question . 61. A 3-yr-old girl with congenital heart disease develops acute
renal failure during a prolonged stay in an intensive care unit. Possible
contributory factors include all of the following except:

E. coli cystitis
Explanation: (See Chapters 515-519 in Nelson Textbook of
Pediatrics, 17th ed.)

Nephrotoxic antibiotics

Hypotensive episodes

Contrast agent used for cardiac catheterization

Congestive heart failure


Question . 62. A full-term newborn infant boy has poor urine output and
demonstrates a rising serum creatinine over the first 4 days of life. Physical
examination reveals an enlarged bladder and no congenital anomalies. The
most likely cause of his acute renal failure is:

Prerenal acute renal failure

Obstructive uropathy
Explanation: (See Chapters 515-519 in Nelson Textbook of
Pediatrics, 17th ed.)

Acute tubular necrosis

Eagle-Barrett syndrome

Chronic glomerulonephritis

Question . 63. A previously healthy 6-yr-old child presents to the emergency


department with a 3-day history of malaise, poor oral intake, diarrhea, and
decreased urine output. Physical examination reveals tachycardia, dry
mucous membranes, and sunken eyes. Results of initial laboratory studies
include BUN 65 mg/dL, potassium 5.5 mEq/L, bicarbonate 14 mEq/L, and
serum creatinine 3.0 mg/dL. The most appropriate next step in treatment is to:

Order a renal ultrasound study

Administer Kayexalate 1 g/kg per rectum

Run intravenous replacement fluid at rate to replace


insensible losses plus urine output

Give NaHCO3 1 mEq/kg IV to correct acidosis

Administer normal saline 20 mL/kg bolus over 30 min


Explanation: (See Chapters 515-519 in Nelson Textbook of
th
Pediatrics, 17 ed.)

Question . 64. A 14-yr-old girl develops acute renal failure related to Henoch-
Sch nlein purpura glomerulonephritis. Laboratory findings may include all of
the following except:

Hypercalcemia
Explanation: (See Chapters 515-519 in Nelson Textbook of
th
Pediatrics, 17 ed.)

Hyperkalemia

Hyponatremia

Hyperphosphatemia

Hypoalbuminemia
Question . 65. A 12-yr-old boy presents with a long-standing history of
polyuria and polydipsia, progressive fatigue, decreased appetite, morning
nausea and emesis, weight loss, and impaired growth velocity. In addition, he
has had no response to a 6-mo course of iron therapy for treatment of anemia.
Initial laboratory evaluation reveals BUN of 125 mg/dL and serum creatinine of
8.7 mg/dL. Other expected laboratory features include all of the following
except:

Elevated parathyroid hormone level

Increased anion gap metabolic acidosis

Decreased levels of growth hormone


Explanation: (See Chapters 515-519 in Nelson Textbook of
th
Pediatrics, 17 ed.)

Small, echogenic kidneys on ultrasonography

Hypocalcemia

Question . 66. A 16-yr-old boy with focal segmental glomerulosclerosis has a


serum creatinine of 1.9 mg/dL and a 24-hr urinary protein excretion of 1800
mg. All of the following strategies may theoretically help to slow the
progression of chronic renal failure except:

Careful control of systemic hypertension

Reduction of proteinuria using ACE inhibitor therapy

Administration of epidermal growth factor


Explanation: (See Chapters 515-519 in Nelson Textbook of
Pediatrics, 17th ed.)

Normalization of serum calcium/phosphorus balance

Treatment of metabolic acidosis with NaHCO3

Question . 67. A 5-yr-old boy with Eagle-Barrett syndrome and renal dysplasia
has a rising serum creatinine of 4.5 mg/dL. The most appropriate statement
regarding patient management is:

At least a short period of dialysis is necessary before the


possibility of renal transplantation is considered

Transplantation offers the best opportunity for full


rehabilitation for children with ESRD
Explanation: (See Chapters 515-519 in Nelson Textbook of
Pediatrics, 17th ed.)

In most cases, ESRD in North American children is treated


with hemodialysis
With his current degree of renal dysfunction, he may
follow an unrestricted diet

Dialysis or transplantation should be considered only


when he becomes symptomatic from ESRD (fatigue,
decline in school performance, anorexia, and pruritus)
Urologic Disorders in Infants and Children

Question . 1. Potter phenotype may be due to:

Renal agenesis

Renal dysplasia

Obstructive uropathy

Severe amniotic fluid leak

None of the above

All of the above


Explanation: Primarily encountered in renal agenesis or
dysplasia, the Potter phenotype (flattened face, broad
nose, low-set ears, receding chin, clubfoot) is also due to
severe oligohydramnios. (See Chapter 529 in Nelson
Textbook of Pediatrics, 17th ed.)

Question . 2. Risk factors for urinary tract infections include all of the
following except:

Uncircumcised penis

Sexual activity

Reflux nephropathy

Double-ureter systems

Chronic use of antibiotics


Explanation: Chronic use of antibiotics is not a risk factor
for urinary tract infection. All the other choices are
significant risks. (See Chapter 530 in Nelson Textbook of
Pediatrics, 17th ed.)

Spina bifida

Question . 3. The presence of renal parenchymal scarring due to


vesicoureteral reflux is best determined by:

DMSA scan
Explanation: A DMSA radionuclide scan helps to define
images that are accurate representations of chronic renal
scarring. (See Chapter 530 in Nelson Textbook of
Pediatrics, 17th ed.)

Renal ultrasonography
VCUG

CT scan

Intravenous pyelography

Question . 4. The most common abdominal mass in a neonate is:

Renal dysplasia-hydronephrosis
Explanation: Renal masses are the most common lesions
in neonates with an abdominal mass. Hydronephrosis and
multicystic-dysplastic lesions are the most common renal
masses. (See Chapter 529 in Nelson Textbook of
Pediatrics, 17th ed.)

Wilms tumor

Neuroblastoma

Meckel diverticulum

Ovarian teratoma

Question . 5. The primary pathology in classic Potter syndrome is best


characterized as:

Oligohydramnios

Renal agenesis
Explanation: Renal agenesis results in no fetal urine
output, resulting in oligohydramnios. The reduced
amniotic fluid produces fetal constraint and compression,
resulting in the abnormal physical features (C, D, E). (See
Chapter 529 in Nelson Textbook of Pediatrics, 17th ed.)

Pulmonary hypoplasia

Facial deformation

Skeletal dysplasia

Question . 6. Multicystic dysplastic kidneys are characterized by all of the


following except:

Usually unilateral

Incidence of 1:2,000
Autosomal dominant inheritance
Explanation: Multicystic dysplastic kidneys are not
inherited. In contrast, polycystic kidneys are bilateral;
inheritance is either autosomal dominant (adult) or
autosomal recessive (child). (See Chapter 529 in Nelson
Textbook of Pediatrics, 17th ed.)

Most common neonatal abdominal mass

No function

Question . 7. A 12-yr-old presents with headaches and a blood pressure of


210/110 mm Hg. There is no history of recent infection. Results of urinalysis
are unremarkable. The most appropriate next step in evaluation is to:

Measure serum complement

Perform a renal ultrasound examination


Explanation: Renal ultrasonography demonstrates a small
left kidney with a deep groove in the lateral convexity of
the kidney. The Doppler scan of blood flow in the renal
arteries appears normal. (See Chapter 529 in Nelson
Textbook of Pediatrics, 17th ed.)

Perform a voiding cystourethrogram

Measure streptococcal titers

Perform a renal arteriogram

Question . 8. The most likely diagnosis for the patient described in Question 7
is:

Polycystic renal disease

Multicystic renal disease

Segmental hypoplasia
Explanation: Segmental hypoplasia, or Ask-Upmark
kidney, produces severe hypertension, usually beginning
at age 10 yr when identified on routine examination.
Nephrectomy is the treatment of choice. (See Chapter 529
in Nelson Textbook of Pediatrics, 17th ed.)

Renal infarction

Neurofibromatosis
Question . 9. All of the following statements concerning the epidemiology of
urinary tract infections in children are true except:

Average age of affected females is 3 yr

Average age of affected males is less than 1 yr

Circumcision reduces the risk

Bladder reflux increases the risk

Staphylococcus saprophyticus is the most common


pathogen in male infants
Explanation: E. coli and other gram-negative enteric
pathogens (e.g., Proteus, Klebsiella) are the most common
pathogens for UTIs at all ages. (See Chapter 530 in Nelson
Textbook of Pediatrics, 17th ed.)

Question . 10. Cystitis is associated with all of the following except:

Urgency

Adenovirus

Fever
Explanation: Cystitis is not usually associated with fever.
With fever, chills, or rigors, suspect pyelonephritis with or
without urosepsis. (See Chapter 530 in Nelson Textbook of
Pediatrics, 17th ed.)

Absence of renal scarring

Sexual activity in females

Question . 11. Risk factors for urinary tract infection include all of the
following except:

Pinworms

Constipation

Pregnancy

Neurogenic bladder

Henoch-Sch nlein purpura


Explanation: HSP is not a risk factor for UTI. Other risk
factors include use of "bubble bath," incorrect perineal
hygiene practices (wiping from back to front), reflux,
instrumentation, wearing tight underwear, and the
potential pathogens listed for Question 9. (See Chapter
530 in Nelson Textbook of Pediatrics, 17th ed.)
Question . 12. From the following list, choose the imaging study or studies
needed to evaluate a 9-wk-old infant with pyelonephritis.
1. Intravenous pyelogram
2. CT scan
3. DMSA scan
4. Renal ultrasound examination
5. Voiding cystourethrogram
6. Cystoscopy

2 only

1, 3, and 6

3, with 2 in selected cases

4, 5, and possibly 3
Explanation: Renal ultrasonography demonstrates renal
anomalies, obstruction, renal enlargement
(pyelonephritis), or renal abscess. Unfortunately, it misses
many renal scars. A VCUG is needed because
vesicoureteral reflux is a common cause of UTI in children
younger than 6 yr. A DMSA scan identifies acute
pyelonephritis (if pyelonephritis is evident clinically, the
role for DMSA is less important). A DMSA scan also shows
scarring and is valuable to follow the progression of
scarring and the possible need for further intervention.
(See Chapter 530 in Nelson Textbook of Pediatrics, 17th
ed.)

Question . 13. Vesicoureteral reflux is associated with all of the following


except:

Contralateral kidney of pair with unilateral dysplasia

Ureteral duplication

Familial inheritance

Ureterocele

Asymptomatic bacteriuria
Explanation: All the rest are important risk factors for
reflux. Of note, approximately 35% of siblings of a child
with reflux have reflux; 50% of the children of a mother
with reflux have reflux. Reflux (primary) is uncommon in
African-American children. (See Chapter 531 in Nelson
Textbook of Pediatrics, 17th ed.)
Question . 14. A 7-mo-old white male infant presents with failure to thrive and
a BUN of 75 mg/dL. He has a history of a poor urinary stream. The most likely
diagnosis is:

Renal artery stenosis

Renal hypoplasia

Urogenic bladder

Posterior urethral valves


Explanation: Posterior urethral valves may be detected by
in utero ultrasound examinations or after birth in children
with a big bladder, poor stream, UTI, or failure to thrive.
The parents of every boy should be questioned about the
baby's urinary stream. (See Chapter 532 in Nelson
Textbook of Pediatrics, 17th ed.)

Nephrolithiasis

Question . 15. A 6-yr-old girl has a long history of urinary frequency and
urgency. She also has nocturnal enuresis. In addition, she has urge
incontinence. The most likely diagnosis is:

Unstable bladder
Explanation: This is a classic presentation of the pediatric
unstable bladder. The bladder is smaller than normal and
exhibits strong uninhibited contractions. Constipation and
UTI may complicate the disorder. Treatment is with
frequent, timed voiding and anticholinergic drugs. (See
Chapter 535 in Nelson Textbook of Pediatrics, 17th ed.)

Wilms tumor

Constipation

Chronic cystitis

Nephrolithiasis

Question . 16. A 10-yr-old boy manifests testicular pain and swelling of acute
onset that is not relieved by acetaminophen. The next step in management is
to:

Apply ice

Check for a history of Chlamydia infection

Immediately refer to a urologist


Explanation: This child has testicular torsion until proven
otherwise. If the torsion is not relieved, testicular
infarction may result in a necrotic organ in a sexually
active male. Gonorrhea and Chlamydia epididymitis must
also be considered. (See Chapter 537 in Nelson Textbook
of Pediatrics, 17th ed.)

Perform laparoscopy

Apply a scrotal support device


Gynecologic Problems of Childhood

Question . 1. Which of the following is the ideal patient position for


examination of the prepubertal child with a vaginal discharge?

Lithotomy

Fowler

Frog-leg
Explanation: The frog-leg position is the least threatening
position and provides the best approach to visualize the
perineum. If this is not satisfactory, the knee-chest
position is employed. (See Chapter 540 in Nelson
Textbook of Pediatrics, 17th ed.)

Prone

Question . 2. Clitoromegaly is defined as a clitoral diameter of:

2 mm

3 mm

5 mm

10 mm
Explanation: Clitoromegaly is defined by width, and in an
adolescent a clitoris is wider than 10 mm is considered
enlarged. Other signs of virilization should be looked for
such as facial hair, baldness, acne, and any palpable
gonads. (See Chapter 540 in Nelson Textbook of
Pediatrics, 17th ed.)

Question . 3. Which of the following is useful in the diagnosis of vaginitis in


the pediatric patient?

Litmus paper

Leukocyte esterase test


Explanation: The leukocyte esterase test will identify the
presence of inflammatory white cells in the discharge. It is
a good screening test. (See Chapter 541 in Nelson
Textbook of Pediatrics, 17th ed.)

Urinalysis

Scotch tape test


Question . 4. Vulvovaginitis is most often associated with:

-Streptococci

Enterococci

Coliform bacteria
Explanation: Nonspecific vulvovaginitis in prepubertal
females is often due to enteric flora, and the specimen
most often contains coliform organisms. That due to
group A -hemolytic streptococci is particularly serious,
accompanied by fever, intense pain, and discharge. (See
Chapter 541 in Nelson Textbook of Pediatrics, 17th ed.)

Pseudomonas

Coagulase-positive staphylococci

Question . 5. Gartner's duct cyst is a remnant of:

The wolffian duct


Explanation: Also called a mesonephric cyst, this remnant
is usually asymptomatic. If it causes dyspareunia, it may
require resection. (See Chapter 545 in Nelson Textbook of
Pediatrics, 17th ed.)

The müllerian duct

The urogenital sinus

A hydatid cyst of Morgagni

Question . 6. Malassezia furfur rash is caused by:

Group B streptococci

Pityrosporum orbiculare
Explanation: Malassezia furfur is the new name for
Pityrosporum orbiculare and causes a rash characterized
by scaly macules on the trunk, face, or genital region. (See
Chapter 541 in Nelson Textbook of Pediatrics, 17th ed.)

Veillonella parvula

Propionibacterium
Question . 7. The agent most commonly associated with folliculitis is:

Staphylococcus aureus
Explanation: Infection of the hair follicles is most often
due to S. aureus. In patients who have recently bathed in
hot tubs, infections with Pseudomonas aeruginosa must
also be considered. (See Chapter 541 in Nelson Textbook
of Pediatrics, 17th ed.)

Escherichia coli

Streptococcus pyogenes

Borrelia

Question . 8. Behçet disease is characterized by:

Recurrent oral ulcers

Recurrent genital ulcers

Uveitis

All of the above


Explanation: Behçet disease may produce painful genital
ulceration in association with other systemic
manifestations such as uveitis and intestinal ulceration.
(See Chapter 151 in Nelson Textbook of Pediatrics, 17th
ed.)

Question . 9. Behçet disease is treated with:

Acyclovir

Topical estrogen cream

Colchicine
Explanation: This inflammatory multisystem disorder
responds to oral colchicine. (See Chapter 151 in Nelson
th
Textbook of Pediatrics, 17 ed.)

Metformin
Question . 10. The most common germ cell tumor of the ovary is:

Seminoma

Dysgerminoma
Explanation: Dysgerminomas are the most common
malignant germ cell tumor of the ovary. Survival
approaches 80%. Benign teratomas are more common.
(See Chapter 545 in Nelson Textbook of Pediatrics, 17th
ed.)

Endodermal sinus tumor

Gonadoblastoma

Question . 11. Dysgerminoma is most often associated with:

XY gonadal dysgenesis
Explanation: Most dysgerminomas are associated with XY
gonadal dysgenesis. Y-DNA chromosome probes help with
this diagnosis. (See Chapter 545 in Nelson Textbook of
th
Pediatrics, 17 ed.)

RNA-based diagnosis to confirm etiology

Bilaterality

D Female genotype

Question . 12. Ovarian malignancies in pediatric patients are evaluated with:

-Fetoprotein (AFP)

Carcinoembryonic antigen (CEA)

CA-125

All of the above


Explanation: Various embryonal probes are available in
the diagnosis and follow-up of ovarian cancers in children.
Because active ovarian cancers may express these
embryonic markers, they are useful in diagnosis and
identification of relapse. (See Chapter 545 in Nelson
Textbook of Pediatrics, 17th ed.)
Question . 13. Endodermal sinus tumor produces a predominance of

CA-125

Carcinoembryonic antigen

-Fetoprotein
Explanation: -Fetoprotein is a useful marker in the
management and follow-up of patients with an endodermal
sinus tumor. (See Chapter 545 in Nelson Textbook of
Pediatrics, 17th ed.)

Human chorionic gonadotropin

Question . 14. The most common system associated with a müllerian anomaly
is:

Skeletal

Gastrointestinal

Urinary
Explanation: Disorders affecting the embryonic urogenital
sinus are common in children with müllerian origin
anomalies. Spina bifida may also occur with müllerian
defects. (See Chapter 546 in Nelson Textbook of
Pediatrics, 17th ed.)

Cardiovascular

Question . 15. Vertical fusion defects can result in:

Septate uterus

Didelphic uterus

Bicornuate uterus

Transverse vaginal septum


Explanation: Patients with a transverse vaginal septum
often present with amenorrhea, cyclic abdominal pain, and
a mass. There is an increased risk for endometriosis. (See
Chapter 546 in Nelson Textbook of Pediatrics, 17th ed.)
Question . 16. A 17-yr-old Tanner stage 2 girl presents with a history of
bilateral spontaneous milky discharge from her breasts for 2 mo. Menarche
was at age 12 yr, and her periods had been regular until 4 mo before this visit
to your office. In addition, she complains of headache on awakening for the
past 2 wk. The most useful screening test is:

Urine pregnancy test

Serum pregnancy test

Serum prolactin level


Explanation: Patients with spontaneous galactorrhea
should be screened by determining their prolactin level.
(See Chapter 543 in Nelson Textbook of Pediatrics, 17th
ed.)

Serum estrogen level

Serum luteinizing hormone level

Question . 17. The prolactin level of the young woman described in Question
16 is 1,000 times higher than normal. The next test in her evaluation should
be:

Cranial MRI study


Explanation: Magnetic resonance imaging (MRI) or
computed tomography (CT) of the cranium demonstrates a
pituitary prolactinoma. Hypothyroidism also produces
hyperprolactinemia and galactorrhea. The treatment of
some small prolactinomas includes bromocriptine
(Parlodel). Larger or persistently symptomatic lesions
require surgery. (See Chapter 543 in Nelson Textbook of
Pediatrics, 17th ed.)

Abdominal CT study

Pelvic ultrasonography

Uterine biopsy

Mammography

Question . 18. A 7-yr-old girl complains of a brown-green discharge on her


underwear. She has no fever or labial tenderness and denies sexual contact.
Her mother states that for the past 4 mo her daughter has been taking ballet
classes and frequently sleeps in her leotards. The most likely diagnosis is:

Nonspecific vaginitis
Explanation: Nonspecific vaginitis most often occurs in
prepubertal girls who wear tight-fitting clothing (leotards)
or are exposed to vaginal irritants (soaps) or have poor
hygiene. It is often due to coliform bacteria or group A
streptococci. (See Chapter 554 in Nelson Textbook of
Pediatrics, 17th ed.)

Gardnerella vaginalis vaginitis

Gonorrhea

Chlamydial vaginitis

Candida vaginitis

Question . 19. Initial therapy for the girl described in Question 18 should
include all of the following except:

Instruction in perineal hygiene

Sitz baths

Use of mild soaps

Avoiding tight clothing

Metronidazole
Explanation: Metronidazole is not indicated for this form of
nonspecific vaginitis. If the process is recurrent,
amoxicillin may be of value in addition to the suggestions
in choices A-D. (See Chapter 541 in Nelson Textbook of
Pediatrics, 17th ed.)

Question . 20. The treatment of choice for labial adhesions is:

Topical erythromycin

Oral erythromycin

Oral estrogens

Topical estrogens
Explanation: Topical estrogen cream each evening for 1
wk is effective in over 90% of cases. Thorough cleansing
followed by application of petroleum ointment for 1-2 mo
helps prevent recurrences. (See Chapter 541 in Nelson
Textbook of Pediatrics, 17th ed.)

Topical progesterone
Question . 21. Mastodynia is best characterized as:

Dependent on breast size

Beginning 18 mo after menarche


Explanation: Breast pain occurs in cyclic patterns at the
time of each menstrual cycle. (See Chapter 543 in Nelson
Textbook of Pediatrics, 17th ed.)

Noncyclic in nature

Unrelated to the menstrual cycle

Unresponsive to nonsteroidal anti-inflammatory agents

Question . 22. Maternal exposure to DES places the female offspring at


increased risk for:

Ovarian cancer

Clear cell adenocarcinoma of the vagina


Explanation: DES is an in utero carcinogen. (See Chapter
545 in Nelson Textbook of Pediatrics, 17th ed.)

Cervical prolapse

Sarcoma botryoides

Melanoma
CNS
Question . 1. To be effective for preventing myelomeningocele, administration of
folic acid needs to begin:

By 3 months of gestation

At the first missed period

Before conception
Explanation: Folic acid supplementation has been a major
public health success in reducing the incidence of neural tube
defects in the United States. Supplementation also reduces the
recurrence rate of neural tube defects in families with a
previously affected child. (See Chapter 585 in Nelson Textbook
of Pediatrics, 17th ed.)

By 30 days of gestation

At 3 months prior to delivery

Question . 2. A newborn with myelomeningocele is being evaluated, and the


parents ask about the likelihood of the development of hydrocephalus. Which of the
following best describes the risk of hydrocephalus in this situation?

The risk increases with higher spine lesions


Explanation: Hydrocephalus is quite common in children with
a myelomeningocele and must be evaluated with a head
ultrasound study or CT. (See Chapter 585 in Nelson Textbook
of Pediatrics, 17th ed.)

The risk decreases with higher spine lesions

The risk is unknown at birth

The risk is less than 25%

The risk is related to the cause of the malformation


Question . 3. A 4-yr-old boy is evaluated for his first generalized tonic-clonic
seizure, which lasted 10 min. There is no history of illness or fever, and findings on
examination an hour after the seizure are completely normal. The most appropriate
management is:

Begin therapy with carbamazepine

Order an EEG
Explanation: An EEG will help define the presence of a
seizure focus and if there is a specific seizure type. A normal
EEG may help the pediatric neurologist determine if
anticonvulsant therapy is needed. (See Chapter 586 in Nelson
Textbook of Pediatrics, 17th ed.)

Order a CT scan of the brain

Order an MRI study of the brain

Order psychometric testing

Question . 4. A 15-mo-old girl is evaluated for a 10-min-long generalized seizure


associated with a temperature of 40oC. Which of the following factors in the history
is most likely to increase the risk of future seizures?

APGAR score of 3 at 5 min

Family history of epilepsy


Explanation: Future nonfebrile seizures can be predicted
when there is a family history of seizures. Up to 30% of children
with febrile seizures have a recurrence when they become
febrile; these are often unpredictable. (See Chapter 586 in
Nelson Textbook of Pediatrics, 17th ed.)

Clinical evidence of roseola

Female gender

Presence of 2 café-au-lait spots


Question . 5. It has been determined that sodium valproate is the best
anticonvulsant medication for a 12-yr-old boy with epilepsy. His parents ask about
the possibility of fatal liver necrosis if he takes the drug. Which of the following
factors markedly increases the risk of this complication?
Male gender

Age younger than 2 yr


Explanation: Young age is a risk factor for hepatic and
perhaps pancreatic complications of valproic acid. It is much
safer in 12-yr-old children than those younger than 2 yr.

Down syndrome

Family history of epilepsy

Allergic reactions to phenobarbital

Question . 6. The chances of severe life-threatening skin reactions such as


Stevens-Johnson syndrome are greatest with which of the following anticonvulsant
medications?
Phenobarbital

Gabapentin

Carbamazepine

Lamotrigine
Explanation: In addition, phenobarbital and carbamazepine
are also associated with an acute drug hypersensitivity
syndrome involving skin, mucous membranes, and the liver.

Sodium valproate

Question . 7. A 3-yr-old boy is being evaluated after an episode at home during


which he lost consciousness for 5 min; he was brought to the emergency
department an hour later. On examination, which of the following factors is most
helpful in distinguishing whether this episode was a seizure or syncope?
Family history

Temperature

Blood pressure

Level of consciousness
Explanation: Syncope usually produces a loss of tone and a
fall, with rapid recovery of consciousness once the affected
person is in a horizontal position. Occasionally there may be a
few tonic-clonic jerking movements with syncope, thus
confusing the assessment.
Size of pupils

Question . 8. A 5-yr-old child is referred with a 6-wk history of morning headaches,


often associated with vomiting. His parents have noted that during this period he
has become irritable and moody. Which of the following would be most likely to be
identified during the physical examination?

Marked elevation of blood pressure

Tenderness on percussion of frontal sinuses

Papilledema
Explanation: This child has a brain tumor and increased
intracranial pressure. Papilledema takes time to develop, must
be looked for in all patients with headaches, and is an ominous
sign. (See Chapter 588 in Nelson Textbook of Pediatrics, 17th
ed.)

Loud orbital bruit

Significant refractive error

Question . 9. A 6-yr-old child with neurofibromatosis (NF1) is found to have an


optic glioma on a routine MRI study, confined to the right optic nerve. Findings on
the neurologic, physical, and retinal examinations are normal. The visual acuity is
20/20 bilaterally, uncorrected. Which of the following is the correct management?

Surgical removal of the tumor

MRI of the optic nerve every 3 mo

Chemotherapy

Annual examination by a pediatric ophthalmologist


Explanation: Patients with NF1 and an optic glioma as in this
case can be observed over time. In many situations the tumor
is slow growing and produces few problems. (See Chapter 589
in Nelson Textbook of Pediatrics, 17th ed.)

Radiation therapy limited to the right optic nerve


Question . 10. A 10-yr-old girl is being evaluated for new onset of school problems,
obsessive-compulsive behavior, and occasional uncontrolled movements of the
hands. She has been healthy and has not taken any medications. Further
evaluation is most likely to reveal:
Brain tumor in the posterior fossa

Partial complex epilepsy

Evidence of streptococcal infection


Explanation: Poststreptococcal obsessive-compulsive disorder
is a well-recognized clinical entity.

Hydrocephalus

Tardive dyskinesia

Question . 11. A 5-yr-old girl is evaluated for progressive difficulty in walking, which
seems to worsen during the day; her walking is much better after a good night's
sleep. Examination during the afternoon shows that she has rigidity in the leg
muscles and dystonic twisting of her feet. This kind of movement disorder often
responds dramatically to which of the following medications?

Sodium valproate

Lorazepam

C. L-Dopa
Explanation: L-Dopa is the treatment of choice for hereditary
progressive dystonia with marked diurnal variation, also called
Segawa disease. This is one of the dopa-responsive dystonias.

Gabapentin

Lithium

Question . 12. A 2-yr-old boy with the spastic diplegia form of cerebral palsy is
being evaluated. MRI of his brain is most likely to show:

Multicystic encephalomalacia

Periventricular leukomalacia
Explanation: PVL is a common observation in children with
CP. It is first observed in the neonatal period in both term and
preterm infants. If identified in the neonatal period and
extensive, it is a very strong predictor of CP.

Normal anatomy
Basal ganglia abnormalities

Agenesis of the corpus callosum

Question . 13. A 6-mo-old female infant is evaluated after the mother notes that
she is strongly left-handed and delayed in sitting and in using the right hand. MRI of
the brain reveals a large porencephalic cyst in the distribution of the left middle
cerebral artery. Which of the following information is most likely to contribute to
establishing the cause of this disorder?

Results of an EEG

Chromosome analysis

Evaluation for thrombophilic disorders


Explanation: The MRI study reveals the sequelae of a
neonatal stroke. Although neonatal strokes are not always due
to thrombosis, the child must be evaluated for
hypercoagulability. (See Chapter 591 in Nelson Textbook of
Pediatrics, 17th ed.)

History of delivery

History of maternal drug ingestion

Question . 14. Which of the following therapies has been found to be effective in
long-term follow-up for some patients with the X-linked neurodegenerative disease
adrenoleukodystrophy?

Vegetable oil as a dietary supplement

Hyperbaric oxygen therapy

Bone marrow transplantation


Explanation: Performed prior to the onset of severe
symptoms, bone marrow transplantation has had some value in
improving the clinical course in some patients with X-linked
adrenoleukodystrophy. (See Chapter 592 in Nelson Textbook
of Pediatrics, 17th ed.)

Total volume exchange blood transfusion

Plasmapheresis
Question . 15. A 5-yr-old girl is evaluated for severe mental retardation,
microcephaly, hand-wringing movements, poor growth and weight gain, and
seizures. Genetic analysis indicates a mutation in a transcription factor called
MeCP2 that normally functions to silence transcription of numerous genes. This
finding indicates that she has which of the following disorders?

Juvenile Huntington disease

Metachromatic leukodystrophy

Rett syndrome
Explanation: Rett syndrome is classically described by the
features of this case. (See Chapter 592 in Nelson Textbook of
Pediatrics, 17th ed.)

Adrenoleukodystrophy

Menkes disease

Question . 16. An 11-yr-old boy had been fighting and wrestling with another boy at
school and sustained several bruises on the face and neck. The next morning he
woke up with inability to speak clearly and weakness on the right side of his body,
greater in the arm than in the leg. The most likely cause for his neurologic disorder
is:

Left-sided brain tumor

Focal seizure disorder

Carotid artery intimal dissection


Explanation: Strokes due to carotid dissection can occur
spontaneously or with neck twisting or by flexion-extension of
the neck and have been reported following roller coaster rides
or visits to the beautician. In most cases the lesion heals
without residual neurologic deficits. (See Chapter 593 in Nelson
Textbook of Pediatrics, 17th ed.)

Venous sinus thrombosis

Cervical spinal cord injury


Question . 17. A 5-yr-old child has a 6-mo history of increasing difficulty in walking
associated with urinary incontinence. Physical examination shows an alert child
with increased deep tendon reflexes, clonus and bilateral Babinski reflexes in the
lower extremities, and absent deep tendon reflexes in the upper extremities
associated with grade 3/5 weakness symmetrically in all extremities. The most likely
diagnosis is:

Myasthenia gravis

Spinal cord tumor


Explanation: The findings described in this patient are typical
of an upper motor lesion. The bilateral nature and the other
features localize the lesion to the spinal cord. (See Chapter 579
in Nelson Textbook of Pediatrics, 17th ed.)

Guillain-Barré syndrome

Metachromatic leukodystrophy

Peripheral neuritis

Question . 18. A 7-mo-old girl is presented to the emergency department with


gradual onset of fever, lethargy, and irritability. Her immunizations are up to date.
Examination reveals a febrile infant who does not interact with the examiner and
cries inconsolably. A lumbar puncture is performed, and the cerebrospinal fluid
contains 1,500 white blood cells/mm3, 84% of which are granulocytes; a glucose
concentration of 12 mg/dL; and a protein concentration of 70 mg/dL. Gram stain is
negative. The most likely etiologic agent for this infection is:

An enterovirus

Herpes simplex virus

Haemophilus influenzae type b

Neisseria meningitides
Explanation: Both meningococci and even pneumococci
(vaccine covers only some but not all pneumococci) are the
most common causes of non-neonatal bacterial meningitis. The
H. influenzae type b vaccine has almost eliminated this
pathogen as a cause of serious bacterial infections in children.
(See Chapter 594 in Nelson Textbook of Pediatrics, 17th ed.)

Group B streptococci
Question . 19. The greatest risk of meningitis following documented occult
bacteremia is associated with infection caused by:

Streptococcus pneumoniae

Haemophilus influenzae type b

Neisseria meningitides
Explanation: Meningococci may rarely cause transient
bacteremia, but a positive blood culture for this pathogen is
usually associated with a localized infection in the meninges,
lung, joint, or elsewhere. (See Chapter 362 in Nelson Textbook
of Pediatrics, 17th ed.)

Salmonella species

Question . 20. The most common sequela associated with bacterial meningitis is

Mental retardation

Chronic seizure disorder

Impaired vision

Impaired hearing
Explanation: Hearing deficits are most common and are
potentially attenuated by steroids. In addition, a higher
incidence of neuropsychiatric disturbances is noted following H.
influenzae type b infection. (See Chapter 594 in Nelson
Textbook of Pediatrics, 17th ed.)

Behavioral disturbance

Question . 21. The most common cause of viral meningoencephalitis is:

An enterovirus
Explanation: Enteroviral disease is quite common in the
summer and fall, producing an aseptic meningitis. PCR
techniques have helped to identify this organism. (See Chapter
594 in Nelson Textbook of Pediatrics, 17th ed.)

Herpes simplex virus

An arbovirus

Mumps virus
A respiratory virus

Question . 22. A 4-yr-old girl has experienced progressive loss of ambulation over
a 2-yr period. On examination, the child is apathetic and uninterested in her
surroundings. She has horizontal nystagmus and optic atrophy. Her voice is
dysarthric. She is hypotonic, and her deep tendon reflexes are absent. A sibling
died at the age of 6 yr with a similar history. The motor nerve conduction velocities
show marked slowing, and computed tomography (CT) of the head shows diffuse
symmetric attenuation of the cerebral and cerebellar white matter. The most likely
diagnosis is:

Multiple sclerosis

Metachromatic leukodystrophy
Explanation: Metachromatic leukodystrophy is a familial
degenerative disease affecting both the central nervous system
(CNS) and peripheral nervous system white matter?hence the
loss of deep tendon reflexes with CNS symptoms. (See
Chapter 592 in Nelson Textbook of Pediatrics, 17th ed.)

GM2 gangliosidosis (Tay-Sachs disease)

Neuronal ceroid lipofuscinosis

Acute demyelinating encephalomyelitis

Question . 23. A 3-yr-old girl has a 2-wk history of fever associated with bifrontal
headache, lethargy, and vomiting. She has a history of perioral cyanosis and
dyspnea with exertion beginning in infancy. She suddenly has a 10-min focal tonic-
clonic seizure. The child is obtunded and has a temperature of 100.8°F (38.2°C),
pulse of 118/min, and blood pressure of 96/70 mm Hg in her right arm, measured
while she is supine. Perioral cyanosis is noted at rest. A harsh pansystolic murmur
is heard best along the left sternal border. Examination of her eye grounds reveals
bilateral papilledema. She has right-sided weakness associated with hyperreflexia
and an extensor plantar reflex. The most likely cause of the hemiparesis is:

Moyamoya disease

A brain tumor

Neurocysticercosis

Methemoglobinemia

A brain abscess
Explanation: This young girl has tetralogy of Fallot and a brain
abscess resulting in part from the right-to-left cardiac shunt.
Predisposing factors for brain abscesses in other patients
include chronic otic and sinus infections. (See Chapter 595 in
Nelson Textbook of Pediatrics, 17th ed.)
Question . 24. Causes of megalocephaly include all of the following except:

Thalassemia

Chronic subdural effusions

Hydrocephalus

Canavan disease

Congenital CMV infection


Explanation: Congenital cytomegalovirus (CMV) infection
usually causes microcephalus, not macrocephalus. Expansion
of the bone marrow (hemolytic anemias), storage diseases
(lysosomal, leukodystrophies), excessive cerebrospinal fluid
(CSF) (hydrocephalus), intracranial bleeding (subdural), and
familial factors contribute to megalocephaly. (See Chapter 585
in Nelson Textbook of Pediatrics, 17th ed.)

Familial factors

Question . 25. A 12-yr-old girl experiences acute monocular blindness of 2 days'


duration. Past medical history reveals that she has had headaches for the past 3 yr
that she cannot characterize, one brief episode of diplopia, and one episode of
paresthesias of the feet. These episodes were not related in time, did not occur in
immediate proximity to the headache, and resolved spontaneously. Findings on
physical examination, including the funduscopic examination, are unremarkable
other than reduced visual acuity. The most important diagnostic step is to perform:

CT

MRI
Explanation: MRI is most useful in confirming the diagnosis of
a possible demyelinating disease such as multiple sclerosis.
MRI demonstrates small 3- to 4-mm plaques compatible with
the disease. The MRI study should include the brain and spinal
cord. (See Chapter 592 in Nelson Textbook of Pediatrics, 17th
ed.)

An electroencephalogram

Peripheral nerve conduction tests

A nerve biopsy
Question . 26. A 12-year-old presents with a history of severe headache, a grand
mal seizure, and sudden collapse with unresponsive flaccid coma. The patient had
a history of intermittent right-sided headaches without an aura and at times without
relief with rest. In addition to coma on physical examination, the patient is afebrile
and has nuchal rigidity. The most likely diagnosis is:

Bacterial meningitis

Tuberculous meningitis

Brain tumor

Arteriovenous malformation
Explanation: Arteriovenous malformation, like an aneurysm,
may rupture, producing hemiplegia or coma. Blood in the
subarachnoid space produces nuchal rigidity and may be
detected by CT or a carefully performed lumbar puncture. (See
Chapter 584 in Nelson Textbook of Pediatrics, 17th ed.)

Pott puffy tumor

Question . 27. Papilledema of acute onset is associated with all of the following
except:

Reduced visual activity


Explanation: Acutely, the visual acuity is usually not reduced
in papilledema. This is in contrast with papillitis noted during
optic neuritis; affected patients have poor visual acuity.

Hyperemia of the optic nerve

Constricted arterioles of the optic disc

Dilated disc veins

Indistinct optic nerve border

Question . 28. The Gowers sign demonstrates:

Poor reflexes

Spinal dysraphism

Tethered cord

Proximal motor weakness


Explanation: Motor weakness, especially of the thighs,
requires the child to stand from a supine position by using the
arms to "climb" up the legs to stand erect.

Hysterical paralysis

Question . 29. Characteristics of simple partial seizures include all of the following
except:

Loss of consciousness
Explanation: There is never a loss of consciousness in simple
partial seizures. Actually, some patients may talk to those
around them during the event. (See Chapter 586 in Nelson
Textbook of Pediatrics, 17th ed.)

Versive seizures

Duration of 10-20 sec

No postictal state

Abnormal EEG findings

Question . 30. Rasmussen encephalitis is characterized by all of the following


except:

Epilepsia partialis continue

Onset before age 10 yr

No sequelae
Explanation: Sequelae may include hemiplegia, hemianopia,
or aphasia. (See Chapter 586 in Nelson Textbook of Pediatrics,
17th ed.)

Abnormal EEG findings

Possible role of CMV infection

Question . 31. Landau-Kleffner syndrome is characterized by all of the following


except:

Onset at age 5 yr

More common in girls


Explanation: The disorder is more common in boys. The
etiology is unknown, and the treatment of choice is valproic
acid. (See Chapter 586 in Nelson Textbook of Pediatrics, 17th
ed.)

Aphasia
Normal hearing

Multiple seizure types

Question . 32. Neurofibromatosis type 1, an autosomal dominant disorder (gene on


chromosome 17), is defined by six or more café-au-lait macules greater than 5 mm
in diameter in prepubertal children or greater than 15 mm in postpubertal children
plus at least one of the following except:

Axillary or inguinal freckling

Lisch nodules of the iris

Two or more neurofibromas or one plexiform neurofibroma

Ash leaf macule


Explanation: The ash leaf-shaped hypopigmented macule is
most typical of tuberous sclerosis (being present in over 90% of
affected children), another autosomal dominant disorder. (See
Chapter 589 in Nelson Textbook of Pediatrics, 17th ed.)

Osseous lesions (sphenoid dysplasia, scoliosis)

Optic gliomas

An affected first-degree relative

Question . 33. A 19-yr-old girl presents with headache, unsteadiness, and poor
hearing that has worsened over the past 5 yr. Her father's medical history includes
some type of brain surgery, and he has been deaf since the age of 35 yr. The most
likely diagnosis is:

Neurofibromatosis type 2
Explanation: NF2 accounts for 10% of all NF cases, has
distinctive chromosomal sites, and is characterized by bilateral
acoustic neuromas. Café-au-lait macules may not be present.
(See Chapter 589 in Nelson Textbook of Pediatrics, 17th ed.)

Optic glioma

Neurofibromatosis type 1

Tuberous sclerosis

Late-onset congenital deafness


Question . 34. Physical features of Sydenham chorea include all of the following
except:

Hypertonia
Explanation: Hypotonia is the rule. (See Chapter 590 in
Nelson Textbook of Pediatrics, 17th ed.)

Milkmaid's grip

Choreic hand

Darting tongue

Emotional lability
NEUROMUSCULAR
Question . 1. A 12-yr-old girl experienced diarrhea, which lasted for 3 days, 2 wk
before manifesting progressive weakness and inability to walk. She has intermittent
tingling of her fingers and toes. Physical examination reveals marked peripheral
muscle weakness without atrophy or fasciculations. The deep tendon reflexes are
absent in her ankles and 1+ at her knees. Findings on the sensory examination are
normal. Motor involvement is symmetric. The most likely diagnosis is:

Transverse myelitis

Guillain-Barré syndrome
Explanation: Guillain-Barré syndrome is an ascending
peripheral polyneuropathy that is predominantly motor but may
have mild sensory symptoms (paresthesias). An upper
respiratory tract infection or diarrhea (often due to
Campylobacter) often precedes the onset of paralysis. (See
Chapter 607 in Nelson Textbook of Pediatrics, 17th ed.)

Polio

Myasthenia gravis

Mononeuritis multiplex

Question . 2. The patient described in Question 1 is admitted to the hospital and


now has progressive weakness and areflexia of the knees and ankles. An important
test to perform is:

Determination of urine specific gravity

Electrocardiogram (ECG)

Serum creatine kinase determination

Muscle biopsy

Pulmonary function testing


Explanation: Pulmonary function tests, such as measurement
of negative inspiratory force, are helpful in detecting impending
respiratory failure due to intercostal or phrenic nerve
involvement. Reductions in inspiratory force often precede
abnormalities of the arterial blood gases (hypercarbia, hypoxia)
and should be monitored frequently in any patient with acute
progressive muscle weakness. (See Chapter 607 in Nelson
Textbook of Pediatrics, 17th ed.)
Qestion . 3. A 15-yr-old boy has lost his ability to walk. On physical examination,
his ankle and knee deep tendon reflexes are noted to be diminished. The weakness
is greatest in peripheral muscles. Cranial nerves all are normal. One week before
these symptoms arose, he returned from a camping trip. The most likely diagnosis
is:

Myasthenia gravis

Organophosphate poisoning

Spinal muscular atrophy

Botulism

Tick paralysis
Explanation: Ticks (wood or dog) may produce a motor-
sensory neuropathy indistinguishable from Guillain-Barré
syndrome. On removal of the tick (often on the scalp), the
paralysis rapidly resolves. (See Chapter 605 in Nelson
Textbook of Pediatrics, 17th ed.)

Question . 4. A 10-yr-old girl has had diplopia and ptosis and weakness of her neck
flexors for 2 mo. Symptoms are worse in the evening and are usually less severe
on awakening in the morning. She has no fasciculations or myalgias, and her deep
tendon reflexes are 1-2+. The most likely diagnosis is:

Hysterical weakness

Muscular dystrophy

Spinal muscular atrophy

Botulism

Myasthenia gravis
Explanation: Myasthenia gravis is characterized by
progressive muscle weakness that is exacerbated by repetitive
muscle use. Classically, the facial and extraocular muscles are
involved. (See Chapter 603 in Nelson Textbook of Pediatrics,
17th ed.)
Question . 5. A 4-yr-old has difficulty in climbing stairs, slow motor development,
and hypertrophied calf muscles. The most likely diagnosis is:

Myasthenia gravis

Myotonia congenita

Duchenne muscular dystrophy


Explanation: Duchenne muscular dystrophy is also called
pseudohypertrophic muscular dystrophy. (See Chapter 600 in
Nelson Textbook of Pediatrics, 17th ed.)

Hypokalemic periodic paralysis

Central core disease

Question . 6. All of the following statements regarding creatine kinase (CK) are true
except:

It is the most useful serum enzyme reflecting damaged muscle


fibers

The MM isozyme is found primarily in skeletal muscle

The MB isozyme is found primarily in cardiac muscle

The BB isozyme is found primarily in smooth muscle


Explanation: Creatine kinase (CK) is one of several lysosomal
enzymes released by damaged or degenerating muscle fibers
and is the most useful in laboratory measurement of these
enzymes in serum. CK is found in only three organs and may
be separated into corresponding isozymes: MM for skeletal
muscle, MB for cardiac muscle, and BB for brain. (See Chapter
598 in Nelson Textbook of Pediatrics, 17th ed.)
Question . 7. All of the following statements regarding congenital neuromuscular
disorders are true except:

Most are hereditary

Most are nonprogressive conditions

The definitive diagnosis is best made by electromyography


(EMG)
Explanation: The definitive diagnosis of congenital
neuromuscular disorders is best determined by histopathologic
findings in the muscle biopsy sample. Most of the congenital
myopathies are hereditary; some are sporadic. In a few
conditions for which the defective gene has been identified, the
diagnosis may be established using the specific molecular
probe on lymphocytes. (See Chapter 599 in Nelson Textbook
of Pediatrics, 17th ed.)

Diagnosis for some disorders may be confirmed by genetic


analyses of lymphocytes

Hypotonia is a common feature

Question . 8. Characteristic features of muscular dystrophies include which of the


following?

They are a primary myopathy

They have a genetic basis

The course is progressive

Degeneration and death of muscle fibers occur at some stage


of disease

All of the above


Explanation: The muscular dystrophies are a group of
unrelated disorders, each transmitted by a different gene and
each differing in its clinical course and expression. The
muscular dystrophies are distinguished from other neurogenic
disorders by the four obligatory criteria listed in the question.
(See Chapter 600 in Nelson Textbook of Pediatrics, 17th ed.)
Question . 9. All of the following statements regarding Duchenne muscular
dystrophy are true except:

It is the most common hereditary neuromuscular disease

It is inherited as an autosomal dominant trait


Explanation: Duchenne muscular dystrophy, the most
common hereditary neuromuscular disorder, is inherited as an
X-linked recessive trait. The gene is on the X chromosome at
the Xp21 locus. (See Chapter 600 in Nelson Textbook of
Pediatrics, 17th ed.)

Symptoms are rarely present at birth or in early infancy

The serum creatine kinase is consistently greatly elevated

It is more common in males

Question . 10. Characteristic clinical manifestations of Duchenne muscular


dystrophy include:

Cardiomyopathy

Intellectual impairment

Weakness of respiratory muscles

Scoliosis

All of the above


Explanation: Cardiomyopathy is a constant feature of
Duchenne muscular dystrophy, although the severity of cardiac
involvement does not necessarily correlate with the degree of
skeletal muscle weakness. Intellectual impairment occurs in all
patients, although only 20-30% have an intelligence quotient
(IQ) of less than 70. Scoliosis is common. (See Chapter 600.1
in Nelson Textbook of Pediatrics, 17th ed.)
Question . 11. All of the following are associated with constant muscle weakness
except:

Hypothyroidism

Hyperparathyroidism

Corticosteroids

Hyperaldosteronism (Conn syndrome)


Explanation: Thyrotoxicosis causes proximal muscle
weakness and wasting accompanied by electromyogram
(EMG) changes. Hypothyroidism, whether congenital or
acquired, consistently produces proximal muscle weakness
and hypotonia. Hyperparathyroidism causes weakness and
reversible muscle wasting. Both natural Cushing disease and
exogenous corticosteroid administration may cause proximal
muscle weakness. Hyperaldosteronism is accompanied by
episodic and reversible muscle weakness. (See Chapter 598 in
Nelson Textbook of Pediatrics,17th ed.)

Question . 12. All of the following statements regarding malignant hyperthermia are
true except:

It is inherited as an autosomal dominant trait

Acute episodes are typically precipitated by intravenous


administration of dyes for radiographic studies
Explanation: Acute episodes of malignant hyperthermia are
precipitated by exposure to general anesthetics and
occasionally to local anesthetic drugs. Acute attacks may be
prevented by administration of dantrolene sodium before an
anesthetic is administered. (See Chapter 602.2 in Nelson
Textbook of Pediatrics, 17th ed.)

Myoglobinuria may result in tubular necrosis and acute renal


failure

Attacks may be prevented by administration of dantrolene


sodium

Metabolic acidosis may be severe


Question . 13. Which of the following is the etiology in most cases of myasthenia
gravis?

Inheritance as a recessive trait

Inheritance as an X-linked trait

Postinfectious, usually after either influenza or chickenpox

Autoimmune disorder
Explanation: Myasthenia gravis is an autoimmune disorder. A
rare familial form is probably an autosomal recessive trait but is
not associated with plasma anti-anti-acetylcholine antibodies.
(See Chapter 603 in Nelson Textbook of Pediatrics, 17th ed.)

Idiopathic

Question . 14. Which of the following is the earliest and most consistent sign of
myasthenia gravis?

Gowers sign

Trendelenburg gait

Ptosis and extraocular muscle weakness


Explanation: Ptosis and some degree of extraocular muscle
weakness are the earliest and most constant signs in
myasthenia gravis. Older children may complain of diplopia.
(See Chapter 603.1 in Nelson Textbook of Pediatrics, 17th ed.)

Respiratory muscle weakness

Head lag
Question . 15. The best method for diagnosis of myasthenia gravis is:

Assay for anti-acetylcholine antibodies

Nerve conduction velocity (NCV) studies

Electromyogram (EMG)
Explanation: Myasthenia gravis is one of the few
neuromuscular diseases in which an electromyogram (EMG) is
more diagnostic than muscle biopsy. A decremental response
occurs in response to repetitive nerve stimulation; the muscle
potentials diminish rapidly in amplitude until the muscle
becomes refractory to further stimulation. Motor nerve
conduction velocity remains normal. Plasma anti-acetylcholine
antibodies should be assayed but are inconsistently found,
being present in only one third of adolescents. (See Chapter
603.1 in Nelson Textbook of Pediatrics, 17th ed.)

Nerve biopsy

Muscle biopsy

Question . 16. Which of the following is a common clinical manifestation of


myasthenia gravis?

Cardiomyopathy

Intellectual impairment

Weakness of respiratory muscles


Explanation: If untreated, myasthenia gravis is usually
progressive and may become life-threatening because of
respiratory muscle weakness. Cardiomyopathy is not a feature
of myasthenia gravis, and electrocardiogram (ECG) findings
remain normal. (See Chapter 603.1 in Nelson Textbook of
Pediatrics, 17th ed.)

Scoliosis

Headache
Question . 17. A 1-day-old newborn, born to a myasthenic mother, has generalized
hypotonia and weakness. Which of the following best describes the prognosis?

Greatly increased risk of the complete picture of myasthenia


gravis

Small but increased risk of the complete picture of myasthenia


gravis

No increased risk of myasthenia gravis


Explanation: After the abnormal (maternally derived)
antibodies disappear, offspring born to myasthenic mothers
have normal muscle strength and are not at increased risk for
myasthenia gravis in later life. (See Chapter 603.1 in Nelson
Textbook of Pediatrics, 17th ed.)

Some residual weakness until puberty but no increased risk of


myasthenia gravis

Episodes of diminishing severity of muscle weakness until


puberty

Question . 18. A 4-yr-old child with a history of poor sucking and swallowing as an
infant recently experienced excessive sweating and blotchy erythema, especially
when excited. Walking is clumsy. There has been new onset of episodes of cyclic
vomiting lasting 24-27 hr, with retching and vomiting every 15-20 min as well as
profuse sweating, blotchy erythema, and evidence of apprehension and irritability.
The most likely diagnosis is:

Duchenne muscular dystrophy

Fabry disease

Chronic organophosphate intoxication

Familial dysautonomia (Riley-Day syndrome)


Explanation: Familial dysautonomia (Riley-Day syndrome) is
usually expressed in infancy as poor sucking and feeding.
Autonomic crises usually begin after age 3 yr. (See Chapter
606.1 in Nelson Textbook of Pediatrics, 17th ed.)

Guillain-Barré syndrome
Question . 19. A 4-yr-old child presents with symmetric weakness that began in the
lower extremities and subsequently progressed over 10-14 days to involve the trunk
and upper limbs. Deep tendon reflexes are absent. There are no sensory deficits or
bowel or bladder dysfunction. Nerve conduction velocity test results are abnormal.
The cerebrospinal fluid shows protein of 78 mg/dL and 5 white blood cells. The
most likely diagnosis is:

Duchenne muscular dystrophy

Fabry disease

Familial dysautonomia (Riley-Day syndrome)

Bell palsy
Explanation: Guillain-Barré syndrome is a postinfectious
polyneuropathy that causes demyelination, primarily in the
motor nerves. The onset is typically gradual, with symmetric
involvement that begins in the lower extremities and
progressively involves the trunk and upper limbs. Bulbar
involvement occurs in about half of cases. Spontaneous
recovery usually occurs in 2-3 weeks. (See Chapter 607 in
Nelson Textbook of Pediatrics, 17th ed.)

Guillain-Barré syndrome

Question . 20. A 9-yr-old boy presents with paresis of the upper and lower portions
of the face and loss of taste on the right side of the anterior portion of the tongue.
On physical examination, the corner of the mouth droops, and he is unable to close
the right eye tightly. Findings on the remainder of the physical examination are
normal. The most likely diagnosis is:

Duchenne muscular dystrophy

Fabry disease

Familial dysautonomia (Riley-Day syndrome)

Bell palsy
Explanation: Bell palsy is an acute unilateral facial nerve palsy
that is not associated with other cranial neuropathies of
brainstem dysfunction. It is a common disorder at all ages and
typically develops about 2 weeks after a systemic infection,
especially herpes simplex virus and Lyme disease. (See
Chapter 608 in Nelson Textbook of Pediatrics, 17th ed.)

Guillain-Barré syndrome
Question . 21. A 5-yr-old boy has been falling frequently and cannot climb stairs as
well as he did 6-12 months ago. He is an only child, and there is no family history of
neurologic disease. On examination, he has proximal weakness of the legs and
enlargement of the calves, but no weakness of cranial or extraocular muscles.
Serum creatine kinase is 16,400 IU/L. You suspect Duchenne muscular dystrophy
and request assay for a molecular marker in blood for dystrophinopathy, but the
result is normal. The next diagnostic test likely to confirm or refute your clinical
diagnosis is:

EMG

Detailed physical therapy evaluation

A repeat of the blood marker assay in the child and also in the
mother

Muscle biopsy

MRI study of calves and thighs


Explanation: (See Chapters 598 and 599 in Nelson Textbook
of Pediatrics, 17th ed.)

Question . 22. A male neonate has respiratory distress, dysphagia, and


generalized muscular hypotonia and weakness. Pregnancy was uncomplicated, but
the mother reports weak fetal movements in the last trimester. Delivery at term was
uneventful, and Apgar scores at 1 and 5 min were 7 and 8. On examination, he has
a dolichocephalic head, high-arched palate, thin muscle mass in trunk and
extremities, and undescended testes. This clinical picture is least suggestive of:

Nemaline rod myopathy

X-linked myotubular myopathy

Duchenne muscular dystrophy


Explanation: (See Chapter 599 in Nelson Textbook of
Pediatrics, 17th ed.)

Myotonic dystrophy, neonatal form

Congenital muscular dystrophy with merosin deficiency


Question . 23. An 8-yr-old girl demonstrates progressive ataxia, deterioration in
speech and academic performance in school, tingling paresthesias of the feet, and
weakness of the ankles with inability to dorsiflex her feet. Nerve conduction velocity
studies show moderate slowing in motor component of the peroneal nerves, and an
MRI study of the brain reveals white matter lesions in the periventricular white
matter and cerebellum. These findings would be compatible with any of the
following except:

Metachromatic leukodystrophy

Hereditary motor-sensory neuropathy type I (Charcot-


Marie-Tooth disease)
Explanation: (See Chapter 604 in Nelson Textbook of
Pediatrics, 17th ed.)

Giant axonal neuropathy

Globoid cell leukodystrophy (Krabbe disease)

Adrenoleukodystrophy

Question . 24. The most frequent toxic neuropathy in children in the United States
currently is due to:

Lead

Arsenic

Organophosphates used in insecticides

Various industrial and agricultural pollutants in drinking water

Drugs used in chemotherapy


Explanation: (See Chapter 605 in Nelson Textbook of
Pediatrics, 17th ed.)
Question . 25. A 15-yr-old boy complains of slowly progressive difficulty with
coordination and intermittent diplopia. He also has mild generalized weakness.
Studies demonstrate normal serum creatine kinase, a mild lactic acidosis, and ECG
evidence of cardiomyopathy. EMG shows nonspecific myopathic features. Your
provisional diagnosis, to be confirmed by other tests, is:

Mitochondrial cytopathy
Explanation: (See Chapter 602 in Nelson Textbook of
Pediatrics, 17th ed.)

Facioscapulohumeral muscular dystrophy

Congenital muscle fiber-type disproportion

Metachromatic leukodystrophy

Multiple sclerosis

Question . 26. The muscle biopsy in mitochondrial cytopathies may show all of the
following histopathologic features except:

Ragged-red fibers with trichrome stain

Lymphocytic infliltrates in perivascular and interstitial


spaces
Explanation: (See Chapter 602 in Nelson Textbook of
Pediatrics, 17th ed.)

Loss of cytochrome c oxidase (COX) activity in some but not all


myofibers

Ultrastructural abnormalities in mitochondrial cristae

Increased lipid within myofibers


Question . 27. An 8-yr-old girl develops progressive proximal weakness, easy
fatigue, a waddling gait, and myalgias of the thighs and shoulders over a period of 6
weeks, and a violaceous rash appears over the malar areas of her face and the
extensor surfaces of the finger joints. Tendor reflexes are preserved. No heart
murmur is heard, and she has no visceromegaly, generalized lymphadenopathy, or
fever. The most likely diagnosis is:

Systemic lupus erythematosus (SLE)

Polymyositis

Facioscapulohumeral muscular dystrophy

Viral myositis with exanthema

Dermatomyositis
Explanation: (See Chapter 149 in Nelson Textbook of
Pediatrics, 17th ed.)

Question . 28. A 13-yr-old girl develops ptosis, diplopia, and difficulty swallowing
but is afebrile and otherwise in good health. On examination, her ptosis is noted to
increase with sustained upward gaze for 30 sec. No weakness of the extremities or
limb girdles is detected, and she has no myalgias or tenderness of muscles to
palpation. The initial diagnostic test most likely to confirm the diagnosis is:

Anti-acetylcholine receptor antibodies in blood

Computed tomography of the thorax for thymoma

Motor nerve conduction velocity studies and EMG

Edrophonium (Tensilon) test


Explanation: (See Chapter 603 in Nelson Textbook of
Pediatrics, 17th ed.)

Muscle biopsy
Neoplastic Diseases and Tumors
Nelson Self Assessments website 17th Edition

Question . 1. Which of the following cancers occurs primarily during childhood?

A. Breast cancer
B. Renal cell cancer
C. Wilms tumor
Explanation: Wilms tumor occurs most commonly among infants and young
children. (See Chapter 483 in Nelson Textbook of Pediatrics, 17th ed.)
D. Prostate cancer
E. Colon cancer

Question . 2. Which of the following cancer types is least common among children?

A. Leukemias
B. Brain and central nervous system tumors
C. Lymphomas
D. Bone tumors
E. Epithelial carcinomas
Explanation: In contrast with tumors among adults, epithelial carcinomas are
uncommon among children. (See Chapter 483 in Nelson Pediatrics, 17th ed.)

Question . 3. All of the following statements regarding adult and pediatric tumors are
true Except:

A. In comparison with adult cancers, childhood cancers are infrequent


B. Death due to childhood cancers results in a much longer loss of potential lifespan
than is the case with cancer in adults
C. Distribution of cancer types differs markedly between adults and children
D. Genetic abnormalities are associated with most pediatric cancers but not with
most adult cancers
Explanation: Specific genetic conditions are believed to account for <5% of all
pediatric malignancies. (See Chapter 483 in Nelson Pediatrics, 17th ed.)
E. Epithelial malignancies (carcinomas) are far more common in adults than in
children

Question . 4. All of the following statements regarding the epidemiology of childhood


cancer are true Except:

A. Childhood cancer accounts for approximately 15-20% of all cases of cancer


Explanation: Only about 1% of new cases of cancer in the United States occur in
children, yet malignancy remains the second leading among children 1-14 yr of
age. (See Chapter 483 in Nelson Textbook of Pediatrics, 17th ed.)
B. Malignant neoplasms are the second leading cause of death among children 1-14
yr of age
C. Leukemia and central nervous system tumors predominate in children
D. Chronic leukemia is more common in adults than children
E. The incidence of cancer during childhood shows a nadir between 7 and 12 yr of
age.
The Neoplastic Diseases and Tumors Nelson Self Assessments website 17th Edition 1
Question . 5. Which of the following cancers has the highest incidence in young children
(<7 yr of age)?

A. Ewing sarcoma
B. Hodgkin disease
C. Testicular cancer
D. Retinoblastoma
Explanation: The incidence of retinoblastoma peaks during the first 2 yr of life.
The other tumors occur with increasing incidence during childhood and
adolescence. (See Chapter 483 in Nelson Textbook of Pediatrics, 17th ed.)
E. Osteosarcoma

Question . 6. A 2-mo-old boy is found to have cryptorchidism. For which of the


following malignancies is he at increased risk?

A. Rhabdomyosarcoma
B. Pelvic osteosarcoma
C. Germ cell tumor
D. Wilms tumor
E. None of the above
Explanation: Cryptorchidism is a risk factor for testicular germ cell tumors. (See
Chapter 484 in Nelson Textbook of Pediatrics, 17th ed.)

Question . 7. Which of the following cancers is most closely associated with Epstein-
Barr virus?

A. Osteosarcoma
B. Non-Hodgkin lymphoma
Explanation: Epstein-Barr virus is associated with African (endemic) Burkitt
lymphoma, nasopharyngeal carcinoma, post-transplantation non-Hodgkin
lymphoma, non-Hodgkin lymphoma in persons with congenital
immunodeficiencies (e.g., X-linked lymphoproliferative syndrome),
leiomyosarcomas in immunocompromised persons, and some cases of Hodgkin
disease. (See Chapter 484 in Nelson Textbook of Pediatrics, 17th ed.)
C. Ewing sarcoma
D. Wilms tumor
E. Hepatoblastoma

Question . 8. A 14-yr-old girl presents with bilateral cervical lymphadenopathy that has
progressed over the last 4 wk. It is slowly worsening despite antibiotic therapy with
cephalexin, which was prescribed 2 wk ago. On physical examination, the lymph nodes
are enlarged, matted, and nontender. There is mild splenomegaly. The most likely
diagnosis is:
A. Human immunodeficiency virus infection
B. Infectious mononucleosis (Epstein-Barr virus infection)
C. Tuberculosis
D. Methicillin-resistant Staphylococcus aureus infection
E. Acute lymphocytic leukemia
Explanation: Leukemias and lymphomas are the most common malignant
neoplasms among young children. (See Box 485-2 and Chapter 485 in)

The Neoplastic Diseases and Tumors Nelson Self Assessments website 17th Edition 2
Question . 9. The definitive diagnostic procedure for the child described in Question 8 is:

A. Urinalysis
B. Lymph node biopsy and culture
Explanation: A lymph node biopsy and culture is the definitive means of
diagnosing lymphoma. (See Chapter 485 in Nelson Pediatrics, 17th ed.)
C. Plain x-ray film
D. Ultrasound study of the neck
E. Complete blood count and differential

Question . 10. Highly specific indicators of cancer in children include:

A. Diffuse enlargement of the pons (brainstem)


B. White reflex emanating from the retina
C. Both A and B
Explanation: Diffuse enlargement of the pons suggests central nervous system
tumor, and white retinal reflex suggests retinoblastoma. (See Box 485-2 and
Chapter 485 in Nelson Textbook of Pediatrics, 17th ed.)
D. Neither A nor B

Question . 11. Of the following signs of cancer, the most likely to occur in children is:

A. Abnormal discharge from a body orifice


B. Change in a mole or wart
C. Focal neurologic deficit
Explanation: Any focal neurologic deficit in the motor or sensory system,
especially a decrease in cranial nerve function, should prompt further
investigation for central nervous system malignancy. (See Box 485-2 and Chapter
485 in Nelson Textbook of Pediatrics, 17th ed.)
D. Cough, hoarseness, or difficulty swallowing
E. Change in urination or defecation pattern

Question . 12. Metabolic complications of anticancer therapy include:

A. Hyperuricemia
B. Hyperkalemia
C. Hyperphosphatemia
D. Hypocalcemia
E. All of the above
Explanation: Anticancer therapy can result in substantial breakdown of tumor
cells (tumor lysis syndrome) with release of large quantities of phosphates and
potassium into the circulation. Hypocalcemia can result in the setting of
inadequate renal function. (See Chapter 486 in Nelson Textbook of Pediatrics,
17th ed.)

The Neoplastic Diseases and Tumors Nelson Self Assessments website 17th Edition 3
Question . 13. A 12-yr-old girl receives cranial, neck, and spinal irradiation for cancer.
All of the following are likely long-term sequelae of the radiotherapy Except:

A. Interstitial fibrosis
B. Scoliosis
C. Impaired cognition and intelligence
D. Pituitary dysfunction
Explanation: Many sequelae of radiation do not become apparent until the child is
fully grown. Irradiation can result in infertility, second cancers, scoliosis,
pulmonary dysfunction (interstitial fibrosis), leukoencephalopathy, impaired
cognition and intelligence, hypothyroidism, isolated growth hormone deficiency,
and panhypopituitarism. Cardiomyopathy is classically associated with the
anthracyclines (doxorubicin and daunomycin) but can also occur with irradiation.
(See Chapter 486 in Nelson Textbook of Pediatrics, 17th ed.)

Question . 14. All of the following statements concerning childhood lymphocytic and
myelogenous leukemias are Except:

A. Leukemias as a group are the most common childhood cancer


B. Acute lymphocytic leukemia accounts for approximately 75% of cases
C. The incidence is higher in white children than in black children
D. The clinical features at presentation are similar
E. The responses to therapy and prognoses are similar
Explanation: The clinical features of the childhood leukemias are similar, because
all involve severe disruption of bone marrow function. There is marked variability
in response to therapy and in the prognosis. (See Chapter 487 in Nelson, 17th ed.)

Question . 15. All of the following statements concerning acute lymphocytic leukemia
(ALL) are true Except:

A. Most cases (about 85%) are derived from T-cell progenitors


Explanation: About 85% of cases of ALL are derived from progenitors of B cells,
about 15% are derived from T cells, and about 1% are derived from mature B
cells. (See Chapter 487 in Nelson Textbook of Pediatrics, 17th ed.)
B. Staging of ALL is based on bone marrow biopsy and cerebrospinal fluid
examination
C. Chromosomal abnormalities are identified in most cases of ALL
D. Exposure to medical radiation is associated with an increased incidence of ALL
E. Childhood ALL was the first form of cancer shown to be curable with
chemotherapy and irradiation

Question . 16. BCharacteristic presenting symptoms and signs of childhood leukemia include:

A. Pallor
B. Petechiae
C. Lymphadenopathy
D. Bone or joint pain
E. All of the above
Explanation: Most children with leukemia present with symptoms of less than 4
weeks' duration. Most children with ALL have pallor, 50% have petechiae or

The Neoplastic Diseases and Tumors Nelson Self Assessments website 17th Edition 4
mucous membrane bleeding, 60% have lymphadenopathy, 25% have fever, and
about 25% have bone pain and arthralgias caused by leukemic infiltration of the
perichondral bone or joint or by leukemic expansion of the marrow cavity. (See
Chapter 487 in Nelson Textbook of Pediatrics, 17th ed.)

Question . 17. Which of the following factors indicates an increased risk of relapse for
childhood ALL?

A. Age greater than 1 yr


B. Age younger than 10 yr
C. Presenting white blood cell count under 100,000/mm3
D. Any chromosomal abnormality
Explanation: Chromosomal abnormalities are found in most cases of ALL; some
indicate a favorable prognosis, some unfavorable, and some have no apparent
influence. (See Table 487-1 and Chapter 487.1 in Nelson Textbook of Pediatrics,
17th ed.)
E. Rapid response to therapy

Question . 18. A 4-yr-old boy has a bone marrow biopsy that shows 4% blasts. No blasts
are seen on peripheral blood smear. Findings on physical examination and other
laboratory studies are normal. The most likely diagnosis is:

A. Acute lymphoblastic leukemia


B. Acute myelogenous leukemia
C. Chronic myelogenous leukemia
D. Preleukemia syndrome
E. None of the above. this is a normal bone marrow biopsy
Explanation: ALL is diagnosed by a bone marrow evaluation that demonstrates
more than 25% of the bone marrow cells as a homogeneous population of
lymphoblasts. (See Chapter 487 in Nelson Textbook of Pediatrics, 17th ed.)

Question . 19. A breakthrough treatment for chronic myelogenous leukemia in patients


with the BCR-ABL chromosomal translocation is:

A. Imatinib
Explanation: Imatinib was designed specifically to inhibit BCR-ABL tyrosine
kinase. (See Chapter 487.4 in Nelson Textbook of Pediatrics, 17th ed.)
B. Umbilical cord blood transplantation
C. Cis-retinoic acid
D. Timed intensive induction chemotherapy
E. Liposomal anthracycline

The Neoplastic Diseases and Tumors Nelson Self Assessments website 17th Edition 5
Question . 20. All of the following statements about leukemia in children with Down
syndrome are true Except:

A. Leukemia occurs more frequently among children with Down syndrome


B. Acute lymphoblastic leukemia is the most common type of leukemia that occurs
in these children
C. Acute myeloid leukemia has a better outcome in children with Down syndrome
than in children without Down syndrome
D. Chemotherapy is easier to administer because of decreased toxicity compared
with the general population
E. Almost all neonates with Down syndrome and transient myeloproliferative
syndrome eventually develop leukemia
Explanation: About 20-30% of neonates with Down syndrome who develop a
transient leukemia or myeloproliferative syndrome will develop typical leukemia
within the first few years of life. (See Chapter 487.3 in Nelson Textbook of
Pediatrics, 17th ed.)

Question . 21. Leukemia in infants is commonly associated with:

A. A better prognosis than in older children


B. Philadelphia chromosome
C. FAB L3 (Burkitt) type
D. A translocation involving chromosome 11
Explanation: More than two thirds of cases of infant leukemia demonstrate
rearrangements of the MLL gene, classically a translocation involving the q23
band of chromosome 11, and it is this subset of patients that largely accounts for
the very high relapse rate. (See Chapter 487.6 in Nelson Pediatrics, 17th ed.)
E. Spontaneous resolution

Question . 22. The most important extramedullary site of relapse in childhood ALL is:

A. Adrenal glands
B. Kidney
C. Lung
D. Heart
E. Central nervous system
Explanation: The most important extramedullary sites of relapse of ALL are the
central nervous system and the testes. Intrathecal therapy is key to prevention of
later central nervous system relapse. Testicular relapse occurs in 1-2% of boys
with ALL, usually after completion of therapy. (See Chapter 487.1 in Nelson
Textbook of Pediatrics, 17th ed.)

The Neoplastic Diseases and Tumors Nelson Self Assessments website 17th Edition 6
Question . 23. A neonate with Down syndrome experiences transient myeloproliferative
syndrome that spontaneously resolves. Which of the following statements best
characterizes the prognosis?

A. This almost never recurs


B. This recurs intermittently during early childhood
C. This indicates an increased risk of leukemia
Explanation: Neonates and infants with Down syndrome may experience transient
myeloproliferative syndrome, which mimics congenital leukemia. These children
have a 20-30% risk of subsequent acute leukemia within the first few years of life.
(See Chapter 487.3 in Nelson Textbook of Pediatrics, 17th ed.)
D. This invariably develops later into leukemia
E. This invariably develops later into malignant histiocytosis

Question . 24. An older child experiences weight loss and night sweats and has a white
blood cell count of 80,000/mm3 and a platelet count of 600,000/mm3. Chromosomal
analysis of bone marrow cells reveals a clonal disorder of stem cells with the specific
translocation, t(9;22)(q34;q11), which is also known as the Philadelphia chromosome.
The diagnosis is:

A. Acute myelocytic leukemia


B. Acute megakaryocytic leukemia
C. Chronic myelogenous leukemia
Explanation: The Philadelphia chromosome is associated with chronic
myelogenous leukemia. (See Chapter 487.4 in Nelson Pediatrics, 17th ed.)
D. Juvenile chronic myelogenous leukemia
E. Hodgkin disease

Question . 25. A 15-yr-old white girl reports that she has had a fever, weight loss, and
night sweats for 3 mo. On physical examination, she is found to have painless swelling of
the left cervical and supraclavicular lymph nodes. Her liver and spleen are not enlarged.
The nearest pediatric oncology center requires a 4-hr drive. Before transfer, the initial
evaluation of the patient should include:

A. Bone marrow aspiration


B. Abdominal CT study
C. Chest radiograph
Explanation: Chest radiography is important for two reasons: first, to document
mediastinal lymph node involvement, and second. to determine whether these
lymph nodes threaten the patency of the airway. (See Chapter 488 in Nelson
Textbook of Pediatrics, 17th ed.)
D. Head CT study
E. Erythrocyte sedimentation rate

The Neoplastic Diseases and Tumors Nelson Self Assessments website 17th Edition 7
Question . 26. The chest radiograph of the patient described in Question 25 reveals
mediastinal lymphadenopathy. The best diagnostic test at the referral center is:

A. Abdominal CT study
B. Head CT study
C. Bone marrow biopsy
D. Lymph node biopsy
Explanation: At this time, a lymph node biopsy can confirm suspicion of Hodgkin
disease. Thereafter, CT scans and bone marrow biopsy are useful in staging the
extent of the lymph node and extranodal involvement. (See Chapter 488 Nelson)
E. Thoracic CT study

Question . 27. A 7-yr-old boy weighing 24 kg presents with painless right


supraclavicular lymphadenopathy. Chest radiograph reveals a mediastinal mass
measuring more than one-third of the thoracic diameter. Biopsy of the lymph node
reveals Hodgkin disease, nodular sclerosing type. Which of the following indicates a
poorer prognosis?

A. Hilar lymphadenopathy
Explanation: For Hodgkin disease, the presence of hilar lymphadenopathy with a
mediastinal mass larger than one third of the thoracic diameter indicates bulky
disease and a poorer prognosis. (See Chapter 488.1 in Nelson, 17th ed.)
B. Unilateral cervical adenopathy
C. Intense mediastinal uptake on gallium 67 scan
D. Pruritus
E. Weight loss of 2 kg

Question . 28. Small non-cleaved cell non-Hodgkin lymphoma (SNCCL) (Burkitt's type)
is characterized by all of the following Except:

A. t(8:14), t(8;22), or t(2;8) translocation


B. B lymphocyte origin
C. Presence of EBV genome
D. Mediastinal lymphadenopathy
Explanation: SNCCL presents as an abdominal tumor in 80% of U.S. cases. Jaw
involvement occurs in <20% of U.S. cases, compared with 70% of younger
patients in equatorial Africa. (See Chapter 488.2 in Nelson Pediatrics, 17th ed.)
E. Jaw involvement

Question . 29. In children with newly diagnosed non-Hodgkin lymphoma, which of the
following laboratory studies provides an indirect measure of tumor burden?

A. Serum uric acid


B. Erythrocyte sedimentation rate
C. Serum lactic dehydrogenase (LDH)
Explanation: Elevation of the level of serum lactate dehydrogenase (>500 U/L)
correlates with tumor mass and has proved useful for stratifying therapy intensity.
(See Chapter 488.2 in Nelson Textbook of Pediatrics, 17th ed.)
D. Serum ALT (alanine aminotransferase)
E. Absolute lymphocyte count

The Neoplastic Diseases and Tumors Nelson Self Assessments website 17th Edition 8
Question . 30. A 5-yr-old boy presents with progressive headache that is associated with
nausea and vomiting. Physical examination reveals bilateral papilledema. The most likely
diagnosis is:

A. Pseudotumor cerebri
B. Central nervous system tumor
Explanation: The classic triad of headache, nausea and/or vomiting, and
papilledema is associated with midline and infratentorial tumors. (See Chapter
489 in Nelson Textbook of Pediatrics, 17th ed.)
C. Brain abscess
D. Cerebrovascular event
E. Central nervous system lupus

Question . 31. For the boy described in Question 30, the most appropriate management is:

A. Lumbar puncture, including measurement of CSF pressure


B. Blood culture and CSF culture and empirical antibiotic therapy
C. Cerebral angiography
D. Emergency MRI scan
Explanation: MRI is the preferred neuroimaging test for suspected central nervous
system tumors. (See Chapter 489 in Nelson Textbook of Pediatrics, 17th ed.)
E. ANA and high-dose methylprednisolone

Question . 32. A 4-yr-old child presents with a hard, fixed abdominal mass that causes
discomfort. Physical examination also shows hypertension. The most likely etiology is:

A. Hodgkin's disease
B. Ewing sarcoma
C. Nephroblastoma
D. Neuroblastoma
Explanation: Most neuroblastomas arise in the abdomen, either in the adrenal
gland or in retroperitoneal sympathetic ganglia. Catecholamine production may
cause hypertension, whereas other vasoactive substances may produce a secretory
diarrhea. (See Chapter 490 in Nelson Textbook of Pediatrics, 17th ed.)
E. Renal clear cell carcinoma

Question . 33. Which of the following is a favorable prognostic factor in neuroblastoma?

A. Age <1 yr
Explanation: Children with neuroblastoma with early-stage disease without
amplified MYCN or chromosome 1p deletion can usually be cured with surgery
alone. Children <1 yr of age have a 95% 3-yr survival rate, compared with 25-
50% for children 1-5 yr of age. (See Chapter 490 in Nelson Pediatrics, 17th ed.)
B. Age > 5yr
C. Stage 3 or 4
D. Amplified MYCN
E. Chromosome 1p deletion in 80-90% of cells

The Neoplastic Diseases and Tumors Nelson Self Assessments website 17th Edition 9
Question . 34. A previously healthy 2-yr-old boy presents with irritability, low-grade
fever for 1 wk, weight loss, and limp. He has no history of vomiting or diarrhea or of
recent trauma. On physical examination, he is pale with a temperature of 38°C. Blood
pressure is 150/95 mm Hg, respiratory rate 24/min, and pulse 130/min. Weight is
decreased 1 kg from his weight at a previous visit at 18 mo of age. Ears and throat are
normal, but there appear to be bruises below both eyes. Lungs are clear to auscultation
and the heart sounds are normal. His abdomen appears distended, but abdominal
examination is unrevealing because of his irritability that prevents adequate palpation.
When he tries to walk, he refuses to bear weight on his left leg, but the leg appears
normal. The most appropriate diagnostic test for this patient is:

A. Blood culture
B. Abdominal CT scan
Explanation: Patients with abdominal neuroblastoma present with a firm, nodular
mass that causes abdominal pain. Some tumors produce catecholamines that can
cause sweating and hypertension. (See Chapter 490 in Nelson Pediatrics, 17th ed.)
C. Coagulation screen including platelet count, PT, PTT, and fibrinogen
D. Synovial fluid analysis of left hip
E. Plain x-ray film of entire left leg

Question . 35. A previously healthy 20-mo-old girl is noted by her mother to be more
irritable and to be falling more frequently than usual when she walks. She seems to be
having trouble with balance but does not appear in pain. There was no decrease in
appetite, vomiting, diarrhea, or fever. Her mother observes her for a week, hoping the
clumsiness will resolve. It does not resolve but worsens, and the child begins to be more
uncooperative and to have random, darting eye movements. She is taken to her
pediatrician, who also notes these abnormalities and performs an MRI study of the brain.
Findings on the MRI study are normal. Which of the following is the most likely
explanation for these clinical findings?

A. Acute hepatic encephalopathy


B. Cysticercosis
C. Behavioral problems
D. Neuroblastoma
Explanation: Opsomyoclonus ("dancing eyes and dancing feet") is a
paraneoplastic syndrome of autoimmune origin that is associated with
neuroblastoma. (See Chapter 490 in Nelson Textbook of Pediatrics, 17th ed.)
E. Acute demyelinating encephalomalacia

Question . 36. A 3-yr-old child presents with an abdominal mass and microscopic
hematuria. The most likely tumor is:

A. Hodgkin disease
B. Ewing sarcoma
C. Wilms tumor
Explanation: Wilms tumor accounts for most renal neoplasms in children. (See
Chapter 491 in Nelson Textbook of Pediatrics, 17th ed.)
D. Neuroblastoma
E. Renal clear cell carcinoma

The Neoplastic Diseases and Tumors Nelson Self Assessments website 17th Edition 10
Question . 37. A 9-yr-old previously healthy white girl manifests progressive painless
proptosis, periorbital edema, and decreased visual acuity of the left eye during a 2-mo
period. The most likely diagnosis is:

A. Pseudotumor cerebri
B. Trichinosis
C. Retinoblastoma
D. Rhabdomyosarcoma
Explanation: Orbital rhabdomyosarcoma is a common site for
rhabdomyosarcoma, which produces local signs as it grows and displaces normal
tissues. (See Chapter 492 in Nelson Textbook of Pediatrics, 17th ed.)
E. Orbital cellulitis

Question . 38. The most common soft tissue sarcoma in children is:

A. Synovial sarcoma
B. Malignant fibrous histiocytoma
C. Fibrosarcoma
D. Rhabdomyosarcoma
Explanation: Rhabdomyosarcoma is the most common soft tissue sarcoma in
childhood. (See Chapter 492 in Nelson Textbook of Pediatrics, 17th ed.)
E. Alveolar (soft part) sarcoma

Question . 39. A 4-yr-old boy presents with urinary obstruction. Ultrasound study reveals
a pelvic mass. There is no history of urinary tract infections or other illnesses. The most
likely diagnosis is:

A. Wilms tumor
B. Neuroblastoma
C. Germ cell tumor
D. Rhabdomyosarcoma
Explanation: Rhabdomyosarcoma is the most common soft tissue sarcoma of
childhood. Wilms tumor is an renal tumor, which would not be found in the
pelvis. (See Chapter 492 in Nelson Textbook of Pediatrics, 17th ed.)
E. Hydronephrosis

Question . 40. Osteosarcoma occurs with the highest incidence in which of the following
age groups?

A. Neonates
B. Children <5 yr of age
C. Children 5-10 yr of age
D. Adolescents
Explanation: The highest risk period for development of osteosarcoma is during
the adolescent growth spurt. (See Chapter 493 in Nelson Textbook of Pediatrics,
17th ed.)

The Neoplastic Diseases and Tumors Nelson Self Assessments website 17th Edition 11
=====================

Figure 493-2 Radiograph of an osteosarcoma of the femur with typical "sunburst"


appearance of bone formation

Question . 41. A 15-yr-old boy presents with a palpable swelling of the humerus that is
associated with pain that awakens him at night. There is a sunburst pattern on the x-ray
film (Figure). The most likely diagnosis is:

A. Ewing sarcoma
B. Osteosarcoma
Explanation: Pain and swelling are the most common presenting symptoms of
osteosarcoma and Ewing sarcoma. Osteosarcoma has a characteristic sunburst
pattern on x-ray films. Ewing sarcoma is associated with periosteal elevation, or
"onion-skinning," on x-ray. (See Chapter 493 in Nelson Pediatrics, 17th ed.)
C. Histiocytosis X
D. Osteochondroma
E. Benign bone cyst

The Neoplastic Diseases and Tumors Nelson Self Assessments website 17th Edition 12
Question . 42. A 15 yr-old female basketball player sustained a knee injury 4 wk
previously. An initial knee x-ray was read as normal. She has been receiving physical
therapy for the past 3 wk with no relief of her pain. Repeat examination shows diffuse
swelling around the knee, which is most prominent in the distal femur. Of the following,
which is the most appropriate next step in management of this patient?

A. Repeat knee x-ray


B. Treatment with a nonsteroidal anti-inflammatory drug
C. Arthroscopy
D. Continuation of physical therapy for another 2 weeks
E. MRI study
Explanation: A bone tumor should be suspected in a child or adolescent with
persistent bone pain and swelling, with or without systemic symptoms. (See
Chapter 493 in Nelson Textbook of Pediatrics, 17th ed.)

Question . 43. A 12-yr-old boy complains of pain and swelling in his right upper arm
with onset 2 wk previously. He has also had night pain. He has had fever with
temperatures to 101°C for the past 2 days. An x-ray film of the humerus shows periosteal
elevation along the shaft. The most appropriate next step is:

A. Blood culture
B. IV antibiotics
C. Erythrocyte sedimentation rate (ESR)
D. MRI study of humerus
Explanation: MRI is the neuroimaging test of choice for bone tumors. (See
Chapter 493 in Nelson Textbook of Pediatrics, 17th ed.)
E. Bone scan

Question . 44. A 12 yr-old boy has completed multiagent chemotherapy with


doxorubicin hydrochloride (Adriamycin), methotrexate, and cisplatin and limb salvage
surgery for nonmetastatic osteosarcoma. Possible late effects of chemotherapy include all
of the following Except:

A. Cardiotoxicity
B. Sterility
C. Hearing loss
D. Second malignancy
E. Cataracts
Explanation: Cardiotoxicity is associated with doxorubicin hydrochloride
(Adriamycin). (See Chapter 493 in Nelson Textbook of Pediatrics, 17th ed.)

The Neoplastic Diseases and Tumors Nelson Self Assessments website 17th Edition 13
Question . 45. A 9-yr-old boy develops local pain and swelling of the left lateral aspect
of the skull. Physical examination reveals marked tenderness and warmth. Plain film
shows a well-defined radiolucent skull lesion. The skeletal survey is otherwise normal.
The most likely diagnosis is:

A. Eosinophilic granuloma
B. Letterer-Siwe disease
C. Hand-Schüller-Christian disease
D. Osteomyelitis of the skull
E. Brain abscess
Explanation: Eosinophilic granuloma is a monostotic or polyostotic disease with
no extraskeletal involvement. This differentiates eosinophilic granuloma from
other forms of Langerhans cell histiocytosis (Letterer-Siwe or Hand-Schüller-
Christian variants). (See Chapter 493 in Nelson Textbook of Pediatrics, 17th ed.)

Question . 46. Which of the following benign bone processes may be associated with
visceral involvement?

A. Osteoid osteoma
B. Aneurysmal bone cyst
C. Osteochondroma
D. Langerhans cell histiocytosis
Explanation: Children with Langerhans cell histiocytosis and bone lesions should
be evaluated for visceral involvement because treatment of Hand-Schüller-
Christian disease and Letterer-Siwe disease is more complex and often requires
systemic therapy. (See Chapter 493 in Nelson Textbook of Pediatrics, 17th ed.)
E. Fibrous dysplasia

Question . 47. Which of the following bone neoplasms is associated with a characteristic
chromosomal translocation?

A. Ewing sarcoma
Explanation: A specific chromosomal translocation, t(11;22), or a variant thereof,
is present in most cases of the Ewing family of tumors. (See Chapter 493 in
Nelson Textbook of Pediatrics, 17th ed.)
B. Osteosarcoma
C. Fibrous dysplasia
D. Eosinophilic granuloma
E. Osteoid osteoma

The Neoplastic Diseases and Tumors Nelson Self Assessments website 17th Edition 14
=======================================

.... sorry

Question . 48. A 5-yr-old boy presents with pain and swelling of the proximal right
lower leg over the past 2 weeks, with limitation of movement of the right knee. A
radiograph (Figure) of the femur is shown. The most likely diagnosis is:

A. Acute osteomyelitis
B. Chronic osteomyelitis
C. Osteoid osteoma
D. Ewing sarcoma
Explanation: Ewing sarcoma has a characteristic radiographic onion-skin
appearance, caused by periosteal reaction around the lytic bone lesion. (See
Chapter 493 in Nelson Textbook of Pediatrics, 17th ed.)
E. Osteosarcoma

Question . 49. A 14-yr-old boy presents with a bony, nonpainful mass in the distal femur
that has been slowly enlarging. There are no associated symptoms. The mass is most
likely to represent which of the following tumors?

A. Ewing sarcoma
B. Osteosarcoma
C. Langerhans cell histiocytosis
D. Osteochondroma
Explanation: Osteochondroma (exostosis) is one of the most common benign
bone tumors in children. Many of these tumors are asymptomatic or are detected
when the child or the parent notes a bony, nonpainful mass. Most arise in the
metaphyses of long bones. (See Chapter 493 in Nelson Pediatrics, 17th ed.)
E. Benign bone cyst

The Neoplastic Diseases and Tumors Nelson Self Assessments website 17th Edition 15
Question . 50. The highest incidence of retinoblastoma has been noted in which of the
following age groups?
A. Infants
Explanation: Retinoblastoma occurs in 1 of 18,000 live births in the United States.
(See Chapter 494 in Nelson Textbook of Pediatrics, 17th ed.)
B. Children 2-5 yr of age
C. Children 5-10 yr of age
D. Adolescents
E. None of the above?the risk of retinoblastoma is similar in all age groups

Question . 51. On routine office ophthalmoscopic examination of a 14-mo-old girl, you


have a hard time seeing the fundus but observe no gross external physical abnormalities.
The extraocular muscles appear intact, and the remaining physical findings are normal. A
red reflex was reportedly present at birth. The most appropriate next step in management
would be to:

A.
Reassure the parents that nothing is wrong
B.
Obtain Toxoplasma and cytomegalovirus titers
C.
Obtain an MRI scan
D.
Refer the patient to an infectious disease specialist
E.
Refer the patient to an ophthalmologist for examination performed with the child
under general anesthesia
Explanation: Difficulty visualizing the fundus raises the suspicion of
retinoblastoma and warrants immediate attention and examination. An MRI scan,
or alternatively a CT scan, may also identify this ocular tumor. (See Chapter 494
in Nelson Textbook of Pediatrics, 17th ed.)
=========================================

Question . 52. An infant presents with the finding shown in the Figure. The cancer most
likely to be associated with this finding is:
A. Wilms tumor
B. Germ cell tumor
C. Retinoblastoma
Explanation: Leukocoria (white pupillary reflex) and strabismus are often the
presenting signs of retinoblastoma. (See Chapter 494 in Nelson Textbook of
Pediatrics, 17th ed.)
D. Rhabdomyosarcoma
E. Leiomyosarcoma

The Neoplastic Diseases and Tumors Nelson Self Assessments website 17th Edition 16
Question . 53. A 10-yr-old boy presents with a painless mass involving the right testicle.
Possible diagnoses include all of the following Except:

A. Leukemia
B. Rhabdomyosarcoma
C. Germ cell tumor
D. Testicular torsion
Explanation: Testicular (spermatic cord) torsion produces acute pain and swelling
of the scrotum. (See Chapter 495 in Nelson Textbook of Pediatrics, 17th ed.)
E. Hydrocele

Question . 54. Which of the following is the most common neoplasm involving the liver
in children?

A. Hepatocellular carcinoma
B. Hepatoblastoma
Explanation: Primary hepatic tumors are rare in children, accounting for 1% of
malignancies in childhood. Approximately two thirds of hepatic tumors in
children are hepatoblastomas. (See Chapter 496 in Nelson Pediatrics, 17th ed.)
C. Neuroblastoma
D. Wilms tumor
E. Rhabdomyosarcoma

Question . 55. An infant has a hemangioma that grows rapidly in size, leading to
thrombocytopenia and microangiopathic hemolytic anemia. Which of the following terms
describes this condition?

A. Kaposi-like form of infantile hemangioma


B. Albright syndrome
C. Kasabach-Merritt syndrome
Explanation: Kasabach-Merritt syndrome is trapping of platelet and red blood
cells within a hemangioma with activation of the clotting system within the
vasculature of the hemangioma. (See Chapter 497 in Nelson Pediatrics, 17th ed.)
D. Maffucci syndrome
E. Ollier disease

Question . 56. An infant has a hemangioma over the eyelid that grows rapidly in size,
obstructing vision. The most reasonable systemic therapy would be administration of:

A. Erythropoietin
B. Heparin
C. Platelet-activating factor
D. Corticosteroids
Explanation: For hemangiomas that are life-threatening or that threaten vital
functions such as eyesight, a trial of oral corticosteroids is warranted.
Approximately 30% respond dramatically and begin to regress within 1 wk, 40%
stabilize or show minimal response, and the remainder do not respond. Other
treatments for hemangiomas that do not respond to corticosteroids include
Interferon- , laser therapy, and surgery. (See Chapter 497 in Nelson, 17th ed.)
E. Vitamin A

The Neoplastic Diseases and Tumors Nelson Self Assessments website 17th Edition 17
Question . 57. A 1-yr-old girl has a history of fever, anemia, severe skin eruption,
generalized lymphadenopathy, and pulmonary infiltrates. A skull radiograph (Figure)
shows multiple lesions. The most likely diagnosis is:

A. Osteosarcoma
B. Osteochondritis
C. Langerhans cell histiocytosis
Explanation: Bone lesions with Langerhans cell histiocytosis may be single or
numerous and are seen most commonly in the skull. (See Chapter 499 in Nelson
Textbook of Pediatrics, 17th ed.)
D. Leukemia
E. Ewing sarcoma

Question . 58. The recommended treatment for the child described in Question 57 is:

A. Curettage of the bone lesions


B. IVIG
C. Interferon-
D. Combination chemotherapy
Explanation: Multisystem Langerhans cell histiocytosis should be treated with
systemic multiagent chemotherapy. (See Chapter 499 in Nelson Textbook of
Pediatrics, 17th ed.)
E. Corticosteroids

Question . 59. Regarding the radiographic finding in Question 57, if the same finding
were seen in a 4-yr-old girl with localized swelling on her skull, and there were no other
symptoms, which of the following would be the recommended treatment?

A. Curettage of the bone lesions


Explanation: Curettage of the bone lesion. Therefore, treatment should be
minimal and directed at arresting the progression of a lesion (e.g., a bone lesion)
that could result in permanent damage before it resolves spontaneously. Curettage
or low-dose radiation therapy may accomplish this goal. (See Chapter 499 in
Nelson Textbook of Pediatrics, 17th ed.)
B. IVIG
C. Interferon-
D. Combination chemotherapy
E. Corticosteroids

The Neoplastic Diseases and Tumors Nelson Self Assessments website 17th Edition 18
Disorders of the Eye
Nelson Self Assessments website 17th Edition

Question . 1. Delayed removal of a congenital and complete unilateral cataract may


lead to:

A. Amblyopia
Explanation: Amblyopia is a vision loss that is due not to a specific organic
lesion but rather to deprivation or disuse of the retina. Even after delayed
removal of a cataract or correction of strabismus, the previously unused retina
"tunes out" the image. (See Chapter 619 in Nelson Textbook of Pediatrics,
17th ed.)
B. Glaucoma
C. Uveitis
D. Strabismus
E. Nyctalopia

Question . 2. Cataracts are noted in all of the following Except:

A. Rubella (congenital)
B. Galactosemia
C. Galactokinase deficiency
D. Neonatal hypoglycemia
E. Hypocalcemia
F. Lowe syndrome
G. Hyperoxygenation
Explanation: Hyperoxia in preterm neonates usually causes retinopathy of
prematurity, which occasionally is associated with cataracts. Oxygen does not
directly cause cataracts. (See Chapter 619 in Nelson Textbook of Pediatrics,
17th ed.)
H. Steroid therapy
I. Child abuse

Question . 3. An 18-mo-old manifests pendular nystagmus, head nodding, and


torticollis. Findings on a cranial MRI scan are normal. This child is most likely to
have:

A. Epilepsy
B. Congenital blindness
C. Neuroblastoma
D. Dysmetria
E. Spasmus nutans
Explanation: The acquired triad of nystagmus, head nodding, and torticollis, in
its classic form, is self-limited and benign. Nonetheless, children with brain
tumors may have signs resembling components of spasmus nutans. (See
Chapter 614 in Nelson Textbook of Pediatrics, 17th ed.)

Disorders of the Eye - Nelson Self Assessments website 17th Edition 1


Question . 4. A 7-yr-old girl experiences fullness of the right upper eyelid and
downward displacement of the eye over a 2-mo period. The right eye also appears to
be proptotic. Which of the following is the most likely diagnosis?
A. Myasthenia gravis
B. Right superior oblique palsy
C. Chalazion
D. Rhabdomyosarcoma
Explanation: Rhabdomyosarcoma of the face, orbit, and sinus often presents
early because of the space-occupying and displacement effects of tumor
growth. (See Chapter 624 in Nelson Textbook of Pediatrics, 17th ed.)
E. Hypothyroidism

Question . 5. The most common presenting sign of retinoblastoma is:


A. Heterophoria
B. Hypopyon
C. Leukocoria
Explanation: The clinical manifestations of retinoblastoma vary, but the initial
sign in the majority of cases is leukocoria (white pupillary reflex, also known
as "cat eye") instead of the normal red pupillary reflex. (See Chapter 621)
D. Coloboma
E. Red reflex

Question . 6. A 17-yr-old girl taking oral contraceptives presents with headache,


nausea, and vomiting. Physical examination reveals papilledema. The most
appropriate action is:
A. Discontinue the oral contraceptives and re-evaluate in 7-14 days
B. Discontinue the oral contraceptives and administer promethazine (Phenergan)
as needed for nausea and vomiting
C. Discontinue the oral contraceptives and administer oral corticosteroids
D. Perform computed tomography (CT) or magnetic resonance imaging (MRI)
study of the head
Explanation: Papilledema constitutes a neurologic emergency. Neuroimaging
should be performed, and if no intracranial masses are found, a lumbar
puncture for determination of cerebrospinal fluid pressure should be
performed. This patient may have pseudotumor cerebri. (See Chapter 622)
E. Perform lumbar puncture for determination of cerebrospinal fluid pressure

Question . 7. A 1-yr-old child presents with increased size of the cornea. Review of
systems reveals history of increased tearing and apparent sensitivity to light. The
cornea appears cloudy. The most likely diagnosis is:
A. Retinoblastoma
B. Glaucoma
Explanation: Symptoms of infantile glaucoma (glaucoma that begins in the
first 3 years of life) include the classic triad of epiphora (tearing), photophobia
(sensitivity to light), and blepharospasm (eyelid squeezing), which are usually
attributed to corneal irritation. An increase in intraocular pressure leads to
expansion of the globe, including the cornea. (See Chapter 623 in Nelson)
C. Chorioretinitis
D. Aniridia
E. Coloboma

Disorders of the Eye - Nelson Self Assessments website 17th Edition 2


Question . 8. Which of the following is the most appropriate initial treatment for the
child described in Question 9?

A. Topical glaucoma medications


B. Oral glaucoma medications
C. Corticosteroids
D. Surgery
Explanation: Unlike glaucoma in adults, in which medications are the first line
of therapy, the treatment of infantile glaucoma is primarily surgical.
Procedures used to treat glaucoma in children include surgery to establish a
more normal anterior chamber angle (goniotomy and trabeculotomy), to create
a route for aqueous fluid to exit the eye (trabeculectomy and seton surgery), or
to reduce aqueous fluid production (cyclocryotherapy and photocyclo-
coagulation). In many children, several operations are required to lower and
maintain the intraocular pressure adequately. Long-term medications may be
necessary as well. (See Chapter 623 in Nelson Pediatrics, 17th ed.)
E. Medications and surgery

Question . 9. A 12-yr-old boy has a small corneal abrasion detected by Wood lamp
examination. Which of the following is the most appropriate treatment?

A. Topical antibiotic ointment


Explanation: Corneal abrasions are treated with frequent applications of a
topical antibiotic ointment until the epithelium is completely healed. The use
of a semi-pressure patch does not improve healing time or decrease pain.
Furthermore, an improperly applied patch may itself abrade the cornea. A
topical cycloplegic agent (cyclopentolate hydrochloride 1%) can relieve the
pain from ciliary spasm in patients with large abrasions. Topical anesthetics
should not be given at home because they retard epithelial healing and inhibit
the natural blinking reflex. (See Chapter 625 in Nelson Pediatrics, 17th ed.)
B. Topical antibiotic ointment and a semipressure patch
C. Topical antibiotic ointment, a semipressure patch, and a topical cycloplegic
agent
D. Topical antibiotic ointment, a semipressure patch, and a topical anesthetic as
necessary for pain
E. Topical antibiotic ointment, a topical cycloplegic agent, and a topical
anesthetic as necessary for pain

Question . 10. All of the following statements concerning the newborn eye are correct
Except:

A. The cornea is smaller than that of an adult


B. Tears are often not present with crying until 1-3 mo of age
C. Ocular alignment may not be perfect in the first several months of life
D. Most infants are myopic (nearsighted)
Explanation: Most infants are hyperopic (farsighted). The cornea is smaller in
infants and grows to reach adult size near the age of 2 yr. Tears are not usually
produced with crying until a few months of age. It is common for children to
have strabismus in the first few months of life. (See Chapter 609 in Nelson)
E. Iris color may change in the first several months of life

Disorders of the Eye - Nelson Self Assessments website 17th Edition 3


Question . 11. Which of the following tests is not part of a routine eye examination?

A. Visual acuity
B. Pupil assessment
C. Preferential looking test
Explanation: Preferential looking tests are done by experienced examiners and
are not part of a routine eye examination. (See Chapter 610 in Nelson)
D. External examination
E. Ocular motility

Question . 12. Which of the following statements concerning abnormalities of


refraction is correct?

A. Children with hyperopia will often squint to see better


B. Astigmatism may be caused by an irregular shape of the cornea
Explanation: Astigmatism is commonly caused by an irregular corneal shape.
Children with hyperopia will accommodate to see better. Children with
myopia will frequently squint to see better. Scientific studies have not proved
that bifocals can cure myopia. Anisometropia occurs when there is a
difference in refractive error between the two eyes. It may lead to amblyopia
and decrease vision in one eye. However, the term anisometropia does not
indicate a difference in vision by itself. High levels of myopia may lead to
retinal detachment. (See Chapter 611 in Nelson Textbook Pediatrics, 17th ed.)
C. Myopia can be cured with the use of bifocal lenses
D. Anisometropia occurs when one eye sees better than the other
E. Myopia is always a benign condition

Question . 13. A 4-yr-old girl presents with a large esotropia and poor vision in one
eye. Both eyes are normal on examination. What is the most likely reason for her poor
vision?

A. Need for glasses


B. Malingering
C. Cataract
D. Suppression
E. Amblyopia
Explanation: This child probably has amblyopia secondary to her strabismus.
Although the need for glasses in only one eye or a cataract in one eye could
present in the same way, these problems are far less common in this age
group. (See Chapter 612 in Nelson Textbook of Pediatrics, 17th ed.)

Disorders of the Eye - Nelson Self Assessments website 17th Edition 4


Question . 14. A newborn is presented with what appear to be large pupils that do not
constrict to light. There is no family history of similar ocular findings. An appropriate
study to be performed in the future is:

A. CT scan of the head and orbit


B. Visual evoked potentials
C. Renal ultrasound study
Explanation: In aniridia, the pupils look large and nonreactive. Because
children so affected are at risk for Wilms tumor, renal ultrasound study is
indicated. (See Chapter 613 in Nelson Textbook of Pediatrics, 17th ed.)
D. Lumbar puncture
E. Cocaine test

Question . 15. A 2-yr-old boy presents with pronounced crossing of his left eye of 2
months' duration. On further examination, he is found to be excessively farsighted for
his age. The most likely diagnosis is:

A. Congenital superior oblique palsy


B. Accommodative esotropia
Explanation: Accommodative esotropia causes a crossing of the eyes
secondary to a high degree of farsightedness. Congenital esotropia does not
develop at this age. (See Chapter 614 in Nelson Textbook Pediatrics, 17th ed.)
C. Intermittent exotropia
D. Duane syndrome
E. Congenital esotropia

Question . 16. Which of the following statements concerning congenital ptosis is


true?
A. Surgery should be performed shortly after birth in all cases
B. Amblyopia will occur only if the lid covers the pupil
C. Amblyopia cannot occur once the lid is surgically elevated
D. Ptosis may occur in association with other ocular or systemic disorders
Explanation: Ptosis is often associated with other systemic disorders. Surgery
does not need to be performed early if the vision is developing normally.
Amblyopia can occur if the ptosis causes a unilateral astigmatism. It can occur
even if the pupil is not completely occluded. Amblyopia can continue to occur
even after successful ptosis surgery. Frontalis suspension is used for cases of
severe ptosis. (See Chapter 615 in Nelson Textbook of Pediatrics, 17th ed.)
E. Frontalis suspension surgery is used for mild cases of ptosis

Question . 17. A 3-mo-old girl is presented with tearing and mucous discharge from
her left eye that has been present since near birth. Her external examination is
otherwise normal. Appropriate forms of treatment would include:
A. Nasolacrimal massage
B. Topical antibiotics for infections if they occur
C. Cleansing of the lids
D. Probing of the nasolacrimal duct if the symptoms do not resolve with time
E. All of the above
Explanation: All of the above are correct. Many cases of nasolacrimal duct
obstruction will resolve within the first year of life. (See Chapter 616)

Disorders of the Eye - Nelson Self Assessments website 17th Edition 5


Question . 18. A 2-day-old infant is presented with bilateral conjunctival redness and
purulent discharge. The most likely diagnosis is:

A. Nasolacrimal duct obstruction


B. Ophthalmia neonatorum
Explanation: Ophthalmia neonatorum occurs shortly after birth. A
nasolacrimal duct obstruction does not usually cause significant conjunctival
redness this early. Congenital glaucoma is often associated with tearing but
not with a purulent discharge. Blepharitis and vernal conjunctivitis do not
generally occur at this age. (See Chapter 617 in Nelson Pediatrics, 17th ed.)
C. Congenital glaucoma
D. Blepharitis
E. Vernal conjunctivitis

Question . 19. All of the following disorders may present as a congenital corneal
opacity Except:

A. Sclerocornea
B. Congenital cataract
Explanation: A cataract is opacity of the lens, not the cornea. (See Chapter 618
in Nelson Textbook of Pediatrics, 17th ed.)
C. Infantile glaucoma
D. Peters anomaly
E. Epibulbar dermoid

Question . 20. A child presents with a dense unilateral cataract. All of the following
are important factors in the visual prognosis following the removal of the cataract
Except:
A. Whether a contact lens or an intraocular lens implant is used
Explanation: The mode of visual rehabilitation is not the most important factor
in the outcome of pediatric cataract surgery. Early diagnosis, age at time of
cataract development, and success with occlusion therapy are the most
important factors. (See Chapter 619 in Nelson Textbook of Pediatrics, 17th
ed.)
B. The compliance with patching for amblyopia
C. The length of time the cataract was present
D. The age of the child at presentation
E. The health status of the posterior segment of the eye

Question . 21. A young girl is found to have bilateral iritis on routine examination. On
further questioning, it is found that she has been limping for the last several weeks.
The most likely diagnosis is:

A. Sympathetic ophthalmia
B. Juvenile rheumatoid arthritis
Explanation: Uveitis in patients with JRA commonly presents without pain or
redness. Sympathetic ophthalmia is unilateral and occurs in the uninjured eye
following a significant ocular injury. Herpes simplex may cause an anterior
uveitis but is usually unilateral and associated with pain. Toxoplasmosis

Disorders of the Eye - Nelson Self Assessments website 17th Edition 6


causes a posterior uveitis and is usually unilateral. Kawasaki disease is not
associated with arthritis and usually causes conjunctivitis. (See Chapter 620 )
C. Herpes simplex
D. Toxoplasmosis
E. Kawasaki disease

Question . 22. A 20-mo-old girl is noted by her parents to have a unilateral white
pupil. Funduscopic examination reveals a very large white mass that fills most of the
eye. All of the following statements regarding this patient are correct Except:

A. Genetic counseling is indicated for this family


B. Removal of the eye may be required
C. A CT scan may demonstrate calcium within the lesion
D. A biopsy should be performed as soon as possible
Explanation: This patient is most likely to have a retinoblastoma. Because it
can be a hereditary tumor, genetic counseling is indicated. Treatment of a
large unilateral tumor may require enucleation. Appropriate work-up may
include a CT scan to demonstrate calcium within the eye. Examination of the
other eye for smaller tumors should always be performed. Biopsy of the
suspected tumor is contraindicated, as it may lead to spread of the tumor cells.
(See Chapter 621 in Nelson Textbook of Pediatrics, 17th ed.)
E. Examination of the other eye should be performed with the patient under
anesthesia

Question . 23. A 4-yr-old boy who is being followed by his primary care doctor for
short stature is found to have poor vision in one eye. On ophthalmic examination, he
is noted to have an afferent pupillary defect on the affected side. His anterior segment
is normal, and his vision cannot be improved with glasses.
The most likely diagnosis is:
A. Strabismus
B. Cataract
C. Optic nerve hypoplasia
Explanation: Optic nerve hypoplasia may be associated with growth hormone
deficiency and poor vision. Patients with unilateral optic nerve abnormalities
will demonstrate an afferent pupillary defect. (See Chapter 622 in Nelson)
D. Glaucoma
E. Retinal detachment

Question . 24. A 3-mo-old boy is brought in by his parents because he appears to be


very light sensitive. On examination, he is tearing, and his corneas appear large and
cloudy. The most likely diagnosis is:
A. Infantile glaucoma
Explanation: The classic findings in infantile glaucoma are enlarged corneas,
cloudy corneas, tearing, and light sensitivity. Although a tear duct obstruction
also causes tearing, it does not cause a change in the size or clarity of the
cornea, and it is not associated with photophobia. (See Chapter 623 in Nelson)
B. Nasolacrimal tear duct obstruction
C. Peters anomaly
D. Infantile cataracts
E. Uveitis

Disorders of the Eye - Nelson Self Assessments website 17th Edition 7


Question . 25. A 2-yr-old child presents with a 1-day history of unilateral proptosis
and fever. Appropriate next steps in the diagnosis and treatment of this child could
include all of the following Except:

A. CT scan of the head, orbit, and sinuses


B. Ophthalmologic consultation
C. Administration of a broad-spectrum antibiotic
D. Surgical drainage of any abscess
E. Determination of TSH and TRH blood levels
Explanation: Although thyroid disease may manifest with unilateral proptosis,
it is not associated with fever and does not usually develop this rapidly. The
other answers are appropriate steps in the management of a child with a
preseptal or orbital cellulitis. (See Chapter 624 in Nelson Pediatrics, 17th ed.)

Question . 26. Which of the following statements regarding children with traumatic
hyphemas is most correct?

A. This is generally a benign condition


B. If secondary bleeding does not occur by the second day, there is little risk of
further complications
C. Children with sickle cell disease are at increased risk for loss of vision
Explanation: Children with sickle cell disease are at increased risk for vision
loss following a traumatic hyphema. Traumatic hyphemas can be a significant
cause for vision loss. Rebleeding is most common 3 to 5 days after the initial
injury. Blood within the anterior chamber can lead to corneal blood staining
and/or glaucoma. Patients with a history of a hyphema are at risk for the
development of glaucoma later in life. (See Chapter 625 in Nelson Textbook
of Pediatrics, 17th ed.)
D. Once the blood leaves the anterior chamber, there is no longer any risk from
the injury causing the hyphema
E. The blood itself produces no ill effects

Disorders of the Eye - Nelson Self Assessments website 17th Edition 8


The Ear
Nelson Self Assessments website 17th Edition

Question . 1. A 10-day-old infant develops irritability and rectal temperature of 38oC


on the day of planned discharge from the NICU after an uneventful recovery from
mild respiratory distress syndrome. Physical examination is unremarkable except for a
bulging, opaque left eardrum. A lumbar puncture reveals normal cerebrospinal fluid.
Diagnostic tympanocentesis is performed. Which of the following organisms is the
one most likely to be found?

A. Chlamydia trachomatis
B. Group A streptococci
C. Haemophilus influenzae type b
D. Escherichia coli
Explanation: E. coli and group B streptococci are common causative agents in
neonatal otitis media. The earlier in the first month of life plus the more
complicated the neonatal course, the more likely it is that these pathogens will
be found. (See Chapter 630 in Nelson Textbook of Pediatrics, 17th ed.)
E. Cytomegalovirus

Question . 2. A previously healthy 8-mo-old infant develops bronchiolitis. On the


fourth day of illness she is noted to have bulging, opaque, white eardrums bilaterally.
Which of the following treatment regimens is the most appropriate to institute?

A. High-dose oral amoxicillin


Explanation: Although respiratory syncytial virus (RSV) may cause otitis
media, it may be a co-pathogen with the typical bacterial causes of otitis
media; thus, RSV infection should be treated as for a bacterial otitis media. In
some European countries, answer D would be appropriate. (See Chapter 630)
B. Intramuscular ceftriaxone
C. Oral cefixime
D. No initial antibiotic treatment; watchful waiting
E. Oral azithromycin

Question . 3. A 2½-yr-old boy presents with an upper respiratory tract infection and
axillary temperature of 38.5oC. He has not complained of ear pain. On pneumatic
otoscopy, his left eardrum is pink, translucent, and retracted, with 3+/4+ mobility; his
right eardrum is reddish, opaque, retracted, and immobile. Which of the following is
the likeliest ear-related diagnosis?

A. Bilateral acute otitis media


B. Normal left ear, right acute otitis media
C. Bilateral otitis media with effusion
D. Normal left ear, right otitis media with effusion
Explanation: The right ear has classic features on physical examination for an
otitis media with effusion. Not all children with otitis media complain of ear
pain, whereas all children who tug at their ears don't have otitis media. (See
Chapter 630 in Nelson Textbook of Pediatrics, 17th ed.)
E. Left otitis media with effusion, right acute otitis media

The Ear - Nelson Self Assessments website 17th Edition 1


Question . 4. All of the following statements regarding otitis media are correct
Except:

A. Otitis media is more prevalent among boys than among girls


B. Otitis media is equally prevalent in poor and in well-to-do children
Explanation: Medically underserved children have a higher risk of otitis media
than that documented for more affluent children. The reason for this is not
well established but could be related to frequent use of day care, the presence
of smoking, and other factors. (See Chapter 630 in Nelson Textbook of
Pediatrics, 17th ed.)
C. The peak prevalence of otitis media is in the first 2 years of life
D. Breast feeding provides protection against otitis media
E. Otitis media tends to run in families

Question . 5. A 12-mo-old infant in day care develops severe bilateral acute otitis
media. Her infection continues despite administration of a succession of antibiotics.
Tympanocentesis reveals the presence of penicillin-resistant Streptococcus
pneumoniae. Which of the following bacterial mechanisms is responsible for the
organism's resistance?

A. Production of -lactamase
B. Production of streptolysin toxin
C. Production of nitric oxide
D. Alteration in penicillin-binding proteins of the cell wall
Explanation: Antimicrobial resistance among pneumococci to penicillin is due
to alterations of penicillin- binding proteins. Treatment with -lactamase
competitors or -lactamase-resistant penicillins is not effective. (See Chapter
630 in Nelson Textbook of Pediatrics, 17th ed.)
E. Alteration of the nucleus

Question . 6. A 2-yr-old boy is seen for his routine check-up, 4 weeks after an
episode of bilateral acute otitis media that resolved uneventfully with antibiotic
treatment. He seems generally well, but his mother reports that he is not hearing as
well as usual. On pneumatic otoscopy, both his eardrums appear amber, opaque, and
retracted, and both are immobile. Otherwise his ENT examination is unremarkable.
Which of the following treatment regimens is the most appropriate to institute at this
time?

A. A 10-day course of amoxicillin


B. A 10-day course of amoxicillin-clavulanate
C. Short-course treatment with azithromycin
D. Observation without treatment for at least 2 more mo
Explanation: The finding of a retracted opaque immobile eardrum after
successful treatment of otitis media is not uncommon and warrants close
observation. In most cases the abnormalities resolve spontaneously. (See
Chapter 630 in Nelson Textbook of Pediatrics, 17th ed.)
E. Referral for consideration of myringotomy and tube insertion

The Ear - Nelson Self Assessments website 17th Edition 2


Question . 7. A 4-yr-old girl has had repeated bouts of acute otitis media since early
infancy. At age 10 mo she underwent bilateral myringotomy with tube insertion. She
remained well for 6 months but then experienced several episodes of tube otorrhea.
The tubes were extruded at age 2 yr, and since then she has averaged 6 episodes of
recurrent acute otitis media each year. At present her eardrums are normal in
appearance except for some atrophic scarring. She breathes easily through her nose
and does not snore. Her internist father is inquiring about the advisability of
adenoidectomy, which was recommended on the golf course by an ENT colleague.
Which of the following statements will best help the parents decide about surgery?

A. Adenoidectomy has shown greater efficacy in preventing recurrent otitis


media in children who have previously received tympanostomy tubes than in
those who have not.
Explanation: Adenoidectomy is one important approach to management of a
child with recurrent otitis media after failure of tympanostomy tubes. This is
true despite the absence of obstruction or significant enlargement of adenoidal
tissue (for age). (See Chapter 630 in Nelson Textbook of Pediatrics, 17th ed.)
B. Adenoidectomy has shown efficacy only during the first 2 yr of life
C. Adenoidectomy has shown efficacy only in children with enlarged adenoids
D. Adenoidectomy has shown greater efficacy in boys than in girls
E. Adenoidectomy has failed to show efficacy in preventing recurrent otitis media

Question . 8. An otherwise healthy 12-mo-old boy develops bilateral otitis media


with effusion in September that persists for 3 mo. An audiogram performed in
December shows a pure-tone average threshold of 30 dB. The parents do not accept a
recommendation for myringotomy and insertion of tympanostomy tubes. The effusion
persists, and a repeat audiogram the following April shows the same results as
previously. The parents again decline surgery. The effusion is still present at a check-
up in July, but in September, 1 yr after onset, the effusion has cleared. Without further
intervention, which of the following outcomes should the parents anticipate as most
likely at the time the child enters school?

A. The child's cognitive development will be unaffected but his speech and
language development will be impaired
B. Both his cognitive and his speech and language development will be impaired
C. Neither his cognitive nor his speech and language development will be
impaired
Explanation: This is an important observation that has been confirmed by
many studies. (See Chapter 630 in Nelson Textbook of Pediatrics, 17th ed.)
D. His cognitive and language development will be unaffected, but his speech
development will be impaired
E. His cognitive and speech development will be unaffected but his language
development will be impaired

The Ear - Nelson Self Assessments website 17th Edition 3


Question . 9. Vertigo in children is:

A. Frequently associated with otitis media


B. Usually related to motion sickness
C. Most likely to be related to Meniere disease
D. An uncommon symptom in children
Explanation: Vertigo, a sense of motion, should not be confused with
dizziness. Vertigo may be associated with nystagmus and may be due to
labyrinthitis, congenital ear defects, trauma, cholesteatoma, neuronitis, benign
paroxysmal vertigo, Ménière disease, or CNS diseases. (See Chapter 626 in
Nelson Textbook of Pediatrics, 17th ed.)
E. Always associated with a hearing loss

Question . 10. Otorrhea (purulent ear drainage) may be associated with all of the
following Except:

A. Presence of a tube in the ear


B. Perforated eardrum
C. Cholesteatoma
D. Lyme disease
Explanation: Lyme disease may cause cranial neuropathies (most often cranial
nerve VII) but should not cause otorrhea. The most common causes of
otorrhea may be otitis externa and perforation of the tympanic membrane in
children with otitis media. (See Chapter 626 in Nelson Textbook of Pediatrics,
17th ed.)
E. Acute otitis media

Question . 11. Conductive hearing loss is:

A. Common in children with chronic ear fluid


Explanation: Otitis media is the most common form of acquired conductive
hearing loss. Any pathologic condition of the pinna, external ear canal,
tympanic membrane, or ossicles can produce congenital or acquired
conductive hearing loss. (See Chapter 627 in Nelson Textbook of Pediatrics,
17th ed.)
B. A permanent condition related to nerve damage
C. Always associated with ossicular abnormalities
D. An uncommon complication of otitis media
E. Unlikely when an intact tympanic membrane is present

Question . 12. Down syndrome (trisomy 21) is commonly associated with:

A. Narrow ear canals


B. Conductive hearing loss
C. Chronic ear fluid
D. Speech delay
E. All of the above
Explanation: Down syndrome may be associated with conductive or sensorineural
hearing loss. (See Chapter 627 in Nelson Textbook of Pediatrics, 17th ed.)

The Ear - Nelson Self Assessments website 17th Edition 4


Question . 13. Pneumatic otoscopy is:

A. Used only to assist in diagnosis of infection


B. Reserved for the operating room to assist in ear tube placement
C. An important office tool for diagnosis of ear fluid and negative middle ear
pressure
Explanation: This is a valuable method that all pediatricians should learn
during residency. It facilitates excellent assessment of the mobility of the
tympanic membrane and any pain associated with the applied pressure. (See
Chapter 630 in Nelson Textbook of Pediatrics, 17th ed.)
D. Performed by an audiologist to assess hearing
E. Not recommended for acute otitis media

Question . 14. Hearing screening in infants is:

A. An elective procedure for diagnosing the presence of middle ear fluid


B. Performed only in healthy infants without risk factors for hearing loss
C. Performed in all hospitals in the first week of life
D. Most commonly performed using otoacoustic emissions (OAE) and/or
auditory brainstem evoked response (ABR)
Explanation: These are the most accurate and reliable tests. Whether all infants
should be screened in the first week of life or before nursery discharge
remains controversial. (See Chapter 627 in Nelson Pediatrics, 17th ed.)
E. Unlikely to detect severe or profound hearing loss

Question . 15. Sensorineural hearing loss in the United States is:

A. Most likely to be inherited in autosomal dominant fashion


B. Most likely to be inherited as an autosomal recessive disorder when
genetically transmitted
Explanation: In about 40% of the cases, sensorineural hearing loss is due to an
autosomal recessive disorder. Mutations of the connexin gene and other
genetic syndromes (long Q-T syndrome), are responsible for many genetic
causes of hearing loss. (See Chapter 627 in Nelson Pediatrics, 17th ed.)
C. Caused by a genetic abnormality in less than 20% of cases
D. Most commonly caused by bacterial meningitis
E. Usually related to chronic ear infections

Question . 16. In moderate to severe SNHL, hearing aids are:

A. Usually placed when the child starts kindergarten


B. Difficult for parents to deal with and require skilled nursing care
C. Not well tolerated and require surgical placement over the mastoid bone
D. Placed as early as age 2-3 mo when the deficit is diagnosed early
Explanation: These are very well tolerated and helpful, especially if started
before age 6 mo. (See Chapter 627 in Nelson Textbook of Pediatrics, 17th ed.)
E. Used only if cochlear implantation cannot be performed

The Ear - Nelson Self Assessments website 17th Edition 5


Question . 17. Otitis externa is:

A. Commonly related to swimming


B. Most commonly caused by Pseudomonas aeruginosa
C. Best treated with topical antibiotic drops
D. Treated with wick placement and drops if severe canal swelling is present
E. All of the above
Explanation: This is the classic swimmer's ear and is due to wetness with
tissue maceration and secondary infection. It can occur without swimming.
(See Chapter 629 in Nelson Textbook of Pediatrics, 17th ed.)

Question . 18. Cochlear implants may be associated with:

A. Pneumococcal meningitis
Explanation: Pneumococcal meningitis may be a complication of cochlear
implants. All patients should be immunized with the currently available
vaccines. (See Chapter 627 in Nelson Textbook of Pediatrics, 17th ed.)
B. Sinusitis
C. Facial cellulitis
D. Septic jugular vein thrombosis
E. Brain abscess

Question . 19. A hearing deficit of moderate loss is associated with an average sound
threshold of 30-50 dB in combination with:

A. Most speech sounds missed at normal conversational levels


B. Language retardation
C. Unvoiced consonant sounds missed
D. Inattention
E. All of the above
Explanation: Unvoiced consonant sounds are missed even with slight hearing
loss. (See Chapter 627 in Nelson Textbook of Pediatrics, 17th ed.)

Question . 20. All of the following indicate the need for referral for audiologic
assessment Except:

A. No differentiated babbling or vocal imitation at age 12 mo


B. No use of single words at age 18 mo
C. Single-word vocabulary of 10 words or fewer at age 24 mo
D. Less than a 100-word vocabulary, or no evidence of two-word combinations,
at age 30 mo
E. All of these indicate the need for referral for audiologic assessment
Explanation: All of these indicate the need for audiologic assessment. (See
Chapter 627 in Nelson Textbook of Pediatrics, 17th ed.)

The Ear - Nelson Self Assessments website 17th Edition 6


Question . 21. A 4-yr-old child who is new to your clinic has a small pitlike
depression anterior to the helix and above the tragus. There are no symptoms. Which
of the following is the recommended initial management?

A. Observation only
Explanation: A pitlike depression just anterior to the helix and above the
tragus may represent a cyst or an epidermis-lined fistulous tract. These are
common, with an incidence of approximately 8 cases in 10,000 children but do
not require surgical removal unless there is recurrent infection. (See Chapter
628 in Nelson Textbook of Pediatrics, 17th ed.)
B. Exploration by probing
C. Computed tomography (CT) or magnetic resonance imaging (MRI) to
evaluate for possible branchial cleft cyst
D. Referral for surgical excision
E. Referral for chromosome analysis

The Ear - Nelson Self Assessments website 17th Edition 7


The Field of Pediatrics

Question . 1. For which age group are unintentional injuries not the leading cause of death?

1-4 yr

5-9 yr

Under 1 yr
Explanation: (See Chapter 1 in Nelson Textbook of Pediatrics, 17th edition.)

10-14 yr
15-19 yr

Question . 2. Which of the following statements is false?

Children (0-18 yr) make up about 50% of the population


Explanation: (See Chapter 1 in Nelson Textbook of Pediatrics, 17th edition.)

In 1999, over 25% of children under 18 yr were living with one parent

Infant mortality in the United States was 6.9/1000 live births in 2000

Postnatal infant mortality in the United States was 2.3/1000 live births in 2000

In 1998, 18% of all children in the United States lived in families with income
below the poverty level ([dollar]16,530 for a family of four persons)

Question . 3. All of the following statements about ethics are true except:

The age at which a competent patient may legally exercise voluntary informed
consent for medical care varies from state to state

The decision for a DNAR order does not imply a decision to withhold other
aspects of providing medical treatment

The decision to withhold life-sustaining medical treatment implies an intent or


choice to hasten a child's death
Explanation: (See Chapter 2 in Nelson Textbook of Pediatrics, 17th edition.)

Medically beneficial treatment can be withheld from a severely disabled infant


who is permanently unconscious

Teenagers may not be able to support themselves but still be competent to


consent to health care
Question . 4. Which of the following statements is false?

The World Health Organization (WHO) has a program called Integrated


Management of Childhood Illness (IMCI), which focuses on a single medical
condition providing all levels of care.
Explanation: (See Chapter 4 in Nelson Textbook of Pediatrics, 17th edition.)

About one-third of children younger than 15 yr in developing countries either


have or are at risk for impaired learning.

About 18,000 children are adopted into American families from abroad each year.

Most disasters occur in developing countries and are due to natural events such
as hurricanes.

Economic conditions and family resources drive health care decision-making.

Question . 5. What is an optimal schedule for health supervision visits of a child during the first year of
life?

1 mo, 6 mo, 12 mo

1 mo, 3 mo, 6 mo, 12 mo

1 wk, 1 mo, 3 mo, 6 mo, 12 mo

1 wk, 1 mo, 2 mo, 6 mo, 9 mo, 12 mo

Prenatal, 1 wk, 1 mo, 2 mo, 4 mo, 6 mo, 9 mo, 12 mo


Explanation: (See Chapter 5 in Nelson Textbook of Pediatrics, 17th edition.)

Question . 6. Which of the following statements is true?

Children are at the forefront of societal and cultural change related to health and
well-being

Ethnicity encompasses culture


Explanation: (See Chapter 3 in Nelson Textbook of Pediatrics, 17th edition.)

Variability within any cultural or ethnic group often exceeds that between groups

The use of complementary and alternative medicine occurs in educated and


uneducated affluent groups

Children are active shapers of their cultural environment rather than passive
recipients of socialization
Question . 7. Which of the following statements is false?

Ninety percent of children in the early 21st century are born into the developing
world

The mortality rate of children under 5 yr worldwide is 80-90/1000

Between 1980 and 2000 the percentage of children immunized in developing


countries stayed constant
Explanation: (See Chapter 4 in Nelson Textbook of Pediatrics, 17th edition.)

Approximately one third of children under 15 yr in developing countries either


have or are at risk for impaired learning

The Integrated Management of Childhood Illness (IMCI) program uses any point of
contact as an opportunity to assess the overall health of the child and family in
developing countries

Question . 8. A 9-mo-old girl has repeated night awakenings after going to sleep and has been unable to
return to sleep without breastfeeding. This behavior has been present for the past month, prior to which
she slept the entire night without waking or requiring middle-of-the-night feedings. Her growth and
development are normal. She breastfeeds 6-7 times/day and eats a variety of solid foods. Of the following,
which is the most likely explanation for this behavior?

Diminishing breast milk production

Increased growth velocity

Separation anxiety
Explanation: (See Chapter 5 in Nelson Textbook of Pediatrics, 17th edition.)

Feeding schedule: trained night feeder

Nightmares

Question . 9. A 2-yr-old boy refuses to hold his mother's hand when walking across the parking lot of a
shopping mall. He attempts to run away from her but she quickly grabs his hand before he darts out in
front of a car. She is exasperated and scared by his actions.
Part A Of the following, which technique is most effective in addressing the behavior?

Verbal reprimand

Spanking

Scolding

Time-out
Explanation: (See Chapter 5 in Nelson Textbook of Pediatrics, 17th edition.)

Slapping his hand


Question . Part B To be effective, time-outs should:

Be accompanied by verbal reprimands

Be accompanied by punishment

Last at least 5-10 min

Last 1 min per year of age


Explanation: (See Chapter 5 in Nelson Textbook of Pediatrics, 17th edition.)

Be done at a point in time removed from the incident

Question . 10. The sensitivity of carefully performed observation, history, and physical examination in the
child with an acute illness is:

70%

60%

90%
Explanation: (See Chapter 6 in Nelson Textbook of Pediatrics, 17th edition.)

100%

50%

Question . 11. Most observation data that pediatricians gather during an acute illness should focus on
assessing which of the following?

Eye behavior

Gross motor ability

Fine motor ability

Work of breathing

Response to stimuli
th
Explanation: (See Chapter 6 in Nelson Textbook of Pediatrics, 17 edition.)
Question . 12. In the acutely ill child younger than 36 mo, which of the following components of the clinical
assessment is best done on the examination table?

Eye examination

Abdominal examination
Explanation: (See Chapter 6 in Nelson Textbook of Pediatrics, 17th edition.)

Pulmonary auscultation

Range and ease of extremity motion

Cardiac auscultation

Question . 13. Paradoxic irritability is a finding from observation that is helpful in diagnosing:

Pneumonia

Urinary tract infection

Septic arthritis

Meningitis
Explanation: (See Chapter 6 in Nelson Textbook of Pediatrics, 17th edition.)

Bacteremia

Question . 14. The white blood cell count and differential is most helpful as a screening test in which of the
following entities?

Meningitis

Bacteremia
Explanation: (See Chapter 6 in Nelson Textbook of Pediatrics, 17th edition.)

Pneumonia

Urinary tract infection

Septic arthritis
Question . 15. Generally, a competent adult can make a decision to accept or reject a physician's
recommendation regardless of the impact on his or her health. A parent must make decisions about a
child's health care based on a physician's recommendations. The range of acceptable decisions that a
parent can make, however, is restricted. Of the following, which is the most important factor for a
pediatrician to consider when evaluating a parent's decision about the health care for a 7-yr-old child?

The expressed wishes of the child

What a reasonable parent would do in that situation

The pediatric clinician's independent professional obligation to act in a child's


best interest
Explanation: (See Chapter 2 in Nelson Textbook of Pediatrics, 17th edition.)

The ability of the parent to pay for the recommended medical care

The opinions of other pediatric clinicians

Question . 16. At times, the assent of a child to a proposed medical intervention is necessary to assure
appropriate treatment. In seeking a child's assent, which of the following is not recommended?

Help a child understand his or her condition

Provide guidance to the child on what is best for him or her


Explanation: (See Chapter 2 in Nelson Textbook of Pediatrics, 17th edition.)

Tell a child what he or she can expect

Assess a child's understanding and whether a child feels pressured to assent

Solicit a child's willingness to participate

Question . 17. A parent has made a decision not to pursue further treatment for a child's condition, given
the medical recommendation that there are no further treatments that offer any hope of extending the
child's life. Of the following treatments, which is not considered an appropriate option to either withdraw or
withhold?

Artificial hydration and nutrition

Adequate sedation and analgesia


Explanation: (See Chapter 2 in Nelson Textbook of Pediatrics, 17th edition.)

Antibiotics

Mechanical ventilation

Inotropic medications
Question . 18. Many health care institutions have established ethics committees made up of a diverse
membership. Of the following, which is generally not a function of the ethics committee?

Drafting a review of institutional policy

Educating health care professionals, patients, and families

Case consulting

Responding to parental requests for ethical advice

Triage of scarce hospital resources


Explanation: (See Chapter 2 in Nelson Textbook of Pediatrics, 17th edition.)

Question . 19. As many adults choose not to be tested for late-onset disorders, we cannot assume that a
child would want or would benefit from similar testing. Genetic testing of childhood and adolescents for
late-onset disorders is generally inappropriate, except under specific circumstances. Of the following,
which is the best reason for genetic testing of late-onset disorders in childhood?

The parents need to make decisions concerning long-term savings for


educational expenses

Scarce family resources require triaging educational support to that child without
the genetic condition

Such testing will result in interventions that have been shown to reduce morbidity
and mortality when initiated in childhood
Explanation: (See Chapter 2 in Nelson Textbook of Pediatrics, 17th edition.)

The parents want to seek suitable marriage partners and need to know the child's
carrier state

The family health insurance is set to expire, and such testing will thus not be
available in the future
Growth and Development, Set 1

Question . 1. Behavioral states in the newborn period include:

Quiet and active sleep

Drowsy and alert states

Fussy and crying states

All of the above


Explanation: Newborn infants have six characteristic organizational
behavioral states. (See Chapter 9 in Nelson Textbook of Pediatrics, 17th
ed.)

None of the above

Question . 2. The best formula to approximate average weight (kg) for a 4-year-old is:

Age (years) × 7 - 5/2

Age (years) × 2 + 8
Explanation: The formula in choice A is used for 7- to 12-yr-old children;
C, for 3- to 12-mo-old infants; D, for 1- to 6-yr-olds in pounds; and E, for
7- to 12-yr-olds in pounds. (See Chapter 15 in Nelson Textbook of
Pediatrics, 17th ed.)

Age (months) + 9/2

(Age [years] × 5 + 17)

(Age [years] × 7 + 5)

Question . 3. A normal infant may cry for up to 3 hr/day during the developmental peak time of
this behavior. This peak is typically at age:

2 wk

4 wk

6 mo

6 wk
Explanation: Crying may or may not be in response to obvious stimuli
(e.g., need for a diaper change). (See Chapter 10 in Nelson Textbook of
Pediatrics, 17th ed.)

4 mo
Question . 4. The best feeding protocol for a temperamentally irregular infant is:

A fixed schedule

One based on the parents' schedule

Every 1-2 hr

One based on demand


Explanation: Demand feedings prevent periods of hunger and episodes
of being fed while not being hungry for a child with an irregular rhythm.
(See Chapter 11 in Nelson Textbook of Pediatrics, 17th ed.)

60 min for each feeding

Question . 5. Object permanence is not present at age 2 mo. The response to a ball dropped in
front of the child is:

Staring momentarily at the spot the ball was dropped from


Explanation: "Out of sight, out of mind" is the characteristic response of
a 2-mo-old. Object permanence appears at approximately 8 mo of age.
This is also called object constancy. (See Chapter 10 in Nelson Textbook
of Pediatrics, 17th ed.)

Eyes lowering as the ball descends

Crying when the ball hits the ground

Smiling at the game of hide-and-seek

None of the above

Question . 6. The ability to manipulate small objects with the pincer grasp is usually noted at
age:

0-2 mo

3-5 mo

6-7 mo

8-9 mo
Explanation: The pincer grasp, which is noted at 8 to 9 mo, along with
increasing mobility, enables an infant to explore the environment. (See
Chapter 10 in Nelson Textbook of Pediatrics, 17th ed.)

10-12 mo
Question . 7. The probable age of a developmentally normal child who is just able to sit without
support, can transfer objects from hand to hand, and speaks in a monosyllabic babble is:

3 mo

4 mo

9 mo

6 mo
Explanation: Each pediatrician should learn key developmental
milestones such as these. (See Chapter 10 in Nelson Textbook of
Pediatrics, 17th ed.)

11 mo

Question . 8. Transitional objects include:

Training underwear

Shoes without laces

Cups with a special drinking spout

Blankets and teddy bears


Explanation: Transitional objects help toddlers (18-24 mo) cope with
separation (e.g., at nighttime for sleep, with baby sitter, at daycare). (See
Chapter 11 in Nelson Textbook of Pediatrics, 17th ed.)

None of the above

Question . 9. Handedness is usually determined by age:

2-4 mo

6-12 mo

15-18 mo

20-24 mo

36-48 mo
Explanation: Handedness should not be attempted to be modified
because this leads to frustration. After age 4 years, a spontaneous
change in handedness should lead to the suspicion of a central nervous
system lesion. (See Chapter 12 in Nelson Textbook of Pediatrics, 17th
ed.)
Question . 10. The best approach for parents to help a preschool child overcome monster fears
is to:

Rationalize that monsters do not exist

Read books that do not have monsters in them

Have the pediatrician explain that monsters are make-believe

Use "great power" like monster spray to keep monsters away


Explanation: It is impossible to rationalize away a preschool child's fear
of monsters. (See Chapter 12 in Nelson Textbook of Pediatrics, 17th ed.)

None of the above

Question . 11. A mother brings her 6½-mo-old circumcised boy to you for a "sick" visit. You saw
the child 2 wk previously for health maintenance, including a DTP immunization, and the child
appeared well. The mother's complaint is that the baby is waking up every night and is fussy
during the day, especially when she leaves him. The child's history is otherwise normal, and
physical examination reveals no problems. The most appropriate approach to management is
to:

Perform urinalysis and obtain a complete blood count to rule out urinary
tract infection

Request that the mother feed the infant more

Reassure the mother that the behavior is normal and will pass in time
Explanation: Waking up at night (if in fact the baby had already slept
through part of the night) at 6 to 8 mo is common behavior. Whether this
is related to separation anxiety or something else (teething?) is not clear
(edentulous babies wake up, too). Choice A would be highly unlikely,
because 6½-mo-old circumcised boys who have grown normally rarely
contract urinary tract infections. Choice B would be unnecessary
because increased food intake does not relieve night fussiness. D is
wrong because DTP reactions occur 4-36 hr after the shot, not 2 wk. (See
Chapter 10 in Nelson Textbook of Pediatrics, 17th ed.)

Reassure the mother that the behavior will pass because it is a reaction
to the DTP shot

Question . 12. The biopsychosocial model of development, when applied to the child's height,
includes all of the following except:

Genetic endowment

Personal eating habits

Access to food
Parents' beliefs

Differences between breast milk and formula


Explanation: In general, all other socioeconomic factors held constant,
both means of infant feeding are equally effective. Nonetheless, in less
advantaged environments, breast milk has specific biologic advantages.
The biopsychosocial model helps combine the nature versus nurture
aspects of previous theories of development. (See Chapter 7 in Nelson
Textbook of Pediatrics, 17th ed.)

Question . 13. All of the following statements regarding a child's temperament are true except:

Temperament is absolute and stable throughout the life span


Explanation: Temperament is only moderately stable over time, and a
mellow 2-yr-old does not always equate to a mellow 22-yr-old. (See
Chapter 7 in Nelson Textbook of Pediatrics, 17th ed.)

Biology influences temperament

It is a pattern of the child's responses

It is relatively resistant to parents' attempts to modify

It helps parents understand the child's behavior without guilt

Question . 14. The parents of a 3-yr-old girl report that "she ran before she walked," "she is
never hungry at the same time," and "she goes from toy to toy." This child is best described as:

Autistic

Having a specific temperament


Explanation: The seven characteristics of temperament have a wide
range of responsiveness; these characteristics are activity level,
rhythmicity, approach and withdrawal, adaptability, threshold of
responsiveness, intensity of reaction, quality of mood, distractibility,
attention span, and persistence. (See Chapter 7 in Nelson Textbook of
Pediatrics, 17th ed.)

Having attention deficit hyperactivity disorder

Having developmental pervasive disorder

Being deaf
Question . 15. The visual acuity of a newborn permits recognition of an object held at a distance
of:

1-2 inches

8-12 inches
Explanation: The near-sighted neonate has a fixed focal length of 8-12
inches. The newborn also has a visual preference for faces. (See Chapter
9 in Nelson Textbook of Pediatrics, 17th ed.)

15-24 inches

24-30 inches

30-36 inches

Question . 16. A newborn infant spends about 40 min with the mother but then falls asleep and
does not respond to the mother's voice. Which of the following statements regarding this lack of
activity is true?

It is suggestive of sepsis

It is suggestive of sedation

It is normal
Explanation: This normal sleep after a 40-min period of social interaction
is a great time to continue the bonding process. (See Chapter 9 in Nelson
Textbook of Pediatrics, 17th ed.)

It represents a seizure

It is due to apnea

Question . 17. The six behavioral states of the neonate include all of the following except:

Quiet sleep

Active sleep

Drowsiness

Alertness

Colic
Explanation: Colic is not a neonatal state and does not even occur
during the neonatal period. (See Chapter 9 in Nelson Textbook of
Pediatrics, 17th ed.)

Fussiness

Crying
Question . 18. The best formula for approximating average weight in kilograms for a 9-mo-old is

Age (mo) + 9/2


Explanation: B is best for 9-mo-olds in pounds. (See Chapter 10 in
Nelson Textbook of Pediatrics, 17th ed.)

Age (mo) + 11

Age (yr) × 2 + 8

Age (yr) × 5 + 17

Age (yr) × 7 + 5

Question . 19. The best formula for approximating average height in centimeters for a 4-yr-old is

Age (yr) × 2.5 + 30

Age (yr) × 6 + 77
Explanation: A is in inches. (See Chapter 10 in Nelson Textbook of
Pediatrics, 17th ed.)

Age (yr) × 7 + 5

Age (yr) × 5 + 7

Age (yr) × 2.5 - 6

Question . 20. All of the following statements regarding growth in the first month of life are true
except:

Weight may decrease 10% in the first week

Weight should equal or exceed birthweight by 2 wk

Once gaining weight, the infant should gain 30 g/day

Preterm infants take longer to regain birthweight

The high fat content of colostrum enhances weight gain in the first week
of life
Explanation: Colostrum has a high protein content. Mature milk has a
higher fat and lower protein content than those of colostrum. (See
Chapter 10 in Nelson Textbook of Pediatrics, 17th ed.)
Question . 21. Crying in the first 2 mo of life is characterized by all of the following except:

Teething
Explanation: Teething does not occur until approximately 5-6 mo. (See
Chapter 10 in Nelson Textbook of Pediatrics, 17th ed.)

Peaking at 6 wk

Peaking at a total of 3 hr/day

Crying in response to obvious stimuli

Crying when no stimulus is obvious

Question . 22. The probable age of a child who scribbles, walks alone, speaks one real word,
and pretends to drink from a cup is:

8 mo

13 mo
Explanation: These milestones correspond respectively to visual-motor
coordination, exploration labeling, and symbolic thought. (See Chapter
10 in Nelson Textbook of Pediatrics, 17th ed.)

16 mo

20 mo

24 mo

Question . 23. The probable age of a child who rolls back to front, has a thumb-finger grasp,
self-inhibits to "no," and bangs two cubes is:

7-8 mo
Explanation: These milestones correspond respectively to control of
exploration, exploration of small objects, response to tone, and
comparison of objects. (See Chapter 10 in Nelson Textbook of Pediatrics,
17th ed.)

10-12 mo

12-15 mo

3-4 mo

15-18 mo
Question . 24. Growth between 3 and 4 mo of age is best characterized as:

Accelerating to a rate of 45 g/day

Slowing to a rate of 10 g/day

Slowing to a rate of 20 g/day


Explanation: This is a normal response that does not cause the child to
cross growth percentiles in a growth chart. (See Chapter 10 in Nelson
Textbook of Pediatrics, 17th ed.)

Accelerating to a rate of 20 g/day

Demonstrating no change compared with rate between 0 and 2 mo

Question . 25. Feeding between 6 and 12 mo of age is characterized by all of the following
except:

Being willing to be fed by a stranger


Explanation: Indeed, stranger anxiety may start to set in at this time. (See
Chapter 10 in Nelson Textbook of Pediatrics, 17th ed.)

Appearing autonomous

Eating finger foods

Turning away from the spoon

Holding a spoon

Question . 26. The probable age of a child who skips, names four colors, and dresses and
undresses is:

15 mo

24 mo

30 mo

18 mo

60 mo
Explanation: This is normal, appropriate motor, language, and social
development. (See Chapter 11 in Nelson Textbook of Pediatrics, 17th ed.)
Question . 27. Early walking suggests:

Preoccupation with objects

Advanced social development

Advanced language development

High-activity type
Explanation: More active types tend to walk early. The other choices are
incorrect. (See Chapter 11 in Nelson Textbook of Pediatrics, 17th ed.)

Spasticity

Question . 28. Between 2 and 5 yr of age, language increases; as a rule, the number of words in
a sentence is:

Based on knowledge of numbers

Equal to the age of the child in years


Explanation: A 2-yr-old has two-word sentences, such that language is
dependent on the environment and verbal interactions with adults. (See
Chapter 12 in Nelson Textbook of Pediatrics, 17th ed.)

Independent of the environment

Independent of the number of questions asked the child by adults

Based on the ABCs

Question . 29. All of the following statements regarding language development are true except:

Deaf children may create their own language

The basics for language may be "hard-wired" in the brain

Language has no role in behavior regulation


Explanation: Language plays a critical part in the regulation of behavior
as children internalize speech. (See Chapter 12 in Nelson Textbook of
Pediatrics, 17th ed.)

Delayed language may signify deafness

Delayed language may signify mental retardation


Question . 30. Growth during the years between 6 and 12 yr is characterized by annual weight
and height increments of:

3.5 kg, 6 cm
Explanation: 3.5 kg (7 lb) and 6 cm (2.5 in) per year are the average
increments in this time period. (See Chapter 13 in Nelson Textbook of
Pediatrics, 17th ed.)

6 kg, 3.5 cm

5 kg, 10 cm

10 kg, 5 cm

1.5 kg, 5 cm

Question . 31. Word-finding difficulties may result in all of the following except:

Difficulty in expressing feelings

Difficulty in verbal self-defense

Frustration

Success in English class


Explanation: Language-based classes produce difficulties for children
with word-finding difficulties. (See Chapter 13 in Nelson Textbook of
Pediatrics, 17th ed.)

Physical acting out

t
Psychologic Disorders, Set 1
Question . 1. Neuroleptic antipsychotic agents produce all of the following unwanted side
effects except:

Bradykinesia

Hyperthermia

Tardive dyskinesia

Inappropriate secretion of ADH


Explanation: Common complications of neuroleptic antipsychotic agents
include extrapyramidal symptoms (Parkinson-like syndrome), sedation,
and anticholinergic symptoms. Neuroleptic malignant syndrome
(malignant hyperthermia) and tardive dyskinesia are rarer complications.
(See Chapter 27 in Nelson Textbook of Pediatrics, 17th edition.)

Sedation

Question . 2. A 4-yr-old boy is noted to have stereotypic body movements and poor verbal and
nonverbal communication, with absence of empathy. At daycare, he has not made any friends.
The most likely diagnosis is:

Attention deficit hyperactivity disorder

Dysthymic syndrome

Deaf-mutism

Autism
Explanation: Autism is a disease of unknown cause and is more common
in males. It is characterized by the symptoms noted in this patient, with
onset usually before 30 mo of age. (See Chapter 27 in Nelson Textbook
of Pediatrics, 17th edition.)

Cerebral palsy

Question . 3. Head banging, hair twirling, rocking, thumb sucking, teeth grinding, and nail biting
all are:

Habit disorders that probably relieve tension


Explanation: The belief that choice B is correct leads to repeated
attempts to "break" children (a distressing term and concept), probably
in effect reinforcing the behavior. (See Chapter 21 in Nelson Textbook of
Pediatrics, 17th edition.)

Behavior problems that are easy to cure in children

Evidence of insecurity in the majority of children and poor parenting by


their parents

Tics
Question . 4. Which of the following statements about Gilles de la Tourette syndrome is true?

It is characterized by tics and coprolalia


Explanation: Gilles de la Tourette syndrome, which has a lifetime
prevalence rate of 0.5 per 1000 persons, is a rare condition in children. It
is characterized by multiple tics, compulsive barking, and shouting
obscene words (coprolalia). It is more common in first-degree relatives of
patients with Tourette syndrome than in the general population and
affects boys three to four times more frequently than girls. The cause is
uncertain, but research has shown that drugs that increase
dopaminergic action precipitate or worsen both tics and Tourette
syndrome. Many environmental precipitants have been noted to serve as
emotional stress sources. The syndrome can be fairly well managed with
haloperidol, a dopaminergic antagonist. Anecdotal reports in the
literature suggest that the serotonin reuptake inhibitors are also
efficacious in its treatment. (See Chapter 21 in Nelson Textbook of
Pediatrics, 17th edition.)

It is characterized by tics and encopresis

It is treated with haloperidol and methylphenidate

It is a common disorder of childhood

It affects girls more often than boys

Question . 5. Night terrors are associated with:

REM sleep

Overeating after 7:00 p.m.

The use of antipsychotic medication

Inception in preschool years and occasional somnambulism


Explanation: Night terrors most commonly occur during stage IV, deep
sleep. Neither the use of antipsychotic medications nor overeating after
7:00 p.m. has ever been shown to be associated with night terrors. They
usually begin in the preschool years. A child having night terrors is
confused and disorientated and shows signs of intense autonomic
activity (labored breathing, dilated pupils, sweating, tachypnea,
tachycardia). Sleepwalking may occur during night terrors and may put a
child at risk for injury. The incidence of night terrors is said to be
between 1% and 4% and is greater in boys. There is a familial pattern in
the development of the symptoms, and febrile illnesses may serve as
precipitating factors. (See Chapter 20.5 in Nelson Textbook of Pediatrics,
17th edition.)

Anger within the family


Question . 6. Conduct disorder in childhood and adolescence is associated with all of the
following except:

Antisocial behavior

Criminality in the father

Physical abuse

Marital discord within the home

Mental retardation
Explanation: Conduct disorder is a distinct clinical entity manifested by
several different antisocial behaviors: stealing, lying, fire setting,
truancy, property destruction, cruelty to animals, rape, use of weapons
while fighting, armed robbery, physical cruelty to others, and repeated
attempts to run away from home. Many argue that conduct disorder is
not a unitary illness but instead comprises three different syndromes
characterized primarily by aggression, intermittent antisocial behaviors,
and delinquency. Little is known about the antecedents of each of these
subtypes or the outcome of patients suffering from them. Risk factors
associated with the development of conduct disorders include antisocial
behavior within family members, criminality in the father, physical abuse
within the home, and marital discord within the home. Many different
approaches have been used in the treatment of children and adolescents
with aggressive behavior, antisocial behavior, and delinquency. The
most effective results have been obtained with parent training
management, in which parents are trained directly to promote prosocial
behaviors within the home and to place reasonable limits on unwanted,
destructive behavior. (See Chapter 25 in Nelson Textbook of Pediatrics,
17th edition.)

Question . 7. Completed suicides in childhood and adolescence may be associated with all of
the following except:

Previous suicide attempts

Alcohol or drug abuse

A history of depression and suicide within the family

Easy access to firearms

Perfectionism within the classroom


Explanation: Previous suicide attempts, substance abuse, easy access
to firearms, and a family history of depression and suicide are all closely
related to both suicide attempts and completed suicides in both children
and adolescents. Fifteen to 40% of completed suicides are preceded by
other suicide attempts. In more than one third of suicides, a parent, a
sibling, or another first-degree relative has previously shown overt
suicidal behavior. Firearms serve as the major method of death in
adolescent suicide. Among preadolescents, jumping from heights is the
most common method. Although perfectionism in the classroom has
been shown to be associated with specific types of anxiety, no
correlation has been shown between perfectionism and suicidal ideation
or suicidal behavior. (See Chapter 24 in Nelson Textbook of Pediatrics,
17th edition.)

Question . 8. A child in the third grade has problems with spelling and reading. She appears
very quiet and confused in class. Her teacher has noticed that this girl has trouble following
directions. Her mind seems to wander whenever the teacher tells a story or explains something
complicated. She is skilled in art and so far has performed well in arithmetic. Which of the
following diagnostic procedures is most likely to yield useful findings in this child?

An attention deficit questionnaire

An intelligence test

A language evaluation
Explanation: The question offers a classic example of a language
problem, particularly in a child with partial understanding. Many children
or parents do not admit to this problem. (See Chapter 29 in Nelson
Textbook of Pediatrics, 17th edition.)

A psychiatric assessment to rule out depression

A neurologic examination

Question . 9. An 11-yr-old child has excellent ideas in a class discussion, but what she records
on paper is primitive and unsophisticated. She can spell well in isolation and understands rules
of punctuation and capitalization, but in her own writing she makes multiple errors and mistakes
in punctuation and capitalization. Her handwriting is legible, but writing is painfully slow. This
girl most likely is having problems with:

Expressive language

Graphomotor production

Ideation

Attention

Simultaneous retrieval memory


Explanation: Simultaneous retrieval memory defects are depicted by the
history of the child described in the question. (See Chapter 29 in Nelson
Textbook of Pediatrics, 17th edition.)
Question . 10. Somatoform disorders, as part of psychosomatic illness, include all of the
following except:

Conversion reaction

Asthma
Explanation: Asthma may be exacerbated by psychologic factors and is
therefore a psychophysiologic disorder. (See Chapter 19 in Nelson
Textbook of Pediatrics, 17th edition.)

Hypochondriasis

Pain disorders

Question . 11. Münchausen syndrome by proxy is characterized by all of the following except:

Recurrent illness that cannot be explained

Statements from experienced pediatricians that they have never seen


such a case

Symptoms that disappear with the parent present


Explanation: In the Münchausen by proxy syndrome, a patient's
symptoms disappear if the parent goes home. Manifestations reappear
when the involved parent returns to the hospital. (See Chapter 35 in
Nelson Textbook of Pediatrics, 17th edition.)

An attentive parent caregiver who never goes home

An unworried parent caregiver

Poor response to therapy

Doctor shopping

Question . 12. When a 7-yr-old child fails to cooperate with care in the hospital, one should
suspect:

Immaturity

Embarrassment

Negativism

Fearfulness
Explanation: Children may be frightened and react to fear with a personal
manner of withdrawal or poor cooperation. (See Chapter 17 in Nelson
Textbook of Pediatrics, 17th edition.)

Oppositionism
Question . 13. If a parent does not appear readily reassured by the diagnosis or treatment plan,
one should suspect:

Hidden anxiety
Explanation: Parents often hold back questions that are highly charged
or that may appear "stupid." They may be angry, ashamed, or
uncomfortable in asking these questions. (See Chapter 17 in Nelson
Textbook of Pediatrics, 17th edition.)

Mistrust

Negativism

Oppositionism

Aggression

Question . 15. Psychosocial problems may manifest as disturbances in:

Feelings

Body function

Behavior

Performance

All of the above


Explanation: Disturbances may manifest as depression, anxiety,
aggression, and learning problems. (See Chapter 27 in Nelson Textbook
of Pediatrics, 17th edition.)

Question . 16. Psychiatric disorders are more common than in the general population of children
in all of the following except:

Smart students
Explanation: An important aspect of these conditions is the capacity of
the parents to adjust and cope. (See Chapter 18 in Nelson Textbook of
Pediatrics, 17th edition.)

Head trauma

Mental retardation

Epilepsy

Prematurity

Encephalitis
Question . 17. Conversion reactions are best characterized by:

Sudden onset

Traceability to a precipitating event

Involvement of special senses

Pseudoseizures

Abrupt end

All of the above


Explanation: The loss or alteration of function without a demonstrable
organic cause defines a conversion reaction, a type of somatoform
disorder. (See Chapter 19 in Nelson Textbook of Pediatrics, 17th edition.)

Question . 18. Enuresis is defined as:

Wetting 2 times per week for 3 consecutive mo


Explanation: Alternatively, it may be defined as wetting that produces
clinically significant distress for the child. (See Chapter 20 in Nelson
Textbook of Pediatrics, 17th edition.)

Wetting 2 times per week for any 3 mo in a year

Not being dry at 3 yr of age

Not being dry at 4 yr of age

Wetting at 5 yr of age on two occasions

Question . 19. Nocturnal enuresis is described by all of the following except:

Being primary or secondary

Having a strong genetic component

Occurring at all stages of sleep

When primary, being associated with emotional disorders


Explanation: There is no higher rate of emotional disturbances in age-
matched enuretic and nonenuretic children. (See Chapter 20 in Nelson
Textbook of Pediatrics, 17th edition.)

Being more common in males


Question . 20. Treatment of enuresis should include all of the following except:

Enlisting the cooperation of the child

Having the child void before retiring

Using alarms

Having the child launder the soiled sheets

Waking the child repeatedly


Explanation: Repeated waking may be beneficial for a few children but
creates added stress and hostility. (See Chapter 20 in Nelson Textbook
of Pediatrics, 17th edition.)

Question . 21. A 5-yr-old is noted by the parents to snore at night. The child has also had
problems staying awake in preschool and has had behavioral problems. The father also snores.
Physical examination of the child reveals large, pink, nonexudative tonsils. The most
appropriate next step is:

Laryngoscopy

Polysomnography
Explanation: Polysomnography reveals episodes of apnea and hypoxia.
(See Chapter 20 in Nelson Textbook of Pediatrics, 17th edition.)

Ambulatory apnea monitoring

Telemetry

Arterial blood gas analysis

Question . 22. The most likely diagnosis of the patient in Question 21 is:

Tonsillitis

Peritonsillar abscess

Obstructive sleep apnea syndrome


Explanation: Obstructive sleep apnea syndrome (OSAS) commonly
presents with airway obstruction during sleep with resultant sleep
disturbances at night and daytime sleepiness. (See Chapter 20 in Nelson
Textbook of Pediatrics, 17th edition.)

Tangier disease

Narcolepsy
Question . 23. The appropriate therapy for severe obstructive sleep apnea syndrome is:

Adenotonsillectomy
Explanation: Removal of the hypertrophied tissue relieves the symptoms
of OSAS. (See Chapter 20 in Nelson Textbook of Pediatrics, 17th edition.)

Tracheostomy

Parapharyngeal muscle surgery

Theophylline

Bilevel positive airway pressure

Question . 24. Risk factors for obstructive sleep apnea syndrome include all of the following
except:

Retroposition of the mandible

Small triangular chin

Long oval face

Long or soft palate

All of the above


Explanation: All of the answers are risk factors as well as the more
obvious such as Pierre Robin syndrome and Prader-Willi syndrome. (See
Chapter 20 in Nelson Textbook of Pediatrics, 17th edition.)

Question . 25. All of the following are considered habit disorders except:

Tics

Bruxism

Trichotillomania

Stuttering
Explanation: Stuttering is often discussed with habit disorders; however,
it is probably not a true habit in that it is not regarded as a tension-
relieving activity. (See Chapter 21 in Nelson Textbook of Pediatrics, 17th
edition.)

Thumb sucking
Question . 26. Tics are characterized by all of the following except:

Difficulty in controlling behavior

Occurrence of brief, transient amnesia after the tic


Explanation: This would be more compatible with a minor seizure. (See
Chapter 21 in Nelson Textbook of Pediatrics, 17th edition.)

Disappearance during sleep

Possible occurrence following encephalitis

Normal EEG

Question . 27. All of the following are true of Tourette syndrome except:

It is made worse by drugs that increase dopaminergic action

Some patients have pediatric autoimmune neuropsychiatric disorder

Some patients have oppositional defiant disorder

Lyme disease may sometimes mimic Tourette syndrome

Management with haloperidol or pimozide is often unsuccessful


Explanation: Both drugs provide a fair degree of relief. (See Chapter 21 in
Nelson Textbook of Pediatrics, 17th edition.)

Question . 28. A third-grade student refuses to go back to school after the winter break. She now
needs her mother to go to sleep with her and complains of headache, bellyache, and muscle
pain. Findings on physical examination are totally normal, but you notice the child is very clingy
to the mother. The most likely diagnosis is:

Stranger anxiety

School anxiety

Stranger reaction

Separation anxiety disorder


Explanation: Parents often consciously or unconsciously encourage this
intense fear of separation because of a fear that something will happen
to the caregiver. (See Chapter 22 in Nelson Textbook of Pediatrics, 17th
edition.)

Narcolepsy
Question . 29. Obsessive-compulsive disorder may be associated with all of the following
except:

Overconcern with body wastes

Prior group A streptococcal infection

Excessive fears

A need for sameness

Increased metabolic activity in the corpus callosum


Explanation: Positron emission tomography may reveal excessive
metabolic activity in the frontal lobes and basal ganglia. (See Chapter 22
in Nelson Textbook of Pediatrics, 17th edition.)

Excessive checking of locks

Question . 30. Major depression is characterized by:

Weight loss

Weight gain

Insomnia

Hypersomnia

Dysphoria

All of the above


Explanation: Major depression may also include manifestations of
fatigue, loss of interest or pleasure (dysphoria), agitation or retardation,
excessive guilt, feelings of worthlessness, and poor concentration. (See
Chapter 23 in Nelson Textbook of Pediatrics, 17th edition.)

Question . 31. Major depression in children may be characterized by:

A strong genetic component


Explanation: Twin studies show a strong genetic component to major
depression. (See Chapter 23 in Nelson Textbook of Pediatrics, 17th
edition.)

Delay of onset until after adolescence

Absence of hallucinations

No risk of depression in adulthood

None of the above


Question . 32. The treatment of choice for childhood-onset major depression is:

Monoamine oxidase inhibitors

Tricyclic antidepressants

Serotonin reuptake inhibitors


Explanation: SSRIs have proven efficacy and have a reasonable safety
profile. (See Chapter 23 in Nelson Textbook of Pediatrics, 17th edition.)

Benzodiazepines

None of the above

Question . 33. All of the following statements about suicide are true except:

15-40% of completed suicides are preceded by attempts

There are 4-5 attempts for each suicide

Access to guns increases the risk of suicide

Alcohol use is unrelated to suicide


Explanation: Use of alcohol and other drugs is associated
epidemiologically with suicide. (See Chapter 24 in Nelson Textbook of
Pediatrics, 17th edition.)

Depression is related to suicide

Question . 34. Important questions after an attempted suicide include all of the following except:

Is the patient less depressed?

Is the patient physiologically stable?

Does the patient still want to die?

Are precipitating events still active?

Does the patient have a future view or orientation?

Have the shame and guilt been moderated?

All of the above


Explanation: These are all helpful in discharge planning and in assessing
recurrence risks. (See Chapter 24 in Nelson Textbook of Pediatrics, 17th
edition.)
Question . 35. A 2-yr-old doesn't get his way in a crowded toy store. He starts to cry and hit and
roll on the floor. The parent should do all of the following except:

Yell and punish him


Explanation: Caregiver anger reinforces the child's behavior and teaches
a vicious cycle of oppositional behavior. (See Chapter 25 in Nelson
Textbook of Pediatrics, 17th edition.)

Acknowledge the child's frustration

Quietly explain that his response is not acceptable

Give him time and space to recover

Nonemotionally place him on a time out

Question . 36. A 4-yr-old is seen hitting his sister. When asked what he has done, he lies. His
lying is most likely to represent:

Pathologic behavior, which needs punishment

Displacement

Reaction formation

Avoidance of an unpleasant punishment

None of the above


Explanation: Lying is common at every developmental age and has
reasons at each level of development. It may include fantasy, avoidance
of punishment, and fear. Pathologic repeated chronic lying often occurs
in combination with other antisocial behavior. At any time, lying should
be approached calmly with an understanding of the fears of the child.
Nonetheless, honesty must be reinforced. (See Chapter 25 in Nelson
Textbook of Pediatrics, 17th edition.)

Question . 37. All of the following statements regarding minor stealing are true except:

All (almost) children steal at least once

It must be overemphasized to avoid a second episode


Explanation: Overemphasis may create an excitement to repeat the
offense. (See Chapter 25 in Nelson Textbook of Pediatrics, 17th edition.)

They should return the stolen item

It may be learned from parents

It may be impulsive
Question . 38. A 26-mo-old boy has a history of poor speech development, tantrum-like rages,
and rocking, repetitive, ritualistic behavior. He attends daycare but spends most of his time in
solitary play. The most likely diagnosis is:

Encephalitis

Latent slow virus infection

Rasmussen disease

Autism
Explanation: Autism begins before 30 mo of age and has a 4:1 male-to-
female ratio. It is characterized by impaired verbal and nonverbal
communication, imaginative activity, and reciprocal social interaction.
(See Chapter 27 in Nelson Textbook of Pediatrics, 17th edition.)

Prader-Willi syndrome

Question . 39. Additional features of the disease affecting the patient described in Question 38
include all of the following except:

Empathy
Explanation: Lack of social relations and absent empathy are typical of
children with autism. (See Chapter 27 in Nelson Textbook of Pediatrics,
17th edition.)

Preoccupation with body parts

Prevalence of 3/1000-4/1000 population

Association with fragile X

Visual scanning of the fingers

Question . 40. All of the following statements regarding attention deficit hyperactivity disorder
are true except:

It is more common in males than in females

Tic disorders may coexist with ADHD

Patients dislike or avoid sustained mental efforts

Oppositional defiant disorders never coexist with ADHD


Explanation: Co-morbidity includes oppositional defiant disorder in 50%,
conduct disorders in 30-50%, anxiety disorder in 20-25%, mood disorders
in 15-20%, and learning disorders in 10-25%. (See Chapter 29 in Nelson
Textbook of Pediatrics, 17th edition.)

Substance abuse occurs in affected adolescents


Question . 41. Possible complications of stimulant drug therapy for attention deficit
hyperactivity disorder include all of the following except:

Jitteriness

Difficulty sleeping

Abdominal pain

Tics

Increased appetite
Explanation: Anorexia is common; this affects growth. (See Chapter 29 in
Nelson Textbook of Pediatrics, 17th edition.)

Question . 42. Which of the following statements is false?

The clinical interview is a tool for gathering information, not for


enhancing behavior
Explanation: (See Chapter 17 in Nelson Textbook of Pediatrics, 17th
edition.)

Most adolescents do not experience more stress during this


developmental stage than at others

Some "symptomatic" actions of children may be part of normal


development

Chronic stresses are more difficult for a child to manage than a single
acute stressful episode

Infants tend to react to stressful situations with impaired physiologic


functions

Question . 43. Which of the following statements is false?

Head injury is relatively common in pediatric populations

Serious brain injuries early in life may be more generally incapacitating


than those that occur later

There is an increased risk of a psychiatric disorder after brain injury

Psychosis is often seen as a result of brain injury in childhood


Explanation: (See Chapter 18 in Nelson Textbook of Pediatrics, 17th
edition.)

Even when head injury is judged to be mild, there may be educational


problems in 25-50% of affected children
Question . 44. Which of the following statements is false?

In somatoform disorders symptoms are associated with unconscious


conflict, but in factitious disorders the unconscious need to be cared for
motivates the falsification of symptoms

Conversion disorder is a type of somatoform disorder that usually


presents in early childhood
Explanation: (See Chapter 19 in Nelson Textbook of Pediatrics, 17th
edition.)

Pseudoseizures are the most common conversion symptom

10-15% of children have ongoing somatic symptoms

Psychoactive medications may be useful in treating anxiety or


depression in childhood

Question . 45. Which of the following choices is not a helpful treatment for enuresis?

Desmopressin acetate

Imipramine

Maintaining a star chart

Bell-and-pad apparatus

Punishment
Explanation: (See Chapter 20 in Nelson Textbook of Pediatrics, 17th
edition.)

Question . 46. Each of the following disorders is often co-morbid with Tourette syndrome
except:

Borderline personality disorder


Explanation: (See Chapter 21 in Nelson Textbook of Pediatrics, 17th
edition.)

Obsessive-compulsive disorder

Attention deficit hyperactivity disorder

Oppositional defiant disorder

Tic disorder
Question . 47. A 7-yr-old boy is referred to you by the school for problems with hyperactivity,
disruptive behavior, and stomach aches. They report that he "acts up" in class and have
requested that the parents pick him up from school early on several occasions. The child
presents to your clinic with his mother. He does not appear fidgety during the evaluation. His
mother reports few problems at home except that he complains of stomach aches when he
wakes up in the morning. His mother appears depressed and "tears up" while sharing that her
husband was recently diagnosed with cancer. You diagnose the child to be suffering from
separation anxiety disorder and recommend the following:

A trial of an SSRI for the child

A trial of dexedrine or methylphenidate

Tell the mother she needs to be strong for the rest of the family

Referral of the family for therapy


Explanation: (See Chapter 22 in Nelson Textbook of Pediatrics, 17th
edition.)

Obtain permission from school officials for the child to be home-


schooled for the rest of the semester

Question . 48. A 12-yr-old girl presents with a history of repetitive handwashing related to
contamination obsessions. After performing an interview you diagnose OCD and begin
treatment with sertraline. After an initial response, you increase the dose to the recommended
maximum dose. She tolerates the medication without side effects but continues to be impaired
by the obsessions and washing rituals. The next step in treatment involves:

Increasing the sertraline

Adding fluvoxamine to the sertraline

Referring the girl for cognitive-behavioral therapy


Explanation: (See Chapter 22 in Nelson Textbook of Pediatrics, 17th
edition.)

Adding risperidone to the sertraline

Obtaining an ASO titer

Question . 49. A 9-yr-old boy who witnessed the murder of his mother by his father is referred to
you by the school psychologist. She has treated the child for 9 mo and is concerned that the
child is quite "jumpy" at school. In addition, she states that his grades have fallen because he
cannot pay attention in class. He reports difficulty falling asleep at night. Which of the following
statements concerning the use of medications for the treatment of post-traumatic stress
disorder (PTSD) in children is true?

Clonidine and guanfacine are helpful for sleep disturbance and


persistent arousal
Explanation: (See Chapter 22 in Nelson Textbook of Pediatrics, 17th
edition.)
Stimulants are the treatment of choice for inattention in PTSD

SSRI-medications are not helpful in the treatment of co-morbid


depression, anxiety, and withdrawal.

Alprazolam and lorazepam are recommended treatments for anxiety and


sleep problems associated with PTSD

Too much time has passed after the trauma, so medications will be of
little help

Question . 50. The lifetime prevalence for depression starting in adolescence is:

2-4%

6-8%

15-20%
Explanation: (See Chapter 23 in Nelson Textbook of Pediatrics, 17th
edition.)

35-50%

60-70%

Question . 51. All of the following statements about adolescent suicide are true except:

It is the leading cause of death in adolescence

Mood problems and violence are major risk factors

Substance abuse is a major risk factor

The suicide rate is higher in younger adolescents than in older


adolescents
Explanation: (See Chapter 24 in Nelson Textbook of Pediatrics, 17th
edition.)

Previous suicide attempts are a major risk factor

Question . 52. Which of the following statements is false?

Breath holding is not unusual during the first year of life

In response to tantrums, parents should be advised to give the child time


and space to recover by turning away briefly
Lying is more common in children with high self-esteem
Explanation: (See Chapter 25 in Nelson Textbook of Pediatrics, 17th
edition.)

Truancy and run-away behavior are never developmentally appropriate

Children exposed to aggressive models on television display more


aggressive behavior than that noted in children not so exposed

Question . 53. Which of the following statements about homosexuality in adolescence is true?

The prevalence is less than 1%

It is a risk factor for suicide, especially in males


Explanation: (See Chapter 26 in Nelson Textbook of Pediatrics, 17th
edition.)

It is designated as a psychologic disorder by the American Psychiatric


Association

Behavioral treatments for homosexuality have been shown to be


effective in changing orientation

It is associated with various psychologic disorders in adulthood

Question . 54. Which of the following statements is false?

Autism is typically diagnosed after 36 mo


Explanation: (See Chapter 27 in Nelson Textbook of Pediatrics, 17th
edition.)

Ritualistic behavior is common with autism

In autism, intelligence measured by conventional psychological testing


usually falls in the functionally retarded range

Genetic factors play a significant role in autism

Early, intensive behavioral therapy, targeted toward speech and


language development, is successful in improving language and social
function of autistic children
Question . 55. Which of the following medications is not used as a mood stabilizer?

Depakote

Lithium carbonate

Carbamazepine

Topiramate

Fluoxetine
Explanation: (See Chapter 28 in Nelson Textbook of Pediatrics, 17th
edition.)

Question . 56. All of the following statements are true except:

Chromosome abnormalities can lead to unique patterns of language


defects

Children who can register and consolidate facts and procedures in


memory may have great difficulty accessing or retrieving these items
when they are needed

Children with special ordering weaknesses do not have problems with


letter and word recognition
Explanation: (See Chapter 29.1 in Nelson Textbook of Pediatrics, 17th
edition.)

Children with special ordering weaknesses may be late in discriminating


between right and left

Dyspraxia in general relates to difficulty in developing an ideomotor plan


and activating coordinated motor actions to complete a task

Question . 57. All of the following statements are true except:

ADHD frequently occurs with other emotional, behavioral, language, and


learning disorders

ADHD is 3 to 4 times more common in males than in females

Several standard behavior rating scales do well in discriminating


between children with ADHD and controls, and these scales are
sufficient to make the diagnosis
Explanation: (See Chapter 29.2 in Nelson Textbook of Pediatrics, 17th
edition.)
Depression and anxiety disorder may present many of the same
symptoms as those of ADHD

Psychosocial interventions, behavior management training, and


medication are effective in treating various components of ADHD

Question . 58. A 20-yr-old college student is referred from the university health service because
of very poor grades during the previous semester. His class participation is good, he seems to
understand the concepts, but he is unable to finish his tests. His admission folder indicates that
he was diagnosed as having a reading problem in fourth grade and received special help in
grades 5-8. He was allowed to take his SATs untimed. Of the following, the most therapeutic
recommendation is:

Intensive phonologic awareness training

Provision of extra time on tests


Explanation: (See Chapter 29.3 in Nelson Textbook of Pediatrics, 17th
edition.)

Systematic, explicit instruction in phonics

Multisensory training

Referral to ophthalmology

Question . 59. An 8-yr-old girl is referred for evaluation of learning problems in school. Her
mother reports that her father, an independent plumbing contractor, had similar problems. The
child's motor milestones were all normal, but she did not say her first words until 18 months.
The most important element in making a diagnosis of dyslexia in this child is:

Verbal IQ tested at a standard score of 94

Poor balance on tandem gait testing

Many errors on a continuous performance test

History of aggressive behavior with classmates

Word reading test at a standard score of 84


Explanation: (See Chapter 29.3 in Nelson Textbook of Pediatrics, 17th
edition.)
Question . 60. Parents of a 12-yr-old girl are concerned because at a parent-teacher conference
they were told that their daughter does not seem to be keeping up with her schoolwork in
language arts, although her abilities in mathematics place her at the top of the class. She had
difficulty in early grades with reading but then seemed to catch up. Currently, she seems to be
able to read words accurately but reads very slowly. She reads the necessary school
assignments but does not read for pleasure. Which of the following is the most likely
explanation for this history?

Poor motivation

Inability to decode words

Low intelligence

Poor reading influence


Explanation: (See Chapter 29.3 in Nelson Textbook of Pediatrics, 17th
edition.)

Parent-child problems in the home


Social Issues

Question . 1. A 6-mo-old boy is brought to the emergency room and is afebrile but responds poorly to
tactile and auditory stimuli. He becomes apneic and unresponsive after a generalized seizure. The
parents state that he was perfectly well in the car on the way to the hospital and that they only brought
him to the emergency room because of constipation. He requires 10 min of cardiopulmonary
resuscitation, after which he is noticed to have a bulging fontanel and bilateral retinal hemorrhages. A
chest film reveals two posterior rib fractures. The most likely diagnosis is:

CPR-induced retinal hemorrhages and rib fractures

Hemorrhagic shock and encephalopathy

Hemophilia

Status epilepticus

Child abuse-shaken baby syndrome


Explanation: In the classic shaken baby syndrome, a computed tomography
scan of the head reveals diffuse cerebral edema and hemorrhage. CPR in
young children does not usually produce retinal hemorrhages and rarely, if
ever, produces rib fractures. (See Chapter 35 in Nelson Textbook of Pediatrics,
17th ed.)

Question . 2. A young child's response to the death of a parent often is characterized by:

Depression and weight loss

Denial and magical wishing


Explanation: Many young children continue in their daily activities and use
denial and magical wishful thoughts for reunion and reappearance. (See
Chapter 33 in Nelson Textbook of Pediatrics, 17th ed.)

Anger and crying

Wishes of death for himself or herself

None of the above

Question . 3. The effect that statements such as "stop it or you'll give me a headache" have on young
children is to:

Teach a child to behave

Give children a pattern of headaches

Create guilt and unrealistic fault


Explanation: Statements such as "stop it or you'll give me a headache" may
cause a child to suffer significant and unrealistic guilt, especially if the parent
leaves for some time or is hospitalized. (See Chapter 33 in Nelson Textbook of
Pediatrics, 17th ed.)
Provide parents with a way to cope

Prepare children for separation

Question . 4. All of the following statements about adoption are true except:

42% are stepparent or relative adoptions

Most adopted children are from foreign countries


Explanation: Five percent of adoptions are from overseas. (See Chapter 30 in
Nelson Textbook of Pediatrics, 17th ed.)

1 million children are adopted in the United States each year

15% are adopted through foster care

2-4% of American families have adopted children

Question . 5. All of the following statements regarding foster care are true except:

Chronic medical illness is present in 35% of children

60% of preschool children in foster care experience developmental delay

42% are white children

A majority of foster children receive EPSDT services


Explanation: Early prevention screening and developmental testing programs
are underutilized in the foster care system. (See Chapter 31 in Nelson
Textbook of Pediatrics, 17th ed.)

Children frequently stay at more than one foster care home

Question . 6. High-quality child care can influence all of the following except:

Child cognition

Future academic achievement

Social development

Sibling rivalry
Explanation: Sibling rivalry and aggression may not be affected as much as
the cognitive issues. (See Chapter 32 in Nelson Textbook of Pediatrics, 17th
ed.)

Scores on standardized tests


Question . 7. A 3-yr-old boy is missing from his mother's house approximately 1 mo after a divorce. The
most likely explanation is:

Sleepwalking

Drug reaction

Running away from his mother

Searching for his father


Explanation: It is not unusual for the young child to keep asking for the
missing parent, to wait at the door or window, or to go outside to look for the
parent. (See Chapter 33 in Nelson Textbook of Pediatrics, 17th ed.)

Anxiety reaction

Question . 8. After divorce, children may demonstrate all of the following except:

A feeling of being overburdened by residence in two homes

Withdrawal

Indifference at times of reunions

Academic deterioration

Expectations that the parents will never get back together


Explanation: Indeed, most children fantasize about the possibility that their
parents will remarry. (See Chapter 33 in Nelson Textbook of Pediatrics, 17th
ed.)

Question . 9. All of the following are age-related behavioral responses to experiencing violence except:

Infants-poor sleep

Adolescents-short-fuse responses

Toddlers-excessive appetite
Explanation: All the rest are true, as well as poor appetite, decreased
exploration of the environment in toddlers, and poor school performance with
hyperactivity in older children. (See Chapter 34 in Nelson Textbook of
Pediatrics, 17th ed.)

Toddlers-clingy behavior

School age-post-traumatic stress syndrome


Question . 10. Munchausen syndrome by proxy is characterized by all of the following except:

10% mortality

Multiple hospitalizations

Induced manifestations by caregiver

Ready admission of abuse by parents


Explanation: Parents often deny their involvement in inducing symptoms in
their children and will rapidly change doctors if it is discussed with them. (See
Chapter 35 in Nelson Textbook of Pediatrics, 17thed.)

Use of medications or toxins

Question . 11. Factors that may be associated with an increased risk of child abuse include:

Poverty

Military base residence

Spouse abuse

Unplanned pregnancy

All of the above


Explanation: Each is a risk factor. Of note, abuse has been reported in all
communities and from all socioeconomic levels. (See Chapter 35 in Nelson
Textbook of Pediatrics, 17th ed.)

Question . 12. A 2-mo-old is admitted with a fracture of the right femur. The mother states that the baby
fell off a low couch onto a plush carpeted floor and did not cry. Thereafter, the baby appeared fine.
Three days later, the grandmother noted that the baby cried when she changed the diaper and that the
leg was swollen. In the emergency department, a bruise was noted over the sternum that was also said
to have occurred during the fall 3 days ago. The mother states that she bleeds easily, but that the
father of the baby is well. An x-ray film reveals a spiral fracture of the child's femur. Features of this
case suggestive of abuse include all of the following except:

Multiple sites of injury

Implausible explanation for injury

Grandmother's deep concern


Explanation: Often, relatives do not know the nature or the cause of the injury,
nor do they readily know or admit to the identity of the person who committed
the trauma. (See Chapter 35 in Nelson Textbook of Pediatrics, 17th ed.)

Injury incompatible with the nature of the fall

Delay in seeking medical attention


Question . 13. Cardiopulmonary resuscitation in a child with head trauma from abuse commonly
results in all of the following except:

Recovery of a pulse

Retinal hemorrhages and broken ribs


Explanation: Retinal hemorrhages and broken ribs rarely follow CPR. The
nature of rib fractures is also different in abuse. (See Chapter 35 in Nelson
Textbook of Pediatrics, 17th ed.)

Recovery of respirations

Normal sinus rhythm

Recovery from cyanosis

Question . 14. Sexual abuse includes all of the following except:

Exposing sexual anatomy

Touching genitals by two preadolescents


Explanation: This is sexual play and is usually normal exploratory behavior if
there is no force or coercion and the children are not more than 4 years apart
in age. (See Chapter 35 in Nelson Textbook of Pediatrics, 17th ed.)

Showing pornography to a child

Use of a child to create pornography

Incest

Question . 15. Pedophiles are best described as:

Being female

Never having repeated experiences

Seeking opportunities to be in contact with children


Explanation: Pedophiles often seek out positions and opportunities to be
around children. Certain children may be particularly vulnerable, such as
those with mental and physical handicaps. (See Chapter 35 in Nelson
Textbook of Pediatrics, 17th ed.)

Preferring females

Being highly violent


Question . 16. A 4-yr-old girl is admitted to the hospital for her third evaluation for vaginal bleeding.
The mother noted bright red blood on the child's underwear. Previous examinations revealed a normal
4-yr-old girl, Tanner stage 1, with normal external genitalia. Pelvic ultrasound results were normal, as
was the serum estradiol level. The hemoglobin and platelet counts were normal, as were the bleeding
time and coagulation studies. Findings on pelvic examination conducted under anesthesia also were
normal. The next step in the examination is to:

Determine the blood type of the blood on the underwear


Explanation: By DNA typing, the blood turned out to be the mother's. The
mother has diabetes and employs home glucose monitoring and would
purposely put blood on her daughter's underwear. (See Chapter 35 in Nelson
Textbook of Pediatrics, 17th ed.)

Interrogate the father

Isolate the parents and child

Determine von Willebrand factor levels

Measure fibronectin in the vagina

Question . 17. The most likely diagnosis for the child described in Question 16 is:

Precocious puberty

Sexual abuse

Vaginitis

Coagulopathy

Munchausen syndrome by proxy


Explanation: Munchausen by proxy brings attention to the child and caregiver.
Often, the perpetrator has some medical background. (See Chapter 35 in
Nelson Textbook of Pediatrics, 17th ed.)

Question . 18. All of the following statements regarding adoption are true except:

Federal law requires that children in foster care who cannot be safely returned
to their families within a reasonable period of time be placed with adoptive
families

Agencies in the United States that arrange international adoptions have no


legal obligation to obtain accurate and complete health histories on children
whom families are considering adopting
th
Explanation: (See Chapter 30 in Nelson Textbook of Pediatrics, 17 ed.)

Families should be encouraged to speak freely and repeatedly about adoption


with the adopted child, beginning in toddler years and continuing through
adolescence
Most adopted children adjust well and lead healthy, productive lives

Many children in foster care waiting to be adopted have "special needs"

Question . 19. Which of the following statements regarding foster care is true?

A permanency plan must be made for a child in foster care no later than 12 mo
from the child's entry into care
Explanation: (See Chapter 31 in Nelson Textbook of Pediatrics, 17th ed.)

A minority of children in foster care have a history of abuse or neglect

An increasing proportion of children entering foster care are adolescents

Children in foster care have low utilization rates for all types of care

A minority of foster care children have behavioral and adjustment problems

Question . 20. All of the following statements regarding child care are true except:

Nearly one half of employed mothers with 3- and 4-yr-old children use center
care as their primary supplemental care

High-quality child care does not influence the cognitive and social
development of disadvantaged children
th
Explanation: (See Chapter 32 in Nelson Textbook of Pediatrics, 17 ed.)

Middle-class children are not protected from the effects of poor-quality child
care

Licensure of child-care providers signifies that minimal health, safety, and


sanitary practices are being followed

Accreditation of a child-care provider suggests that a program is of sufficient


quality to promote children's development

Question . 21. All of the following statements are true except:

Recurrent separations tend to accustom children to separations and make


them less wary and guarded about reestablishing the relationship with an
absent parent
Explanation: (See Chapter 33 in Nelson Textbook of Pediatrics, 17th ed.)

Most bereaved families remain socially connected and expect that life will
return to some new sense of normalcy

School-aged children think more concretely than younger children, recognize


the permanence of death, and begin to understand biologic processes of the
human body

No specific grief sign, symptom, or cluster of behaviors identifies the child or


family in need of help with bereavement after the death of a family member

Medication, as a first line of treatment, rarely proves useful in normal or


uncomplicated grief reactions

Question . 22. All of the following statements regarding children and violence are true except:

The source of first exposure to violence for children is often television


Explanation: (See Chapter 34 in Nelson Textbook of Pediatrics, 17th ed.)

The violence children experience and witness has a profound impact on health
and development

High levels of witnessing violence place children at risk for psychological,


social, academic, and physical problems

Occasional wife battering is estimated to occur in 16% of all families

The most ubiquitous source of exposure to violence for children in the United
States is television

Question . 23. The source of first exposure to violence for children is often:

Community violence

War violence

School violence

Domestic violence
Explanation: (See Chapter 34 in Nelson Textbook of Pediatrics, 17th ed.)

Question . 24. The violence children witness affects their development in all of the following ways
except:

It influences how they view the world and their place in it

Children become more capable in motor function from the constant threat of
violence
Explanation: (See Chapter 34 in Nelson Textbook of Pediatrics, 17th ed.)

Fear may thwart their exploration of the world, which is essential in learning in
childhood

Higher exposure to violence correlates with poorer performance in school,


symptoms of anxiety and depression, and lower self-esteem

Question . 25. A 6-mo-old child is brought to your office with the chief complaint of leg swelling and
decreased leg movement of 3 days' duration. The mother, a single parent, said that the child awoke 3
days ago with the swelling and guarding. She was unaware of any trauma to the child but said that her
2-yr-old "plays rough" with the 6-mo-old. She said that she delayed coming to see you because she
had no transportation. An x-ray film reveals a new (less than 7 days old) spiral fracture of the femur.
Which of the following should be your next action?

Refer the child to an orthopedist for casting

Perform a skeletal series, refer the child to an orthopedist for casting, examine
and do a skeletal survey of the sibling

Perform a skeletal series, refer the child to an orthopedist for casting, examine
and do a skeletal survey of the sibling, and report the spiral fracture to
appropriate authorities as suspected physical abuse after screening the
mother for risk factors and informing the mother of your plan
Explanation: (See Chapter 35 in Nelson Textbook of Pediatrics, 17th ed.)

Tell the mother you suspect physical abuse and interrogate her until you get a
history; when the mother tells you that the child fell from the crib to a carpeted
floor and sustained the injury, you conclude that the finding is in keeping with
the fracture, with no need to report suspected child abuse

Because this fracture is probably due to osteogenesis imperfecta, send the


child for genetics evaluation and connective tissue biopsy for a definitive
diagnosis

Question . 26. The parents of a 4-yr-old girl are in the process of a divorce. The child returned from a
visit with the father yesterday. The mother, who has primary custody of the child, examined the child's
genitalia because the child complained of dysuria. She found the genitalia to be red and open to the
"size of a dime." She asked the girl if the father had done anything to her and the child remained silent.
The child has been reluctant to visit the father. The mother brings the child to you because she is
concerned that the child has been abused. Which of the following constitutes the most appropriate
next step in management?

With the mother in the room for support, interview the child by asking if the
father has ever done anything to her genitalia. If the child answers in the
affirmative, make a report to appropriate authorities.

Report what the mother has told you to appropriate authorities.

Ask the father to come to your office so that you can interview him. It is likely
that the mother has prompted the child.

Speak to the child with the mother out of the room. Begin with general
questions. Determine knowledge of body parts and private parts. Ask if
anything has happened to various body parts. If the child answers in the
affirmative, ask what happened. Do a complete examination including
inspection of the anus and genitalia. If the child gives any history suggestive
of abuse or has findings suggestive of trauma to the anus or hymen that are
unexplained, make a report to the appropriate authorities.
Explanation: (See Chapter 35 in Nelson Textbook of Pediatrics, 17th ed.)

Question . 27. A 6-mo-old child has a 2-mo history of daily projectile vomiting. She has been
hospitalized on two occasions. Findings on laboratory studies including an upper GI series and on
swallowing studies have been normal. The mother had similar symptoms as a child. A 3-yr-old sister
has been treated for gastroesophageal reflux. You are the fourth physician that the mother, a nurse,
has contacted to diagnose and treat these persistent symptoms. Inspection of hospital records
indicates that no vomiting took place during either hospital admission. The most appropriate next step
in management is:

Conduct a complete and detailed work-up for cyclic vomiting with referrals for
consultation to gastroenterology, neurology, and nephrology. Persist in your
evaluation until a diagnosis is made.

Have the mother bring you a sample of the vomitus. Test it for the presence of
ipecac. Perform toxicology studies. If results are positive, report as suspected
Munchausen syndrome by proxy to proper authorities.
Explanation: (See Chapter 35 in Nelson Textbook of Pediatrics, 17th ed.)

Place the child on treatment for gastroesophageal reflux in light of the high
rate of false-negative results on laboratory testing for this condition. If the
condition persists despite treatment, hospitalize the child and obtain surgical
consultation.

Confront the mother with lack of medical findings for the condition. Indicate
that you suspect she is lying about the symptoms. Request a psychiatric
consultation for the mother. Report your suspicions to proper authorities with
a request that the child remain in the home under weekly supervision by a
public health nurse.

Consult an allergist and request a work-up for food allergy. Put the child on a
restrictive diet and observe for symptoms.
Children with Special Health Needs, Set 1

Question . 1. A mentally retarded child with microphthalmia, microcephaly, chorioretinitis, and


a history of a neonatal petechial rash is most likely to have:

A chromosomal syndrome

TORCH infection
Explanation: This pattern of abnormalities is most compatible with a
congenital TORCH (toxoplasmosis, other, rubella, cytomegalovirus,
herpes simplex) infection. In addition, intrauterine and postnatal growth
retardation may be evident. (See Chapter 37.2 in Nelson Textbook of
Pediatrics, 17th ed.)

Fetal alcohol syndrome

Galactosemia

Hyperammonemia

Question . 2. Frequent problems of children with common chronic illnesses include all of the
following except:

Unpredictability

Pain

Expense

Multiple providers

Failure to graduate high school


Explanation: Many children with common chronic diseases of
childhood (e.g., asthma, seizures, diabetes, arthritis, cystic fibrosis,
sickle cell anemia) attend high school and graduate. (See Chapter 37 in
Nelson Textbook of Pediatrics, 17th ed.)

Isolation

Psychologic or behavioral problems


Question . 3. An infant with multiple grotesque congenital anomalies dies on the third day of
life. Her mother has not had an opportunity to see her before death owing to postpartum
complications. When informed of the baby's death, she says she wants to see her. She cannot
be moved from where she is receiving intensive care. Which of the following is the most
appropriate response to her request?

Tell her that she is too sick to see the baby

Tell her that she will be able to see the baby later

Take the baby to her bedside


Explanation: Parents who are not able to see their deformed neonate
may greatly exaggerate the perceived severity of any anomaly and may
have excessive feelings of guilt. Most parents benefit from seeing their
child with anomalies and often identify aspects of beauty or normalcy
with the help of a nurse or physician. (See Chapter 37 in Nelson
Textbook of Pediatrics, 17th ed.)

Tell her she would not want to see the baby

Tell her that it is too late for her to see the baby

Question . 4. The parents of a 10-yr-old girl with mental retardation are seeking information on
what to expect for her future. The youngster is in a mainstreamed educational program, is just
beginning to master simple reading skills, and has one close friend. The difficulty of long-term
prognostication aside, possible life goals for this child include:

Holding a regular job

Getting married

Having children

All of the above


Explanation: This child demonstrates important educational and social
milestones that are partially predictive of future achievements with
respect to these life goals. (See Chapter 37.2 in Nelson Textbook of
Pediatrics, 17th ed.)

Question . 5. A preschooler with Down syndrome is seen for a routine health supervision visit.
A knowledgeable clinician will pay particular attention to screening for problems that are
known to occur with increased frequency in children with this condition. Which of the
following conditions is least likely to be found in this child?

Atlantoaxial instability

Neurogenic bladder
Explanation: Neurogenic bladder is not typically encountered in Down
syndrome, but Hirschsprung disease may occur. (See Chapter 37.2 in
Nelson Textbook of Pediatrics, 17th ed.)

Hypothyroidism

Conductive hearing loss


Question . 6. A 3-yr-old boy with a limited vocabulary is referred for formal psychometric
testing and is found to have an IQ of 60. Findings on physical examination are essentially
unremarkable except for mild hypotonia. Appropriate initial laboratory studies include all of
the following except:

Karyotype, including test for fragile X

Audiologic evaluation

Cranial CT scans
Explanation: Cranial computed tomography is not indicated until other
evaluations are completed unless a patient has macrocephaly,
microcephaly, abnormal neurologic findings, or significant
dysmorphology. (See Chapter 37.2 in Nelson Textbook of Pediatrics,
17th ed.)

Formal speech and language evaluation

Question . 7. The general approach to management of the child with psychosocial failure to
thrive includes all of the following except:

Keeping meal time brief

Offering solid foods before liquids

Forcing the child to eat


Explanation: Forced feeding exacerbates abnormal psychosocial
tension between a child and his or her caregiver. (See Chapter 36 in
Nelson Textbook of Pediatrics, 17th ed.)

Minimizing environmental distractions

Minimizing the intake of water and juice

Question . 8. From the following list, choose the most appropriate way(s) to assess growth in
premature infants to diagnose failure to thrive.
1. Use corrected age (subtract weeks premature) until age 1-2 yr
2. Determine whether two major growth percentiles are crossed
3. Add additional weight as if the child had been born at term
4. Determine the weight to length ratio
5. Do not use head circumference until age 24 mo

1 only

1 and 2
Explanation: Corrected age rather than chronologic age is one solution.
The "real" gestational age of a 24-wk premature infant who is now 20 wk
old is 44 wk (1 month corrected age may also be valuable). Determining
whether growth percentiles are crossed is the other solution. (See
Chapter 36 in Nelson Textbook of Pediatrics, 17th ed.)
3 and 4

3 and 5

1 and 5

Question . 14. Major causes of failure to thrive include all of the following except:

Formula feeding
Explanation: In infants who are fed formula in sufficient amounts,
malabsorption is not a major cause of failure to thrive. (See Chapter 36
in Nelson Textbook of Pediatrics, 17th ed.)

Failure to provide sufficient calories

Failure to ingest sufficient calories

Failure to retain sufficient calories

Malabsorption

Question . 15. The leading cause of failure to thrive in infants between 0 and 3 mo of age is:

TORCH infection

Psychosocial pathology
Explanation: All must be considered, but B is No. 1. (See Chapter 36 in
Nelson Textbook of Pediatrics, 17th ed.)

Gastrointestinal reflux

Cystic fibrosis

Inborn errors of metabolism

Question . 16. Snoring and mouth breathing as a cause of failure to thrive suggest:

Streptococcal pharyngitis

Mononucleosis

Obstructive sleep apnea


Explanation: Adenoid hypertrophy or poor oropharyngeal motility
(possibly related to cerebral palsy) can cause failure to thrive from
obstructive sleep apnea. Mononucleosis is rare in infants. (See Chapter
36 in Nelson Textbook of Pediatrics, 17th ed.)

Anterior meningocele

Cerebral palsy
Question . 17. An 8-mo-old presents with failure to thrive. The past medical history includes
severe thrush and Candida diaper rash and recurrent otitis media with perforation. On physical
examination the patient has generalized lymphadenopathy and hepatosplenomegaly; there is
also bilateral parotitis. The most likely diagnosis is:

Mononucleosis

Familial histiocytosis

X-linked combined immunodeficiency

AIDS
Explanation: Babies with AIDS usually present with failure to thrive and
recurrent infections. Lymphadenopathy, parotitis, and
hepatosplenomegaly are classic features of AIDS in infants. (See
Chapter 36 in Nelson Textbook of Pediatrics, 17th ed.)

Psychosocial failure to thrive

Question . 18. All of the following are true regarding children with chronic illness except:

Children with disabilities rarely survive to adulthood


Explanation: Indeed, most survive to be adults. (See Chapter 37 in
Nelson Textbook of Pediatrics, 17th ed.)

6-7% of children has some limitation of activity

1-2% of children meet the definition of severe disability

Of disabled children, 40% have learning and developmental disorders

Of disabled children, 35% have chronic physical conditions

Question . 19. Principles of care for children with chronic diseases include all of the following
except:

Early detection

Amelioration of functional consequences

Designation of children with asthma as "asthmatics"


Explanation: Labeling depersonalizes the child and marks the child for
life. A disease should not define a child. (See Chapter 37 in Nelson
Textbook of Pediatrics, 17th ed.)

Prevention of secondary psychosocial handicaps

Treatment in the context of the family


Question . 20. The Individuals with Disabilities Education Act includes all of the following
except:

Support of state programs providing early intervention

Providing cash assistance for children with disabilities


Explanation: Supplemental security income programs provide cash
support for people of all ages with disabilities. (See Chapter 37 in
Nelson Textbook of Pediatrics, 17th ed.)

Education in the least restrictive manner

Support of the state programs providing special education

Question . 21. A sixth-grade child with chronic arthritis typically views the cause of chronic
illness as due to:

Germ theory
Explanation: At this age, germ theory is an easily comprehended but
unfortunately not always accurate view of chronic illness. Nonetheless,
it does help with compliance in taking medicine. (See Chapter 37 in
Nelson Textbook of Pediatrics, 17th ed.)

Punishment for bad behavior

Physiologic mechanisms

Not following rules

Failure to take medicines

Question . 22. Mental retardation is best classified by:

A system designating the degree as mild, moderate, severe, profound

The designators imbecile, retarded, functional

IQ percentiles

Support needs (intermittent, limited, extensive, pervasive)


Explanation: This functional approach places the child's needs for
support into the context of the environment. (See Chapter 37 in Nelson
th
Textbook of Pediatrics, 17 ed.)

None of the above


Question . 23. Common identifiable causes of mental retardation include all of the following
except:

Trisomy 21

Hypothyroidism

Fetal alcohol syndrome

Fragile X syndrome

Cystic fibrosis
Explanation: Cystic fibrosis itself does not cause mental retardation. A-
D are common; hypothyroid-induced retardation is preventable by early
screening and rapid therapy. (See Chapter 37 in Nelson Textbook of
Pediatrics, 17th ed.)

Question . 24. All of the following statements about mental retardation are true except:

About 3% of the population have an IQ less than 2 standard deviations


below the mean

Severe retardation is inversely related to socioeconomic status


Explanation: Mild retardation is related to low socioeconomic status,
whereas more severe retardation is equally distributed in all such
groups. (See Chapter 37 in Nelson Textbook of Pediatrics, 17th ed.)

Up to 5% of children with mental retardation are profoundly affected

The reported incidence increases at entry to school

Language development is a first clue to mild retardation

Question . 25. Adaptive skill areas to assess in children with mental retardation include all of
the following except:

Communication

Sports
Explanation: Sports per se are not an adaptive skill but could be
categorized into leisure activities. (See Chapter 37 in Nelson Textbook
of Pediatrics, 17th ed.)

Self-care

Home living

Social skills

Community use
Question . 26. In male patients with mental retardation without an obvious etiology, the next
step in evaluation should be:

Plasma ammonia assay

Blood lead level determination

EEG

Chromosome analysis
Explanation: Fragile X syndrome manifests in males as mental
retardation, large ears, and large testes. It is relatively common. (See
Chapter 37 in Nelson Textbook of Pediatrics, 17th ed.)

Cranial CT

Question . 27. A 3-yr-old girl presents with microcephaly and mental retardation. Her mother
had a flu-like illness during the second month of pregnancy. At birth, the baby had petechiae
and hepatosplenomegaly, which have resolved. The most likely diagnosis is:

Congenital HIV infection

Congenital rubella
Explanation: Congenital rubella was once a common cause of mental
retardation. Thanks to active immunization programs, congenital
rubella is rare. (See Chapter 37 in Nelson Textbook of Pediatrics,17th
ed.)

Congenital parvovirus infection

Isoimmune neonatal thrombocytopenia

Subacute sclerosing panencephalitis

Question . 28. A 4-yr-old's concept of death may include all of the following except:

It may be reversible

It is like sleep

Dead people still eat and breathe

They understand causality


Explanation: Causality is a late developmental state of understanding. It
is important not to reinforce the immature child's belief that death is like
sleep because the child will be fearful of sleeping lest he or she may
die. (See Chapter 38 in Nelson Textbook of Pediatrics, 17th ed.)

It is a functional state
Question . 29. The fear of dying in young children is best described as fear of:

The afterlife

The unknown

Separation
Explanation: Separation from loved ones is the dominant and often only
concern of young children. (See Chapter 38 in Nelson Textbook of
Pediatrics, 17th ed.)

Transcendentalism

Existentialism

Question . 30. Perpetuating the myth of "everything is going to be all right" with a dying child
will:

Help reassure the child

Hide "bad things" from the child

Enhance an awareness of eventual death

Alleviate fears

Prevent exploration of fears


Explanation: Most chronically ill children have a sense of impending
death and need to articulate this with parents or caregivers who can
discuss fears and provide some reassurance (at least for comfort). (See
Chapter 38 in Nelson Textbook of Pediatrics, 17th ed.)

Question . 31. Giving an estimate of how long a child with a life-threatening condition will
survive will:

Be inaccurate
Explanation: Population-based statistics are poor for predicting the
time of death of an individual child. (See Chapter 38 in Nelson Textbook
of Pediatrics, 17th ed.)

Help accept the diagnosis

Avoid unnecessary expenses

Provide time to remove the child from school

Enhance communication between siblings and the child


Question . 32. All of the following statements are true except:

In the United States the most common presentation of failure to thrive


is poor growth detected in the ambulatory setting

The laboratory evaluation of children with failure to thrive is usually


helpful
Explanation: (See Chapter 36 in Nelson Textbook of Pediatrics, 17th ed.)

The history, physical examination, and observation of the parent-child


interaction usually suggest the diagnosis of failure to thrive

Failure to thrive in the first year of life regardless of cause is


particularly ominous

Approximately one third of children with psychosocial failure to thrive


are developmentally delayed and have social and emotional problems

Question . 33. All of the following statements about mental retardation (MR) are true except:

Mild MR is defined as an IQ of 80 or below


Explanation: (See Chapter 37 in Nelson Textbook of Pediatrics, 17th ed.)

In children with severe MR, a biologic cause can be identified in over ¾


of cases

Statistically, 2.5% of the population can be expected to have MR

About 0.5% of the population have severe MR

The brains of 10-20% of individuals with severe MR are normal by


standard neuropathologic studies

Question . 34. All of the following statements are true except:

The BSID II is used to test children 1 mo-3½ yr of age

The BSID II is used to identify infants with severe mental retardation

The WPPSI-R is used to test children with mental ages of 3-7 yr

The Stanford-Binet Intelligence Scale measures verbal abilities, abstract


thinking, quantitative reasoning, and adaptive functioning
Explanation: (See Chapter 37 in Nelson Textbook of Pediatrics, 17th ed.)

The WISC III is used for children who function above a mental age of 6
yr
Question . 35. All of the following statements are true except:

Surveys indicate that 15-18% of children and adolescents have some


form of chronic condition

About 6-7% of all children and adolescents have some limitation of


activity due to a chronic condition

At least 90% of children with severe long-term illnesses survive to


young adulthood

About 2-4% of children with severely long-term illnesses account for at


least 35% of child health care expenditures

Children face a relatively small number of common chronic conditions


and a few rare diseases
Explanation: (See Chapter 37 in Nelson Textbook of Pediatrics, 17th ed.)

Question . 36. John is 12 yr old and has Down syndrome. He has recently transitioned from an
elementary school that used an inclusion model to a middle school program that continues to
focus on reading and math. John has been spending more time in his room and less time with
the family. He is less lively and more irritable. The most likely diagnosis is:

Dementia

Hypothyroidism

Adjustment disorder
Explanation: (See Chapter 37 in Nelson Textbook of Pediatrics, 17th ed.)

Depression

Atlantoaxial subluxation
Question . 37. Of the following causes of severe mental retardation, the most common is:

Inborn errors of metabolism

Developmental brain anomalies

Perinatal causes

Chromosomal abnormalities
Explanation: (See Chapter 37 in Nelson Textbook of Pediatrics, 17th ed.)

Postnatal causes
Question . 38. The most common reason for people with mental retardation to be placed out of
the home is:

Single-parent family

Greater degree of mental retardation

Greater intensity of medical needs

Lack of an appropriate school, necessitating residential placement

Behavioral disturbance
Explanation: (See Chapter 37 in Nelson Textbook of Pediatrics, 17th ed.)

Question . 39. In addition to the deficits in cognition and onset before age 18 yr, the diagnosis
of mental retardation requires deficits or impairments in:

Adaptive behavior
Explanation: (See Chapter 37 in Nelson Textbook of Pediatrics, 17th ed.)

Stereotypic behavior

Self-stimulatory behavior

Play

Moor behavior

Question . 40. Jane is an 18-mo-old girl who presents because of delayed walking. Findings on
the motor examination are normal, save for mild, diffuse hypotonia. Further evaluation should
include:

Chromosome analysis

Assessment of language skills


Explanation: (See Chapter 37 in Nelson Textbook of Pediatrics, 17th ed.)

EMG and nerve conduction studies

MRI

Audiologic evaluation
Question . 41. All of the following statements are true except:

Provision of respite services is especially important to families caring


for children with complex chronic conditions over prolonged periods of
time

Good end-of-life care cannot be effectively carried out in a hospital


setting
Explanation: (See Chapter 38 in Nelson Textbook of Pediatrics, 17th ed.)

Children's fear of death is centered on the concrete fear of being


separated from parents and other loved ones

In some families and cultures, truth telling and autonomy are much less
valued than family integrity

Many dying children do not have their pain successfully treated

Question . 42. All of the following statements are true except:

Physicians should ask parents of the patient about their economic


circumstances

The medical problems of children of migrant farm workers are similar to


those of children of homeless families

By age 20-30 yr, few children who were at special risk have made
moderate success of their lives
Explanation: (See Chapter 39 in Nelson Textbook of Pediatrics, 17th ed.)

Most states authorize treatment of homeless youth

Recurrent otitis media is an especially frequent problem among Native


American children
Nutrition

Question . 1. The breast-fed infant of a mother who is a strict vegan may experience deficiency
of which of the following vitamins if the mother is not receiving supplements of the vitamin?

B6

B12
Explanation: A strict vegan diet contains no eggs, meat, or milk products
and is thus deficient in vitamin B12. (See Chapters 41, 42, and 44 in
Nelson Textbook of Pediatrics, 17th ed.)

Folate

Biotin

Question . 2. The best source of iron for 1-mo-old infants is:

Iron-fortified cereals

Yellow vegetables

Fruits

Breast milk
Explanation: Although breast milk contains relatively less iron by weight,
the iron is more bioavailable than the iron in cereals. Fruits, yellow
vegetables, and cow's milk are poor sources of iron. (See Chapters 40
and 41 in Nelson Textbook of Pediatrics, 17th ed.)

2% low-fat cow's milk

Question . 3. A 4-mo-old child with vitamin D deficiency rickets would be expected to show all of
the following except:

Craniotabes

Bowleg
Explanation: In rickets, parathyroid hormone level is elevated, which
results in low serum phosphate levels. Low serum phosphate levels
result in abnormal osteoblastic activity, which may result in craniotabes
and a rachitic rosary with enlargement at the costochondral junctions.
Even though osteoid of the legs is uncalcified, bowing does not occur
until weight is borne on the legs. (See Chapter 44 in Nelson Textbook of
Pediatrics, 17th ed.)

Rosary
Low serum phosphate levels

High alkaline phosphatase levels

Question . 4. The estimated average requirement (EAR) of a nutrient is best defined as:

A dietary reference index

The recommended dietary allowance

Age- and gender-specific to meet the needs of 50% of persons


Explanation: The EAR is one form of nutrient evaluation. The RDA
(answer B) is the daily dietary intake sufficient to meet 97-98% of
individual nutrient needs in age and gender groups. (See Chapter 40 in
Nelson Textbook of Pediatrics, 17th ed.)

The range encompassing the lower and upper limits of a nutrient

A daily average calculated once each year

Question . 5. The AI (adequate intake) in infants is estimated from:

Bomb calorimetry

The EAR

The RDA

The intake of nutrients from human milk


Explanation: The AI is one component of the DRI (dietary reference
intakes). The DRI also considers the EAR, RDA, and tolerable upper level
(UL). (See Chapter 40 in Nelson Textbook of Pediatrics, 17th ed.)

Intakes that will prevent deferrable nutrient deficiencies

Question . 6. All of the following are advantages of breast-feeding except:

Reduced incidence of allergy

Reduced incidence of otitis media

Reduced incidence of colic

Increased psychologic comfort

Vitamin K content
Explanation: Vitamin K must be given (intramuscularly at birth) to all
infants. Breast-fed infants whose diet is not supplemented with vitamin K
are at risk for bleeding. (See Chapter 41 in Nelson Textbook of Pediatrics,
17th ed.)

Utility for preterm infants weighing less than 2000 g

Question . 7. Problems associated with breast-feeding include all of the following except:

Less than optimal nutrients for infants <1000 g

Vitamin K content

Transmission of live viruses

Hyperbilirubinemia

Contraindication in erythroblastosis fetalis


Explanation: Antibodies in mother's milk are inactivated in the infant's
intestines and do not contribute to intravascular hemolysis. (See Chapter
41 in Nelson Textbook of Pediatrics, 17th ed

Question . 8. Atypical features of infant colic include all of the following except:

Fever

Onset in the first week of life

Onset at age 6 mo

Sudden onset

Crying mainly in the early morning


Explanation: Colic usually recurs in the early evening. (See Chapter 41 in
Nelson Textbook of Pediatrics, 17th ed.)
Question . 9. All of the following observations regarding infant feeding are true except:

Self-feeding with a spoon at age 12 mo should be encouraged

Infants will select a balanced diet

Consistent rejection of one food group should suggest food allergy

Infants should be put to bed with a bottle of milk


Explanation: Baby bottle caries syndrome is a significant problem
because the milk bathes the teeth, setting up multiple caries. (See
Chapter 41 in Nelson Textbook of Pediatrics, 17th ed.)

The likes and dislikes of infant tastes should be respected


Question . 10. All of the following are clinical manifestations of kwashiorkor except:

The presence of edema

Rash in sun-exposed areas


Explanation: The rash of kwashiorkor is in areas of irritation. Sun-
exposed dermatitis is typical of pellagra. (See Chapter 42 in Nelson
Textbook of Pediatrics, 17th ed.)

Hypochromotrichia

Muscle weakness

An increased susceptibility to infection

Question . 11. All of the following are laboratory manifestations of kwashiorkor except:

Persistent ketonuria
Explanation: Ketonuria is present early but does not persist into the later
stages. (See Chapter 42 in Nelson Textbook of Pediatrics, 17th ed.)

Hypoalbuminemia

Hypoglycemia

Potassium deficiency

Low serum amylase levels

Question . 12. All of the following statements about obesity in children are true except:

Obese children eat more junk food


Explanation: Obese children do not necessarily eat more food or more
junk food than leaner children. (See Chapter 43 in Nelson Textbook of
Pediatrics, 17th ed.)

Single-gene disorders are rare causes of obesity

Obesity may be associated with insulin resistance

The highest prevalence of obesity in the United States is in the Northeast

Menarche may be earlier in obese girls

Question . 13. Complications of obesity in childhood include all of the following except:

Angina
Explanation: Although hyperlipidemia secondary to obesity may be
present, coronary artery disease does not manifest in children. (See
Chapter 43 in Nelson Textbook of Pediatrics, 17th ed.)

Blount disease

Slipped capital femoral epiphysis

Sleep apnea

Glucose intolerance

Question . 14. Physical features of vitamin D deficiency rickets include all of the following
except:

Bitot spots
Explanation: Bitot spots are seen in vitamin A deficiency (dry plaques on
the bulbar conjunctiva). (See Chapter 44 in Nelson Textbook of
Pediatrics, 17th ed.)

Craniotabes

Enlargement of the costochondral junctions

Thickening of the ankles and wrists

Large anterior fontanel

Bowleg

Question . 15. Clinical features of vitamin E deficiency include all of the following except:

Cerebellar ataxia

Muscle weakness

Peripheral neuropathy

Hemolysis

Hepatosplenomegaly
Explanation: A-C often occur in patients with malabsorption. D has been
reported in premature infants. (See Chapter 44 in Nelson Textbook of
Pediatrics, 17th ed.)
Question . 16. The pediatric patient with obesity typically presents with:

Tall stature
Explanation: (See Chapter 43 in Nelson Textbook of Pediatrics, 17thed.)

Delayed bone age

Negative family history of obesity

Delayed puberty

Elevated serum cholesterol

Question . 17. The group that has exhibited the most striking increase in the prevalence of
obesity in the United States is:

Caucasian males

Caucasian females

African-American and Hispanic males

African-American and Hispanic females


Explanation: (See Chapter 43 in Nelson Textbook of Pediatrics, 17thed.)

Asian-American males and females

Question . 18. The risk of developing obesity in adulthood correlates negatively with:

High birthweight

Number of obese parents

Months of breast-feeding
Explanation: (See Chapter 43 in Nelson Textbook of Pediatrics, 17thed.)

Hours of television viewing

Weight during adolescence


Pathophysiology of Body Fluids and Fluid Therapy

Question . 1. Diabetes insipidus may be due to all of the following except:

Pituitary adenoma

Renal epithelial ADH reception defect

Hypokalemia

Hypercalcemia

Adrenal deficiency
Explanation: Adrenal deficiency may cause renal salt wasting and
usually does not affect free water excretion. (See Chapter 45.3 in Nelson
Textbook of Pediatrics, 17th ed.)

Question . 2. A 1-mo-old boy presents with severe failure to thrive, emesis, and a temperature of
41°C. Serum electrolyte measurements reveal a sodium level of 185 mEq/L, and the urine
specific gravity is 1001. The most likely diagnosis is:

Adrenal insufficiency

Salt poisoning

Hypernatremic dehydration

Malignant hyperthermia

Nephrogenic diabetes insipidus


Explanation: Nephrogenic diabetes insipidus is a sex-linked recessive
disorder due to deficient binding of ADH to the renal tubular cell.
Exogenous administration of ADH is therefore ineffective. (See Chapter
45.3 in Nelson Textbook of Pediatrics, 17th ed.)
Question . 3. A well-grown 6-mo-old presents with a tonic-clonic seizure lasting 30 min. The
child is found to be hypothermic and remains lethargic. The diet history reveals that the mother
is a participant in the WIC program. Because it is the end of the month, she has begun to dilute
the remaining formula with water as there is not enough to last until she receives her next
allotment of formula next week. The most likely diagnosis is:

Hypocalcemia

Hyponatremia
Explanation: Hyponatremia due to feeding diluted formula or excessive
amounts of sodium-free fluids (especially water) is relatively and
unfortunately common among poor families who run out of formula. (See
Chapter 46 in Nelson Textbook of Pediatrics, 17th ed.)

Hypoglycemia

Hypernatremia

Hypokalemia

Question . 4. Hyperkalemia may be associated with all of the following except:

Succinylcholine use

Burns

Trauma

Chemotherapy

Metabolic alkalosis
Explanation: Metabolic alkalosis produces hypokalemia. (See Chapters
45.4 and 45.8 in Nelson Textbook of Pediatrics,17th ed.)

Digitalis toxicity

Uremia

Question . 5. A normal anion gap acidosis is most likely to be due to:

Diabetes mellitus

Renal tubular acidosis


Explanation: Renal tubular acidosis with renal bicarbonate loss and
diarrhea-induced stool losses of bicarbonate are the common causes of
a normal anion gap acidosis. (See Chapter 45.8 in Nelson Textbook of
Pediatrics,17th ed.)

Nephrotic syndrome

Uremia

Shock
Question . 6. A 10-mo-old infant presents with vomiting and diarrhea, tachycardia, normal blood
pressure, dry mucous membranes, a capillary refill time of 2 sec, deep respirations, and
irritability. The percent dehydration for this patient is:

0-3%

3-5%

6-9%
Explanation: Dehydration of 6-9% represents moderate dehydration and
early shock. Tachycardia reflects the intravascular volume loss, and
deep respirations represent the pulmonary response to metabolic
acidosis. (See Chapters 47 and 48 in Nelson Textbook of Pediatrics, 17th
ed.)

10-12%

12-15%
Question . 7. A serious complication of the treatment of hypernatremic dehydration is:

Cerebral thrombosis

Cerebral edema
Explanation: Cerebral edema occurs if free water is given in excessive
amounts, if the serum sodium falls more than 10 mEq/L/day, and if
idiogenic osmoles remain in neurons during rehydration. Cerebral
thrombosis may occur before therapy is started and may be associated
with inherited hypercoagulable conditions. (See Chapters 45.3 and 47 in
Nelson Textbook of Pediatrics, 17th ed.)

Hyperchloremia

Hypoglycemia

None of the above

Question . 8. The best method to reduce the potassium level during hyperkalemia, by reducing
the body burden of potassium, is:

Sodium bicarbonate infusion

Glucose and insulin infusion

Calcium infusion

Albuterol aerosol

Kayexalate enema
Explanation: Kayexalate, a potassium-binding resin, and dialysis are the
only methods to remove potassium from the body. Other methods shift
potassium from the extracellular to the intracellular space. (See Chapter
45.4 in Nelson Textbook of Pediatrics, 17th ed.)
Question . 9. The finding of marked metabolic alkalosis with acidic urine indicates:

Marked sodium depletion

Marked potassium depletion


Explanation: Marked potassium depletion is more likely, although
laboratory error is possible. When in doubt, repeat laboratory tests, but
such a repeat in this case would confirm the finding. (See Chapter 45.4 in
Nelson Textbook of Pediatrics, 17th ed.)

Hyperventilation

Diabetes mellitus

Laboratory error

Question . 10. From the following list, choose the route(s) by which insensible water loss may
occur.
1. Sweat
2. Fecal loss
3. Evaporative loss from skin
4. Respiratory water loss
5. Obligate water for urinary solute excretion

1 and 3

1, 2, and 3

3 only

3 and 4
Explanation: Insensible water losses usually occur independently of total
body water homeostasis. (See Chapter 45.2 in Nelson Textbook of
Pediatrics, 17th ed.)

2 and 5

Question . 11. Which of the following drugs or agents may inhibit antidiuretic hormone release?

Demerol

Barbiturates

Alcohol
Explanation: All the others may stimulate ADH release. (See Chapter 45.3
in Nelson Textbook of Pediatrics,17th ed.)

Nicotine

-Adrenergic drugs
Question . 12. Hypernatremia may be induced by all of the following except:

Hyperglycemia
Explanation: Indeed, hyperglycemia may produce pseudohyponatremia.
(See Chapter 45.3 in Nelson Textbook of Pediatrics, 17th ed.)

Adipsia

Insufficient breast-feeding

Gastroenteritis

Nephrogenic diabetes insipidus

Question . 13. The most common cause of nutritional hyponatremia is:

Salt substitutes

Low-salt diets

The WIC syndrome


Explanation: The WIC syndrome in families receiving formula from the
Women's, Infants', and Children's nutritional supplementation program
occurs when families run out of formula toward the end of the month and
begin to dilute the formula with water. Another common cause is feeding
excess amounts of pure water in general. (See Chapter 45.3 in Nelson
Textbook of Pediatrics, 17th ed.)

Furosemide (Lasix) therapy

Vegan diets

Question . 14. Manifestations of hyperkalemia include all of the following except:

Paresthesias

Weakness

Paralysis

Wide QRS complex

Tetany
Explanation: A-D are noted in hyperkalemia. The first ECG change is
peak T waves. Lengthening of the P-R interval and QRS complex occurs
later. (See Chapter 45.4 in Nelson Textbook of Pediatrics, 17th ed.)
Question . 15. Potential causes of hyperkalemia include all of the following except:

Succinylcholine

Digitalis toxicity

Acute renal failure

Albuterol overdose
Explanation: -Agonists often cause hypokalemia. (See Chapter 45.4 in
Nelson Textbook of Pediatrics, 17th ed.)

Captopril overdose

Question . 16. An increased anion gap occurs in all of the following except:

Diabetic ketoacidosis

Renal tubular acidosis


Explanation: RTA produces a non-anionic gap acidosis due to loss of
bicarbonate by the kidney and not due to net gain of new acid in the
circulation. (See Chapter 45.8 in Nelson Textbook of Pediatrics, 17th ed.)

Salicylate poisoning

Methylmalonicacidemia

Ethylene glycol poisoning

Question . 17. A preterm infant born to a mother with severe preeclampsia is noted to be
hypotonic and apneic in the delivery room. After resuscitation and stabilization, she remains
hypotonic with decreased deep tendon reflexes in the arms and knees. The mother's treatment
included hydralazine, magnesium sulfate, and indomethacin. The laboratory evaluation of this
patient should include:
1. Serum calcium determination
2. Arterial blood gas analysis
3. Serum magnesium determination
4. CBC
5. Anion gap measurement

1 and 3

2 only

1, 2, and 3
Explanation: 1, 2, and 3 would be helpful in the management of these
immediate problems. A CBC would be of some use if anemia or infection
were suspected. (See Chapters 45.5 and 45.6 in Nelson Textbook of
Pediatrics, 17th ed.)

2 and 4

2, 3, and 5
Question . 18. The serum magnesium level for the patient in Question 17 is 6.5 mg/dL. From the
following list, select the most appropriate next step(s) in treatment.
1. Continue mechanical ventilation
2. Infuse normal saline
3. Add calcium to the intravenous solution
4. Begin chelation therapy
5. Administer KCl

1, 2, and 3
Explanation: 1, 2, and 3 are correct, although with supportive care and
time, most patients do quite well. (See Chapters 45.5 and 45.6 in Nelson
Textbook of Pediatrics, 17th ed.)

3 only

4 only

1 and 5

2 and 4

Question . 19. Possible consequences of hypophosphatemia include all of the following except:

Hypocalcemia
Explanation: Indeed, hypercalcemia is more common as a compensatory
mechanism to release phosphate from bone. (See Chapter 45.7 in Nelson
Textbook of Pediatrics, 17th ed.)

Hemolysis

Rhabdomyolysis

Paresthesias

Confusion
The Skin
Nelson Self Assessments website 17th Edition

Question . 1. A 10-day-old infant develops an annular, scaly, erythematous eruption


on the forehead. He was born at term after an uncomplicated delivery, with a past
medical history significant only for hyperbilirubinemia treated with phototherapy.
Results of a KOH examination are negative. His mother has intermittent episodes of
joint pain. The most appropriate next step in diagnosis is:

A. Liver function tests


B. Fungal culture
C. Skin biopsy
D. Autoimmune work-up
Explanation: This vasculitic-type rash is compatible with neonatal lupus.
Maternal antibodies cross the placenta (IgG) and produce autoimmune
phenomena in the baby. These include hemolytic anemia, thrombocytopenia,
neutropenia, cutaneous vasculitis, and congenital heart block. All
manifestations except heart block resolve with time. E would also be correct.
Neonatal lupus should be considered diagnostically based on the age at
presentation, morphology of rash, and history of UV light exposure. NLE
babies can have ECG abnormalities, but only 1 in 10 has both ECG changes
and a rash. Therefore, because a rash is present, the next best step
diagnostically would be serum assay for autoantibodies, which should be
diagnostic. (See Chapter 637 in Nelson Textbook of Pediatrics, 17th ed.)
E. Electrocardiogram

Question . 2. A 2-mo-old girl has a rapidly growing hemangioma involving the


perioral region bilaterally, with symmetric extension onto the chin and anterior neck.
She was born 8 wk premature with an otherwise benign medical history, although her
parents wonder if she may be developing asthma. The most appropriate next step in
patient management is:

A. Laser surgery
B. Skin biopsy
C. Reassurance that hemangiomas will go away on their own
D. Inhaler prescription
E. ENT evaluation
Explanation: A beard area hemangioma may herald a subglottic lesion that
could be life-threatening and may be the source of the perceived asthma
symptoms. (See Chapter 640 in Nelson Textbook of Pediatrics, 17th ed.)

The Skin - Nelson Self Assessments website 17th Edition 1


Question . 3. A previously healthy 1-yr-old boy presents to the emergency
department with seizures. He has had no recent illnesses or high fevers, and has had
no ingestion history and is taking no new medications. On examination he is alert,
interactive, and afebrile with normal vital signs. His skin examination is significant
for confetti-type white spots on his right lower extremity his parents state is a
"birthmark." The most likely diagnosis is:

A. Neurofibromatosis type 1
B. Tuberous sclerosis
Explanation: These hypopigmented macules resemble ash leaves and are best
identified with a Wood lamp. (See Chapter 643 in Nelson Pediatrics, 17th ed.)
C. Waardenburg syndrome
D. McCune-Albright syndrome
E. Vitiligo

Question . 4. A 6-mo-old infant is presented for evaluation of a tuft of thick black


hair located on the sacrum. The child's mother states that the lesion has been present
since birth and seems to be asymptomatic, and the child is otherwise healthy. The
most appropriate next step in management is:

A. Reassurance that the lesion will resolve spontaneously in time


B. MRI of the spinal cord
Explanation: Lumbar or sacral hair tufts or hemangiomas may be associated
with spinal dysraphisms, such as a tethered cord. In younger infants, an
ultrasound study is a useful examination, but as infants grow older, MRI is
indicated. (See Chapter 637 in Nelson Textbook of Pediatrics, 17th ed.)
C. Referral to a plastic surgeon for excision
D. Counseling of the parents regarding melanoma risk
E. Watchful waiting

Question . 5. Hypohidrotic ectodermal dysplasia is associated with all of the


following Except:

A. Partial or complete absence of sweat glands


B. Hypertrichosis
Explanation: Patients usually have hypotrichosis. Because of the inability to
sweat, these children often have many episodes of high fever; therefore, this
entity is included in the differential diagnosis of fever of unknown origin. (See
Chapter 639 in Nelson Textbook of Pediatrics, 17th ed.)
C. Anomalous detention
D. X-linked recessive inheritance
E. Mutations of ectodysplasin

The Skin - Nelson Self Assessments website 17th Edition 2


Question . 6. PHACES syndrome is associated with all of the following Except:

A. Phalangeal dysplasia
Explanation: PHACES is associated with posterior fossa defects such as
Dandy-Walker cysts or cerebellar hypoplasia. Strokes are a major source of
morbidity. (See Chapter 640 in Nelson Textbook of Pediatrics, 17th ed.)
B. Facial hemangiomas
C. Aneurysms and stroke
D. Coarctation of the aorta
E. Ocular anomalies
F. Sternal anomalies

Question . 7. Kasabach-Merritt syndrome is seen most often with:

A. Hemangiomas
B. Nevus flammeus
C. Tufted angioma
Explanation: This syndrome, associated with thrombocytopenia,
microangiopathic hemolytic anemia, and consumption coagulopathy, is due to
a tufted angioma or a kaposiform hemangioendothelioma. (See Chapter 640 in
Nelson Textbook of Pediatrics, 17th ed.)
D. Port-wine stain
E. Capillary hemangiomas

Question . 8. Nevus sebaceus (Jadassohn sebaceous nevus) is associated with all of


the following Except:

A. Yellow-orange-appearing plaque
B. Cardiac failure
Explanation: These initially benign-appearing plaques, usually seen on the
head or neck of infants, may develop into benign or malignant tumors in
adulthood. They should be resected before adulthood. (See Chapter 641 in
Nelson Textbook of Pediatrics, 17th ed.)
C. Risk of benign tumors
D. Risk of malignancy
E. Verrucous changes during adolescence

Question . 9. Recurrent erythema multiforme is associated with:

A. Human herpesvirus 6
B. Anxiety
C. Herpes simplex
Explanation: Both HSV herpes labialis and HSV herpes genitalis have been
implicated in the majority of cases of recurrent EM. (See Chapter 644 in
Nelson Textbook of Pediatrics, 17th ed.)
D. Psychosis
E. Mononucleosis

The Skin - Nelson Self Assessments website 17th Edition 3


Question . 10. Complications of Stevens-Johnson syndrome include all of the following Except:

A. Bullae and exfoliation


B. Corneal ulceration
C. Polyarthritis
D. Osteomyelitis
Explanation: Secondary local bacterial infections of the skin or sepsis may
occur. Antibiotics are indicated for proven infections or if there is a high index
of suspicion for infection. Steroids may increase the risk of infection.
Treatment for patients with Stevens-Johnson syndrome is similar to that for
patients with extensive burns. (See Chapter 644 in Nelson Pediatrics, 17th ed.)
E. Myocarditis
F. Hepatitis

Question . 11. A 15-yr-old girl who had new-onset seizures was started on
carbamazepine 4 wk ago. She now has a diffuse erythematous macular rash, fever,
lymphadenopathy, eosinophilic leukocytosis, and elevated values on liver function
tests. The most likely cause of these abnormalities is:

A. Erythema multiforme
B. Stevens-Johnson syndrome
C. Hypersensitivity syndrome
Explanation: The rash may look like any of the other choices (more often
choices B and E). However, hepatitis and lymphadenopathy are uncommon
findings with choice B or E. Hypersensitivity reactions to anticonvulsant drugs
occur 1-3 mo after initiation of therapy and probably represent a cell-mediated
immunologic reaction to the drug or a drug metabolite. (See Chapter 644)
D. Serum sickness
E. Toxic epidermal necrolysis

Question . 12. Onychomycosis is associated with all of the following Except:

A. T. rubrum invasive infection


B. T. mentagrophytes superficial infection
C. A good response to treatment with oral terbinafine
D. A good response to topical ketoconazole
Explanation: Topical therapy is of little value in fungal infections of the nail.
Prolonged oral therapy is needed to penetrate the growing nail, making it
resistant to fungal infections. (See Chapter 656 in Nelson Textbook of
Pediatrics, 17th ed.)
E. A good response to oral itraconazole

Question . 13. The treatment of choice for scabies is:


A. Crotamiton lotion
B. Permethrin 5% cream
Explanation: One percent lindane cream (or lotion) is also recommended but is
potentially neurotoxic. (See Chapter 658 in Nelson Pediatrics, 17th ed.)
C. Ultraviolet light
D. Salicylate baths
E. Topical gentamicin

The Skin - Nelson Self Assessments website 17th Edition 4


Question . 14. The treatment of choice for pediculosis capitis is:

A. Application of 0.5% malathion


Explanation: In addition, 1% permethrin cream is also a treatment of choice.
Malathion is not indicated in infants or neonates. Permethrin plus oral
trimethoprim-sulfamethoxazole is a promising therapy for hard-to-treat cases.
(See Chapter 658 in Nelson Textbook of Pediatrics, 17th ed.)
B. Application of 10% permethrin ointment
C. Brushing and combing
D. Hydrogen peroxide
E. Selenium-based shampoo

Question . 15. Giant congenital pigmented nevi are associated with all of the
following Except:

A. An incidence of less than 1 case in 20,000 births


B. Leptomeningeal involvement
C. Malignant melanoma
D. Hydrocephalus
E. Male predominance
Explanation: Giant pigmented nevi are difficult-to-manage lesions that affect
males and females with equal frequency. (See Chapter 641 in Nelson
Textbook of Pediatrics, 17th ed.)

Question . 16. Incontinentia pigmenti is associated with all of the following Except:

A. Lethality in females
Explanation: Incontinentia pigmenti is X-linked dominant and is lethal in
males during fetal life. (See Chapter 642 in Nelson Textbook of Pediatrics,
17th ed.)
B. Erythematous linear streaks and vesicles
C. Alopecia
D. Hypodontia
E. Microphthalmos
F. Seizures

Question . 17. Stevens-Johnson syndrome (erythema multiforme major) is associated


with all of the following Except:

A. A good response to prednisone


Explanation: There is no agreed-on indication for systemic steroids in Stevens-
Johnson syndrome. Topical steroids are indicated for ocular involvement to
prevent scarring. (See Chapter 644 in Nelson Textbook of Pediatrics, 17th ed.)
B. Involvement of two mucous membranes
C. Esophageal stricture
D. Corneal scarring
E. Infectious causes
F. Drug-related causes

The Skin - Nelson Self Assessments website 17th Edition 5


Question . 18. A well-appearing newborn has a lacy, reticulated, red and/or blue
cutaneous pattern over most of the body, which is prominent when the neonate is in a
cool environment. The correct diagnosis is:

A. Mongolian spots
B. Salmon patch
C. Cutis marmorata
Explanation: Cutis marmorata is an accentuated physiologic vasomotor
response that disappears with increasing age during the first year of life. (See
Chapter 637 in Nelson Textbook of Pediatrics, 17th ed.)
D. Harlequin color change
E. Erythema toxicum

Question . 19. A 2-day-old well-appearing full-term white neonate experiences


multiple firm, yellow-white, 1- to 2-mm papules or pustules with a surrounding
erythematous flare on the trunk. Wright stain of the lesions shows numerous
eosinophils. The most likely diagnosis is:

A. Erythema toxicum
Explanation: Erythema toxicum is a benign, self-limited, evanescent eruption
that occurs in approximately 50% of full-term infants; preterm infants are
affected less commonly. The lesions are firm, yellow-white, 1- to 2-mm
papules or pustules with a surrounding erythematous flare. Lesions may be
sparse or numerous and clustered in several sites or widely dispersed over
much of the body surface. Palms and soles are usually spared. Peak incidence
is on the second day of life, but new lesions may erupt during the first few
days as the rash waxes and wanes. (See Chapter 637 in Nelson Pedia, 17th ed.)
B. Pustular melanosis
C. Acropustulosis
D. Eosinophilic pustular folliculitis
E. Herpes simplex virus infection

Question . 20. The recommended therapy for port-wine nevus is:

A. Tattooing
B. Excision and skin grafting
C. Cryosurgery
D. Laser therapy
Explanation: The most effective treatment for port-wine stains is the
flashlamp-pumped-pulsed dye laser. This therapy is targeted at the lesion and
avoids thermal injury to the surrounding normal tissue. Alternative therapies
include masking with cosmetics, cryosurgery, excision, grafting, and tattooing.
(See Chapter 640 in Nelson Textbook of Pediatrics, 17th ed.)
E. Interferon-

The Skin - Nelson Self Assessments website 17th Edition 6


Question . 21. An 8-yr-old girl presents with large café-au-lait spots with irregular
borders and precocious puberty. X-ray shows polyostotic fibrous dysplasia of bone.
The most likely diagnosis is:

A. Neurofibromatosis
B. McCune-Albright syndrome
Explanation: Large, often asymmetric café-au-lait spots with irregular borders
are characteristic of McCune-Albright syndrome. This disorder also includes
polyostotic fibrous dysplasia of bone, leading to pathologic fractures;
precocious puberty; and numerous hyperfunctional endocrinopathies. (See
Chapter 642 in Nelson Textbook of Pediatrics, 17th ed.)
C. Tuberous sclerosis
D. Maffucci syndrome
E. Normal child

Question . 22. A black teenager presents with a sharply demarcated, dense, firm,
rubbery growth on the face at the site of a previous smaller laceration that was
incurred long ago. Which of the following is the most likely diagnosis?

A. Granuloma annulare
B. Keloid
Explanation: Keloids are usually induced by trauma and commonly follow ear
piercing, burns, scalds, and surgical procedures. Certain individuals, especially
blacks, seem predisposed to keloid formation. In both keloids and
hypertrophic scars, new collagen forms over a much longer period than in
wounds that heal normally. Unlike keloids, hypertrophic scars remain
confined to the site of injury and gradually involute over time. (See Chapter
649 in Nelson Textbook of Pediatrics, 17th ed.)
C. Necrobiosis lipoidica
D. Anetoderma
E. Mastocytosis

Question . 23. A young child has unusual skin hyperelasticity and joint hypermobility
of gradual onset. The skin snaps back into place when pulled. Which of the following
is the most likely diagnosis?

A. Cutis laxa
B. Ehlers-Danlos syndrome
Explanation: Ehlers-Danlos syndrome is a group of genetically heterogeneous
connective tissue disorders. Affected children appear normal at birth, but skin
hyperelasticity, fragility of the skin and blood vessels, and joint hypermobility
develop over time. The essential defect is a quantitative deficiency of
collagen. Ehlers-Danlos syndrome has been classified into 10 clinical forms.
(See Chapter 649 in Nelson Textbook of Pediatrics, 17th ed.)
C. Anetoderma
D. Pseudoxanthoma elasticum
E. Elastosis perforans serpingosa

The Skin - Nelson Self Assessments website 17th Edition 7


Question . 24. A child presents with multiple, discrete, skin-colored, 1- to 3-mm
dome-shaped smooth papules on the face with central umbilication. The most likely
diagnosis is:

A. Verruca vulgaris (common warts)


B. Verruca plana (flat warts)
C. Condylomata acuminata
D. Molluscum contagiosum
Explanation: The lesions of molluscum contagiosum are discrete, pearly, skin-
colored, dome-shaped, smooth papules that may range in size from 1 to 5 mm.
They typically have a central umbilication from which a plug of cheesy
material can be expressed. Papules of molluscum contagiosum may occur
anywhere on the body, but the face, eyelids, neck, axillae, and thighs are areas
of predilection. (See Chapter 657 in Nelson Textbook of Pediatrics, 17th ed.)
E. Keloid

Question . 25. Which of the following is the most serious adverse effect associated
with use of isotretinoin?
A. Hepatitis
B. Cardiomyopathy
C. Pseudotumor cerebri
D. Teratogenicity
Explanation: Isotretinoin has many adverse effects. It is teratogenic and is
contraindicated for use during pregnancy; pregnancy should be avoided for 1
mo after discontinuation of therapy. Most patients experience cheilitis, xerosis,
periodic epistaxis, and blepharoconjunctivitis. Increased serum triglyceride
and cholesterol levels are also common. Less common adverse effects include
arthralgias, myalgias, depression, temporary thinning of the hair, paronychia,
increased susceptibility to sunburn, formation of pyogenic granulomas, and
colonization of the skin with Staphylococcus aureus leading to impetigo,
secondarily infected dermatitis, and scalp folliculitis. (See Chapter 659 in
Nelson Textbook of Pediatrics, 17th ed.)
E. Carcinogenicity

Question . 26. A 2-yr-old child presents with symmetric psoriasiform skin lesions
distributed in the perioral, acral, and perineal areas and on the cheeks, knees, and
elbows. There is mild alopecia and a history of chronic diarrhea. The most likely
diagnosis is:
A. Psoriasis
B. Juvenile xanthogranuloma
C. Acrodermatitis enteropathica
Explanation: Acrodermatitis enteropathica is caused by an inability to absorb
sufficient zinc from the diet. Initial signs and symptoms occur during infancy
and consist of a rash in the perioral, acral, and perineal areas and on the
cheeks, knees, and elbows. There is often alopecia and chronic diarrhea. Some
patients with cystic fibrosis present with a similar rash. Biotinidase deficiency
should be in the differential diagnosis. Treatment of acrodermatitis
enteropathica is with oral zinc compounds. (See Chapter 661 in Nelson)
D. Scurvy
E. Pellagra

The Skin - Nelson Self Assessments website 17th Edition 8

You might also like